Anda di halaman 1dari 3495

FAMILY MEDICINE BOARD REVIEW EXPRESS

Live Course Syllabus

Board Review Express March 13-16, 2014 Indianapolis, Indiana


Day 1 - Thursday, March 13
7:00 - 8:00 am 8:00 - 8:30 am 8:30 - 9:30 am 9:30 - 9:45 am 9:45 - 10:00 am 10:00 - 10:30 am 10:30 - 11:00 am 11:00 - 11:30 am 11:30 - 12:00 pm 12:00 - 12:15 pm 12:15 - 1:15 pm 1:15 - 1:45 pm 1:45 - 2:15 pm 2:15 - 2:45 pm 2:45 - 3:00 pm 3:00 - 3:15 pm 3:15 - 3:45 pm 3:45 - 4:15 pm 4:15 - 4:45 pm 4:45 - 5:00 pm 5:00 - 5:45 pm 5:45 pm Breakfast Provided/Registration Welcome & Overview - David Weismiller, MD, ScM, FAAFP Health Promotion & Screening - David Weismiller, MD, ScM, FAAFP Q&A Break Obesity & Metabolic Syndrome - Belinda Vail, MD, FAAFP Maternity Care I - David Weismiller, MD, ScM, FAAFP Diabetes - Belinda Vail, MD, FAAFP Maternity Care II - David Weismiller, MD, ScM, FAAFP Q&A Lunch STI's, Vaginitis, and Vaginosis - David Weismiller, MD, ScM, FAAFP Preoperative Examination & Surgical Management - Belinda Vail, MD Common ENT Problems - William Geiger, MD, FAAFP Q&A Break Challenging Issues in Hematology - William Geiger, MD, FAAFP Fever and Infectious Diseases in Children - Belinda Vail, MD, FAAFP Common Neurological Disorders - William Geiger, MD, FAAFP Q&A Guide to Exam Preparation (no CME) - David Weismiller, MD, ScM, FAAFP Recess

Board Review Express March 13-16, 2014 Indianapolis, Indiana


Day 2 - Friday, March 14
7:00 - 8:00 am 8:00 - 8:30 am 8:30 - 9:00 am 9:00 - 9:30 am 9:30 - 9:45 am 9:45 - 10:00 am 10:00 - 10:30 am 10:30 - 11:00 am 11:00 - 11:30 am 11:30 - 12:00 pm 12:00 - 12:15 pm 12:15 - 1:15 pm 1:15 - 1:45 pm 1:45 - 2:15 pm 2:15 - 2:45 pm 2:45 - 3:15 pm 3:15 - 3:30 pm 3:30 - 3:45 pm 3:45 - 4:15 pm 4:15 - 4:45 pm 4:45 - 5:15 pm 5:15 - 5:45 pm 5:45 - 6:00 pm 6:00 pm Breakfast Provided Pulmonary Infections & Pneumovax - Dana King, MD, FAAFP Musculoskeletal Medicine - Joseph Garry, MD, FACSM, FAAFP COPD, Lung Cancer, OSA - Dana King, MD, FAAFP Q&A Break Review of the Diseases of the Upper GI Tract - David Weismiller, MD Fracture Care in Family Medicine - Joseph Garry, MD, FACSM, FAAFP Asthma: Pediatrics & Adult - Dana King, MD, FAAFP Sports Medicine - Joseph Garry, MD, FACSM, FAAFP Q&A Lunch Review of the Diseases of the Lower GI Tract - David Weismiller, MD Pediatric Orthopedics - Joseph Garry, MD, FACSM, FAAFP Common Newborn Issues - David Weismiller, MD, ScM, FAAFP Management of Chronic Pain - Gary Levine, MD, FAAFP Q&A Break Managing Common Cutaneous Problems Part 1 - Gary Levine, MD, FAAFP Well Child Care and Adolescent Issues - David Weismiller, MD, ScM Managing Common Cutaneous Problems Part 2 - Gary Levine, MD, FAAFP Selected Issues in Women's Health - David Weismiller, MD, ScM, FAAFP Q&A Recess

Board Review Express March 13-16, 2014 Indianapolis, Indiana


Day 3 - Saturday, March 15
7:00 - 8:00 am 8:00 - 8:30 am 8:30 - 9:00 am 9:00 - 9:30 am 9:30 - 9:45 am 9:45 - 10:00 am 10:00 - 10:30 am 10:30 - 11:00 am 11:00 - 11:30 am 11:30 - 12:00 pm 12:00 - 12:15 pm 12:15 - 1:15 pm 1:15 - 1:45 pm 1:45 - 2:15 pm 2:15 - 2:45 pm 2:45 - 3:15 pm 3:15 - 3:30 pm 3:30 - 3:45 pm 3:45 - 4:15 pm 4:15 - 5:00 pm 5:00 - 5:30 pm 5:30 - 5:45 pm 5:45 pm Breakfast Provided Abnormal Uterine Bleeding - David Weismiller, MD, ScM, FAAFP Renal I - Gary Levine, MD, FAAFP Common Issues in the Elderly Part 1 - Laurence Robbins, MD Q&A Break Common Issues in the Elderly Part 2 - Laurence Robbins, MD Renal II - Gary Levine, MD, FAAFP Acute & Chronic Cognitive Diseases - Laurence Robbins, MD Peripheral Vascular Disease - Robert Dachs, MD, FAAFP Q&A Lunch Acute CVA & TIA - Robert Dachs, MD, FAAFP Pharmacology in the Elderly - Laurence Robbins, MD The Surgical Abdomen - Robert Dachs, MD, FAAFP Managing Dysrhythmias - Jonathon Firnhaber, MD, FAAFP Q&A Break Trauma & Wound Care - Robert Dachs, MD, FAAFP ACS & Hyperlipidemia - Jonathon Firnhaber, MD, FAAFP Bites, Stings & Other Emergency Things - Robert Dachs, MD, FAAFP Q&A Recess

Board Review Express March 13-16, 2014 Indianapolis, Indiana


Day 4 - Sunday, March 16
6:45 - 7:15 am 7:15 - 7:45 am 7:45 - 8:15 am 8:15 - 8:45 am 8:45 - 9:15 am 9:15 - 9:30 am 9:30 - 9:45 am 9:45 - 10:15 am 10:15 - 10:45 am 10:45 - 11:15 am 11:15 - 11:45 am 11:45 - 12:00 pm 12:00 pm Breakfast Provided Behavioral Medicine I - Stanley Oakley, Jr, MD, FAPA Hypertension - Jonathon Firnhaber, MD, FAAFP The Major Arthritides - BW Blount, MD, FAAFP Heart Failure - Jonathon Firnhaber, MD, FAAFP Q&A Break Behavioral Medicine II - Stanley Oakley, Jr, MD, FAFP Endocrine Disease - BW Blount, MD, MPH, FAAFP Behavioral Medicine III - Stanley Oakley, Jr, MD, FAFP Urologic Problems - BW Blount, MD, MPH, FAAFP Q&A Adjourn

Instructions for Notetaking on a PDF file


1. You must have a current version of Adobe Reader to use the note taking function. If you do not have a current version of Adobe Reader, a free download at http://get.adobe.com/reader/ . Please proceed to download by following the prompts. 2. If the slides are too small on your screen, you may enlarge the slide by going to the zoom feature in your Adobe Reader toolbar. 3. You must install the files to your hard drive and work from those files if you want to save your notes. Open the presentation from your hard drive and right-click on the slide where you want to note to appear. You may type as small or as large of a note as you please. You will want to click on Save to save your notes before closing. It is recommended to do this periodically so that you are assured that your notes are preserved. 4. When you go back to review your slides, a note icon will appear where you have entered a note. Double-click the note icon and your note will open.

Faculty List
David Weismiller, MD, ScM, FAAFP, Course Chair, Professor, Department of Family Medicine, Brody School of Medicine, East Carolina University, Greenville, North Carolina B Wayne Blount, MD, MPH, FAAFP, Professor, Department of Family and Preventative Medicine, Emory University School of Medicine, Atlanta, Georgia

Robert Dachs, MD, FAAFP, Vice Chairman, Department of Emergency Medicine, Ellis Hospital, Schenectady, New York; Clinical Assistant Professor, Ellis Hospital Family Medicine Residency Program, Albany Medical college, Albany, New York Jonathon Firnhaber, MD, FAAFP, Assistant Professor, Residency Director, Department of Family Medicine, Brody School of Medicine at East Carolina University, Greenville, North Carolina

Faculty List (cont.)


Joseph Garry, MD, FAAFP, FACSM, Associate Professor, Sports Medicine Division, Department of Family Medicine and Community Health, University of Minnesota, Minneapolis William J Geiger, MD, FAAFP, Associate Program Director, Grant Family Medicine Residency, OhioHealth, Columbus, Ohio Dana King, MD, MS, FAAFP, Professor and Chair, Department of Family Medicine, West Virginia University Health Science Center, Morgantown, West Virginia Gary I Levine, MD, FAAFP, Associate Professor, Department of Family Medicine, Brody School of Medicine at East Carolina University, Greenville, North Carolina

Faculty List (cont.)


Stanley Oakley, MD, FAPA, Professor and Director of Continuing Medical Education, Department of Psychiatric Medicine, Brody School of Medicine, Greenville, North Carolina Laurence J. Robbins, MD, Associate Professor of Medicine, University of Colorado School of Medicine, Denver, Colorado Belinda Vail, MD, FAAFP, Vice Chair, Department of Family Medicine, University of Kansas Medical School, Kansas City, Kansas

AAFP Learner Assurance Statement/AAFP Policy on Disclosure and Resolution of Conflict of interest.
It is the policy of the AAFP that all CME planning committee/faculty/authors/staff disclose relationships with commercial entities upon nomination/invitation of participation. Disclosure documents are reviewed for potential conflict of interest and, if identified, they are resolved prior to confirmation of participation. Only those participants who have no conflict or interest or who agree to an identified resolution process prior to their participation were involved in this CME activity.

Disclosures
The AAFP has selected and provides funding for all faculty appearing in this program. According to AAFP policy, all relationships between speakers and proprietary entities that may have a direct interest in the subject matter of their presentation will be disclosed.ill be

Disclosures
The following faculty and staff have returned disclosure forms indicating that they have no affiliation of financial interest in any organization(s).

B Wayne Blount, MD - Faculty Robert Dachs, MD - Faculty Jonathon Firnhaber, MD - Faculty William Geiger, MD Faculty Dana King, MD - Faculty Gary Levine, MD - Faculty Stanley Oakley, MD - Faculty Laurence Robbins, MD - Faculty Belinda Vail, MD - Faculty David Weismiller, MD - Faculty

Gary Bird- AAFP Staff Elizabeth Campbell AAFP Staff Ann Karty AAFP Staff Terry Thompson AAFP Staff Sherri Woodward AAFP Staff

Disclosures
The following faculty and staff have returned disclosure forms indicating that they have or an immediate family member has a financial relationship with or interest in a commercial interest: Joseph Garry, MD Stock/Bond Holdings, Pfizer (Analgesics, Bone Health); Stock/Bond Holdings, Merck (Analgesics, Bone Health)

Course Disclaimer
The material presented at this course is being made available by the American Academy of Family Physicians for educational purposes only. This material is not intended to represent the only, nor necessarily best, methods or procedures appropriate for the medical situations discussed, but rather is intended to present an approach, view, statement or opinion of the faculty which may be helpful to others who face similar situations. The AAFP disclaims any and all liability for injury or other damages resulting to any individual attending this course and for all claims which may arise out of the use of the techniques demonstrated therein by such individuals, whether these claims shall be asserted by a physician or any other person. Every effort has been made to assure the accuracy of the data presented at this course. Physicians may care to check specific details such as drug doses and contraindications, etc., in standard sources prior to clinical application. This material may contain recommendations/guidelines developed by other organizations. Please note that although these guidelines may be included, this does not necessarily imply the endorsement by the AAFP.

Learning Objectives
At the end of the AAFP Board Review Express course, you should be able to: 1.Demonstrate an understanding of common clinical problems seen in family medicine. 2.Analyze your training needs on conditions or elements within specific body systems, as defined by the ABFM. 3.Outline effective test-taking strategies to prepare for the ABFM exam.

Core Competencies
The AAFP's CME Programs department provides numerous opportunities for family physicians to acquire knowledge and skills that are critical in today's health care system. Professional competencies for family physicians include medical knowledge, interpersonal and communication skills, patient care, professionalism, practice-based learning and improvement and systems-based practice. All CME activities allow family physicians to add to their expertise and knowledge in one or more competency areas.

CME Credit
This activity, the Family Medicine Board Review Express, with a beginning date of March 13, 2014 has been reviewed and is acceptable for up to 28.75 Prescribed credits by the American Academy of Family Physicians. Physicians should claim only the credit commensurate with the extent of their participation in the activity.

Epidemiology Video
Please follow the link [below] to access the Epidemiology lecture. You will be able to take a short quiz and claim credit upon completion.
https://lms.aafp.org/course/epidemiology-0

Now Online: CME Credit Reporting and Evaluations


Bookmark it!
www.aafp.org/reportcme/boardrev/indianapolis

1.Log in with your username and password. 2.Complete all attended session evaluations and the course evaluation. 3.Once you submit, your CME credits will be added to your record instantly.

Board Review Express March 13-16, 2014 Indianapolis, Indiana


Day 1 - Thursday, March 13
7:00 - 8:00 am 8:00 - 8:30 am 8:30 - 9:30 am 9:30 - 9:45 am 9:45 - 10:00 am 10:00 - 10:30 am 10:30 - 11:00 am 11:00 - 11:30 am 11:30 - 12:00 pm 12:00 - 12:15 pm 12:15 - 1:15 pm 1:15 - 1:45 pm 1:45 - 2:15 pm 2:15 - 2:45 pm 2:45 - 3:00 pm 3:00 - 3:15 pm 3:15 - 3:45 pm 3:45 - 4:15 pm 4:15 - 4:45 pm 4:45 - 5:00 pm 5:00 - 5:45 pm 5:45 pm Breakfast Provided/Registration Welcome & Overview - David Weismiller, MD, ScM, FAAFP Health Promotion & Screening - David Weismiller, MD, ScM, FAAFP Q&A Break Obesity & Metabolic Syndrome - Belinda Vail, MD, FAAFP Maternity Care I - David Weismiller, MD, ScM, FAAFP Diabetes - Belinda Vail, MD, FAAFP Maternity Care II - David Weismiller, MD, ScM, FAAFP Q&A Lunch STI's, Vaginitis, and Vaginosis - David Weismiller, MD, ScM, FAAFP Preoperative Examination & Surgical Management - Belinda Vail, MD Common ENT Problems - William Geiger, MD, FAAFP Q&A Break Challenging Issues in Hematology - William Geiger, MD, FAAFP Fever and Infectious Diseases in Children - Belinda Vail, MD, FAAFP Common Neurological Disorders - William Geiger, MD, FAAFP Q&A Guide to Exam Preparation (no CME) - David Weismiller, MD, ScM, FAAFP Recess

2014

Board Review Express

Course Objectives
Discuss common clinical problems in family medicine Summarize an evidence-based approach to current advances in the diagnosis and treatment of common clinical problems Demonstrate successful study and test-taking techniques

Board Review Express


Engaged/Active
ARS Answers to embedded questions

Presentation handouts
Flashdrive

Faculty

Registration Packet
Course Evaluation/CME Reporting Form
Link listed on copy of program and at end of PDF file for each day Be constructive

Badge

Recertification Examination
ABFM
Exam Format
All multiple choice 4 to 5 options
Stem and options (A, B, C, D, E)

Five sections Three general portions


Section 1 - 120 Multiple Choice Questions (120 minutes) Section 4 80 multiple choice questions (95 minutes) Section 5 - 80 multiple choice questions (95 minutes)

Two module choices


Selected on exam day during the exam 45 questions each (45 minutes/module)

ABFM Certification/Recertification Examination Content


Cardiovascular Endocrine Gastrointestinal Heme/Immune Integumentary Musculoskeletal Nephrologic Neurologic Nonspecific Psychogenic ReprodFemale ReprodMale Respiratory 9% 6% 5% 2% 5% 9% 2% 2% 7% 5% 3% 1% 10% Special Sensory Population-based Care 2% 3%
Epidemiology, EBM, prevention, health policy & legal issues, bioterror, quality improvement; geographic/urban/rural issues

Patient-based Systems

3%

Clinical decision-making, communication and doctor-patient interaction, family & cultural issues, ethics, palliative care; end-of-life care

Module 1* Module 2*

13% 13%

Link to Epidemiology
Epidemiology Evidence-based medicine Clinical decision-making

Exam Modules*
Ambulatory Family Medicine Child and Adolescent Care Geriatrics Womens Health Maternity Care Emergent/Urgent Care Hospital Medicine Sports Medicine

Recertification Examination
ABFM
Exam Format
All multiple choice 4 to 5 options Two general portions Two module choices

Exam Tutorial
www.theabfm.org Preview of the look and feel of a computerized examination

Recertification Examination
ABFM
Exam Format
All multiple choice 4 to 5 options Two general portions Two module choices

Exam Tutorial
www.theabfm.org Preview of the look and feel of a computerized examination

Guide to Exam Preparation

Exam Preparation
Board Review Express
Listen loudly Tables and summary slides (Study Folder)

Study the material when you return home (Primary)


AAFP Recommended Curriculum Guidelines for Family Medicine Residents
http://www.aafp.org/medical-schoolresidency/program-directors/curriculum.html

Review questions are secondary


www.aafp.org/boardreview/questions

Other Housekeeping
Questions
Write Pass to center aisle Put speakers name on outside

Silence cell phones End of day


Pass your ARS keypads to the outside aisle

Write your name in your syllabus Issues


Material/Exam - Faculty Course AAFP Staff

SORT Levels of Evidence USPSTF

GUIDANCE WITH NOMENCLATURE

SORT
Strength of Recommendation Taxonomy

Category A: Recommendation based on consistent and good-quality patient-oriented evidence. Category B: Recommendation based on inconsistent or limited quality patient-oriented evidence. Category C: Recommendation based on consensus, usual practice, opinion, diseaseoriented evidence-based series for studies of diagnosis, treatment, prevention, or screening.

Levels of Evidence
A
Meta Analysis of RCT A statistical analysis that combines or integrates the results of several independent clinical trials considered by the analyst to be "combinable" usually to the level of reanalyzing the original data, also sometimes called: pooling, quantitative synthesis. Review of a body of data that uses explicit methods to locate primary studies, and explicit criteria to assess their quality. Systematic Review of RCT

High quality RCT

Individuals are randomly allocated to a control group and a group who receive a specific intervention. Otherwise the two groups are identical for any significant variables. They are followed up for specific end points.
True positive rates and true negative rates for diagnostic tests. Groups of people are selected on the basis of their exposure to a particular agent and followed up for specific outcomes. "Cases" with the condition is matched with "controls", and a retrospective analysis used to look for differences between the two groups. Survey or interview of a sample of the population of interest at one point in time

Sensitivity and specificity (test)

B B

Cohort study Case control study Cross sectional study

B
C C

Case report or case series


Expert opinion Anecdote

A report based on a single patient or subject; sometimes collected together into a short series.
A consensus of experience from the good and the great A conversation

Definitions of USPSTF Recommendation Grades


Grade Definition Suggestions for Practice

A B

The USPSTF recommends the service; there is high certainty that the net benefit (i.e., benefits minus harms) is substantial The USPSTF recommends the service; there is high certainty that the net benefit is moderate or there is moderate certainty that the benefit is moderate to substantial The USPSTF recommends against routinely providing the service; there may be considerations that support providing the service in an individual patient; there is moderate or high certainty that the service has no net benefit or that the harms outweigh the benefits

Offer/provide this service Offer/provide service

Offer/provide the service only if there are other considerations in support of offering/providing the service in an individual patient

The USPSTF recommends against the service; there is moderate or Discourage the use of this high certainty that the service has no benefit or that the harms service outweigh the benefits The USPSTF concludes that the current evidence is insufficient to assess the balance of benefits and harms of the service; evidence is lacking, of poor quality, or conflicting, and the balance of benefits and harms cannot be determined If offered, patients should understand the uncertainty about the balance of benefits and harms

Health Promotion and Screening


David G Weismiller, MD, ScM, FAAFP Department of Family Medicine The Brody School of Medicine at East Carolina University
weismillerd@ecu.edu

AAFP Reprint No. 267 Health Promotion and Disease Prevention

Disclosure Statement
Dr Weismiller has nothing to disclose.

The AAFP has selected all faculty appearing in this program. It is the policy of the AAFP that all CME planning committees, faculty, authors, editors, and staff disclose relationships with commercial entities upon nomination or invitation of participation. Disclosure documents are reviewed for potential conflicts of interest and, if identified, they are resolved prior to confirmation of participation. Only those participants who had no conflict of interest or who agreed to an identified resolution process prior to their participation were involved in this CME activity.

Learning Objectives
1. Describe the differences between health promotion, prevention, and screening. 2. Recognize the three leading causes of morbidity in the United States. 3. Counsel patients on necessary lifestyle modifications to maintain health. 4. Reinforce the necessity of patient education and counseling for health promotion, including healthy diets, exercise, and smoking cessation.

Health Promotion and Screening


Effective health promotion
Lifestyle modification: 3 leading causes of morbidity in the US Counseling Quaternary
Set of health activities to mitigate or avoid the consequences of unnecessary or excessive intervention of the health system. It is the practice of first do no harm.

Prevention
Primary
Immunizations

Secondary
Breast cancer

Screening
Done in asymptomatic persons, typically secondary prevention

Tertiary
Congestive heart failure

HEALTH PROMOTION

Topic
Access to Health Services Clinical Preventive Services

Healthy People 2020 Indicator


Persons with medical insurance Persons with a usual primary care provider Adults who receive a colorectal cancer screening based on the most recent guidelines Adults with hypertension whose blood pressure is under control Adult diabetic population with an A1c value greater than 9 percent Children aged 19 to 35 months who receive the recommended doses of DTaP, polio, MMR, Hib, hepatitis B, varicella, and PCV vaccines

Environmental Quality Injury and Violence Maternal, Infant, and Child Health Mental Health

Air Quality Index (AQI) exceeding 100 Children aged 3 to 11 years exposed to secondhand smoke Fatal injuries Homicides Infant deaths Preterm births Suicides (MHMD-1) Adolescents who experience major depressive episodes

Nutrition, Physical Activity, and Obesity

Adults who meet current federal physical activity guidelines for aerobic physical activity and musclestrengthening activity Adults who are obese Children and adolescents who are considered obese Total vegetable intake for persons aged 2 years and older
Persons aged 2 years and older who used the oral healthcare system in past 12 months Sexually active females aged 15 to 44 years who received reproductive health services in the past 12 months Persons living with HIV who know their serostatus Students who graduate with a regular diploma 4 years after starting 9th grade Adolescents using alcohol or any illicit drugs during the past 30 days Adults engaging in binge drinking during the past 30 days Adults who are current cigarette smokers Adolescents who smoked cigarettes in the past 30 days

Oral Health Reproductive and Sexual Health

Social Determinants Substance Abuse Tobacco

Health Promotion
Risk stratification
Age, gender, family history, SES, lifestyle choices, environmental factors, and medical issues

Counseling
Reading the patient correctly
Soft-sell Direct approach

USPSTF recommends that prevention be discussed at each patient visit.


http://www.ahrq.gov/clinic/pocketgd.htm

Patient education

Steps in Administering Health Promotion Counseling


Define health risks. Determine the stage of readiness of the patient. Advocate and commend behavior change. Assist in identification of a target behavior; identify barriers versus benefits. Reinforce health benefits of behavior change. Offer resources, strategies, and support; create plan of action and monitoring mechanisms.
Hensrud DD. Clinical preventive medicine in primary care: background and practice: 2. Delivering primary preventive services. Mayo Clin Proc. March 2000;75:255-64.

Barriers
Practicalities of organizing staff and practice to systematically implement Reaching affected patients in a practice or community; limited systems to address prevention during every visit with every patient Time and reimbursement for prevention remain major issues improving

USPSTF
The USPSTF was convened by the Public Health Service to rigorously evaluate clinical research in order to assess the merits of preventive measures, including screening tests, counseling, immunizations, and preventive medications. www.uspreventiveservicestaskforce.org/uspstopics.htm
Topic Index (A to Z) Recommendations for adults Recommendations for children and adolescents Affordable Care Act: USPSTF A and B Recommendations Topics in Progress

Definitions of USPSTF Recommendation Grades


Grade Definition Suggestions for practice

A B

The USPSTF recommends the service; there is high certainty that the net benefit (ie, benefits minus harms) is substantial. The USPSTF recommends the service; there is high certainty that the net benefit is moderate or there is moderate certainty that the benefit is moderate to substantial.

Offer/provide this service. Offer/provide service.

The USPSTF recommends against routinely providing the service; Offer/provide the service only if there may be considerations that support providing the service in an there are other considerations in individual patient; there is moderate or high certainty that the service support of offering/providing the has no net benefit or that the harms outweigh the benefits.service in an individual patient. The USPSTF recommends against the service; there is moderate or Discourage the use of this high certainty that the service has no benefit or that the harms service. outweigh the benefits. The USPSTF concludes that the current evidence is insufficient to assess the balance of benefits and harms of the service; evidence is lacking, of poor quality, or conflicting, and the balance of benefits and harms cannot be determined. If offered, patients should understand the uncertainty about the balance of benefits and harms.

1. The number one cause of morbidity in the United States today is:
A. Poverty B. Tobacco C. Unprotected sex D. Alcohol dependence E. Overweight/Obesity

We All Know It!


Smoking, obesity trim life expectancy
USA Today, January 26, 2011
(http://www.usatoday.com/yourlife/health/2011-01-25-smoking-obesity-life-expectancy_N.htm)

US Major Health Indicators


Indicator Adults who are current smokers (2010) Obese adults (2010) Physically inactive adults (2009) Incidence of syphilis, gonorrhea, and chlamydial cases per 100,000 (2008) Adults with alcohol and illicit drug abuse or dependence (2006-2007) Uninsured (ages 19-64 years) (2010)

Prevalence % 17.3 27.5 49 517.4/100,000


9.2 17.8

From: QuickStats: Number of Deaths from 10 Leading CausesNational Vital Statistics System, United States, 2010 JAMA. 2013;309(15):1582-1582.

MMWR. 2013;62:155.
Date of download: 4/22/2013

Copyright 2012 American Medical Association. All rights reserved.

Highest Ranked Services with the Lowest Delivery Rates


Tobacco cessation counseling to adults Screening older adults for undetected vision impairment Offering adolescents an anti-tobacco message or advice to quit Counseling adolescents on alcohol and drug abstinence Screening adults for colorectal cancer Screening young women for chlamydial infection Screening adults for problem drinking Vaccinating older adults against pneumococcal disease

USPSTF: Smoking (2009)


The USPSTF recommends that clinicians ask all adults about tobacco use and provide tobacco cessation interventions for those who use tobacco products. Grade: A recommendation The USPSTF recommends that clinicians ask all pregnant women about tobacco use and provide augmented, pregnancy-tailored counseling for those who smoke. Grade: A recommendation

USPSTF: Tobacco Use


August 2013
Recommends that primary care clinicians provide interventions, including education or brief counseling, to prevent initiation of tobacco use in school-aged children and adolescents. Grade: B recommendation

Prevalence of Interest in Quitting


National Health Interview Survey, United States, 2010
Interested in quitting Past year quit attempt % 52.4 (95% CI) (50.7-54.0) Recent smoking cessation % (95% CI) 6.2 (5.4-7.0)

Characteristic Overall

% 68.8

(95% CI) (67.2-70.5)

Of the 52.4% who had tried: 68.3% did so without evidence-based cessation counseling or medications. Only 48.3% who had visited a healthcare provider in the past year received advice to quit smoking. Only 31.7% had used counseling and/or medications when they tried to quit. 30% had used medications. 5.9% had used counseling.

Obesity
Adults
BMI > 25 is overweight. BMI > 30 is obese.

Pediatrics
Risk for overweight: BMI at or above the 85th percentile and lower than the 95th percentile for children of the same age and sex. Overweight: BMI at or above the 95th percentile for children of the same age and sex.

Obesity Trends* among US Adults BRFSS, 1990, 2000, 2010


(*BMI 30, or about 30 lbs overweight for 54 person) 1990 2000

2010

No Data

< 10%

10%14%

15%19%

20%24%

25%29%

30%

Weight of the Nation


CDC (National Premier. May 14 and 15, 2012)

Public health campaign from CDC in conjunction with new IOM report (May 8, 2012) on solutions to the obesity crisis
THE WEIGHT OF THE NATION, A MULTI-PART, MULTI-PLATFORM SERIES ADDRESSING THE NATIONAL OBESITY EPIDEMIC (HBO) The feature films and the 10 shorts will stream free of charge on HBO.com (http://HBO.com). The films are also available on YouTube for embedding and sharing.

IOM Goals
Weight of the Nation
Integrating physical activity into peoples daily lives Making healthy food and beverage options available everywhere Transforming marketing and messages about nutrition and activity Making schools a gateway to healthy weights Galvanizing employers and healthcare professionals to support healthy lifestyles

IOM
Specific Strategies
Requiring at least 60 minutes per day of physical education and activity in schools Industry-wide guidelines on which foods and beverages can be marketed to children and how Expansion of workplace wellness programs Taking full advantage of physicians roles to advocate for obesity prevention with patients and in the community Increasing the availability of lower-calorie, healthier childrens meals in restaurants

2. The 2008 Physical Activity Guidelines for Americans published by the US DHHS recommend which one of the following for adults?
A. Spreading out physical activity over the course of 2 weeks B. Alternating between muscle strengthening exercise and aerobic exercise every other week C. A weekly minimum of 120 minutes of moderateintensity aerobic activity (eg, brisk walking) if that is the type of physical activity chosen D. Working all major muscle groups on 2 or more days during weeks when muscle-strength training exercise is performed

US Physical Activity, 2009


Indicator Prevalence 51%

Recommended physical activity among adults aged > 18 years


Recommended physical activity among youth

18.4%
32.8%

Television viewing among youth (students in grades 9-12 who report watching television for 2 or fewer hours on an average school day)

The 2008 Physical Activity Guidelines for Americans


Recommendations Aerobic activity every week for adults (SOR C)
Moderate aerobic activity: 150 min/wk (brisk walking), or Vigorous aerobic activity: 75 min/wk (jogging or running), or An equivalent mix of moderate and vigorous-intensity aerobic activity If meeting the minimums, slowly increasing the amount of time will increase the benefits. ***Even 10-minute intervals can be beneficial.
2008 Physical Activity Guidelines for Americans. US Dept of Health and Human Services, 2008, ODPHP pub no U0036.

The 2008 Physical Activity Guidelines for Americans


Recommendations

Muscle strengthening activity every week (SOR C)


All major muscle groups 2 or more days/wk
Legs Hips Back Abdomen Chest Shoulders Arms

US Preventative Services Task Force


Recommendations (2002)
Insufficient evidence to determine whether routine counseling to promote physical activity for all patients in primary care settings leads to sustained increases in physical activity among adult patients (Grade I).
Controlled trials were variable and with mixed results.

No trials with children or adolescents Did not look at how physical activity reduced chronic disease (well documented)

American Cancer Society Guidelines on Nutrition and Physical Activity for Cancer Prevention, 2012
Achieve and maintain a healthy weight throughout life.
Be as lean as possible throughout life without being underweight. Avoid excess weight gain at all ages. For those who are currently overweight or obese, losing even a small amount of weight has health benefits and is a good place to start. Engage in regular physical activity and limit consumption of high-calorie foods and beverages as key strategies for maintaining a healthy weight. Adults should engage in at least 150 minutes of moderate intensity or 75 minutes of vigorous intensity activity each week, or an equivalent combination, preferably spread throughout the week. Children and adolescents should engage in at least 1 hour of moderate or vigorous intensity activity each day, with vigorous intensity activity occurring at least 3 days each week. Limit sedentary behavior such as sitting, lying down, watching television, or other forms of screen-based entertainment. Doing some physical activity above usual activities, no matter what one's level of activity, can have many health benefits. Choose foods and beverages in amounts that help achieve and maintain a healthy weight. Limit consumption of processed meat and red meat. Eat at least 2.5 cups of vegetables and fruits each day. Choose whole grains instead of refined grain products. Drink no more than 1 drink per day for women or 2 per day for men.

Adopt a physically active lifestyle.


Consume a healthy diet, with an emphasis on plant foods.


If you drink alcoholic beverages, limit consumption.

Kushi LH, Doyle C, McCullough M, Rock CL, Demark-Wahnefried W, Bandera EV, Gapstur S, Patel AV, Andrews K, Gansler T, American Cancer Society 2010 Nutrition and Physical Activity Guidelines Advisory. American Cancer Society guidelines on nutrition and physical activity for cancer prevention: Reducing the risk of cancer with healthy food choices and physical activity. CA Cancer J Clin 2012. Jan-Feb;62(1):30-67.

Pediatrics and Overweight


Body mass index (BMI) correlates well with laboratory measurements of body fat. Its main use is in tracking whether a child is underweight or overweight. Unlike weight, it does not steadily increase with age. It is gender- and age-specific and declines to a nadir in the first 4-6 years of life; it steadily increases after that.

BMI
Weight status category Underweight Healthy weight Risk for overweight* Overweight* Percentile range < 5th percentile 5th percentile to < 85th percentile 85th to < 95th percentile > 95th percentile

* In this population, because of the possible negativity associated with the word obese, the term overweight is used instead.

When to Measure BMI?


Current recommendations: Yearly in children starting at age 2
This is used to identify excessive rates of weight gain relative to linear growth; can indicate the need to counsel parents and children on healthy eating and physical activity (SOR C).

No evidence that tracking BMI, or any specific measures, prevents childhood obesity (SOR A)

Overweight/Obesity
Prevalence of obesity and overweight is increasing in child populations throughout the world, impacting short- and long-term health.
Increased energy content of the diet Decreased levels of physical activity Increasingly sedentary lifestyles

USA
25% in 85th to 95th percentile BMI 7% > 95th percentile BMI

Being overweight or obese can have a significant effect on both the physical and emotional health of children.

Risks of Childhood Obesity


Physical Complications
Type 2 diabetes Metabolic syndrome Hypercholesterolemia Hypertension Asthma/breathing problems Sleep disorders including apnea Early puberty or menstruation

Social/Emotional Complications
Low self-esteem Bullying Behavior and learning problems Depression

Interventions
Benefit of behavior therapy may be increased if parents, rather than the child, are given the primary responsibility for behavior change. There have been many trials that focused on changing levels of physical activity and/or sedentary behavior, but they have been too small to provide conclusive evidence. While physical activity is universally recommended because of its proven health benefits, the contribution to weight loss is not as clear in childhood.
Children should be encouraged to increase their levels of physical activity, even if there is no great benefit in terms of weight reduction.

USPSTF: Obesity
Recommends screening all adults for obesity (Grade B Recommendation) 2012
Clinicians should offer or refer patients with a body mass index (BMI) of 30 kg/m2 or higher to intensive, multicomponent behavioral interventions.

Recommends that clinicians screen children aged 6 years and older for obesity and offer them or refer them to comprehensive, intensive behavioral interventions to promote improvement in weight status (Grade B Recommendation) 2010

ALCOHOL

3. Which of the following statements is true regarding alcohol abuse counseling?


A. The CAGE but NOT the AUDIT tool has been validated as a screening instrument for adult alcohol abuse. B. The US Preventive Services Task Force (USPSTF) recommends screening and counseling adolescents on the risks of alcohol misuse. C. The USPSTF recommends screening and counseling adults on the risks of alcohol misuse. D. While the USPSTF found that screening can accurately identify adults at risk for alcohol misuse, they found insufficient evidence of effectiveness for brief, office-based interventions.

Highest-Ranked Services with the Lowest Delivery Rates


Tobacco cessation counseling to adults Screening older adults for undetected vision impairment Offering adolescents an anti-tobacco message or advice to quit Counseling adolescents on alcohol and drug abstinence Screening adults for colorectal cancer Screening young women for chlamydial infection Screening adults for problem drinking Vaccinating older adults against pneumococcal disease

Alcohol Use
Definitions of patterns of drinking alcohol
From: Google Images

Excessive drinking includes heavy drinking, binge drinking, and any drinking by pregnant women or underage youth. Acceptable
Men < 2 drinks per day Women < 1 drink per day

Heavy
For women, more than 1 drink per day on average For men, more than 2 drinks per day on average

Binge, the most common form of excessive alcohol consumption


For women, 4 or more drinks during a single occasion For men, 5 or more drinks during a single occasion

Most people who binge drink are not alcoholics or alcohol dependent.

Alcohol and Adults


USPSTF 2013
Recommends that clinicians screen adults aged 18 years or older for alcohol misuse and provide persons engaged in risky or hazardous drinking with brief behavioral counseling interventions to reduce alcohol misuse. Grade: B Recommendation Concludes that the current evidence is insufficient to assess the balance of benefits and harms of screening and behavioral counseling interventions in primary care settings to reduce alcohol misuse in adolescents. Grade: I Statement

Validated Instruments
Alcohol Abuse

The CAGE and AUDIT tools are two of several validated instruments that can be used in primary care settings to screen for alcohol abuse (SOR A).

Acceptable limit of alcohol: Men < 2 drinks per day, women < 1 drink per day

CAGE
CAGE Questionnaire (PDF) CAGE test scores > 2 had a sensitivity of 93% and a specificity of 76% for the identification of problem drinkers.
1. Have you ever felt you needed to Cut down on your drinking? 2. Have people Annoyed you by criticizing your drinking? 3. Have you ever felt Guilty about drinking? 4. Have you ever felt you needed a drink first thing in the morning (Eye-opener) to steady your nerves or to get rid of a hangover?
John A. Ewing. Detecting Alcoholism: The CAGE Questionnaire. JAMA. 1984.

AUDIT
AUDIT: The Alcohol Use Disorders Identification Test: Guidelines for Use in Primary Care, second edition, by Thomas F Babor, John C Higgins-Biddle, John B Saunders, and Maristela G Monteiro.

Ten-question test developed by the World Health Organization to determine if a person's alcohol consumption may be harmful Test designed to be used internationally; validated in a study using patients from six countries. Questions:
1-3 deal with alcohol consumption 4-6 relate to alcohol dependence 7-10 consider alcohol-related problems

Scoring
A score of 8 or more in men (7 in women) indicates a strong likelihood of hazardous or harmful alcohol consumption. A score of 20 or more is suggestive of alcohol dependence.

Adverse Effects of Excessive Alcohol


Long-term health risks: Over time, excessive alcohol use can lead to the development of chronic diseases, neurological impairments, and social problems.
Neurological problems, including dementia, stroke, and neuropathy Cardiovascular problems, including myocardial infarction, cardiomyopathy, atrial fibrillation, and hypertension Psychiatric problems, including depression, anxiety, and suicide Social problems, including unemployment, lost productivity, and family problems Cancer of the mouth, throat, esophagus, liver, colon, and breast In general, the risk of cancer increases with increasing amounts of alcohol. Liver diseases, including:
Alcoholic hepatitis Cirrhosis, which is among the 15 leading causes of all deaths in the United States Among persons with hepatitis C virus, worsening of liver function and interference with medications used to treat this condition

Other gastrointestinal problems, including pancreatitis and gastritis

Alcohol Use and CVA


Effects of alcohol on stroke risk are controversial but:
The negative effects of heavy use (> 5/d) are well documented. Heavy use increases the risk for all forms of stroke not just ischemic.

Secondary Stroke Prevention


The American Heart Association/American Stroke Association 2006 guideline on stroke prevention in patients with a previous stroke or TIA, lists elimination or reduction of alcohol consumption in heavy drinkers as one of the primary goals.

Heavy Alcohol Consumption: CVA


Mechanisms for the negative effects
More vulnerable to cerebral atrophy Atrial fibrillation Reduced cerebral blood flow Alcohol-induced hypertension Hypercoagulable state

Light to Moderate Alcohol Consumption: CVA


Mechanisms for reduced risk of stroke
Increases in HDL Decreases in platelet aggregation Lower plasma fibrinogen concentration

Primary Secondary (Screening) Tertiary Quaternary

PREVENTION

Prevention
Primary
Avoids the development of a disease. Most population-based health promotion activities are primary preventive measures. Immunizations Estimated that 50,000 lives could be saved per year if the ACIP immunization schedule was followed

General Principles
Serious side effects are exceedingly rare. Several studies: Physicians recommendations can make a huge difference in whether a patient/child is immunized. Every visit is an opportunity for primary prevention. Immunization series do not need to be restarted. Breastfeeding is NOT a contraindication to vaccines. Never restart a vaccine series.

General Principles
Successful dialogue
Take time to LISTEN. Solicit and welcome questions. Keep the conversation going.

www.aafp.org/immunizations
Ages 0-18 Adult
Medical/other indications

ACIP 2014 (0-18 years)

ACIP 2014 Immunization Schedule Adult


Tdap (2013)
Each and every pregnancy; third trimester, preferably between 27 and 36 w Recommended universally for all adults, including those aged > 65 y

Pneumococcal vaccination (2014)


Patients with immunocompromising conditions, HIV, asplenia, or end-stage renal disease are indicated to receive both 23-valent pneumococcal polysaccharide vaccine (PPSV23; Pneumovax 23, Merck) and the 13-valent pneumococcal conjugate vaccine (PCV13; Prevnar 13, Pfizer). Schedule emphasizes that PCV13 be given first, followed by PPSV23

Addition of the Haemophilus influenzae type b vaccine for immunocompromised adults, particularly for those after stem cell transplantation (2014)

2014 ACIP Adult Immunization Schedule

2014 ACIP Adult Immunization Schedule

Hepatitis B Vaccine: Diabetics


ACIP, 25 October 2011; CDC Approved, 23 December 2011. MMWR. 60(50);1709-1711.

Routine vaccination of unvaccinated adults with diabetes < 60 years of age (SOR A)
Increased risk because of shared testing equipment > 60 permissive use recommendation because vaccine is more effective in younger patients (SOR B)

Vaccination Coverage in Adults*


CDC. Non-influenza vaccination coverage among adults: United States, 2011. MMWR Morb Mortal Wkly Rep. 2013;62.
*NOTE: Childrens vaccination coverage is about 90%.

Vaccine
Tdap (ages 19-64) Herpes zoster HPV Women ages 19-26 > 1 Men ages 19-26 > 1 Pneumococcal Ages 19-64 Age > 65 Hepatitis B High risk, ages 19-49 Ages 19-59 with diabetes Healthcare professionals

2011 Coverage
12.5% (Healthcare workers (26.8%) 15.8% 29.5% <3% 20.1% 62.3%

Health People 2020


30%

60% 90%

42% 22.8% 63.8 10.7%


42.8% 68.6% 47%

90%
80% 80%

Hepatitis A (ages 19-49)


Influenza > 6 m of age 65 y of age Pregnant women

Implications for Public Health Practice


Implementing reminder-recall systems Use of standing order programs for vaccination Assessment of practice level vaccination rates with feedback to staff members

4. Which immunization would be considered to be safely administered during pregnancy?


A. MMR B. HPV C. Tdap D. Varicella E. HPV

Vaccines and Pregnancy


Safe
Tdap* Influenza TIV Hepatitis A, if at risk Hepatitis B, if at risk Meningococcal, if indicated Pneumococcal polysaccharide, if indicated

Wait until after pregnancy


MMR Varicella HPV Influenza LAIV

*ACIP Recommendations for Pregnant Women: Administer a dose of Tdap during each pregnancy, irrespective of the patients prior history of receiving Tdap. Guidance for Use: To maximize maternal antibody response and passive antibody transfer to the infant, optimal timing for Tdap administration is between 27 and 36 weeks gestation although Tdap may be given at any time during pregnancy. Women not previously vaccinated with Tdap: If Tdap is not administered during pregnancy, Tdap should be administered immediately postpartum.

Prevention
Secondary
Activities are aimed at early disease detection, thereby increasing opportunities for interventions to prevent progression of the disease and emergence of symptoms.
Breast cancer

Screening
Guideline resources
American Cancer Society US Preventive Services Task Force Institute for Clinical Systems Integration

Screening Tests Effectiveness


The disease must have serious consequences, a long preclinical phase, and effective treatment. The screening test must have high sensitivity and specificity, be low in cost, and be acceptable to patients. The risks and costs of false (+) and false () results must be low, there must be a consensus on management of patients with (+) results, and there must be a system in place for referral and treatment.

Screening Tests Sequence


A highly sensitive (and usually relatively inexpensive) test should be used first, almost guaranteeing the detection of all cases of the disease (albeit at the expense of including a number of false-positive results). This should be followed by a more specific test (and usually more expensive test) to eliminate the false-positive results.
Eg, this is the usual sequence if testing for HIV, hepatitis B, and many other common but serious diseases.

Breast Cancer
Screening

Breast Cancer
Most common cause (with exception of skin) of cancer in women and the 2nd leading cause of cancer death
1/8 women will develop breast cancer. 1/30 will die.

Presence of dominant inherited cancer susceptibility genes (BRCA 1 and BRCA 2) occur in about 1/300-500 of general population
Screening for inherited risk (ACOG 2009)
Assessment of risk for significant BRCA mutations Genetic testing of high-risk women (Level A)

USPSTF
24 December 2013
Recommends that primary care providers screen women who have family members with breast, ovarian, tubal, or peritoneal cancer with 1 of several screening tools designed to identify a family history that may be associated with an increased risk for potentially harmful mutations in breast cancer susceptibility genes (BRCA1 or BRCA2). Women with positive screening results should receive genetic counseling and, if indicated after counseling, BRCA testing. (B recommendation) The USPSTF recommends AGAINST routine genetic counseling or BRCA testing for women whose family history is not associated with an increased risk for potentially harmful mutations in the BRCA1 or BRCA2 genes. (D recommendation)

Screening Tools Evaluated by the USPSTF


http://www.uspreventiveservicestaskforce.org/uspstf12/brcatest/brcatestfinalrst ab.htm#tab1

Tool

Ontario Family History Assessment Tool Manchester Scoring System Referral Screening Tool* Pedigree Assessment Tool FHS 7*
* Simplest and quickest to administer Since 2005, family history risk stratification tools have been developed and validated for use in primary care practice to guide referral for BRCA genetic counseling. In addition, the potential benefits and harms of medications for breast cancer risk reduction have been studied for longer follow-up periods, and more information is available about the potential psychological effects of genetic counseling and risk-reducing surgery.

High-Risk Women
ACOG 2009
Personal history of breast and ovarian cancers (+) ovarian cancer and a close relative (first- or seconddegree relative) with ovarian cancer, premenopausal breast cancer, or both (+) ovarian cancer who are of Ashkenazi Jewish ancestry 50 years and younger with breast cancer and a close relative with ovarian cancer or male breast cancer at any age Women of Ashkenazi Jewish ancestry in whom breast cancer was diagnosed at 40 years or younger Women with a close relative with a known BRCA1 or BRCA2 mutation
ACOG Practice Bulletin No. 103: hereditary breast and ovarian cancer syndrome. Obstet Gynecol. 2009;113(4):958.

BRCA1 or BRCA2 Mutation


Can be considered for prophylactic oophorectomy and mastectomy
Prophylactic therapy
Decreases incidence of breast and ovarian cancer Inadequate evidence for mortality benefits

Cancer Genetics Studies Consortium Recommendations for Screening


Monthly BSE: Age 21 CBE q 6-12 m starting at age 25-35 years Annual mammograms starting at age 25-35 years Ovarian cancer screening (US, CA-125 levels) q 6-12 months starting at age 25-35 years

5. Which of the following statements about breast self-examination is true?


A. BSE has been confirmed to reduce breast cancer mortality. B. BSE should be performed by women in the shower just prior to the onset of menses. C. BSE has detected many more cancers when performed by properly trained women. D. BSE has a Grade D recommendation from the US Preventive Services Task Force.

Breast Cancer
Screening Methods
Breast self-examination (BSE) Studies have not clearly demonstrated BSE as beneficial for cancer screening. Any benefits must be balanced against potential harms such as excessive invasive procedures performed as a result of the discovery of noncancerous lesions.

Breast Cancer
Screening Methods
Clinical breast exam (CBE)
Insufficient evidence to recommend it as a singular screening modality. RCTS demonstrate varying detection rates: 3%-57%. Most advocates have supported CBE as a complementary technique to mammography. About 5% of screening-detected cancers are found using CBE alone.

Screening for Breast Cancer


USPSTF 2009 (Updates 2002 Rec)
Biennial screening mammography for women aged 50-74 (Grade B) Biennial screening before age 50 should be an individual decision and take patient context into account (Grade C). > 75 years of age: Insufficient evidence to assess additional benefits and harms from mammogram (Grade I) Recommends against teaching BSE (Grade D) CBE in women > 40 (Grade I) Digital mammography or MRU (Grade I)
http://www.ahrq.gov/clinic/uspstf09/breastcancer/brcanrs.htm

Women Aged 40-49


Individualize decision to begin biennial screening according to the patients context and values The recommendation applies to women who are NOT at increased risk by virtue of a known genetic mutation or history of chest radiation.

Decision Analysis
Reduction of Mortality
Biennial Screening
Age 50-69 40-69 50-79 Reduction in mortality (compared with no screening) [Range] 17% [15%-23%] 20% (considered a minor improvement) 24% (additional 7%)

Extending the age range produced only minor improvements: Additional 3% reduction starting at age 40 years and 7% extending to age 79 years.

Decision Making
How many 40-year-old women who start having screening mammograms every two years will die from breast cancer in the next 10 years?
2 per 1000

How many 40-year-old women who DO NOT start having screening mammograms every two years will die from breast cancer in the next 10 years?
2.5 per 1000

Timing of Screening
Evidence indicates that biennial screening is optimal. Biennial schedule preserves most of the benefit of annual screening AND cuts the harms nearly in half.

Special Considerations
Estimated lifetime risk > 20% or who have a BRCA mutation
Screening begins at age 25 or at the age that is 5-10 years younger than the earliest age that breast cancer was diagnosed in the family.

Screening Breast MRI


The American Cancer Society recommends screening breast MRI (impact on breast cancer mortality is uncertain): Women with BRCA1 or BRCA2 gene mutations Women with a first-degree relative with BRCA1 or BRCA2 gene mutations who have not as yet had genetic testing Women with a lifetime risk of more than 25% as defined by risk assessment tools largely dependent on family history Women who underwent radiation to the chest between ages 10-30 for Hodgkins disease Women known to have a hereditary breast cancer syndrome, ie, Li Fraumeni, Cowden, and Bannayan-Riley-Ruvalcaba, and their first-degree relatives

Cancer Screening 2010


CDC. MMWR. January 27, 2012;61(3).
Data from the 2010 National Health Interview Survey
Breast cancer screening rate: 72.4% (Healthy People 2020 target: 81.1%) Other breast cancer screening rates
No usual source of health care: 36.2% No health insurance: 38.2%

Overall, the proportion of women aged 50-74 years who reported having had a mammogram in the past 2 years remained stable during 2000-2010.

Recommendations of Others
Organization Year
ACS 2003

Recommendation
Annual mammography beginning at age 40 years and continuing for as long as the woman is in good health; annual CBE after the age of 40 years. Insufficient evidence to recommend BSE. Similar to ACS, except for inclusion of a positive recommendation for BSE Endorsed the USPSTF recommendation

AMA AAFP ACOG

2002 2009

Mammography (Level B) and CBE (Level C) annually starting at the age of 40. No consensus on 2011* upper age limit of mammograms. All women should be encouraged to practice breast self-awareness.
2009 Mammography q 1-2 years (age 50-59). Does NOT recommend CBE or BSE.
* Obstet Gynecol. 2011;118:372-382.

WHO

Informed Decision Making


Bellizzi et al. Arch Intern Med. 2011;171:2031-7.
Screening for Target population Age recommended to stop screening Target population screening Screened and age 75-79 Screened and age > 80 Reported advised by physician (age 75-79)

Breast Prostate

50-74 50-74

75 75 75 65

74% 40% 48% 83%

62% 57% 57% 53%

50% 42% 47% 38%

62% 62% 65% 48%

Colorectal 50-74 Cervix 21-64

National Health Interview Survey (2005 and 2008); 49,575 adults


~50% of these older adults report their physicians recommended the cancer screening. Physician recommendation was the strongest predictor of obtaining the screening.

Critical role for healthcare providers to make informed screening decisions for older adults
Functional status, comorbidities, life expectancy, personal preferences

Breast Cancer Screening


Conclusions
Has resulted in an increase in diagnosis of localized disease without a commensurate decrease in the incidence of more widespread disease It cannot predict which of the discovered cancers are more aggressive, and cannot accurately detect premalignant lesions. The decrease in the mortality rate of breast cancer is due BOTH to earlier detection and better followup medical care.

Summary of 2009 ASCO Recommendations on Pharmacologic Interventions to Reduce Breast Cancer Risk
Agent
Tamoxifen

Recommendation

Dosage

May be offered to reduce the risk of ER (+) invasive 20 mg/day breast cancer in pre- and postmenopausal women with a for 5 years 5-year projected cancer risk > 1.66% or with lobular carcinoma in situ; risk reduction benefit continues for at least 10 years; impact on breast cancer-related mortality is unknown.

Raloxifene May be offered to reduce the risk of ER (+) invasive breast cancer in postmenopausal women with a 5-year projected breast cancer risk = 1.66% or with lobular carcinoma in situ; impact on breast cancer-related mortality is unknown. Should not be used for breast cancer risk reduction in premenopausal women. May be used longer than 5 years in women with osteoporosis in whom breast cancer risk reduction is a secondary benefit.

60 mg/day for 5 years

Prevention
Tertiary
Reduces the negative impact of an already established disease by restoring function and reducing disease-related complications.

6. A 74-yo female with New York Heart Association class II heart failure and a left ventricular ejection fraction of 34% is on optimal dosages of an ACE inhibitor, a -blocker, and rosuvastatin (Crestor). Her past medical history is notable only for a long history of hypertension. She is a nonsmoker and reports that she has a small glass of blush wine with dinner each evening. On examination she has a blood pressure of 126/72 mm Hg and a BMI of 28.2 kg/m2. Her chest is clear and her cardiac examination is notable only for an S4. Self-help measures recommended for patients such as this include which one of the following?

A. B. C. D. E.

A sodium intake 4000 mg/day Strict avoidance of alcohol consumption Avoiding exercise Avoiding NSAID use A weight-loss program with a goal BMI of 25 kg/m2 or less

Heart Failure
Daily weight Low sodium diet
< 2300 mg per day

Self Help

Medications
Beta blocker ACE inhibitor Diuretic (+/) Digoxin

Echocardiogram
ACC/AHA 2005 Guideline for the Diagnosis and Management of Chronic Heart Failure in the Adult

Self-Care
CHF patient must deal with his/her condition on a daily basis; help from clinicians is not always available.
Partnership model of care Responsibility shifts from the physician to the patient, encouraging shared decision making and steering away from the passive patient/expert doctor paradigm.

Patients have been found to have better outcomes simply by wielding more power in the doctor/patient encounter. Educating patients to self-manage their chronic diseases has been shown to lead to increased levels of functioning, reduced pain, and decreased health care costs (Hibbard, 2003).

Self-Care
Advocated as a Method of Improving Outcomes in Patients with Heart Failure

Sodium Intake < 2300 mg daily (AHA)


Same amount recommended for healthy adults

Fluid restriction to < 2 L/day may be appropriate for patients with hyponatremia or persistent or recurrent fluid retention; more liberal intake appropriate for stable HF patients.

Self-Care
Advocated as a Method of Improving Outcomes in Patients with Heart Failure

Avoid NSAID use.


Shown to increase the risk for renal insufficiency and hospitalization

Available studies indicate that survival is highest in patients with a BMI of 30-32 kg/m2; no studies have demonstrated a survival benefit from weight loss in patients with heart failure.
AHA guidelines currently recommend that weight loss be encouraged only in patients with a BMI > 40 kg/m2.

Self-Care
Advocated as a Method of Improving Outcomes in Patients with Heart Failure

Several epidemiologic studies have failed to demonstrate a correlation between alcohol consumption and the development of heart failure.
Exception: Patients with alcoholic cardiomyopathy, who should abstain from alcohol use Heart failure patients who choose to drink should be advised to limit their alcohol intake to no more than 12 drinks a day.

Self-Care
Advocated as a Method of Improving Outcomes in Patients with Heart Failure

Avoidance of physical exertion has been advised in the past; it is now thought that a reduction in physical activity leads to physical deconditioning and an unnecessary worsening of symptoms. Exercise training 3-5 days a week should be considered in all stable outpatients with chronic heart failure.

Symptoms of Heart Failure


Recommended Therapy
Known structural heart disease AND shortness of breath and fatigue, reduced exercise tolerance

Therapy
Drugs for routine use Diuretics ACE I Beta blockers Drugs in selected patients Aldosterone antagonist ARBs Digitalis Hydralazine/nitrates Devices in selected patients Biventricular pacing Implantable defibrillators

Goals
Treat hypertension. Encourage smoking cessation. Treat lipid disorders. Encourage regular exercise. Discourage alcohol intake, illicit drug use. Control metabolic syndrome.

CRT
Keeps the right and left ventricles pumping together by sending small electrical impulses through the leads

AHA Heart Failure Guidelines


2009
Cardiac resynchronization therapy (CRT) and implantable cardioverter defibrillator (ICD) for HF patients with either sinus rhythm (SOR A) or atrial fibrillation (SOR B) who meet the following criteria:
LVEF 35% NYHA class III or ambulatory class IV heart failure symptoms despite optimal medical therapy QRS interval of 0.12 seconds

Resynchronization-Defibrillation for Ambulatory Heart Failure Trial


N Engl J Med. 2010;363(25):2385-2395. Ann Intern Med. 2011;154(6):401-412.

Addition of CRT to ICD resulted in reduced rates of hospitalization and death among patients with NYHA class II or III heart failure, a wide QRS complex, and an LVEF 30% (SOR A). Meta-analysis has confirmed that CRT improves LVEF and reduces all-cause mortality and HF hospitalization in all patients with a reduced LVEF, symptoms of HF, and a prolonged QRS interval, regardless of NYHA class.

Refractory Heart Failure


Patients who have marked symptoms at rest despite maximal medical therapy Options:
Compassionate end-of life care, hospice Extraordinary measures Heart transplant Chronic inotropes Permanent mechanical support Experimental surgery or drugs

Heart Transplantation
Generally not performed in patients over the age of 65-70 No shortage of recipients; primary limiting factor is lack of donors. Recipients need lifelong immunosuppressant therapy.

Summary
Effective health promotion
Lifestyle modification: 3 leading causes of morbidity in the US
Smoking Overweight /obesity Alcohol abuse

Quaternary
Set of health activities to mitigate or avoid the consequences of unnecessary or excessive intervention of the health system. It is the practice of first do no harm.

Counseling

Screening
Done in asymptomatic persons; typically secondary prevention

Prevention
Primary
Immunizations

Secondary
Breast cancer

Tertiary
Congestive heart failure

Thank you!

Obesity and Metabolic Syndrome


Belinda Vail, MD University of Kansas School of Medicine Kansas City, Kansas

Disclosure Statement
Dr. Vail has nothing to disclose.

It is the policy of the AAFP that all individuals in a position to control content disclose any relationships with commercial interests upon nomination/invitation of participation. Disclosure documents are reviewed for potential conflicts of interest. If conflicts are identified, they are resolved prior to confirmation of participation. Only participants who have no conflict of interest or who agree to an identified resolution process prior to their participation were involved in this CME activity.

Learning Objectives
Upon completion of this lecture, participants should be able to: 1. Define the metabolic syndrome and its link to overweight and obesity 2. List the diagnostic criteria for metabolic syndrome 3. Describe a comprehensive evaluation for the metabolic syndrome: medical nutrition therapy, referral for bariatric surgery, self-management education, physical activity recommendations, and psychosocial assessment and care 4. Assemble a suitable treatment plan for the patient with metabolic syndrome: combination of lifestyle modifications, pharmacotherapy, and surgery

Obesity/Metabolic Syndrome
1/3 of Americans are obese and/or have metabolic syndrome

CDC data

Overweight and Obesity


Overweight: 25 kg/m2 Obesity: 30 kg/m2 In children > 85% for age and gender*
Girls: http://www.cdc.gov/growthcharts/data/set1clinical/cj4 1l024.pdf Boys: http://www.cdc.gov/growthcharts/data/set1clinical/cj4 1l023.pdf

Obesity and the Metabolic Syndrome


Development of obesity involves:
Genetics and metabolic factors Physical inactivity and calorie-dense foods Psychological and behavioral
PTSD 18-30% increased risk

Possibly gut bacteria

Metabolic syndrome
Abdominal obesity and insulin resistance Atherogenic dyslipidemia and elevated BP Proinflammatory and prothrombotic state

1. Which of the following criteria are used to define metabolic syndrome?


A. Insulin resistance, LDL cholesterol, and BMI B. Hypertension, LDL cholesterol, and BMI C. Hypertension, HDL cholesterol, and abdominal girth D. Insulin resistance, triglycerides, and LDL cholesterol

Metabolic Syndrome*
National Cholesterol Education Programs Adult Treatment Panel III (NCEP/ATP III) guidelines Combination of 3 of the following: Fasting glucose 110 mg/dL Waist circumference > 40 men, > 35 women HDL < 50 mg/dL women, < 40 mg/dL men Triglycerides 150 mg/dL Blood pressure 130/85 mm Hg WHO includes microalbuminuria AACE diagnosisclinical judgment based on risk factors Incidence increases with age and BMI

Implications
Doubles risk of cardiovascular disease (CVD) Increases 5 times the risk of developing diabetes Also increases risk for:
PCOS, abnormal menses, infertility Fatty liver and cholesterol gallstones Asthma and other respiratory disturbances Sleep disturbances (sleep apnea) Osteoarthritis Cancers (breast, uterine, colon)

Prediabetes
Based solely on plasma glucose Increased risk of developing diabetes
A1c 5.7-6.4% Impaired fasting glucose (100-125 mg/dL) Impaired glucose tolerance (140-199 mg/dL after 75 gm load)

2. The USPSTF recommends screening for obesity in everyone over what age?
A. 6 years B. 10 years C. 15 years D. 18 years E. 21 years

Prevention and Screening


Preventing obesity from conception
Limit gestational weight gain No maternal smoking Breast feed at least 12 months Infants should sleep at least 12 hours daily

Preventing obesity in childhood


Delay solid foods until at least 4 months of age Daily activity for at least one hour Limit screen time to 2 hours/day

USPSTF (and AAFP) recommends screening everyone age 6 for obesity

Treatment
Obesity is primary target for treatment of metabolic syndrome
Weight reduction Increased physical activity

Benefits of weight loss


Lowers cholesterol and triglycerides Raises HDL Lowers blood pressure Decreases glucose and insulin resistance Decreases CRP and PAI-1 Improves sexual function and sex hormones

Weight Loss/Lifestyle
The USPSTF recommends*:
Intensive behavioral dietary counseling for adult patients with hyperlipidemia and other known risk factors for cardiovascular and diet-related chronic disease delivered by primary care clinicians or by referral B recommendation.

Intensive counseling: > 1 session/mo for > 3 mo* Individualized lifestyle interventions better than group Weight loss (cut calories 500-1000 kcal/d)
3500 kcal = 1 pound** (20 oz soda = 250 kcal)

Diets
Low carb are fastest and reduce insulin resistance; that edge is lost by 6 mo Structured plans (Weight Watchers, Biggest Loser, Jenny Craig, etc.) good long term, but only if they stay on the plan (can be very costly) Satiety plans (AmIHungry.com, Volumetrics) successful long term Raw food, vegetarian, paleo diets are all effective because fewer calories are consumed No diet significantly better than the others

Glycemic Index
Foods with an increased glycemic index release glucose more rapidly from carbohydrates* Low glycemic load diets better for high insulin secretors Higher consumption of whole fruit associated with lower diabetes risk (blueberries, grapes/raisins, apples, pears, bananas, grapefruit)

Nutrition
Therapeutic Lifestyle Changes (TLC) National Heart, Lung, and Blood Institute ADA, AHA, Obesity Society Designed to decrease risk of developing heart disease
http://www.nhlbi.nih.gov/health/public/heart/chol/chol_tlc.pdf

TLC dietary guidelines < 7% of daily calories from saturated fat (no trans fats) 25-35% of total daily calories from fat < 200 mg of dietary cholesterol daily < 2400 mg of sodium daily 2 grams/day of plant stanols or sterols 10-25 grams/day of soluble fiber Just enough calories to maintain a healthy weight

Other Dietary Changes for Diabetes


Protein: < 20% of total daily energy increase variety 0.8-1.0g/kg for microalbuminuria and 0.8g/kg for proteinuria Increase amount and variety of seafood Whole grains, fruits, veggies, low-fat dairy 2 servings whole grains = 21% diabetes Limit alcohol Increase potassium, calcium, vitamin D, magnesium Increase B12 as patients age and with metformin 5% weight loss decreases fasting glucose

Exercise
5-7 times/wk, 30 minutes (45-60 min for weight loss)
150 min/wk of moderate aerobic activity 55-69% predicted max. heart rate Resistance training - 5 major muscle groups twice weekly At least 1 hour daily for children

Aerobic and resistance training best approach Structured: 2-5 supervised sessions for 12-16 weeks Exercise before and during pregnancy reduces risk of developing gestational diabetes In type 1 diabetes: Athletes should not participate in strenuous activity if glucose > 300 mg/dL or > 250 mg/dL with urine ketones*

3. Which of the following medications is licensed only for short-term use?


A. B. C. D. E. Topiramate (Topamax) Phentermine (Adipex) Orlistat (Xenical) Lorcaserin (Belviq) Phentermine and topiramate (Qsymia)

Weight-Loss Medications
Orlistat (OTC as Alli60mg)
Blocks fat breakdown and absorption

Phentermine and topiramate (Qsymia)


Lower dose of both, approved for long-term use

Lorcaserin (Belviq) 5-HT2c receptor agonist


Controlled substance, promotes satiety Possible serotonin syndrome with SSRI

Phentermine (15-30 mg)


Appetite suppressant Also: benzphetamine, diethylpropion, phendimetrazine

Pharmacologic Treatment of Metabolic Syndrome


Metformin and pioglitazone
Decrease insulin resistance No data on CVD risk reduction

Statins reduce CVD risk Fibrates modify atherogenic dyslipidemia Antihypertensives (ACE inhibitors) Low-dose aspirin is promising
No medications target PAI-1 and fibrinogen Treat prothrombotic state with antiplatelet meds

Bariatric Surgery
Most effective form of weight loss** Indications:
BMI > 40 kg/m2 or > 35 kg/m2 with comorbid conditions Other weight loss methods have failed High risk for obesity-related morbidity and mortality

Other qualifications
Generally indicated ages 18-65 Psychological evaluation and motivation

3. What is the most common cause of death following bariatric surgery?


A. Sepsis B. Pulmonary embolus C. Malnutrition D. Myocardial infarction

Complications
< 0.3% mortality after bariatric surgery
Lowest with gastric banding Highest with gastric bypass #1 cause of death following bariatric surgery is pulmonary embolus**

Most common complication: iron deficiency anemia (15% after malabsorptive surgery) Reoperations 8% Gallstones are common with weight loss Rates of serious complications are inversely associated with hospital and surgeon procedure volume.

Gastric Banding
Restrictive procedure Most effective for overeaters with lower BMI Band can be adjusted by changing amount of saline Complications
Gastritis GERD Band slippage Port infection

Sleeve Gastrectomy
Restrictive procedure More weight loss than gastric banding Otherwise similar complications

Redundant area of stomach removed

Roux en Y
Both restrictive and malabsorptive
Requires significant follow up Labs every 3-4 months for 1st year, then twice yearly
Ferritin CBC Lipids PTH B12 Electrolytes Vitamin D Thiamine

Supplement with:
Calcium B12 Folic acid Vitamin D Iron

Psychological Evaluation
Reasons for seeking surgery Weight and diet history Current eating behaviors
10% to 25% meet criteria for binge eating disorder Attitudes and feelings toward exercise

Understanding of the surgery Social supports and history Psychiatric history


5 times the rate of depression 23% to 47% are using psychotropic medications Millon Behavioral Medicine Diagnostic

Efficacy of Bariatric Surgery


Weight loss 3 years post surgery
Gastric banding: 16-55% Gastric bypass: 32-71%

Resolution of diabetes:
Gastric banding: 31-77% Gastric bypass: 72-100%

Others
Improved quality of life Reduction in medication use

Bariatric Surgery and Diabetes


Up to 75% of obese patients have complete resolution of diabetes following bariatric surgery Mortality rates 3-4x higher in those treated with oral medications vs. surgery Best chance for diabetes remission:
Young Lower A1c concentrations Not using insulin Not using sulfonylureas or insulin sensitizers

Summary
Provide intensive diet and exercise counseling Multidisciplinary team approach (dietitian, personal trainer, psychologist, behavioral therapist) Intervention should last at least 3-4 months Consider adding medication when patients are at a plateau Consider bariatric surgery when the above fail, particularly if BMI > 40 or with comorbid conditions.

5. Which of the following is true


regarding screening for diabetes?
A. The USPSTF recommends screening all persons with a BMI > 30 kg/m2 B. The ADA recommends screening for all obese persons beginning at age 35 C. According to a consensus panel, all children who are > the 85th percentile for weight should be screened D. The USPSTF recommends screening adults with a blood pressure > 135/80 mm Hg

Adult Screening Recommendations


USPSTF: screen asymptomatic adults with sustained blood pressure (either treated or untreated) greater than 135/80 mm Hg
(level B)
http://www.ahrq.gov/professionals/clinicians-providers/guidelinesrecommendations/guide/section2a.html#diabetest2

Screen at age 45, then every 3 years Screen if BMI 25 and 1 additional risk factor

American Diabetes Association Screening Recommendations


Physical inactivity Family history of diabetes (esp. in 1st degree relative) High-risk ethnic population Previous gestational diabetes or baby > 9 lbs Hypertension History of vascular disease Dyslipidemia (HDL < 35 / triglycerides > 250) History of impaired glucose tolerance Clinical conditions associated with diabetes (acanthosis nigricans) PCOS (polycystic ovary syndrome)

Screening in Children*
Every 2 years at age 10 or puberty if:
BMI or weight > 85% (> 120% of ideal)* 2 of the following risk factors Family history 1st or 2nd degree relative High risk ethnic/racial group Signs or symptoms of insulin resistance (acanthosis nigricans, hypertension, dyslipidemia, polycystic ovary disease)

Screening for Comorbid Conditions


Blood pressure at every visit Lipids yearly Screen for hypothyroidism as it can contribute to dyslipidemia* Screen for tobacco use Screen for depression (more prevalent in patients with chronic disease) In type 1 screen for hypothyroidism and celiac disease

References
Adams TD, et al. Health benefits of gastric bypass surgery after 6 years. JAMA 2012;308(11):1122031. Allen RH, et al. Nutritional deficiencies after Roux-en-Y gastric bypass for morbid obesity often cannot be prevented by standard multivitamin supplementation. Am J Clin Nutrition 2008;87(5)1128-33. Biggs ML et al. Association between adiposity in midlife and older age and risk of diabetes in older adults. JAMA 2010;303(24):2504-12. Birkmeyer NJO, et al. Hospital complication rates with bariatric surgery in Michigan. JAMA 2010;304(4):435-442. Church TS, et al. Effects of aerobic and resistance training on hemoglobin A1c levels in patients with type 2 diabetes. JAMA 2010;304(20):2253-62.

References
Courcoulas AP, et al. Weight change and health outcomes at 3 years after bariatric surgery among individuals with severe obesity. JAMA 2013;310(22):2416-25. Gillman MW, Ludwig DS. How early should obesity prevention start? Nov 13, 2013DOI: 10.1056/NEJM p1310577 Grief SN, Miranda RL. Weight loss maintenance. Am Fam Physician 2010;2(6):630-4. Mueller PS, et al. Bariatric surgery is superior to medical management for weight loss. BMJ 2013;347:5934. Rao G. Office-based strategies for the management of obesity. Am Fam Physician 2010;81(12):1449-56.

Answers
1. 2. 3. 4. 5. C A B B D

Maternity Care I
David G. Weismiller, MD, ScM, FAAFP
Department of Family Medicine The Brody School of Medicine at East Carolina University
weismillerd@ecu.edu

Maternity and Gynecologic Care AAFP Reprint No. 261 http://www.aafp.org/dam/AAFP/documents/medical_education_residency/prog ram_directors/Reprint261_Maternity.pdf

Disclosure Statement
Dr Weismiller has nothing to disclose.

The AAFP has selected all faculty appearing in this program. It is the policy of the AAFP that all CME planning committees, faculty, authors, editors, and staff disclose relationships with commercial entities upon nomination or invitation of participation. Disclosure documents are reviewed for potential conflicts of interest and, if identified, they are resolved prior to confirmation of participation. Only those participants who had no conflict of interest or who agreed to an identified resolution process prior to their participation were involved in this CME activity.

Learning Objectives
1. Cite the content and sequence of routine prenatal care. 2. Recall the laboratory and imaging tests commonly used in prenatal care. 3. Define the diagnosis of diabetes in pregnancy.

1. Which one of the following is recommended for routine prenatal care? A. Serologic testing for herpes simplex virus infection B. Parvovirus antibody testing C. Cystic Fibrosis carrier screening D. Only at-risk women should be tested for HIV infection. E. All pregnant women should be screened for asymptomatic bacteriuria by urine culture.

Initial Prenatal Visit Lab Tests


Blood group/Rh

Antibody screen
H/H Rubella antibody titer

2001 CDC. Interval to 1 month for pregnancy after rubella vaccine


Syphilis screen

CDC and USPSTF three times if high risk


Hepatitis B virus surface antigen Cervical cytology (as needed)

STI GC/chlamydia Urine culture Asymptomatic bacteriuria Sickle cell screen African American or Caribbean descent HIV screening Varicella screening +/ CF screening GD screening Intake (if high-risk) Routine 24-28 weeks

Anemia in Pregnancy ACOG Practice Bulletin July 2008


Iron supplementation decreases the prevalence of anemia at delivery (Level A). Iron deficiency anemia during pregnancy has been associated with an increased risk of (Level B): LBW Preterm delivery Perinatal mortality All pregnant women should be screened for anemia (Level C). (+) iron deficiency anemia (Hgb < 11 mg/dL (first trimester), < 10.5 (second), < 10 (third): treat with supplemental iron, in addition to prenatal vitamin.
Obstetrics and Gynecology. 2008;112(1) 201-207.

USPSTF Recommendations for STI Screening 2005


Pregnancy All women: hepatitis B, HIV, syphilis Women < 25 and those engaging in high-risk sexual behaviors also screened for chlamydia and gonorrhea Does not recommend routine screening for chlamydia in pregnant women not at increased risk; it notes that individual circumstances may support screening. No recommendation made about screening for gonorrhea in pregnant women not at increased risk (Category I)

CDC Recommendations for STI Screening 2010


Chlamydia: annual screening for sexually active women < 25 Nucleic acid amplification samples from urine, vagina, or endocervix Gonorrhea: reference recommendations of USPSTF http://www.uspreventiveservicestaskforce.org/u spstf/uspsgono.htm

Gonorrhea USPSTF 2005


All high-risk, sexually active women should be screened. Age < 25 Multiple sex partners No barrier contraception Incarcerated Illicit drug use Insufficient evidence for or against routine screening in high-risk men Recommend against routine screening of low-risk men and women

Asymptomatic Bacteriuria
Defined > 100,000 cfu/mL of a single bacterial species E. coli is most commonly isolated. Lactobaccilli and Staphylococcus species (other than S saprophyticus) may be presumed to be contaminants. Present in 2%-7% of pregnant women Higher preterm delivery rate than women without bacteriuria Culture at first prenatal visit. Unusual for women who do not have Asx bacteriuria at initial visit to develop later in pregnancy Exception? Sickle cell trait q trimester urine screen

2. Which of the following antibiotics should be used in the empiric treatment of asymptomatic bacteriuria?
A. Ampicillin B. Clindamycin C. Cephalexin D. Avelox

Treatment
Seven-day course Cephalexin, 250 mg po QID Single-dose regimens not widely studied in pregnancy Ampicillin should not be used high rates of resistance. Urine culture after therapy to ensure cure Culture will also identify patients with persistent or recurrent bacteriuria. Consider antibiotic suppressive dose until delivery. Cephalexin 250 po QHS

Antibiotics in Pregnancy
Considered Safe
Amoxicillin Ampicillin Clindamycin Erythromycin Penicillin Cephalosporins

Typically Avoid
Tetracyclines Nitrofurantoin* Sulfonamides*
* Prescribing sulfonamides or nitrofurantoin in the first trimester is still considered appropriate when no other suitable alternative antibiotics are available. During the second and third trimesters, sulfonamides and nitrofurantoin may be used as first-line agents for infections.

ACOG Committee Opinion #494: Sulfonamides, Nitrofurantoin, and Risk of Birth Defects. Obstet Gynecol. 2011;117(6):1484-1485.

Treatment of Asymptomatic Bacteriuria


Outcome Clearing asymptomatic bacteriuria Incidence of pyelonephritis Incidence of low birthweight babies Incidence of preterm delivery

Antibiotic treatment compared to placebo RR (95% CI)


0.25 (0.14-0.48) 0.23 (0.13-0.41) 0.66 (0.49-0.89)

NNT 1.6 6.7 20

Antibiotics for asymptomatic bacteriuria in pregnancy. Cochrane Database of Systematic Reviews. 2001;(2):CD000490.

Herpes
All patients and their partners should be asked about a history of genital and orolabial HSV infection. Rates of vertical transmission at delivery Primary HSV infection 50% Non-primary first episode 33% Recurrent 0-3% Genital herpes acquired during pregnancy does not seem to increase rates of neonatal illness or congenital HSV infection as long as HSV seroconversion has completed by time labor begins.

Suppressive Therapy > 36 Weeks ACOG 2007


Women with active recurrent genital herpes should be offered suppressive viral therapy at or beyond 36 weeks of gestation (Level B).
Suppressive treatment after 36 weeks Decreased 75%

Risk of recurrent genital herpes at delivery


Rate of C/S for recurrent genital herpes Viral detection at delivery using culture or PCR Indication Acyclovir

40%
90% Valacyclovir

Daily suppression

400 mg po TID daily from 36 weeks until delivery

500 mg po BID daily from 36 weeks until delivery

HIV Testing
CDC 2006, 2008; ACOG 2008; USPSTF 2005 All pregnant women offered HIV testing at the first prenatal visit and in the third trimester Routine opt-out HIV testing Decreased maternal-fetal transmission with detection and treatment Rapid testing in labor if undocumented HIV status Positive test initiate antiretroviral prophylaxis (with consent) without waiting for confirmatory test.
Centers for Disease Control and Prevention. US Public Health Service Task Forces Recommendations for Use of Antiretroviral Drugs in Pregnant HIV-1-Infected Women for Maternal Health and Interventions to Reduce Perinatal HIV-1 Transmission in the United States, July 2008. CDC. Revised recommendations for HIV testing of adults, adolescents, and pregnant women in health care settings. MMWR. 2006;55(RR-14):117. USPSTF. Screening for HIV: Recommendation statement. Ann Intern Med. 2005; 143:32-37. ACOG Committee Opinion Number 418, September 2008.

Evidence of Immunity to Varicella


Screening for Varicella in pregnancy (CDC 2006)
Recommend routine screening for varicella through history. If negative/unsure history, obtain varicella titer. Recommend offering vaccination postpartum, if varicella is non-immune.

Cystic Fibrosis Recommendations


Information about CF screening made available to all couples Reasonable to offer carrier screening to all couples regardless of race or ethnicity as alternate to selective screening CF carrier screening should be offered before conception or early in pregnancy both partners are of Caucasian, European, or Ashkenazi Jewish ethnicity. May elect to use either sequential or concurrent carrier screening
ACOG Committee Opinion No. 325. Update on Carrier Screening for Cystic Fibrosis. Obstet Gynecol. 2005;106:1465-1468.

No Routine Screening
Hepatitis C CDC screen women at increased risk HPV Per USPSTF guidelines < 30 reflex (ASCUS) q 3 years > 30 routine with Pap q 5 years Parvovirus B-19

Parvovirus B-19
Embryotoxic, not teratogenic Infection 1st trimester miscarriage Peaks early 2nd trimester fetal anemia, hydrops fetalis, and stillbirth Fetus at greatest risk 3-6 weeks after maternal infection Screening: Maternal IgG and IgM Fetal monitoring: Weekly U/S and middle cerebral artery Doppler If hydrops and anemia are present, RBC transfusion may be needed.

3. Per ACOG 2013, at 24 weeks gestation you perform a 1-hour Glucola test, which is elevated at 166 mg/dL. You would next:
A. Repeat the 1-hour Glucola in one week. B. Have the patient see the dietician for diabetic diet information. C. Order a 3-hour glucose tolerance test. D. Start low-dose metformin.

Detection of GDM
Organization
ACOG (2013)

Recommendation
Use a 2-step method at 24-28 weeks (Level B).

Comments
Use a blood glucose level of either 135 or 140 mg/dL with factors such as community prevalence rates of GD determining the cutoff. Goal was not to look at the performance or whether one method was better than another for screening. Found treating can significantly reduce the risk of preeclampsia, macrosomia, shoulder dystocia.

USPSTF (January 14, 2014)

Screen asymptomatic women after 24 weeks (Grade B). Current evidence is insufficient to assess the balance of benefits and harms of screening for GD in asymptomatic pregnant women before 24 weeks of gestation (Grade I).

ADA (2014)

Screen for undiagnosed type 2 diabetes at the first prenatal visit in those with risk factors; screen at 24-28 weeks if not previously known to have diabetes.

Updated Guidelines: Use either: 1. 1-step method 2. 2-step method

Detection and Diagnosis ADA 2014


Screen for undiagnosed type 2 diabetes at the first prenatal visit in those with risk factors using standard diagnostic criteria A1C > 6.5% FPG > 126 1-step method: 2-h plasma glucose during on OGTT (75 g glucose load) 2-step method: 50-g 1-h glucose challenge followed by fasting 100-g, 3-hour OGTT Patient with classic symptoms of hyperglycemia a random plasma glucose > 200 mg/dL

High Risk of GD
Clinical characteristics Marked obesity Personal history of GD or large-for-gestational-age infant Glycosuria Diagnosis of PCOS Strong family history of type 2 diabetes Ethnic background (Hispanic, Native American, South or East Asian, African American, Pacific Islands descent) If negative screening test, retest between 24 and 28 weeks.

ADA 2008

Screening for and Diagnosis of GD ADA 2014


Not previously known to have diabetes Screen at 24-28 weeks. Two methods One-step: Use a 75-g 2-h OGTT. Plasma glucose (PG): fasting, 1 and 2 h Diagnosis when any of the following PG values are exceeded: Fasting: > 92 mg/dL 1 h: > 180 mg/dL 2 h > 153 mg/dL Two-step: Use a 50-g 1-h glucose challenge followed by fasted 100-g, 3-h OGTT.

Two-Step Approach in USA


ACOG 2013, ADA 2014
Initial screening: 50 g oral glucose load (glucose challenge test) > 135 or > 140 mg/dL* 3-hour OGTT Note: > 190, > 90% abnormal 3-hour 3-hour OGTT 2 or more abnormal values = (+) GDM Overnight fast, 100 g glucose polymer Abnormal plasma blood glucose: > fasting 95 mg/dL, 1h 180, 2h 155, 3h 140
*Either threshold acceptable, ACOG 2013 (Level C). Can also be used as a 1-step method for high-risk women or in areas in which the prevalence of insulin resistance is 5% or higher (eg, southwestern and southeastern US).

Gestational Diabetes
Condition is increasing as obesity and older age at pregnancy become more common. Increased risk: Gestational hypertension Preeclampsia Cesarean delivery 7-fold increased risk of developing diabetes later in life

Screening Later in Pregnancy


Open neural tube defects Maternal serum alpha fetoprotein Aneuploidy Quad screen Screening sonogram Anemia Group B Streptococcus

Summary MSAFP and Quad Screen


Maternal blood sample High: Open neural tube defect Low Trisomy 21 Low MSAFP, high hCG, low estriol, high DIA Trisomy 18 Low levels of all four analytes Most accurate when performed between 16 and 18 weeks EGA Most common reason for false positive is incorrect assignment of EGA.

Detection Rates
Screening test Detection rate (%)
79-89 67-81

False positive rate (%)


< 3-5 < 3-5

Trisomy 21
First trimester NT measurement, PAPP-A and -hCG Second trimester quad screen

Integrated first plus second trimester screen


Stepwise sequential first plus second trimester screen Contingent sequential first plus second trimester screen

94-96
95 92

< 3-5
5 5

Abbreviations: NT: nuchal translucency, PAPP-A: pregnancy-associated plasma protein A, quad: quadruple.

4. According to the CDC, all women of childbearing age should consume how much folic acid on a daily basis?
A. B. C. D. 4 mg 0.4 mg 1 mg 0.8 mg

Folic Acid Supplementation


All women of childbearing age: 0.4 mg/day (1992 CDC) Peri-conceptional use: Estimated to reduce risk of NTD by 70%-80% History NTD: 4 mg/day 1-3 months before, continue through first 3 months of pregnancy. Note: IOM recommends 600 mcg/day for adult women aged 19 and older and 800 mcg daily for adolescents aged 14-18 years.
Wald et al. Quantifying the effect of folic acid. Lancet. 2001;358:2069-73.

Folic Acid Supplementation with Epilepsy


4 mg/day Valproate Carbamazepine 800 g/day Other AEDs 1-3 months prior to conception and continue through first trimester.
Low serum folate levels in women with epilepsy are independently associated with an increased risk of major fetal malformations. Not conclusively determined if folic acid supplementation prevents NTDs in women receiving antiepileptic drugs (AEDs).

5. According to ACOG, which of the following statements best characterizes the use of sonography in pregnancy?
A. It has the ability to diagnose minor fetal anomalies. B. It is best carried out at 12 weeks estimated gestational age. C. It is an accurate method of determining placental location. D. Routine use is recommended.

Screening Ultrasound Defined


Sonographic assessment of the pregnant abdomen between 16 and 20 weeks EGA (second trimester), in the absence of specific indications for a second-trimester ultrasound

37

Screening Ultrasound
The basic sonographic examination should provide the following information: Fetal number Fetal presentation Documentation of fetal life Placental location Fetal biometry Assessment of amniotic fluid volume Assessment of gestational age Survey of fetal anatomy for gross malformations Evaluation for maternal pelvic masses

Ultrasound in Pregnancy
Summary ACOG February 2009
US examination is an accurate method of determining GA, fetal number, viability, and placental location (Level A). Ability to diagnose major fetal anomalies established (Level A) Specific indications are the best basis for use in pregnancy. Optimal timing in the absence of specific ACOG indications is at 18-20 weeks.
ACR AIUM
ACOG Practice Bulletin No. 101. Ultrasopnography in Pregnancy. Obstet Gynecol. 2009;113:451-61.

6. An 18-yo G1 P0 at 40 5/7 weeks presents stating that she thinks her water broke 2 days ago. An exam confirms ROM. The patient is afebrile with a non-tender uterus; FHR tracing reassuring. A GBS culture obtained 4 weeks ago was (). Patient with NKDA. In addition to induction of labor, which one of the following is the most appropriate management for this patient?

A. No antibiotic prophylaxis B. Ampicillin, 2 g IV initially, then 1 g IV q 4 h until delivery C. Clindamycin, 900 mg IV q 8 h until delivery D. Vancomycin, 1 g IV q 12 h until delivery

GBS
Universal screening recommended MMWR. 2002;51(RR-8):1-52 GBS culture at 35-37 weeks Swab lower vagina (introitus) followed by rectum (sweep over anus) using same swab. Order susceptibility testing on isolates. Women with negative GBS cultures within 5 weeks of delivery do not require antibiotics even if risk factors develop.

Delivery at < 37 weeks of gestation


Amniotic membrane rupture > 18 hours Intra-partum temperature > 100.4 F (> 38.0 C)

Intra-partum Prophylaxis for GBS CDC 2010

7. A 26-yo female presents with lower abdominal pain and vaginal bleeding. Her last menstrual period was 7 weeks ago. A urine pregnancy test is positive, and a quantitative -hCG level is 2500 mIU/mL. Transvaginal ultrasonography shows no evidence of an intrauterine gestational sac. Baseline laboratory tests, including a CBC, liver function tests, and renal function tests, are all normal. She is treated with a single dose of IM methotrexate (Trexall) at 50 mg/m2 of body surface. Four days later the patient presents for reevaluation, and her quantitative -hCG level is found to be 3800 mIU/mL. 7 days later the quantitative -hCG level is found to be 4400 mIU/mL. Which of the following is the most appropriate next step?

A. A repeat dose of methotrexate, 50 mg/m2 of body surface B. Repeat transvaginal ultrasonography to evaluate for a viable intrauterine pregnancy. C. Laparoscopy with salpingostomy D. Expectant management
Answer Now

Ectopic Pregnancy
Ruptured ectopic pregnancy accounts for 10%-15% of all maternal deaths. Most strongly associated risk factors with ectopic pregnancy: Previous ectopic pregnancy In utero DES exposure Increased risk for ectopic pregnancy: History of genital infections or infertility Current smoking Early diagnosis critical: Linear salpingostomy is a tube-saving procedure. Pregnancy outcomes with methotrexate are comparable to those with surgery.

Diagnosis
Present with abdominal pain and vaginal bleeding; typically ~7 weeks after LNMP Ultrasound: Diagnostic test of choice Suspect ectopic: No intrauterine gestational sac and: Transabdominal US: -hCG > 3500 mIU/mL Transvaginal US: -hCG > 1800 mIU/mL -hCG Normal IUP increases by 53% q 48 hours (doubles) Cannot alone differentiate between an ectopic and IUP Chorionic villi (diagnostic curettage) If not detected, ectopic pregnancy should be suspected. Only considered when -hCG levels are falling or when levels are elevated and US does not show intrauterine pregnancy

Villi Absent on D&C

Decreasing hCG

Rising or stable hCG Mass > 4 cm

Rising or stable hCG Mass < 4 cm

Follow hCG titers Laparoscopy or laparotomy Medical treatment

Adapted from Apgar and from Speroff.

Serum Progesterone Levels


Can detect pregnancy failure and identify patients at risk for ectopic pregnancy Not diagnostic of ectopic pregnancy Sensitivity is 15%. Therefore, 85% of patients with ectopic pregnancy will have normal serum progesterone levels. Algorithms for diagnosing ectopic pregnancy that include progesterone levels miss more ectopic pregnancies and require more surgeries than do algorithms without progesterone.

Treatment
Hemodynamically unstable patient Laparotomy Early diagnosis and patient stable Laparoscopic salpingostomy Most cost-effective treatment Medical management with Methotrexate (MTX) in a single or multi-dose regimen Multi-dose more effective than surgery, but more expensive Single-dose has a higher failure rate than laparoscopic salpingostomy, especially in patients with higher -hCG levels.

Absolute Contraindications to Single-Dose MTX


Breastfeeding Overt or lab evidence of immunodeficiency Alcoholism, alcoholic liver disease, or other chronic liver disease Preexisting blood dyscrasias, such as bone marrow hypoplasia, leucopenia, thrombocytopenia, or significant anemia Known hypersensitivity for methotrexate Acute pulmonary disease Peptic ulcer disease Hepatic, renal, or hematologic dysfunction, and several metabolic diseases

Monitoring MTX therapy for Ectopic Pregnancy


Obtain a CBC with differential and liver and renal function tests before starting any regimen
Regimen Single dose* MTX, 50 mg/m2 IM Surveillance Measure -hCG level on days 4 and 7. If difference > 15%, repeat weekly until undetectable. If difference < 15%, repeat MTX dose and begin new Day 1. If fetal cardiac activity present on Day 7, repeat MTX dose and begin new Day 1. Refer for surgical treatment if -hCG level is not decreasing or fetal cardiac activity persists after 3 MTX doses. Follow up as for single-dose regimen.

2 dose MTX, 50 mg/m2 IM, Days 0, 4 Multi-dose (up to 4 doses) MTX, 1 mg/kg IM, Days 1, 3, 5, 7 Leucovorin, 0.1 mg/kg IM, Days 2, 4, 6, 8

Measure -hCG level on days 1, 3, 5, and 7 Continue alternate-day therapy until -hCG level decreases > 15% in 48 hours or 4 doses of MTX have been given. Then, weekly -hCG measurement until undetectable.

Adapted from Seeber BE, et al. Obstet Gynecol. 2006; and Little SH, et al. J Fam Prac. 2012;61(11):684.

Single-Dose MTX
Easier to administer and monitor Best prognostic indicator of successful treatment is initial -hCG level. Lower initial level the higher the success rate. Contraindications: Ectopic pregnancy > 3.5 cm and fetal cardiac activity

Diagnosis of Ectopic Summary


Failure of -hCG to double in 48-72 hours Ultrasound (transvaginal) IUP rules out ectopic No gestational sac + -hCG > 1800 highly suggestive Gestational sac/embryo outside of uterus confirms ectopic. Pitfalls: Pseudogestational sac, ruptured corpus luteum Laparoscopy: Gold standard

Ectopic Expectant Management


Criteria Minimal pain or bleeding Reliable follow-up No evidence of tubal rupture -hCG < 1000 and falling Adnexal mass < 3 cm, or not detected No embryonic heartbeat

First-Trimester Bleeding
Differential diagnosis Ectopic pregnancy Spontaneous pregnancy loss Chromosomal abnormalities are causative ~50% of the time. No bed rest or drug therapy including progestogens will correct the common etiologies (Cochrane 2004). Idiopathic bleeding in a viable pregnancy Cervical abnormalities Infection of the vagina or cervix Subchorionic hemorrhage Vaginal trauma Molar pregnancy

Criteria for Diagnosing Early Pregnancy Failure


Society of Radiologists in Ultrasound
Definitive pregnancy failure can be diagnosed in a woman with an IUP of uncertain viability when TVUS reveals any of the following: Embryonic crown-rump length > 7 mm and no heartbeat Mean gestational sac diameter > 25 mm and no embryo present No embryo with heartbeat > 2 weeks after TVUS showed a gestational sac without a yolk sac No embryo with heartbeat > 11 days after TVUS showed a gestational sac with a yolk sac
N Engl J Med. 2013 October 10;369:1443. (http://dx.doi.org/10.1056/NEJMra1302417)

8. An indication for the administration of anti-D immune globulin (RhoGAM) to an unsensitized, D-negative patient is:
A. Routinely at 16-20 weeks B. Induced abortion C. Delivery of a D-negative infant D. Amniocentesis only when the placenta is traversed

Anti-D Immune Globulin


Unsensitized, D-negative patient, if she has had one of the following conditions or procedures: Ectopic gestation Abortion (either threatened, spontaneous, or induced) Procedure associated with possible fetal-to-maternal bleeding, such as chorionic villus sampling or amniocentesis Condition associated with fetal-maternal hemorrhage (eg, abdominal trauma, ECV, abruptio placentae) Routinely at ~28 weeks Delivers a D-positive baby

Answers
1. E 2. C 3. C 4. B 5. C 6. A 7. A 8. B 9. C 10. A

10. A 22-yo G1 female at 9 weeks gestation sees you for follow-up after an episode of vaginal bleeding. Her -hCG levels are rising, and ultrasonography confirms a viable 8-week IUP with no apparent source of bleeding. Appropriate information and advice for this patient would include which of the following? A. She is at increased risk for miscarriage. B. She should remain at bed rest for 1 week beyond the end of the bleeding episode. C. She should take prenatal vitamins to reduce the risk of miscarriage. D. She should take a progestogen to reduce the risk of miscarriage.

Pregnancy Loss
Spontaneous pregnancy loss (loss of a pregnancy without outside intervention < 20 wks gestation) Threatened Inevitable Incomplete Missed Ectopic Complete Recurrent spontaneous (habitual) Ultrasonography is very helpful in the diagnosis. Chromosomal abnormalities are causative ~50% of the time.

Pregnancy Loss
20% of pregnant women will have some bleeding < 20 wks gestation. ~50% will end in a spontaneous pregnancy loss. 20% of recognized pregnancies will end in miscarriage or loss. No bed rest or drug therapy including progestogens will correct the common etiologies (Cochrane 2004). At the time of onset of the bleeding, viability is most likely already determined.

First Trimester Bleeding


Differential diagnosis Cervical abnormalities Ectopic pregnancy Idiopathic bleeding in a viable pregnancy Infection of the vagina or cervix Molar pregnancy Spontaneous pregnancy loss Subchorionic hemorrhage Vaginal trauma

First Trimester Bleeding


Laboratory tests should include KOH and wet prep of vaginal secretions CBC ABO and Rh testing Quantitative hCG GC and Chlamydia testing Ultrasonography Crucial in identifying status of the pregnancy Location of the pregnancy Transvaginal with hCG > 1,800 mIU/mL (should see IUP) Transabdominal with hCG > 3,500 mIU/mL (should see IUP)

Pregnancy Loss
Biochemical predictors of Successful Outcomes Doubling of the -hCG level in 48 hours Serum progesterone level > 25 ng/mL Predictors of Failure of Successful Outcomes Subchorionic hemorrhage on US If present, continuation rate of pregnancy is decreased even if heartbeat is present. Serum progesterone level < 5 ng/mL Failure of the -hCG to double at 48 hrs

Adapted from Apgar, 2001

hCG Titer Transvaginal Ultrasound


Abnormally rising hCG No gestational sac Normally rising hCG

Villi present on D&C


Gestational sac No yolk sac Gestational sac Yolk sac present

Incomplete abortion
Nonviable pregnancy Normal pregnancy

First-Trimester Bleeding and Preterm Delivery


Compared with controls, women presenting with first-trimester bleeding or threatened pregnancy loss were more likely to deliver prematurely. Bleeding group rate was 11.9%. Most likely between 34-37 wks gestation Control group rate was 5.6%.

Diabetes:
Treatment and Complications
Belinda A. Vail, MD University of Kansas School of Medicine Kansas City, Kansas

Disclosure Statement
Dr. Vail has nothing to disclose.

It is the policy of the AAFP that all individuals in a position to control content disclose any relationships with commercial interests upon nomination/invitation of participation. Disclosure documents are reviewed for potential conflicts of interest. If conflicts are identified, they are resolved prior to confirmation of participation. Only participants who have no conflict of interest or who agree to an identified resolution process prior to their participation were involved in this CME activity.

Learning Objectives
Upon completion of this lecture, participants should be able to: 1. State the criteria for the diagnosis of Diabetes mellitus 2. Summarize a disease management plan including appropriate performance improvement measures, lipid management, blood pressure management, hemoglobin A1c management, urine protein screening, dilated eye exam, foot exam, and aspirin use 3. Recommend appropriate pharmacologic methods of controlling blood sugar including oral agents and selection of type and regimen of insulin

Diagnosis of Diabetes
Hb A1c 6.5% Fasting plasma glucose 126 mg/dL 2-hr plasma glucose 200 mg/dL (75g glucose) Random plasma glucose 200 mg/dL with classic symptoms of hyperglycemia

Type 1 vs Type 2
< 10% of diabetics Onset usually in childhood or adolescence Loss of beta cells in the pancreas: insulin requiring Weight loss is prevalent Sudden onset Screening for complications begins 5 years after diagnosis > 90% of diabetics Onset now common in adolescence Insulin resistance so high insulin levels Obesity is prevalent Insidious onset Screening for complications begins at diagnosis

ADA Management Goals


A1C < 7%**
< 6.5% for new diagnosis, long life expectancy < 8% for longstanding diabetes, advanced complications, limited life expectancy Monitor every 3 months History of hypoglycemia is predictive of future episodes and increases morbidity and mortality*

Limit hypoglycemia

Blood pressure < 140/80 mm Hg (< 130/80 preferred)* ** LDL cholesterol < 100 mg/dL*

< 70 mg/dL with cardiovascular disease Or reduction of 30-40% if cant reach above goal

Self Management
Self management that works
Motivational interviewing, collaborative problem solving, and negotiating individualized goals Cognitive behavioral therapy Nurse- and dietitian-led training Type 1 (3x/d if multiple insulin injections or pump) During illness When oral medications are adjusted During corticosteroid use When postprandial hyperglycemia is a concern When using oral sulfonylureas

Self monitoring indicated for:

Benefit in type 2 disappears by 1 year

Pharmacological Therapy
Biguanide: (metformin) Thiazolidinedione (TZD): (pioglitazone) Sulfonylureas: (glyburide, glipizide, glimepiride) Meglitinides: (repaglinide, nateglinide) Alpha-glucosidase inhibitors: (acarbose, miglitol) Insulin (short and long acting; human and synthetic) GLP-1 receptor agonists: (exenatide, liraglutide, pramlintide) DPP-4 inhibitors: (sitagliptin, saxagliptin, linagliptin, vildagliptin) SGLT2 inhibitors: (canagliflozin) Bile acid sequestrant: (colesevelam)

1. Which of the following medications should be continued if the patient is moving to insulin therapy?
A. B. C. D. E. Metformin Acarbose Glipizide Repaglinide Glyburide

Metformin
This should be the #1 choice unless there are contraindications Insulin sensitizer**, decreases glucose production in the liver Lowers insulin and lipid levels; no hypoglycemia or weight gain Improves cardiovascular outcomes in overweight and newly diagnosed type 2 diabetics** decreases mortality** Only oral medication for use in children and adolescents* Inexpensive

Metformin
Use with caution in the elderly (> 65), renal dysfunction**, cardiopulmonary disorders (OK with stable CHF), and hepatic disease Stop prior to IV contrast** and 48 hours after** (angiography/pyelography) Must check creatinine prior to use* Rare cases of lactic acidosis* (recommended to stop at creatinine > 1.5 men, 1.4 women) Widely used for polycystic ovary disease* Check for B12 deficiency* Category B in pregnancy

Thiazolidinediones
Pioglitazone (Actos), rosiglitazone (Avandia) Insulin sensitizers** (decrease insulin resistance)* gluconeogenesis Use with caution in the elderly due to declining ventricular function Caution in cardiopulmonary disorders (volume overload)*. Black Box warning for class III or IV heart failure Monitor LFTs, avoid in hepatic dysfunction including nonalcoholic fatty liver disease*

Thiazolidinediones
Increase ovulation Category C in pregnancy (growth retardation in animals) * Pioglitazone risk of MI, stroke, and death, triglycerides, HDL, serious heart failure Increase in distal limb fractures in women Increased risk of bladder cancer > 1yr use
(28/100,000 person/yrs)

? Increased risk of diabetic macular edema Expensive

2. 55 y/o with newly diagnosed DM and HTN with chronic renal failure and a creatinine of 2.4. Which medication should you start? A. Acarbose (Precose) B. Glyburide (DiaBeta) C. Metformin (Glucophage) D. Glipizide (Glucotrol)

Sulfonylureas
Glipizide (Glucotrol), glyburide (Micronase), glimepiride (Amaryl) Stimulate pancreatic beta cells to release insulin Weight gain and hypoglycemia Can be used in low doses in the elderly (avoid glyburide) OK in mild renal dysfunction (except glyburide has an active metabolite eliminated by the kidneys)* OK in cardiopulmonary comorbidities (sleep apnea, CHF) Glyburide in gestational diabetes Inexpensive

Meglitinides
Helpful for erratic eating schedule Repaglinide (Prandin), nateglinide (Starlix)
Rapid acting (half-life < 1 hr) insulin secretagogues Moderate cost and moderate decrease in glucose May be used in elderly, renal failure, and cardiopulmonary disorders

Alpha-glucosidase Inhibitors
Helpful for erratic eating schedules Acarbose (Precose) and miglitol (Glyset)
Delay carbohydrate absorption in gutdecrease peak glucose levels, no hypoglycemia as monotherapy* (Treat hypoglycemia in combination Rx with glucose or glucagon) Reduces risk of cardiovascular events* (STOP-NIDDM) Monitor LFTs; avoid in cirrhosis, GI disease Not for use in renal dysfunction (creatinine > 2)* Weight neutral Category B in pregnancy

3. Which medication is most effective at lowering postprandial blood sugars?


A. Exenatide (Byetta) B. Pioglitazone (Actos) C. Metformin (Glucophage) D. Nateglinide (Starlix)

GLP-1 Receptor Agonists


All given by subcutaneous injection May produce lower A1c than DPP-4 inhibitors but no change in micro- or macrovascular outcomes Exenatide (Byetta): GLP-1 agonist
Dose 5-10 mcg bid or 2 mg once weekly Now OK to give with insulin (not in same syringe)

Liraglutide (Victoza): GLP-1 agonist


Dose 0.6, 1.2, 1.8 mg daily Contributes to weight loss

Pramlintide (Symlin): Synthetic analogue-amylin


Given with insulin (not in same syringe) Dose 60-120 mcg with meals

GLP-1 Receptor Agonists


Mechanism of action
Potentiate insulin secretion Suppress postprandial glucagon secretion Slow gastric emptying Promote satiety (no weight gain)

Side effects
Nausea, vomiting, diarrhea, weight loss* Pancreatitis* Hypoglycemia (with sulfonylurea) Thyroid C-cell tumor risk

Decrease dose in renal failure (except liraglutide) Avoid if creatinine clearance < 30 mL/min Cost ~ $200 / mo Category C in pregnancy

DPP-4 Inhibitors
Choices
Sitagliptin (Januvia) 100 mg daily (25-50 mg renal dose) Saxagliptin (Onglyza) 2.5-5 mg daily (2.5 mg renal dose) -- HF Vildagliptin (Galvus) 50-100 mg daily Linagliptin (Tradjenta) 5 mg daily

Block dipeptidyl peptidase-4 (the enzyme that breaks down natural incretins) Better insulin release and blood sugar control particularly postprandial Dont add to sulfonylurea in the elderly* Linagliptin not renally excreted and good choice in elderly Side effects minimal: URI, sore throat, diarrhea, pancreatitis Weight neutral Expensive ~$150-200/mo

SGLT2 Inhibitors
Canagliflozin (Invokana), Dapagliflozin (Farxiga) Block reabsorption of glucose in the kidney Increase urinary excretion of glucose Decrease weight and blood pressure Increase HDL cholesterol Increase UTIs and vaginal yeast infections

Other Medications
Salsalate lowers A1c but increases weight and albuminuria Colesevelambile acid sequestrant
May increase incretins

Obeticholic acidnot approved


Promise for fatty liver in improved insulin sensitivity Well tolerated

Combination Therapy
Best if different mechanisms of action are combined Available combinations
Metformin with almost all other classes Sitagliptin and simvastatin (Juvisync) Pioglitazone and glimepiride (Duetact)

Usually minimal cost savings because metformin is cheap As beta cell function declines or inability to gain control, add insulin (all can be used with insulin)*

Insulin
Weight gain outweighed by glucose control Average dose 0.6-0.8 units/kg body weight/day
~ 40-50 units for a 70 kg man Half for basal needs and half with meals

Bioavailability changes with site of injection*


Faster in abdomen, slower in thigh

Exercise accelerates absorption in thigh*


Arm reduces exercise-induced hypoglycemia by 60% Abdomen reduces exercise-induced hypoglycemia by 90%

Best combination: long-acting basal and rapid-acting syntheticmost closely mimics normal* Commonly used in geriatrics when other medications are contraindicated*

Intensive Insulin Therapy


Early use of insulin to reach blood sugar goal Consistent increase in insulin dose until blood sugar is at goal Results
Decreased progression to renal disease in type 1 diabetes In type 2 diabetes, no difference in eventual progression to dialysis Does not prevent cardiovascular events More hypoglycemia

Rapid-Acting Analogues
Lispro (Humalog), aspart (Novolog), glulisine (Apidra) Analogs of human insulin; all similar Onset 15 min, peak 1-3 hr, duration 2-5 hr May need to adjust long-acting regimen Particularly well-liked by type 1 diabetics (~ 1/3 of daily insulin requirement) Available in 75/25 mix with longer acting protamine form

Long-Acting Insulins
NPH duration 16-24 hr (2/3 in AM, 1/3 in PM) Glargine (Lantus) 24 hr (human analog)
Cannot mix with other insulins Solution must remain clear Used in types 1 and 2 Initiate dose at 80% of prior total insulin dose Split dose when > 60 units Best approach for geriatric patients in long-term care facilities (predictable control)*

Detemir (Levemir) similar to glargine


Not supposed to increase weight* Length of activity increases as dose increases

Insulin Pump/Transplant
High patient satisfaction Improved glucose control Uses only short-acting insulin Requires motivated patient to do frequent glucose checks Pancreatic transplant still primarily experimental Early diagnosis of type 1immunosuppression and hematopoietic stem cell transplant increases beta cell function and prolongs insulin independence

4. Which of the following medications would not be recommended for a 15-y/o with type 2 diabetes, hypertension, and hyperlipidemia? A. Metformin B. Januvia C. Glargine insulin D. Lisinopril E. Pravastatin

Treatment in Children
Treatment in children: metformin and insulin* Screening for complications in type 1
Microalbumin yearly beginning age 10 Retinopathy beginning at age 10 and 3-5 years after onset then yearly Screen for celiac disease Screen for hypothyroidism Screen for hypertension

Hypertension
ACE inhibitor for HTN or elevated albumin/creatinine

Lipids
Check if positive family history Use statins if > age 10

Endocrinologists Algorithm for Initiation of Therapy, Based on A1c Levels


6.5-7.5%: monotherapy (usually metformin) 7.6-9.0%: dual therapy: metformin plus
Sulfonylurea or TZD: Pioglitazone or Glinide: Repaglinide/nateglinide or DPP-4: Sitagliptin/saxagliptin/linagliptin or GLP-1 agonist: Exenatide/pramlintide/liraglutide Metformin plus DPP-4 or GLP-1 plus/or Sulfonylurea or glinide plus/or TZD

> 9.0%: Insulin or triple therapy:

Metformin + exenatide + pioglitazone at onset reduced treatment failure by 84% at 2 years

New Aspirin Recommendations


Diabetics: 2-4x the risk for cardiovascular complications Stroke risk doubles within 5 years of diagnosis Bleeding risk increases in patients with diabetes on aspirin Low-dose aspirin reasonable in adults with diabetes and no history of vascular disease whose 10-year risk of CHD events is > 10% and no increased risk of bleeding
Males > 50 and Females > 60 with 1 additional risk factor Smoking, hypertension, dyslipidemia, albuminuria, family history of premature death

Secondary prevention for diabetes and known cardiovascular disease


Clopidogrel (Plavix) 75 mg/d if allergic to aspirin

Immunizations
Influenza yearly Pneumococcal once and repeat at age 65 (5 years after the 1st) Hepatitis B now recommended for all age 60 (after age 60, at high risk for hep B) Tdap (replaces Td one time only) Zoster vaccine at 60

Complications
Macrovascular
Heart disease Stroke

Diabetic foot problems


Ulcers Osteomyelitis Charcot foot

Microvascular
Retinopathy Neuropathy Nephropathy

Ketoacidosis and Hyperosmolar hyperglycemia Hypoglycemia

Treatment of Comorbidities and Complications

5. In which of the following is the risk of peripheral vascular disease the highest?

A. B. C. D. E.

African-American male Asian male White female Hispanic male Native American female

Cultural Competence
Asian-Americans: Develop diabetes at a lower body mass
(BMI = 24, compared to African American = 26, Caucasian = 30)

Develop diabetes at a younger age Develop more end-stage renal failure African-Americans Insulin resistance is higher Develop retinopathy at a lower A1c level Latino-Americans Peripheral vascular disease 80% more common Mortality rate 2x whites 50% of Latino children will develop diabetes and the rate in Latinos will double in the next 10 years

Screening for Complications


Yearly dilated eye exam Yearly urine microalbumin and creatinine Foot screening to prevent amputation:
Yearly monofilament, pulses, vibratory Visual inspection of feet at every visit

Ask about autonomic neuropathies: erectile dysfunction, postural hypotension, gastroparesis (best test is gastric emptying time*) Screening for cardiac disease with stress echo/thallium if patient is symptomatic, develops microalbuminuria, or for high index of suspicion

Prevention of Complications
Glycemic control
1% reduction in A1c = 21% decrease in risk of developing a complication

Better outcomes with tight control early in the disease process (no evidence for tight control as the disease progresses)

Blood pressure control Correction of dyslipidemia (statins) Smoking cessation Healthy diet and active lifestyle Aspirin to prevent cardiovascular complications

6. 19 y/o CM with ketoacidosis. K+ 6.8, pH 7.12. What fluid should you use?
A. B. C. D. NS until K+ is 4.0, then add K+ NS until K+ is 5.0, then add K+ NS until K+ is 5.5, then add K+ NS until K+ is 4.0, then add K+

Ketoacidosis
Insufficient insulin; increased gluconeogenesis and fatty acid oxidation resulting in metabolic acidosis Uncommon in type 2 unless African-American or Hispanic Criteria
Glucose > 250 pH < 7.3 Bicarbonate < 18 Serum and urine ketones

Ketoacidosis Treatment
Volume replacement
1 L NS/hr until dehydration resolved Then NS at 150-500 ml/hr* (usually down 5-8 L)

Insulin drip (1-2 units/hr0.1 U/kg/h) Hourly monitoring of electrolytes, glucose, and pH Bicarb only for pH < 7.2 or HCO2 < 10 mEq/L* Replace K+ as soon as it starts to fall* Continue insulin drip until acidosis is resolved* Add D5 when glucose is ~ 250 mg/dL*
Reduce but do not stop drip if hypoglycemic

Hyperosmolar, Hyperglycemic State*


Hospitalization (may need ICU)
Mortality >>> DKA

Insulin infusion (oral and SQ are inadequate)* IV fluids (normal saline) Replace K+ as it falls near normal range Oral medications and/or subcutaneous insulin is restarted after blood sugars return to the 200 range Delirium or altered mental status usually clears with correction of metabolic abnormalities

Microvascular Complications
Nephropathy, retinopathy, neuropathy All treated with:
Glycemic control BP control Lipid control Smoking cessation

No evidence that ASA is helpful*

Treatment
Retinopathy
Refer to ophthalmologist for any retinopathy Laser photocoagulation Ranibizumab (Lucentis) for macular edema

Neuropathy
1st: amitriptyline, nortriptyline, gabapentin, pregabalin, duloxetine, venlafaxine, 5% lidocaine patch 2nd: topiramate, lamotrigine, carbamazepine, capsaicin cream 3rd: opioids, tramadol Also try: L carnitine, acupuncture Symptomatic treatment

Nephropathy Treatment
Avoid NSAIDS: they acutely reduce renal blood flow and may cause interstitial nephritis* Aggressive management of blood sugar and BP Treat with increased doses of ACE inhibitors or switch to ARB if creatinine is increasing despite ACE therapy Dont use ACE inhibitor and ARB together* Continue albumin measurement to assess response to therapy and progression Limit dietary protein When hypoglycemia occurs in previously well controlled type 2, most likely cause is progressing renal failure* Refer: rapid decline, difficulty managing, advanced disease

7. 57 y/o CM with diabetic foot ulcer. What is the best indicator of its ability to heal?

A. B. C. D. E.

Size of ulcer Patients pulse Signs of infection Patients blood sugar Patients blood pressure

Diabetic Foot
Leading cause of non-traumatic foot amputation Neuropathy, altered foot structure, vasculopathy Best test for sensation is a monofilament* Best treatment is aggressive prevention Diabetic foot ulcer: remove pressure; good wound care and debridement; no antibiotics if not infected; Osteomyelitis usually occurs in the foot*: best test is MRI** Best indicator for successful healing: intact vascular supply (pulses)* Assess decreased pulse with noninvasive vascular studies (ankle-brachial index)* Etiology:
Untreated: aerobic Gm+ staph and -hemolytic strep Treated: polymicrobial Cover MRSA and strep (dicloxacillin, cephalexin, augmentin, doxycycline, trimethoprim/sulfa)

References
Aoki TT, White RD. Initiating insulin in patients with type 2 diabetes. J Fam Practice. 2007;Suppl:S4-S12. Cordts S. Selfmonitoring of blood glucose in patients with diabetes not using insulin. Cochrane for Clinicians Am Fam Physician 2012;85:866-7. Executive summary: Standards of medical care in diabetes 2014. Diabetes Care. 2014;37(Supp1):S5-S65. Graham DJ. Risk of acute myocardial infarction, stroke, heart failure, and death in elderly medicare patients treated with rosiglitazone or pioglitazone. JAMA 2010;304(4):411-18.

Graham L. ADA releases updated recommendations on standards of medical care in diabetes. Am Fam Physician 2010;82(2):206-7.
Lindsay TJ, et al. Treating diabetic peripheral neuropathic pain. Am Fam Physician 2010;82(2):151-158.

Reference
Peterson K, Silverstein J, Kaufman F, Warren-Boulton E. Management of type 2 diabetes in youth: an update. Am Fam Physician. 2008;76:658-66. Robard HW, et al. Statement by an American Association of Clinical Endocrinologists/American College of Endocrinology Consensus Panel on type 2 diabetes mellitus: an algorithm for glycemic control. Endocrine Practice 2009;15(6):540-59. Sawin G, Shaughnessy AF. Glucose control in hospitalized patients. Am Fam Physician 2010;81(9):1121-24. Shakil A, Church RJ, Shobha SR. Gastrointestinal complications of diabetes. Am Fam Physician. 2008;77(12):1697-1704. Trojian TH, Chu SM. Dupuytrens disease: Diagnosis and treatment. Am Fam Physician. 2007;76(1):86. Yehia BR, Epps KC, Golden SH. Diagnosis and management of diabetic ketoacidosis in adults. Hospital Physician. March, 2008;21-26.

Answers
1. 2. 3. 4. 5. 6. 7. A D A B A C B

Other Complications
Dupuytrens disease prevalence up to 33%* Diabetes is a risk factor for developing necrotizing soft-tissue infections* Perinephric abscess*
Fever and persistent flank pain Fever > 4 days after initiation of antibiotics *CT for peri-renal fluid or enlargement of psoas muscle Peri-renal gas on CT is diagnostic Treatment: drainage and antibiotics

Fatty liver: 25% progress to cirrhosis


Metformin can improve metabolic problems

Acrochordons* (skin tags) common finding

Integrative Options for Diabetes


Chromium and fiber: beneficial effect on glycemic control (SOR: C). Maybe magnesium. Lower vitamin D levels increase risk of DM but no evidence that supplementation works Alpha lipoic acid shows promise for diabetic neuropathy Evidence on cinnamon is inconclusive Acupuncture can be helpful for peripheral neuropathy and bladder dysfunction (SOR: B) Biofeedback and meditation best for stress reduction to improve glycemic control (SOR: C) Yoga and Tai Chi questionable

Evidence
Intensive treatment early in diabetes with tight control of blood sugar and blood pressure can decrease complications (neuropathy, retinopathy, nephropathy, and foot infections) and improve long-term outcomes*.
(Ebell in AFP)

Tight control in type 1 improves cardiovascular outcomes. In type 2 and elderly, tight control may be detrimental.* (Nathan, Diab Care, 2009;32:193) Lowering blood pressure below conventional standards reduces the incidence of cardiovascular events and mortality. (AHRQ) Lifestyle interventions for overweight individuals with impaired glucose tolerance reduces the incidence of progression to diabetes. (Bandolier)

Evidence
Both vigorous exercise and moderate exercise reduce the risk of type 2 diabetes in women. The more exercise taken, the greater the risk reduction. Bandolier: RCT There is fair evidence to recommend acarbose treatment for overweight individuals with impaired glucose tolerance to prevent cardiovascular events or hypertension (level B). Metformin should be considered as the first-line oral hypoglycemic agent in overweight patients with diabetes***.
National Guideline Clearinghouse: (SOR:A)

Metformin corrects fatty liver by activating AMP-activated protein kinase, decreasing acetyl-CoA carboxylase, and reducing fatty acid oxidation. There is no specific creatinine level beyond which ACE inhibitors or ARBs cannot be used.* National Guideline
Clearinghouse: (SOR:A)

Maternity Care II
David G. Weismiller, MD, ScM, FAAFP
Department of Family Medicine
The Brody School of Medicine at East Carolina University
weismillerd@ecu.edu

Maternity and Gynecologic Care AAFP Reprint No. 261 http://www.aafp.org/dam/AAFP/documents/medical_education_residency/ program_directors/Reprint261_Maternity.pdf

Disclosure Statement
Dr Weismiller has nothing to disclose.

The AAFP has selected all faculty appearing in this program. It is the policy of the AAFP that all CME planning committees, faculty, authors, editors, and staff disclose relationships with commercial entities upon nomination or invitation of participation. Disclosure documents are reviewed for potential conflicts of interest and, if identified, they are resolved prior to confirmation of participation. Only those participants who had no conflict of interest or who agreed to an identified resolution process prior to their participation were involved in this CME activity.

Learning Objectives
1. Identify appropriate techniques to recognize and manage premature onset of labor. 2. Define the treatment of DM and the diagnosis and treatment of hypertension in pregnancy. 3. Describe management of induction and augmentation of labor including post-term pregnancy. 4. Recall the recognition and treatment of late pregnancy bleeding including abruptio placentae, placenta previa, and postpartum hemorrhage.

1. A 20-year-old female is seen for follow-up 6 weeks after delivery. Her pregnancy was complicated by preeclampsia. Her examination is unremarkable. This patient will be at increased risk for which of the following in mid-life?

A. Breast cancer B. Diabetes C. Hypothyroidism D. Ischemic heart disease

Definitions
Gestational BP
> 140/90 mm Hg on two occasions at least 4 hours apart >160 mm Hg systolic or > 110 mm Hg diastolic Hypertension can be confirmed within minutes to facilitate timely antihypertensive therapy.

Nomenclature
Preeclampsia Eclampsia Chronic hypertension Gestational hypertension Transient hypertension of pregnancy Chronic hypertension

Preeclampsia-Eclampsia
ACOG Task Force on Hypertension in Pregnancy. Obstet Gynecol. 2013;122(5):1122-1133.

Diagnosis
Gestational BP elevation after 20 weeks of gestation in a woman with previously normal BP Proteinuria > 0.3 g protein in a 24-hour urine specimen
No longer REQUIRED to make the diagnosis of preeclampsia when hypertension occurs with one or more severe features. When proteinuria IS used among other diagnostic criteria for preeclampsia, a protein:creatinine ratio of at least 0.3 is sufficient.

Severe features
SBP > 160 mm Hg; DBP > 110 mm Hg on two occasions at least 4 hours apart while the patient is on bed rest (unless antihypertensive therapy is initiated before this time) Plt count < 100,000/microliter Impaired liver function
Abnormally elevated transaminases (> twice normal) Severe persistent RUQ pain or epigastric pain unresponsive to medication and not accounted for by alternative diagnosis, or both

Progressive renal insufficiency (serum Cr > 1.1 mg/dL or a doubling of the serum creatinine concentration in the absence of other renal disease) Pulmonary edema New-onset cerebral or visual disturbances

Preeclampsia-Eclampsia
ACOG Task Force on Hypertension in Pregnancy. Obstet Gynecol. 2013;122(5):1122-1133.

Diagnosis
Gestational BP elevation after 20 weeks of gestation in a woman with previously normal BP Proteinuria > 0.3 g protein in a 24-hour urine specimen
No longer REQUIRED to make the diagnosis of preeclampsia when hypertension occurs with one or more severe features When proteinuria IS used among other diagnostic criteria for preeclampsia, a protein:creatinine ratio of at least 0.3 is sufficient

Severe features

Patients who have had preeclampsia have a 4X increased risk of hypertension and a 2X increased risk of ischemic heart disease, stroke, and VE.
No association between preeclampsia and cancer

Risk Factors for Preeclampsia


Pregnancy-associated factors Maternal-specific factors
Chromosomal abnormalities Hydatidiform mole Hydrops fetalis Multifetal pregnancy Oocyte donation or donor insemination Structural congenital abnormalities Urinary tract infection Age < 20 and > 35 Black race Family history Nulliparity Preeclampsia in previous pregnancy Medical conditions
Diabetes
Type 1 GD

Paternal-specific factors

First-time father Previously fathered a preeclamptic pregnancy in another woman

Obesity Chronic hypertension Renal disease Thrombophilias

Chronic Hypertension
Gestational BP elevation before the 20th week of gestation Hypertension that is diagnosed for the first time during pregnancy and does not resolve postpartum is also classified as chronic hypertension. Chronic hypertension is the most common cause of IUGR.

Management of Chronic Hypertension


ACOG Task Force on Hypertension in Pregnancy. Obstet Gynecol. 2013;122(5):1122-1133.

Blood pressures persistently above 160/105 should be treated to goals between 120/80 and 160/105.
Labetalol Nifedipine Methyldopa

Ultrasound screening for fetal growth restriction is appropriate. Antenatal fetal surveillance* should be performed in women who:
Require antihypertensive therapy Have superimposed preeclampsia Have fetal growth restriction
*Fetoplacental assessment: Biophysical profile or modified biophysical profile to include umbilical artery Doppler velocimetry

Gestational Hypertension
Gestational blood pressure elevation without proteinuria that is detected for the first time after 20 weeks gestation Two types
Transient hypertension of pregnancy: Preeclampsia has not developed and BP has returned to normal by 12 weeks postpartum. Chronic hypertension: BP elevation persists.

Prevention
Low-dose ASA (60-80 mg)
Women without risk factors no benefit At risk, 19% reduction (NNT = 69); begin at the end of first trimester
Histories of preeclampsia that is recurrent Previously developed before 34 weeks

Calcium supplementation

No data that dietary supplementation with calcium will prevent preeclampsia in low-risk women in the US.
Helpful:
High risk of gestational hypertension (teenagers, previous preeclampsia, women with increased sensitivity to angiotensin II, preexisting hypertension) Communities with low dietary calcium intake (mean intake < 900 mg/d) demonstrate significant reductions in incidence of preeclampsia.

Bedrest is NOT recommended for prevention OR treatment of preeclampsia.

2. 19-yo primigravida at approximately 40 weeks EGA comes to the hospital with painful contractions. She has received no prenatal care. Examination: Cervix is 4 cm dilated and 80% effaced at station -1. Blood pressure is 164/111 mm Hg and a urine dipstick shows 3+ protein. She reports that she has had severe headaches for 3 days and has noticed a lot of swelling in her hands and feet. Moments after her blood is drawn and IV access is obtained, she has a generalized tonic-clonic seizure and fetal heart tones drop to 60 BPM. Which of the following is the most appropriate immediate course of action?

A. Emergency cesarean section B. Lorazepam (Ativan), 2 mg IV push, repeated in 2 minutes if necessary C. Magnesium sulfate, 4 g loading dose IV, followed by a drip at 2 g/hr D. Terbutaline, 0.25 mg subcutaneously

Management
ACOG Task Force on Hypertension in Pregnancy. Obstet Gynecol. 2013;122(5):1122-1133.

Anticonvulsive therapy
MgSO4 Reduces the risk of first or subsequent seizures in women with severe preeclampsia
Now ONLY recommended for women with preeclampsia with severe features

Reduces risk of subsequent seizures in women with eclampsia


Antidote for toxicity: Calcium gluconate 1 g IV over 3 minutes

Antihypertensive therapy Vaginal delivery is preferred!

Antihypertensive Therapy
Acute Hypertension in Preeclampsia
BP dangerously high or rises suddenly in women with preeclampsia, especially intrapartum.
Antihypertensive medications should not be administered unless BPs are persistently > 160/110.
Pharmacologic Agent
Arterial vasodilator
eg, Hydralazine

Dose
5 mg IV over 1-2 minutes, or IM 20-40 mg IV bolus or 1 mg/kg infusion (maximum 220 mg) 10 mg po and repeat in 30 minutes, if necessary; oral nifedipine lowered BP more quickly than did IV labetaolol. (Shekhar S et al. Obstet Gynecol. 2013;Nov;122:1057.) 0.25 ug/kg/min to a maximum of 5 ug/kg/min

Beta-blocker
eg, Labetalol

Calcium antagonists eg, Nifedipine

Sodium nitroprusside

Postpartum Management
ACOG Task Force on Hypertension in Pregnancy. Obstet Gynecol. 2013;122(5):1122-1133.

In women with gestational hypertension and/or preeclampsia, BP should be monitored for at least 72 hours postpartum and again 7-10 days after delivery. Greatest risk for postpartum hypertension Antenatal preeclampsia particularly with higher urinary protein, serum uric acid, and BUN

3. Gestational diabetes has been associated with all of the following perinatal complications except:
A. Increased frequency of maternal hypertensive disorders B. Increased risk of operative delivery C. Increased frequency of neonatal hyperglycemia D. Increased risk of intrauterine fetal death during last 4-8 weeks of gestation

Why All the Fuss


Adverse Outcomes
Maternal
Increased frequency
Maternal hypertensive disorders Cesarean delivery

Fetal
Excessive fetal growth (macrosomia)
Increased risk for operative delivery Shoulder dystocia Birth trauma

Increased risk of intrauterine fetal death during last 4-8 weeks of gestation
Fasting hyperglycemia (> 105 mg/dL)

Neonatal morbidity
Hypoglycemia
Hypocalcemia Hyperbilirubinemia Polycythemia

The most common cause of neonatal death in children of mothers known to have DM before pregnancy is congenital anomalies.

4. 26-yo G1 P0 at 28 weeks gestation has a 1-hour plasma glucose level of 145 mg/dL on a 50 g/hr glucose challenge test. A 3-hour glucose tolerance test confirms gestational diabetes. You initially recommend glucose monitoring and treatment with diet and physical activity. You would recommend insulin therapy if her 2-hour postprandial blood glucose levels are not consistently below which of the following target levels?

A. 100 mg/dL B. 120 mg/dL C. 140 mg/dL D. 160 mg/dL

GD Treatment
ACOG 2013
Initial management (Level A)
Nutritional counseling by registered dietician Advice on moderate exercise program (if possible)

No conclusive evidence for the threshold value at which clinicians should start medical therapy Pharmacologic treatment
Insulin and oral medications (eg, glyburide and metformin) equivalent in efficacy (Level A) Either can be an appropriate first-line therapy.
ACOG Practice Bulletin: Management of Gestational Diabetes Mellitus. Obstet Gynecol. 2013;122:406-416.

Gestational Diabetes
Maternal Surveillance Glucose Monitoring
Optimal frequency in GD not established
Fasting whole blood glucose
< 95 mg/dL (< 105 plasma) 2 hour postprandial BG < 120 mg/dL (1 hour < 140)

1-2 times per week versus daily

Human insulin therapy (if on more than 3 occasions)


> 95 mg/dL fasting whole blood glucose or > 120 mg/dL 2 h postprandial Daily glucose monitoring

ACOG 2013

Diabetes
Timing of Delivery?
GD: Manage expectantly
As long as glucose values normal (Level C) Council regarding option of scheduled cesarean delivery when EFW > 4500 g (Level B)

Pre-gestational diabetes (well controlled)


Recommend delivery after 39 weeks and by the EDD

Long-Term Considerations
Increased risk for recurrence of GD
33%-50% likelihood

Increased risk for development of diabetes after pregnancy


Up to 1/3 will have diabetes or impaired glucose metabolism at postpartum screening. 35% of women 5-10 years after parturition Usually type 2

Offspring increased risk


Obesity Glucose intolerance Diabetes in late adolescence and young adulthood

Postpartum
Reclassification of maternal glycemic status at least 6 weeks after delivery (preferred 6-12 weeks, ACOG) [Level C]
FPG or 2-hr OGTT

Reassessment of glycemia every one (USPSTF) to three years (ADA) [SOR:C], if above normal; yearly assessment (ADA) if impaired fasting glucose or impaired glucose tolerance at 6-12 weeks

5. A 26-yo G2P1 presents at 30 weeks gestation with a complaint of severe itching. She has excoriations from scratching in various areas. She says that she had the same problem during the last pregnancy, and her medical records reveal a diagnosis of intrahepatic cholestasis of pregnancy. Elevation of which of the following is most characteristic of this disorder?

A. B. C. D.

GGT Bile acids Direct bilirubin Prothrombin time

Intrahepatic Cholestasis of Pregnancy (ICP)


Most common pregnancy-related liver disorder Etiology ambiguous
Abnormalities in the metabolism and disposition of sex hormones and/or bile acids ? Genetic predisposition ? Environmental factors

Intrahepatic Cholestasis of Pregnancy: Diagnosis


Historic
Pruritus at night, pruritus continually (with no rash) and mild jaundice during the 3rd trimester; elevation of serum level of total bile acids
Clinical jaundice in 50% of cases; pruritus worsens with onset of jaundice.

Tends to recur (60%-70%); severity varies. Seldom exhibits before 25 weeks

Hepatitis
More than 20% of women known to be infected with hepatitis C develop ICP, so testing is indicated.

Liver biopsy
Pathologic findings are not specific, and a liver biopsy is rarely indicated.

Laboratory Presentation
Parameter Finding
Serum alkaline phosphatase Bilirubin Increased 5-10 fold (hepatic > placental) Elevated bilirubin (conjugated, < 5mg/dL)

Serum transaminases
Serum bile acids (fasting)

Usually normal
> 3 times upper limit of normal

GGT

Usually normal or modestly elevated may help to differentiate this condition from other cholestatic liver diseases
Usually normal but if elevated may reflect a vitamin K deficiency from malabsorption

PT

Pharmacologic Treatment

* Kondrackiene J et al. Gastroenterology 2005;129(3): 894-901

Goal: Decrease maternal symptomatology and enhance fetal outcome. Main drug prescribed: Most benefit for both mother and infant Ursodeoxycholic acid*
10 mg/kg q day

Pharmacologic Treatment

Outcomes ICP
Maternal usually benign Impaired quality of life Pruritus and abnormal laboratory tests resolve within 1-2 weeks of delivery, no sequelae. Fetal may be serious Higher bile acid levels and impaired transport of bile acids from the fetus through the uteroplacental circulatory system
Preterm labor, preterm delivery Fetal compromise
Increased risk of cesarean delivery

Meconium-stained amniotic fluid IUFD

Bleeding in Late Pregnancy


Placenta previa Placenta abruption Vasa previa Cervical trauma (eg, intercourse) Vaginal infections Bloody show

6. Which of the following is not a risk factor for placenta previa?


A. B. C. D. Young maternal age Prior cesarean delivery Advanced maternal age Tobacco use

Placenta Previa
Occurs in 0.3%-0.6% of all births Painless vaginal bleeding Bleeding patterns first bleeding episode:
Before 30 weeks Between 30 and 35 weeks After 36 weeks 33% 33% 33%

Each successive episode of bleeding may be heavier, more unpredictable.

Placenta Previa Etiology


Usually none is found. Risks:
Prior C/S or history of uterine curettage
? Damage to myometrium or endometrium

Cocaine Advanced maternal age Tobacco

Recurrence rate of 6-12x in subsequent pregnancy Malpresentation is the result rather than the cause of placenta previa.

Placenta Previa Diagnosis


Antenatal by ultrasound Before 24 weeks, only a cautionary significance is attributed to a placenta close to the os.
90% have moved away from the os at term. Need to confirm movement in 3rd trimester. In cases where the bulk of the placenta is over the os after 24 weeks, it is less likely to be clear of the os at term.

7. Placental abruption is associated with each of the following maternal conditions except:
A. B. C. D. Trauma Tobacco use Hypertensive disorders Diabetes

Placenta Abruption
Separation of the placenta from implantation site Occurs in approximately 1% of all deliveries
Incidence increases with gestational age. Most serious complications are due to hypovolemia acute renal failure.

Etiology
Trauma, cocaine abuse, acute decompression of amniotic fluid, preterm rupture, hypertensive disorders, tobacco use

Risk of recurrence high (20-30x)

Placenta Abruption Management


Assess fetal viability
Fetal demise here placental detachment usually > 50%
Deliver the fetus. Prepare to transfuse.

Live fetus + rigid uterus again > 50% detachment


90% fetal demise C/S

Live fetus + soft uterus induction of labor

Antepartum Hemorrhage Summary


Symptom/Sign Pain Contractions Blood Coagulation Concealed hemorrhage Placenta previa Minimal Absent or mild Bright red Normal Almost never Abruptio placentae Severe Hard, rapid, often tetanic Port wine Abnormal > 20% of time

Preterm Labor Diagnosis


Gestation > 24 weeks, < 37 weeks Contractions and cervical change

8. Which of the following is the greatest risk factor for preterm labor?
A. B. C. D. Concurrent STD Low socioeconomic status Uterine anomaly History of mid-second-trimester spontaneous loss of pregnancy

Risk Factors for Preterm Labor Greatest Risk


Prior preterm birth* Pre-pregnancy Recurrence risk of weight < 50 kg* 17%-37% Bleeding* Concurrent STD* African-American race*

9. Regarding the risks and benefits of corticosteroid therapy for fetal lung maturation, which of the following is not true?
A. Corticosteroid therapy is recommended for all pregnant women between 24 and 34 weeks gestation who are at risk of preterm delivery within 7 days. B. Corticosteroid therapy has been associated with an increased risk of neonatal infection. C. Multiple courses of corticosteroids have been associated with fetal adrenal suppression. D. Corticosteroids accelerate the appearance of pulmonary surfactant in the fetal lungs.

Preterm Labor
Symptomatic Management
Intervention
Bed rest Hydration Tocolytic agents

Recommendation
Not effective for the prevention of preterm birth and should NOT be routinely recommended (Level B) Not effective for the prevention of preterm birth and should NOT be routinely recommended (Level B) First line: Beta-adrenergic agonist, calcium channel blockers, NSAID for short-term prolongation of pregnancy; maintenance therapy is INEFFECTIVE for preventing preterm birth and improving neonatal outcomes (Level A) Single course between 24 and 34 weeks who are at risk of preterm delivery with in 7 days (Level A) (Betamethasone 12 mg q 24hX2 OR Dexamethasone 6 mg q 12hX4) Should NOT be used to prolong gestation or improve neonatal outcomes in women with POL and INTACT membranes (Level A) Reduces the severity and risk of CP in surviving infants if administered when birth is anticipated before 32 weeks of gestation (Level A)

Corticosteroids

Antibiotics Magnesium Sulfate

Prevention of Preterm Labor


17-hydroxyprogesterone caproate
Trade name: Makena

Only one indication


Pregnant women with a history of at least one spontaneous preterm birth

Weekly injection (hip)


Begin at 16 weeks and no later than 21 weeks.

10. Which of the following statements is most accurate regarding postpartum hemorrhage (PPH)?
A. PPH occurs in 10%-15% of vaginal deliveries. B. PPH is defined as a 10% change in hematocrit between admission and postpartum period. C. Active management of the third stage does not impact the risk of PPH. D. PPH is an uncommon cause of maternal morbidity and mortality.

Postpartum Hemorrhage Definition


ACOG Practice Bulletin, October 2006
10% change in hematocrit between admission and postpartum period Need for erythrocyte transfusion

Early: First 24 hours after delivery Late: After 24 hours but before 6 weeks after delivery

Early Postpartum Hemorrhage Etiology


Uterine atony (Tone) Uterine overdistention
Hydramnios Multiple gestation Fetal macrosomia

Retained placental fragments (Tissue)


Including abnormal placentation
Accreta

Oxytocin use High parity Rapid or prolonged labor Intra-amniotic infection Halogenated anesthetic

Increta
Percreta

Early Postpartum Hemorrhage Etiology


Lacerations of vagina and cervix (Trauma)
Forceps Fetal macrosomia Precipitous labor and delivery Episiotomy

Coagulopathy (Thrombin)
Hereditary DIC
Sepsis Abruption

HELLP

Uterine rupture Uterine inversion

Treatment Approach
Uterine massage
Bimanual

Oxytocics (medical uterotonic therapy) Inspect for lacerations Surgical intervention


Anderson JM and Etches D. Prevention and Management of Postpartum Hemorrhage. Am Fam Physician. 2007;75:875-82.

Oxytocin

Postpartum Hemorrhage Treatment Medical Uterotonic Therapy

Misoprostol (Cytotec, PGE1) Methylergonovine


Per rectum (400 mcg after placenta and 100 mcg at 4 and 8 hours postpartum, total 800-1000 mcg)* Contraindicated in presence of hypertensive disease state IM or intramyometrial

IV, IM, or intramyometrial

15-methyl PGF2-alpha (Carboprost or Hemabate)


Dinoprostone (PGE2)

* Caliskan E et al. Am J Obstet Gynecol. 2002; 187:1038-45.

Postpartum Hemorrhage Prevention


Correct anemia. Avoid routine episiotomy. Infant to breast Routine use of pitocin after delivery of the placenta Active management of third stage

Third Stage of Labor Management


Expectant
Await separation. Leave cord uncut. Spontaneous placental delivery Oxytocin/breast after placental separation

Active
Oxytocin with shoulder delivery Cord clamped and cut early (2-3 minutes) Controlled cord traction

Accurately date everyone.

Postdates Pregnancy > 42 weeks

Early sonograms help (Level A).

Antenatal fetal testing between 41 and 42 weeks (Level C)


q 4-7 days depending on method Most do MBPP q 3-4 days.

Induction: Risk of routine induction (C/S) in the era of cervical ripening agents is lower than previously reported.
Cervical ripening: Improves success of inductions (Level B) Bishop score of 8 Oxytocin Non-pharmacologic methods of labor induction may be helpful.

Cervical Ripening Agents


(Induction is indicated, cervix is unfavorable)
Total score > 8 (Max score 15); probability of vaginal delivery after labor induction is similar to that after spontaneous labor. Dilatation 0 cm 1-2 cm 3-4 cm > 5 cm (Points) 0 1 2 3 Effacement Consistency Position Station* 0%-30% 0 Firm 0 Post 0 40%-50% 1 Medium 1 Mid 1 60%-70% 2 Soft 2 Anterior 2 -1/0 2 +1/+2 3 > 80% 3

-3 -2 0 1 * Station reflects a -3 to +3 scale.

Modified from Bishop EH. Pelvic scoring for elective induction. Obstet Gynecol. 194;24:267.

Induction
Level A Evidence
Post-term pregnancies with unfavorable cervix can undergo induction OR expectant management. Prostaglandins (PG) can be used to promote cervical ripening AND induce labor. Delivery should be effected if there is evidence of fetal compromise or oligohydramnios.

Level C Evidence
Reasonable to initiate fetal monitoring between 41-42 weeks.
Many practitioners use twice weekly. NST and AFI (MBPP) should be adequate.

Many recommend induction or prompt delivery in patients with a favorable cervix and no other complications.

Additional Resource
Syllabus Advanced Life Support on Obstetrics

Answer Key Maternity Care II


1. 2. 3. 4. 5. 6. 7. D C C B B A D 8. A 9. B 10.B 11.C 12.A 13.D

Additional Material
Thyroid Disease Seizure Disorder Trauma Shoulder Dystocia Breech

11. A 32-yo Asian female presents to your office for a 6-month follow-up for hypothyroidism. She informs you that she is about 8 weeks pregnant. She currently takes levothyroxine (Synthroid), 100 g daily. Which one of the following statements is true regarding her medication regimen?
A. Levothyroxine should be discontinued until after delivery. B. A beta-blocker should be added to her regimen during pregnancy. C. The dosage of levothyroxine will most likely need to be increased during the pregnancy. D. The dosage of levothyroxine will most likely need to be decreased during the pregnancy.

Thyroid Gland and Pregnancy


Glandular hyperplasia and increased vascularity result in moderate thyroid enlargement but not thyromegaly.
Thyroid function tests are not indicated in asymptomatic pregnant women with slightly enlarged thyroid glands (Level B).

Any goiter or nodule recognized during pregnancy should be considered pathologic.

Hypothyroidism in Pregnancy
Difficult to diagnose
Subclinical disease more common than overt disease

Untreated hypothyroidism associated with:


Pregnancy loss Prematurity Low-birth-weight infants Medically indicated preterm delivery, preeclampsia, placental abruption ? IUGR (not clear if independent of other complications) Poor cognitive function

Correlation between severity of maternal hypothyroidism and the severity of the cognitive disability in the offspring

Hypothyroidism
Treatment
Begin full dose replacement with 0.1 to 0.15 mg per day. Goal is to maintain the TSH in the low normal range. Monitor TSH every 4-6 weeks.
In stable patients, prudent to check TSH levels q trimester.

Thyroxine requirement usually increases as the pregnancy advances. Fetal surveillance????

Hyperthyroidism
Treatment
Propylthiouracil (PTU) 100-150 mg q 8 hours (300-450 mg/day)
May require doses of 600-900 mg per day

Methimazole
10 to 20 mg daily in a single dose
Increase to control thyrotoxicosis. Monitor: Free T4 and TSH q month, LFTs at baseline 2m and 5m. Titrate downward as soon as possible.

Beta blockers may be used for rapid control of adrenergic symptoms.


Patil-Sisodia K and Mestman JH.. Graves Hyperthyroidism and Pregnancy: A Clinical Update. Management Endocrine Practice. 2010;16(1):118-129.

Maternal thyrotoxicosis
Inadequately treated associated with a greater risk: Preterm delivery
Including medically indicated preterm deliveries

Severe preeclampsia Heart failure Miscarriage (no data to support this claim) Fetal
LBW

Seizure Disorder
Optimize seizure control. Folic acid supplement
0.8-4 mg per day

Referral to genetic counselor Avoid multiple anticonvulsants. Use the best single agent for seizure type at lowest protective dose.
No single drug of choice Older agents FDA Risk category D Newer agents limited data

Seizure free for > 2 years: Consider for AED withdrawal six months or more prior to planned conception.

Seizure Disorder
Offspring
4%-8% risk of congenital anomalies
Anticonvulsant medication Increased genetic risk

Role of hypoxia associated with maternal seizure is unclear. Increased risk of developing epilepsy

12. A 26-yo G2P1 at 36 weeks fell off of her bicycle 2 hours ago. She says that she has felt no movement since the fall. She has abrasions on the left forearm and left leg but otherwise appears uninjured. Which one of the following would be the most sensitive indicator of fetal compromise in this patient? A. B. C. D. Continuous electronic fetal monitoring Vaginal bleeding Uterine ultrasonography Vaginal bleeding

Management of the Pregnant Woman after Trauma


Stabilize the pregnant woman using ABC protocols
Supplemental oxygen IV lactated Ringers solution in 3:1 ratio based on blood loss Laboratory studies CBC Blood type and Rh Kleihauer-Betke
Not predictive of fetal outcome

Injuries are severe consider:


Coagulation studies Chemistry panel ABG

Deflect uterus off of great vessels if more than 20 weeks gestation. Radiologic studies as needed

Secondary Survey
Examine for nonobstetrical injury.
Treat as needed.

Speculum examination to rule out spontaneous rupture of membranes, vaginal bleeding Check for fetal heart tone.

Fetal heart tone ABSENT Maternal treatment only; no fetal resuscitation

Fetal heart tone PRESENT Estimate gestational age (history, fundal height, Leopolds maneuvers, ultrasonography

< 24 weeks of gestation*


Document fetal heart tone No fetal resuscitation

* Determination of fetal viability varies by institution. Some centers use estimated fetal weight limits typically 500 g; others use a gestational age of 24-26 weeks.

> 24 Weeks of Gestation*


YES

Electronic fetal monitoring for four hours

EFM for 24 hours, intervene if needed for NRFHR tracing.

Vaginal bleeding SROM FH tone abnormality Uterine contractions for > 4 hours High-risk mechanism of injury Uterine tenderness Abdominal pain Maternal anesthesia

NO

D/C monitoring and discharge to home, return to L and D: Vaginal bleeding Decreased fetal movement Loss of fluid vaginally Repetitive uterine contractions Abdominal pain or tenderness

13. Following a prolonged second stage of labor, your primigravid patient at 42 weeks gestation delivers the fetal head, but the anterior shoulder does not release with a normal amount of downward traction. The statement that most accurately is associated with this complication is:
A. A history of shoulder dystocia in a previous pregnancy does not confer any greater risk of the complication in this pregnancy. It is important to stop and suction the newborn before attempting to deliver the anterior shoulder in cases where this complication is anticipated Routine induction of labor for macrosomia has been proven to improve perinatal outcomes. After delivery of the fetal head, pH can be expected to drop by 0.04 per minute due to cord compression.

B.

C. D.

Shoulder Dystocia
Caused by the impaction of the anterior shoulder above the symphysis pubis Elective induction of labor or elective cesarean delivery for all women suspected of carrying a fetus with macrosomia is not appropriate (Level B).
Planned C/S to prevent SD may be considered (Level C)

EFW > 5000 g with no DM EFW > 4500 g with DM

Shoulder Dystocia Complications


Maternal
Lacerations Perineum Vagina Rectum Hemorrhage Uterine atony Lacerations

Fetal
Brachial plexus (10%-20%) Erbs palsy Majority are bruising/stretching; complete recovery in 6-12 months (80+ percent) Cervical spine Asphyxia Cord pH drops 0.04/minute Fractures Clavicle Humerus

Shoulder Dystocia
A true obstetric emergency; studies suggest the majority are relieved by maneuvers that are safe for fetus and mother. Manual maneuvers will facilitate resolution of most of these cases.
Sequence in which maneuvers are performed will depend on the experience/preference of the individual provider. No evidence that one maneuver is superior to another in releasing an impacted shoulder or reducing the chance of injury.

Fundal pressure is counterproductive and not indicated.

ACOG Committee Opinion Term Singleton Breech July 2006


Decision regarding mode of delivery should depend on experience of healthcare provider.
Cesarean will be preferred mode for most physicians because of diminishing expertise in vaginal breech delivery.

Offer and perform ECV whenever possible. Planned vaginal delivery may be reasonable.
Great caution, detailed patient informed consent

Inform woman that the risk of perinatal or neonatal mortality or short-term serious neonatal morbidity may be higher than if cesarean delivery is planned.

Sexually Transmitted Infections, Vaginitis, Vaginosis


David G Weismiller, MD, ScM, FAAFP Department of Family Medicine
The Brody School of Medicine at East Carolina University
weismillerd@ecu.edu

Disclosure Statement
Dr Weismiller has nothing to disclose.

The AAFP has selected all faculty appearing in this program. It is the policy of the AAFP that all CME planning committees, faculty, authors, editors, and staff disclose relationships with commercial entities upon nomination or invitation of participation. Disclosure documents are reviewed for potential conflicts of interest and, if identified, they are resolved prior to confirmation of participation. Only those participants who had no conflict of interest or who agreed to an identified resolution process prior to their participation were involved in this CME activity.

Learning Objectives
1. Discuss the common clinical presentations for patients experiencing STIs. 2. Demonstrate the contemporary use of testing to diagnose STIs and treatment of STIs. 3. Review major infections with identification of appropriate therapy.

Case
22-yo G0 woman presents for an annual well-woman evaluation; c/o vaginal discharge for two days. Pelvic: No lesions on the vulva or vagina; cervix appears reddened, almost strawberry texture. There is a generous amount of yellowish, malodorous leukorrhea, but no notable pus at the cervical os. Bimanual exam: Questionable cervical tenderness and fullness in both adnexa; exam is limited due to the patients obesity.

1. Of the following, which is the most likely diagnosis?


A. B. C. D. Herpes simplex virus (HSV) infection Trichomonas vaginalis infection Candida albicans infection Bacterial vaginosis

Vaginal Discharge
Herpes simplex virus (HSV) infection Painful vesicles that ulcerate Trichomonas vaginalis infection Malodorous discharge, occasional vulvar and vaginal irritation From: CDC Cervicitis can be somewhat tender to touch; patients often c/o non-specific pelvic pain Candida albicans infection Very itchy with thick white discharge, typically Bacterial vaginosis (mixed vaginal flora) Shift/overgrowth in bacterial flora Malodorous discharge, vulvar itching; NO cervicitis or pelvic pain

Trichomoniasis
Pap smears and T. vaginalis
Conventional
Warrants treatment if high-risk patient (Am J Med. 2000;108)

Liquid-based cytology

From: CDC

Warrants treatment without further testing (Torre. AJOG. 2003;188)

2. Trichomonas vaginal infection is best treated with which of the following agents?
A. Metrogel-Vaginal (topical vaginal metronidazole) 5 g applied nightly for 5 days B. Zithromax (azithromycin) 1 g orally in a single dose C. Flagyl (metronidazole) 2 g orally in a single dose D. Levaquin (levofloxacin) 250 mg orally in a single dose

Trichomonas: CDC 2010


Recommended regimens
Metronidazole (resistance is rare)
2 g single oral dose Allergy: Use Clindamycin 300 mg BID for seven days.

Tinidazole
2 g single dose

Alternative regimen
Metronidazole 500 mg po BID for 7 days

Metronidazole gel considerably less efficacious (< 50%) compared to oral preparations Treatment failure
Repeat oral dose.

Treat the sexual partner.

Is There Necessary Follow-up?


High rates of reinfection 17% in one series
Rescreen women at 3 months benefit of this approach has not been fully evaluated. No data support rescreening in men.

Most recurrent infections thought to result from having sex with an untreated partner
Limited number due to low-level metronidazole resistance (2%-5%)
Use higher dose metronidazole or tinidazole.

Treatment Failure and Reinfection Is Excluded?


First choice
Metronidazole 500 mg po BID for 7 days

IF that fails
Tinidazole or metronidazole 2 g po q day for 5 days

Trichomoniasis: CDC 2010 Pregnancy


Women can be treated with 2 g metronidazole in a single dose at any stage of pregnancy. Multiple studies and meta-analyses have not demonstrated an association between metronidazole use during pregnancy and teratogenic or mutagenic effects in infants.

3. A 30-yo AA presents with vaginal discharge. On examination the discharge is homogeneous with a pH of 5.5, a (+) whiff test, and many clue cells. Which one of the following findings in this patient is most sensitive and specific for the diagnosis of bacterial vaginosis?

A. B. C. D.

pH of the discharge Presence of clue cells Character of the discharge Whiff test

Bacterial Vaginosis
Current clinical criteria (Amsel Criteria, 3 of 4) Discharge homogeneous grayishwhite pH > 4.5
Greatest sensitivity; lowest specificity Need vaginal pH paper

Clue cells
> 20% on HPF microscopy MOST SPECIFIC AND SENSITIVE SIGN OF BV
From: Google Images

Whiff test (+) amine


Volatilized amines released after 10% KOH (semen does the same thing)
From: CDC

BV Is Not an Annoyance Ailment


Premature rupture of membranes Preterm delivery Postpartum endometritis Salpingitis and PID Postoperative infections Vaginitis Acquisition of HIV

Bacterial Vaginosis Treatment


Primary Treatment
Metronidazole 500 mg po BID times 7 days
Avoid alcohol during treatment and 24 hours after.

CDC, 2010

Metronidazole vag gel (0.75%) 5 g in vagina q day times 5 days Clindamycin vag cream (2%) 5 g in vagina q day times 5 days

Alternative treatment
Clindamycin 300 mg po BID times 7 days Clindamycin ovules 100 g in vagina q HS times 3 days

Recurrent
Metronidazole vag gel (0.75%) 5 g in vagina 2x/week for 4-6 months

Probiotics Preventing Recurrent Vaginitis?


Lactobacillus
Eating yogurt
Daily for two months

Vaginal suppositories
Daily for a week, stop for week, then daily for second week

So
Bacterial vaginosis YES [SOR B] Candidiasis NO [SOR B] No adverse effects [SOR A]
Jurden L, Buchanan M, Kelsburg G, and Safranek S. Journal of Family Practice. 2012; 60(6):357, 368.

4. Which of the following statements is true when considering a patient with vulvovaginal candidiasis (VVC)? A. Diaphragm use is a predisposing factor to uncomplicated VVC. B. 80% of patients with symptomatic VVC will have candida documented on culture. C. Complicated VVC is typically associated with a non-albicans species of candida. D. Clinical and mycological cures are greater with oral as opposed to topical treatment.

VVC CDC 2010


Uncomplicated
Infrequent Mild to moderate symptoms Candida albicans Immunocompetent, nonpregnant

Complicated

Recurrent Severe symptoms Non-albicans species Women with uncontrolled diabetes, debilitation, immunosuppression, pregnancy

Predisposing Factors Associated with VVC


OCP use Sponge and IUD Tight clothing Antibiotic use Pregnancy Diabetes
Poorly controlled*

Immunosuppression* HIV*

* Best Evidence

Non-albicans Species
Prevalence of 17%
C. glabrata, C. tropicalis, C. krusei

? Increased OTC use or incomplete courses of therapy


Elimination of more sensitive albicans and selection for more azole-resistant nonalbicans species

No hyphae on wet prep


Buds present, but can be missed

Uncomplicated VVC
Cochrane Systematic Review: 2002 Clinical and mycological cures are the same oral OR topical treatment. All topical agents are highly effective.
No evidence that one formulation is superior to others

Oral fluconazole present in vagina for at least 72 hours


More side effects, but preferred by women

Recurrent VVC
Defined
Four specific episodes occurring in 12 months Or: At least three episodes unrelated to antibiotic therapy within 12 months

Recurrent VVC CDC 2010


Induction therapy Fluconazole 150 mg q 72 hours x 3 initially or 7-14 days of topical treatment Maintenance therapy Fluconazole 150 mg once weekly for 6 months Long-term cure remains elusive.

5. Which of the following statements is true regarding herpes simplex virus? A. The USPSTF recommends offering screening for HSV to asymptomatic patients. B. Antiviral therapy is not indicated for the first clinical episode of HSV. C. Latex condoms are effective in preventing transmission of HSV. D. Following a primary HSV outbreak, the typical time to recurrence is 36 months.

Genital Herpes Simplex Virus (HSV)


Most infections are subclinical or mild. HSV is a potent facilitator of HIV. Screening USPSTF 2002 Routine screening of asymptomatic persons is not recommended. Latex condoms are effective in preventing transmission.
37% of couples do not use condoms as advised.

Primary HSV
No antibodies to HSV 1 or 2 Lesions appear 2-14 days following exposure. Tender vesicles, ulcers Fever, HA, myalgias 75% asymptomatic Viral shedding for 12 days
From: Wikipedia

First Clinical Episode of Genital Herpes


Most patients should receive antiviral therapy.

Agent Acyclovir Acyclovir* Famciclovir Valacyclovir

Dose 400 mg po TID 200 mg po 5x/d 250 mg po TID 1 g po BID

Duration 7-10 days 7-10 days 7-10 days 7-10 days


* CDC 1998, 2002, 2006, 2010

Recurrent HSV
50% of patients will develop prodromal symptoms such as tingling or shooting pains into the hips or buttocks.
Usually unilateral and much smaller than primary consistent area of outbreak

90% of persons with primary HSV-2 will have at least one recurrence in the next 12 months.

Episodic Therapy for Recurrent Genital Herpes: CDC 2010


Initiate therapy within 1 day of lesion onset or during prodrome that precedes some outbreaks.
Agent
Acyclovir Acyclovir Acyclovir Famciclovir Valacyclovir

Dose
400 mg po TID 800 mg po TID 800 mg po BID 125 mg po BID 500 mg po BID

Duration
5 days 2 days 5 days 5 days 3-5 days

Valacyclovir

1 g q day

5 days

HSV: Suppressive Therapy CDC 2010


Reduces frequency of recurrences by 70%-80% (with frequent recurrences > 6/year) Reduces but does not eliminate subclinical viral shedding therefore ? prevention of transmission Agent Dose Acyclovir 400 mg po BID Famciclovir 250 mg po BID Valacyclovir* 500 mg po q day Valacyclovir 1 g q day
*Valacyclovir 500 mg q day might be less effective in patients who have very frequent recurrences (ie, > 10 episodes per year).

HSV Management in Pregnancy


If 1 or more outbreaks, oral acyclovir (no adverse fetal effects known) Suppression @ 36 wks (400 TID, Valtrex 500 qd)
Decreases risk of genital HSV at delivery (OR 0.25) Decreases need for C/S (OR 0.30)

Vaginal delivery if no active lesions or prodromal symptoms BUT C/S if active lesions present, especially if primary infection
Decreases neonatal transmission rate (1/85 or 1.2% vs 9/117 or 7.7%; OR 0.4)

6. Which of the following is the best strategy regarding an asymptomatic patient with a positive gonorrhea test and negative chlamydia test?
A. Repeat the test in 2 weeks. B. Prescribe cefixime 400 mg daily for 7 days. C. Administer ceftriaxone 125 mg IM in the office and prescribe doxycycline 100 mg twice daily for 7 days. D. Administer ceftriaxone 250 mg IM and azithromycin 1 g po.

Gonorrhea Rates by County, United States, 2010

Rate per 100,000 population

< 19.0 19.1100.0

(n = 1408) (n = 1107)

> 100.0

(n = 627)

From: CDC

Uncomplicated Neisseria Gonorrhea (any Anatomic Site)*


Ceftriaxone 250 IM plus
Azithromycin 1 g po (preferred) or doxycycline 100 mg BID
Note: 8/12 regimen is in response to an increase in multidrug-resistant gonorrhea; fluoroquinolone-resistant gonorrhea is continuing to spread and is now widespread in the US. This class of antibiotics is no longer recommended for the treatment of gonorrhea in the United States.
* This regimen is recommended for all adult and adolescent patients, regardless of travel history OR sexual behavior.

CDC: August 2012

Treatment of Gonorrhea at ANY Anatomic Site


Alternatives
If ceftriaxone is unavailable, single oral dose of cefixime 400 mg and either:
A single dose of azithromycin or Doxycycline 100 mg orally twice per day for 7 days

If the patient is allergic to cephalosporins, a single dose of azithromycin 2 g can be used.

Patients who receive either of the alternatives to ceftriaxone should return for a test of cure in 1 week. CDC advises that clinicians should perform susceptibility testing in patients who fail to respond to treatment and notify their local public health STI program.

Neisseria Gonorrhea
MMWR/July 8, 2011/Vol.60/No.26
Declining cephalosporin susceptibility
Ceftriaxone Cefixime
Not preferred as treatment because higher number of reported failures AND less efficacy against pharyngeal disease August 10, 2012 MMWR (CDC)
Developing resistance No longer recommends cefixime at ANY dose as a first-line regimen for treatment of gonococcal infections Hoping the switch may delay emergence and spread of resistance to cephalosporins

Treatment failures reported in Asia and Europe

Additional Guidance
CDC: August 10, 2012
Patients who have persistent symptoms after treatment should be retested by culture.
If culture is (+) for gonococcus, isolates should be submitted for resistance testing.

Test of cure one week after RETREATMENT. Ensure that sex partners from the preceding 60 days are promptly evaluated and treated.

Neisseria Gonorrhea Treatment in Pregnancy


Cephalosporin regimen
Women who cant tolerate cephalosporin regimen may receive 2 g spectinomycin IM. No tetracycline regimen

Azithromycin or amoxicillin for presumptive or diagnosed chlamydial infection

Gonorrhea USPSTF 2005


All high-risk sexually active women should be screened
Age < 25 Multiple sex partners No barrier contraception Incarcerated Illicit drug use

Insufficient evidence for or against routine screening in high-risk men Recommend AGAINST routine screening of low-risk men and women

7. A 21-yo had a pelvic examination and a normal Pap test 1 week ago. Her screening test for Chlamydia returned positive.
She is now being treated for chlamydia cervicitis with azithromycin, 1 g in a single dose. When should she have a test of cure for Chlamydia?

A. 1-2 weeks B. 3-4 months C. 12 months (at her next routine examination) D. No test of cure is required.

ChlamydiaRates by County, United States, 2010

Rate per 100,000 population

< 300.0 > 400.0

(n = 1962) (n = 762)

300.1400.0 (n = 418)

From: CDC

Test of Cure?
Except in pregnant women, test of cure (repeat testing 3-4 weeks after completing therapy) is not recommended when treated with the recommended or alternative regimens. Exceptions:
Therapeutic compliance is in question. Symptoms persist. Reinfection is suspected.

Pregnant women diagnosed with a chlamydial infection during the first trimester should not only receive a test to document chlamydial eradication, but be retested 3 months after treatment.

Test of Reinfection?
A high prevalence of C. trachomatis infection is observed in women who were treated for chlamydial infection in the preceding several months. Recently infected women are a major priority for repeat testing; consider advising all women with chlamydial infection to be retested approximately 3 months after treatment. Limited evidence is available on the benefit of retesting for chlamydia in men previously infected.

Chlamydia Trachomatis USPSTF 2007


Strongly recommends (Grade A)
Screening of all sexually active non-pregnant women aged < 25 years AND Sexually active non-pregnant women > 25 years with risk factors (new partner, multiple sexual partners)

Recommends against (Grade C)


Routine screening for women > 25 years (pregnant or not) if they are not at increased risk

Evidence is insufficient to assess the balance of benefits and harms of screening for chlamydial infection for men (Grade I).

CDC: High-Risk Sexual Behaviors


Previous STIs New or multiple sex partners Inconsistent condom use Engage in commercial sex work and drug use Those living in communities with a high prevalence of disease

Non-Culture Tests for Chlamydia and GC


DNA-amplification tests (gold standard now)
More sensitive than culture (detects 20% more) and 100% specific for GC and chlamydia (CDC 2007)

Appropriate specimens
Endocervical secretions or urine samples in females Urethral swab or urine in males
Gram stain of a male urethral sample for intracellular gram () diplococci is highly sensitive and specific; diagnostic in symptomatic men.

Chlamydia Treatment: CDC 2010


Recommended equally effective, no test of cure required
Doxycycline 100 mg po BID x 7 d Azithromycin 1 g po single dose

Alternative regimens
Ofloxacin 300 mg po BID x 7 d Levofloxacin 500 mg q D X 7 d Erythromycin base 500 mg QID for 7 d* Erythromycin ethylsuccinate 800 mg QID for 7 *
*Less effective; consider TOC 3 weeks after treatment.

CDC 2006 Males with Urethritis


Mucopurulent or purulent discharge or dysuria or urethral pruritus
Test now, treat now, diagnose later.

Empiric treatment recommended if:


Gram stain of urethral secretions > 5 WBC per oil immersion field Positive leukocyte esterase on first void urine or > 10 WBC per high power field High risk who are unlikely to return

8. Which of the following statements characterizes pelvic inflammatory disease the best?
A. The diagnosis of pelvic inflammatory disease is based on laboratory evaluation. B. Empiric antibiotic therapy prior to confirming PID impairs the diagnosis of other important causes of lower abdominal pain. C. Mucopurulent discharge is a necessary criteria to make the diagnosis of PID. D. Cefotetan plus doxycycline is first-line parenteral treatment of PID in the patient requiring hospitalization.

Pelvic Inflammatory Disease


Clinical diagnosis, not laboratory-based Untreated PID has significant morbidity and mortality empiric treatment is recommended if the patient meets the following minimal diagnostic criteria: Uterine/adnexal tenderness or Cervical motion tenderness, and No other cause for illness identified Empiric therapy unlikely to impair diagnosis and management of other important causes of lower abdominal pain do not delay treatment while pursuing additional evaluation if history and physical are suggestive of PID.

Other Helpful (but Not Necessary) Criteria Include:


Mucopurulent discharge Temperature > 38.3 C (101 F) WBC on wet prep ESR C-reactive protein Laboratory evidence of GC or chlamydia

Management of PID
Increased risk of ectopic and infertility
Delay > 3 days = 3x risk (observational data)

Start empiric therapy if minimal criteria present. Polymicrobial infection is likely present if the patient has symptoms.
Broad-spectrum regimen necessary

Treat sexual partner if had sex with patient during 60 days preceding onset of symptoms.

PID: Oral Treatment


Mild to Moderate Disease: CDC 2007
Regimen A
Ceftriaxone 250 mg IM in a single dose PLUS Doxycycline 100 mg orally BID x 14 days WITH OR WITHOUT metronidazole 500 mg po BID x 14 days

Regimen B
Cefoxitin 2 g IM in a single dose and probenecid 1 g po administered concurrently in a single dose PLUS Doxy and metronidazole as above

Criteria for Admission for the Treatment of PID


Uncertain diagnosis Surgical emergencies like appendicitis cannot be excluded. Suspected pelvic abscesses Concurrent pregnancy (due to high risk of maternal mortality, fetal wastage, and preterm delivery) Adolescent patient with uncertain compliance with therapy Severe illness Patient cannot tolerate outpatient regimen (eg, severe vomiting) Lack of response to 72 hours of treatment Concurrent HIV infection Clinical follow-up cannot be arranged within 72 hours.

PID
Parenteral Treatment: CDC 2007
Regimen A
Cefotetan 2 g IV q 12 hours OR cefoxitin 2 g IV q 6 hours PLUS Doxycycline 100 mg po or IV q 12 hours

Regimen B
Clindamycin 900 mg IV q 8 hours PLUS Gentamicin load followed by 1.5 mg/kg q 8 hours. (Single dosing may be substituted.)
Discontinue 24 h after clinical improvement and complete therapy with doxycycline 100 mg po BID or clindamycin 450 mg po QID x 14 days.

HIV
CDC Recommendations: 2010
Screening should be offered to ALL individuals in healthcare settings (must be voluntary).
Estimated 1.1 million people in US with HIV, and 25% are undiagnosed; only 36.6% of adults have had an HIV test. (+) Enzyme immunoassay (EIA) must be confirmed by Western blot or immunofluorescence assay. HIV-1/2 antibody detectable in 95% of patients within 3 months after infection Prevention strategies should be discussed at the time of diagnosis. Partner notification should be encouraged.
Branson BM, Handsfield HH, Lampe MA, et al. Revised recommendations for HIV testing of adults, adolescents, and pregnant women in health-care settings. MMWR Recomm Rep. 2006;55(RR-14):1-17.

HIV Screening
USPSTF: 30 April 2013
Screen all patients aged 15-65 for HIV, regardless of risk level (Level A).
Substantially decrease the HIV disease burden across the country. Dramatically reduce transmission of the virus. Markedly curtail infected patients progression to AIDS and death.

2005 8 years later, new, convincing evidence shows "with high certainty" that identifying and treating HIV infection in asymptomatic people will substantially benefit public health as well as the health of those individual patients. Insufficient evidence to establish optimal interval for screening
Reasonable to rescreen at-risk people at 1-year intervals:
Engage in high-risk behaviors. Live in or receive medical care in high prevalence settings, eg, correctional facilities, homeless shelters, TB clinics, STI clinics, and clinics that serve men who have sex with men.

Update 2013
Three Key Research Findings
20% of people infected with HIV are not aware they are infected.
236,400 people in US do not know that they should take precautions against transmitting the virus and begin treatment to limit HIV-related illness and end-organ damage.

Proven that initiation of antiretroviral therapy before carriers become symptomatic (CD4 counts are between 0.200 and 0.500 x 109 cells/L) markedly reduces progression to AIDS and death Overall harms of screening the general population and treating those who are found to be HIV-positive are now considered small.
Both conventional and rapid screening tests are highly accurate, with sensitivities and specificities topping 99.5% so the potential harms of falsenegative and false-positive results are minimal. Treatment is highly effective, with benefits clearly outweighing adverse effects.

HIV Negative
Doesnt require direct personal contact Reinforce HIV prevention/risk reduction Give concrete recommendations about when to re-test.

HIV Positive
Must give results confidentially in person! Brief HIV disease education Psychosocial support services Communicable disease reporting: All new diagnoses must be reported to local/state health authorities. Partner notification (screen for DV) Linkage to care; confirm contact information and insurance status!

HIV References
Carolyn Chu, MD, MSc, and Peter A Selwyn, MD, MPH. Complications of HIV Infection: A Systems-Based Approach. Am Fam Physician. 2011;83(4):395-406. http://www.aafp.org/afp/2011/0215/p395.pdf Carolyn Chu, MD, MSc, and Peter A Selwyn, MD, MPH. Diagnosis and Management of Acute HIV Infection. Am Fam Physician. 2010;81(10):1239-1244 http://www.aafp.org/afp/2010/0515/p1239.pdf

9. All of the following infections cause genital ulcers EXCEPT:


A. B. C. D. Human papillomavirus Treponema pallidum Haemophilus ducreyi Herpes simplex virus

Genital Ulcers
Think herpes (HSV), syphilis, chancroid Specific tests
Serology for T. pallidum Culture or antigen for HSV Culture for H. ducreyi HIV testing

Biopsy ulcers that do not respond to initial therapy

Primary and Secondary Syphilis Rates by County, United States, 2010

Rate per 100,000 population

< 0.2 0.212.2 > 2.2

(n = 2,167) (n = 383) (n = 592)

NOTE: In 2010, 2167 (69.0%) of 3141 counties in the United States reported no cases of primary and secondary syphilis.

From: CDC

Diagnosis of Syphilis
Definitive test
Dark-field microscopy
CDC 2010 still recommends nontreponemal followed by treponemal for screening/diagnosis.

Nontreponemal serology
RPR and VDRL (screening) Correlates with disease activity (4-fold decline in titer by 6 months); rarely (+) for life

Treponemal antibody
FTA-ABS (confirmation test)
Correlates poorly with disease activity; not used to assess treatment response; may remain (+)

VDRL-CSF for neurosyphilis


Highly specific, low sensitivity

USPSTF 2004 Syphilis


Strongly recommends that persons at increased risk for syphilis be screened
Sex workers IVDA HIV+ Other STIs Known partner with active syphilis

Increase in MSM (high rates of co-infection with HIV)

Recommends AGAINST screening persons NOT at increased risk

Syphilis Treatment: CDC 2010


Parenteral PCN G
2.4 million units IM Preferred drug for treatment of ALL stages EXCEPT late latent/tertiary neurosyphilis

Desensitization if PCN allergy


Documented limited data on alternatives doxycycline/tetracycline advise 14-day course Emergence of azithromycin-resistant T. pallidum should not be routinely used to treat

Jarisch-Herxheimer
Acute febrile reaction occurring in first 24 hrs after treatment

Syphilis Treatment: CDC 2010


Late latent/tertiary
Benzathine PCN G
7.2 million units administered as 3 doses of 2.4 million units IM q week

Neurosyphilis
Aqueous crystalline PCN G
18-24 million units/day, administered as 3-4 million units IV q 4 hours or continuous infusion for 10-14 days Procaine penicillin
2.4 million units IM once daily plus probenecid 500 mg po QID for 10-14 days

Chancroid
Painful genital ulcers, painful inguinal lymphadenopathy 10% co-infected with HIV/syphilis H. ducreyi
Co-factor for HIV transmission (more Tx failures if HIV [+]) Check HIV, VDRL

Primary treatment
Azithromycin Ceftriaxone Ciprofloxacin Erythromycin
From: CDC

Granuloma Inguinale
Painless, ulcerative lesions
No lymphadenopathy Highly vascular lesions

Klebsiella granulomatis
Donovan bodies on biopsy

Primary treatment
Doxycycline

Alternatives
Ciprofloxacin Erythromycin Azithromycin Trimethoprim-sulfa

From: CDC

Lymphogranuloma Venereum
Rare in USA Chlamydia trachomatis Painful lymphadenopathy Primary treatment
Doxycycline

Alternative
Erythromycin
From: CDC

10. In considering hepatitis B, which of the following statements is true?


A. The USPSTF recommends routinely screening the general asymptomatic population for chronic hepatitis B virus infection. B. The USPSTF does not recommend screening for HBV infection in pregnant women at their first prenatal visit. C. Hepatitis B vaccine is no longer part of the routine immunizations in the United States. D. Newborns of mothers infected with HBV should be given hepatitis B immune globulin after delivery and start on the hepatitis B vaccine series.

Screening
The USPSTF strongly recommends screening for HBV infection in pregnant women at their first prenatal visit. Grade: A Recommendation The USPSTF recommends against routinely screening the general asymptomatic population for chronic hepatitis B virus infection. Grade: D Recommendation
Screening for Hepatitis B Infection, Topic Page. February 2004. US Preventive Services Task Force.

Prevention
Hepatitis B vaccine is part of the routine immunizations in the United States. A Cochrane review confirmed that hepatitis B vaccination decreased HBV infection in healthcare professionals.

Hepatology. 2009;49(5 suppl):S4-S12. Cochrane Database Syst Rev. 2005;(4):CD000100.

Prevention
Healthcare professionals exposed to HBsAg-positive patients
Give hepatitis B immune globulin after the exposure and start on the hepatitis B vaccine series if not previously vaccinated.

Newborns of mothers infected with HBV


Give hepatitis B immune globulin after delivery and start on the hepatitis B vaccine MMWR Recomm Rep. 2006;55(RR-16):1-33. series. Cochrane Database Syst Rev. 2006;(2):CD004790.

Hepatitis C
In the US, estimated 4.1 million persons have been infected with hepatitis C virus (HCV), of whom an estimated 3.2 (95% confidence interval [CI] = 2.73.9) million are living with the infection. New infections continue to be reported, particularly among persons who inject drugs and persons exposed to HCV-contaminated blood in healthcare settings with inadequate infection control.

Hepatitis C Screening
CDC 1998, 2003

Recommended HCV testing for persons with risks for HCV infection

Hepatitis C Screening
CDC: 10 November 2011
Proposal: Recommend one-time screening of all baby boomers (1945-1965).
Baby-boomers account for about three-quarters of those who die with HCV infection (liver cancer or fibrosis). Estimate 80 million baby boomers would be screened and about 3.2 million infections diagnosed.

Efforts coincide with May 2011 approval of two new protease inhibitors for HCV:
Boceprevir (Victrelis, Merck & Co.) Telaprevir (Incivek, Vertex Pharmaceuticals)

CDC 2013 and HCV


May 7, 2013, MMWR. Early Release on the MMWR website (http://www.cdc.gov/mmwr)

Update in guidance
Changes in availability of certain commercial HCV antibody tests Evidence that many persons identified as reactive by an HCV antibody test might not subsequently be evaluated to determine if current HCV infection Significant advances in the development of antiviral agents

Previous guidance has focused on strategies to detect and confirm HCV antibody.
Reactive results from HCV antibody testing cannot distinguish between persons whose past HCV infection has resolved and those who are currently HCV infected. Persons with current infection who are not identified as currently infected will not receive appropriate preventive services, clinical evaluation, and medical treatment.

Testing strategies must ensure the identification of those persons with current HCV infection.

Recommended Testing Sequence for Identifying Current Hepatitis C Virus (HCV) Infection: CDC 2013

May 7, 2013, MMWR. Early Release on the MMWR website (http://www.cdc.gov/mmwr)

USPSTF 2013
June 21, 2013

The USPSTF recommends screening for hepatitis C virus (HCV) infection in persons at high risk for infection. Grade: B Recommendation The USPSTF also recommends offering one-time screening for HCV infection to adults born between 1945 and 1965. Grade: B Recommendation

HPV/Genital Warts Condyloma acuminata


Caused by various types of HPV (6 and 11 most commonly 90%) Average time to development of new anogenital warts after infection is ~ 2-3 months. Young sexually active people
Range of manifestations
Small, unnoticed, regress without therapeutic intervention (immunocompetent) Multiply, expand in size, produce symptoms
Itching, irritation, bleeding Mass effect Interfere with hygiene, function, sexual activity

Range of emotions
Psychological sequelae typical concerns about STI

Summary: CDC 2010


Location
Anogenital (keratinized epithelium)

Possible Treatments
Podophyllotoxin, cryotherapy DCA/TCA, surgical removal Interferon alpha Imiquimod Sinecatechins DCA/TCA (can be repeated weekly, if necessary) Cryotherapy with liquid nitrogen; avoid cryo-probe Laser vaporization Exclude HSIL, cryotherapy Cryotherapy with liquid nitrogen DCA/TCA Surgical removal

Vagina (mucosal epithelium) Cervix Anal

HPV in Pregnancy
Treatment Options
Acceptable Therapies
Trichloroacetic acid Cryotherapy Electrosurgical excision Laser ablation

Unacceptable Therapies
Podophyllin Podofilox solution and gel 5-Fluorouracil Imiquimod

Typically NOT used in pregnancy: Interferon (intralesional injection Class C)

Cervical Cancer
Cervical Cancer Is an STI Caused by HPV!
Worldwide the fourth leading cause of cancer deaths in women
Developing countries second most common cause of cancer deaths in women

Screening programs have dramatically reduced the rate in screened population.


70% reduction in the US over the past five decades

Condoms appear to be an effective barrier against HPV transmission.


HPV infections are primarily transmitted through contact with infected skin or mucosal surfaces.

So Who Is Getting Cervical Cancer?


Median age is 45-50 years.
Older women from lack of screening Younger women from rapidly progressing disease

Profile of woman with invasive cancer:


50% have never had a pap smear. Another 10% have not been screened within 5 years prior to their diagnosis.
Cervical Cancer. NIH Consensus Statement. 1996 Apr 13;14:1-38.

11. Which of the following is a risk factor for cervical cancer?


A. B. C. D. Exogenous hormones Family history Smoking Pregnancy

Assessing the Patients Risk for Cervical Cancer


Risk Factors
Onset of sexual activity before age 20 Multiple sexual partners Low SES

NON-Risk Factors
Family history Menstrual history Exogenous hormones Pregnancy

Co-factors
Other STIs HIV infection Smoking* Nutritional deficiency

*Cigarette smoking is the only non-sexual behavior consistently and strongly correlated with cervical dysplasia and cancer, independently increasing the risk two- to four-fold.

References

Summary Chart of 2006 CDC Treatment Guidelines for STDs. Pharmacists Letter/Prescribers Letter. 2006;22(9):220912. Updated recommended treatment regimens for gonoccoccal infections and associated conditions United States, April 2007. http://www.cdc.gov/STD/treatment/ Meyers D, Wolff T, Gregory K, Marion L, Moyer V, Nelson H, Petitti D, Sawaya GF. USPSTF Recommendations for STI Screening. Am Fam Physician. 2008;77(6):819824. Screening for Chlamydial Infection. Topic Page. June 2007. US Preventive Services Task Force. Agency for Healthcare Research and Quality, Rockville, MD. http://www.ahrq.gov/clinic/uspstf/uspschlm.htm Screening for Gonorrhea. Topic Page. May 2005. US Preventive Services Task Force. Agency for Healthcare Research and Quality, Rockville, MD. http://www.uspreventiveservicestaskforce.org/uspstf/uspsgono.htm Screening for Genital Herpes. Topic Page. March 2005. US Preventive Services Task Force. Agency for Healthcare Research and Quality, Rockville, MD. http://www.uspreventiveservicestaskforce.org/uspstf/uspsherp.htm CDC Sexually Transmitted Diseases Treatment Guidelines 2010. http://www.cdc.gov/std/treatment/2010/STD-Treatment-2010-RR5912.pdf

Answers
1. B 2. C 3. B 4. C 5. C 6. D 7. D 8. D 9. A 10. D 11. C

Preoperative Examination and Surgical Management


Belinda Vail, MD, FAAFP Department of Family Medicine University of Kansas, School of Medicine Kansas City, Kansas

Disclosure Statement
Dr. Vail has nothing to disclose.

It is the policy of the AAFP that all individuals in a position to control content disclose any relationships with commercial interests upon nomination/invitation of participation. Disclosure documents are reviewed for potential conflicts of interest. If conflicts are identified, they are resolved prior to confirmation of participation. Only participants who have no conflict of interest or who agree to an identified resolution process prior to their participation were involved in this CME activity.

Learning Objectives
1. Screen patients for risk factors before they undergo surgical procedures and report subsequent risks to the patients surgical care team. 2. Evaluate patients for preoperative cardiac risk based on the risk and complexity of the surgery and the functional status of the patient. 3. Understand the perioperative management of diabetes.

Preoperative Evaluation Goal


To identify and manage medical conditions that may impact perioperative morbidity and mortality Accurate communication to the patient and the surgical team Medical optimization (rather than clearance)

Preoperative Evaluation
Thorough medical history Assessment of perioperative risk factors Physical exam
BMI, BP, O2 sat, cardiac, pulmonary

Ancillary tests Interventions recommended to mitigate risks Social support


Care giver, home health, rehab?

Issues Pertinent to Surgery


Current medical problems Cardiac status Pulmonary status Functional status ( 4 METs) Hemostasis status (Hx of abnormal bleeding) Possibility for symptomatic anemia Possibility of pregnancy History of anesthesia problems Smoking

Preoperative Evaluation
1. Assess the risk/complexity of the surgery 2. Assess the functional status of the patient 3. Decide on cardiac evaluation 4. Consider other testing

1. A 72 y/o patient of yours with


diabetes and hypertension is having cataract surgery and she comes to you for a pre-op evaluation. What will you order?
A. ECG B. CBC C. BUN/creatinine and electrolytes D. Thallium stress test E. Nothing

Highest risk procedures (> 5% risk of MI) Aortic and peripheral vascular surgery* Emergent major operations (especially if patient > 75 years of age) Intermediate risk (1-5% risk) Head and neck surgery (carotid endarterectomy) Intraperitoneal or intrathoracic surgery Orthopedic Prostate Low risk (< 1% risk)* Endoscopic procedures Cataract surgery* Breast surgery

Evaluating for Risk of Perioperative MI

2. You have an 87 y/o patient who stays


active in his steeply sloped yard (raking, mowing, planting). What could be said about his functional capacity. A. His METS are less than 4. B. He would require an ECG prior to a laparoscopic hernia repair. C. His METS are greater than 4. D. He would not qualify for hernia surgery

Functional Status of the Patient


Poor functional status
< 4 METS (metabolic equivalents assessment of energy expenditure)
Can do light housework (dusting, dishwashing) Can walk a block or two on level ground Cannot climb a flight of stairs, walk up a hill, or run

Should have noninvasive testing for moderate or high-risk surgery


ECG and exercise or pharmacologic stress testing

Functional Status
Examples of activities requiring > 4 METS and indicating good functional status:
Golf Bowling Dancing Doubles tennis Heavy work around the house

Good functional statusnon-invasive testing only for high-risk surgeries

Cardiac Evaluation: ECG


1. Indicated for high-risk patients:
Active cerebro/cardiovascular signs or symptoms (CVA/TIA, CHF, ischemic cardiac disease) Renal insufficiency (Cr > 2.0 mg/dL) Diabetes requiring insulin

2. Indicated for high-risk procedure 3. Indicated for intermediate-risk procedure if patient has poor functional capacity 4. Not indicated for intermediate-risk procedures* if patient has good functional capacity 5. Not indicated for low-risk procedures

Cardiac Evaluation
Noninvasive testing first* - any of the following:
ECG Thallium stress test Exercise testing if patient is capable

Invasive testing for positive results


Cardiac catheterization

3. Which of the following is true regarding coagulant management of a 65 y/o with hypertension and hyperlipidemia prior to a right colectomy?
A. Order an INR and platelets B. Stop his baby aspirin a week prior to surgery C. Order a bleeding time D. Obtain a bleeding history and continue aspirin

Aspirin
Safe to continue low-dose aspirin, as it reduces CV complications Increases bleeding risk by 20% Continue if history of MI or coronary stents
Discontinuation results in a 3-fold increase in the risk of adverse cardiac events Cessation results in a 90-fold increase in complications in patients with coronary stents

No difference in severity of bleeding events (except intracranial and maybe prostatectomy) Discuss with surgeon

Beta-Blockers
Continue beta-blockers before, during, and after surgery
If they have been used for at least 4 weeks prior to surgery (SOR: A) Used for known ischemic HD undergoing vascular surgery Reduce cardiac oxygen demand Reduce risk of nonfatal MI and cardiac death

If beta-blockers have not been used for at least 1 week, initiation may be harmful (SOR: B)

Revised Cardiac Risk Index (RCRI) or Goldman Classification


1 point each for:
History of CVA/TIA CHF Renal insufficiency (Cr > 2.0 mg/dL) Diabetes requiring insulin Ischemic cardiac disease Suprainguinal vascular, intrathoracic, intraabdominal surgical site RCRI = 0 NNT with beta-blocker = 2349, NNH = 208 RCRI 4 NNT with beta-blocker = 33 Use beta-blockers only if RCRI is 2

Statins
Statin benefits
Lipid-lowering Reduce vascular inflammation Improve endothelial function Stabilize atherosclerotic plaques (reduce 30-d MI & death) Statin therapy (lovastatin and fluvastatin longer acting) Even beneficial within days of procedure Ideally started several weeks prior to surgery Without regard to lipid levels Decreases significantly the CV risk for vascular surgery Risk of CV events sharply increases if stopped

Coronary Artery Revascularization Prophylaxis


Accomplished by PCI or CABG CARP Trial (follow up 2.7 years)
No difference in all-cause mortality No difference in incidence of postoperative MI

DECREASE-V
Preoperative revascularizationno benefit

Surgery during the dual antiplatelet therapy period substantially increases risk
Clopidogrel (Plavix) and aspirin Bare 6 wk; Drug-eluting 12 mo

4. Which of the following


procedures is an indication for a pre-operative chest x-ray?
A. Hernia repair B. Emergency surgery for a ruptured diverticuli C. Cataract surgery D. Patient > age 50

Chest X-ray
No outcomes evidence for routine CXR Indications for CXR
New or unstable cardiopulmonary signs or symptoms

Risk factors for pulmonary complications:


COPD Age > 60 years Functional dependence Hypoalbuminemia CHF Emergency or prolonged procedure Certain surgical sites (head, neck, upper abdomen)

Routine Laboratory Tests


Urinalysis: only for implantation of foreign material (hip replacement, heart valve) or urologic procedures

Electrolyte and creatinine testing: PMH of HTN, CHF, CKD, complicated DM, liver disease Medications: diuretics, ACE-I/ARB, NSAIDs, digoxin A1c: indicated only for patients at very high risk or signs and symptoms of undiagnosed diabetes CBC: at risk for anemia (chronic kidney or liver disease, or inflammatory diseases) or excessive blood loss Coagulation tests: based on bleeding history or if taking anticoagulants.

Laboratory Testing
Pregnancy testing in patients of childbearing age
Sexually active and delayed menses Concerned about pregnancy Possibility of pregnancy is uncertain

> $30 billion annually on preop exams


60% are unnecessary 30-95% of unexpected lab abnormalities not addressed

5. Your 57 y/o patient has rheumatoid


arthritis and is having a hip replacement surgery. Which x-ray should you order?

A. The contralateral hip B. Chest x-ray C. Cervical spine film D. KUB E. Bone density

Rheumatoid Arthritis
Patients with rheumatoid arthritis require C-spine imaging for atlantoaxial subluxation prior to intubations*
Prevent spinal cord injury during intubation May require cervical fusion prior to surgery

Perioperative Management

Medication Management
Medications to stop:
Clopidogrel/ticlopidine: 5-7 days prior NSAIDs: 1-3 days prior COX-2 agents: 2-3 days prior Dabigatran (Pradaxa): 2-5 days prior Rivaroxaban (Xarelto): at least 24 hours

Medications to give:
Parenteral antibiotics: 30 min prior Long-acting insulins: Morning of surgery Steroids: usual daily dose

Warfarin Management
Low thromboembolic risk
A-fib with no CVA or embolism in past 12 months Biological heart valves > 3 months out Vascular grafts DVT > 3 months outno hypercoagulable No current systemic arterial embolism

Management
Stop 5 days pre-op Restart post-op when taking PO

Warfarin Management
High thromboembolic risk
Mechanical heart valve DVT/PE with hypercoagulable state History of DVT/PE < 3 months ago

Management
Stop 4 days pre-op and start LMWH Stop LMWH 12-18 hours pre-op Restart LMWH 6 hours post-op Restart warfarin when taking PO Stop LMWH when INR = 2.0

Herbal Medications
70% of patients fail to disclose use of herbal medicines 8 most commonly used
Echinacea, ephedra, garlic, ginkgo, ginseng, kava, St. Johns wort, valerian Alteration of the actions of absorption, distribution, metabolism, and elimination of conventional drugs

Surgery and Coronary Stents


Bare stents
Surgery should be avoided for at least 4 wks

Drug-eluting
Surgery should be avoided for 1 yr

If surgery is unavoidable
Dual anti-platelet Rx (clopidogrel and aspirin) should be continued perioperatively unless strongly contraindicated. Procedures associated with high risk for clinically significant bleeding (intracranial or prostatic surgery)

Perioperative Issues
Majority of perioperative problems:
Cardiac Pulmonary Renal Infectious Hematologic

Assess for Risks of Delayed Healing and Infection


Risks for surgical-site infection
Smoking Diabetes Obesity Malnutrition Chronic skin disease

Perioperative Management
Diabetes:
Increased risk of infection Increased post-op cardiovascular morbidity and mortality Poor preoperative control leads to poor outcomes, so control should be addressed prior to surgery Continue usual diabetes regimen* and minimize fasting

Instructions
STOP SMOKING
Its so important it gets its own slide Some surgeons will not do elective surgery if the patient smokes

Instructions: Fasting
2-4-6-8 hour rule
2 hours for clear liquids (water, pulp-free fruit juice, carbonated beverages, clear tea, and coffee) 4 hours of breast milk 6 hours for non-human milk and light meals such as toast 8 hours for regular meals; fried, fatty foods; meat

6. Which of the following is the most important risk factor for postoperative pulmonary complications?
A. COPD B. General anesthesia C. Peripheral vascular procedure D. Diabetes E. Obesity

Postoperative Complications
Risk factors for pulmonary complications
Advanced age Functional dependence COPD Heart failure Serum albumin < 30 g/L High-risk surgery (vascular, emergent, AAA, prolonged, neurosurgery, abdominal)

Pulmonary Complications
Common pulmonary complications
Atelectasis Pneumonia Respiratory failure Bronchospasm Exacerbation of underlying disease

Prevention
Incentive spirometer Chest physiotherapy Preoperative corticosteroids for COPD, etc

Pulmonary Risks
Procedure-related risk factors are more predictive of pulmonary complications than patient-related factors
Greatest risk is how close surgery is to the diaphragm (thoracic) Surgery > 3 hours significantly increases risk Pre-op O2 sat 91%

Need to quit smoking 8 weeks prior to surgery

Obstructive Sleep Apnea


All patients should be screened for OSA (SOR C) Patients with OSA who have an oral appliance or CPAP equipment should bring these with them on the day of the surgery (SOR A)
Should be in pre-op recommendations

Renal Status
Patients with CRF are at increased risk
Surgery well tolerated if GFR > 25 mL/min GFR 10-15 mL/min complications rise 55-60% Consider preoperative dialysis

Postoperative acute kidney injury (AKI) has a 58% mortality rate (develops in 1% of surgical patients)

Minimizing Perioperative Risk


Ensure preoperative euvolemia and normal osmolar status Minimize exposure to nephrotoxins Avoid perioperative hypotension Consider preoperative dialysis if GFR < 15 mL/min

Infection
Pneumonia is most prominent remote infection
Prevent with early ambulation, incentive spirometry, tight glycemic control

Surgical site infections = 37% of post-op infections


Prevent with tight glucose control Treat preexisting infections Provide nutritional supplementation 7-14 days preoperatively Smoking cessation

Preventing MRSA
8% of nosocomial infections Universal frequent hand washing and room cleaning Use of good isolation techniques

Thromboembolism Prophylaxis
Low risk: early mobilization Medium risk: intermittent pneumatic device or graduated compression stockings, low molecular weight heparin (LMWH) -- or fondaparinux [Arixtra] or warfarin High risk: LMWH (or as above), graduated compression stockings and intermittent pneumatic device

Pediatrics
Consider delaying surgery in a pediatric patient with a URI if using general anesthesia and 1 or more of the following are present
Asthma History of prematurity Copious secretions A parent who smokes Planned use of an endotracheal tube Procedure involving the airway

Joint Management
Consider agreement with surgeon to order standard tests depending on the surgery to be performed
Eg, ultrasound for cholecystectomy

Local Anesthesia
Decrease pain from infiltration of local anesthetics:
Warm solution Small needles Sow infiltration Sodium bicarbonate Inject through edge of wound Pretreat with topical anesthetics.

References
Dhatariya K, et al. NHS diabetes guideline for the perioperative management of the patient with diabetes. Diabet Med. 2012 Apr;29(4):420-33. Feely MA, et.al. Preoperative testing before noncardiac surgery: guidelines and recommendations. Am Fam Physician. 2013;87(8):414-8. Holt NF. Perioperative cardiac risk reduction. Am Fam Physician. 2012;85(3):239-46. Lindsley KL, et.al. Routine preoperative medical testing for cataract surgery. Cochrane Database Syst Rev. 2012:(3):CD007293

Answers
1. 2. 3. 4. 5. 6. E C D B C A

Common ENT Problems


William J. Geiger, M.D. Associate Program Director Grant Family Medicine Residency OhioHealth Columbus, Ohio

Disclosure
Dr. Geiger has nothing to disclose

The AAFP has selected all faculty appearing in this program. It is the policy of the AAFP that all CME planning committees, faculty, authors, editors, and staff disclose relationships with commercial entities upon nomination or invitation of participation. Disclosure documents are reviewed for potential conflicts of interest and, if identified, they are resolved prior to confirmation of participation. Only those participants who had no conflict of interest or who agreed to an identified resolution process prior to their participation were involved in this CME activity.

Learning Objectives
Recognize the diagnosis and management of acute and chronic otitis media, acute and chronic sinusitis, and vertigo. Identify ancillary tests including tympanometry and imaging studies in the appropriate clinical situation.

Question #1
A 12 month old male patient of yours is brought into your office by his mother for fever, cough and pulling on his left ear. He is afebrile in your office, playful and interactive. When you examine his ear, the tympanic membranes are slightly erythematous, but he is uncooperative for pneumatic otoscopy. Your diagnosis is:

A. B. C. D.

Acute Otitis Media Otitis media with effusion URI, Acute Otitis Media uncertain Ramsay-Hunt syndrome

Diagnosis of Acute Otitis Media


Must use stringent criteria
Assure appropriate treatment Avoid overuse of antibiotics

Three required elements to make the diagnosis


Acute onset of symptoms of otalgia Presence of a middle ear effusion Acute signs of middle ear inflammation
http://pediatrics.aappublications.org/content/early/2013/02/20/peds.2012-3488

SORT B

Symptoms of Acute Otitis Media


Onset within 48 hours Severe symptoms
Moderate to severe pain Fever >39oC (102.2oF) Non-verbal child (<6 months old)
Holding, tugging, or rubbing of ear Poor eating, irritability
http://pediatrics.aappublications.org/content/early/2013/02/20/peds.2012-3488 SORT B

Diagnosis of Middle Ear Effusion


Decreased or absent mobility of TM
Do not diagnose without demonstrating MEE Pneumatic otoscopy standard of care Must have an air seal in canal and no air leaks in the system
Squeezing the bulb -> positive pressure Releasing the bulb -> negative pressure

Tympanometry
http://pediatrics.aappublications.org/content/early/2013/02/20/p eds.2012-3488 SORT B

Diagnosis of Middle Ear Inflammation


Bulging and fullness of the TM Bullae on the TM Acute purulent otorrhea (not from otitis externa) Marked erythemaBUT
An erythematous TM in the absence of bulging or immobility of the TM has only a 15% PPV for AOM An erythematous TM can be caused by fever, crying, and URI

Otitis Media with Effusion (OME)


Signs of middle ear fluid
Impaired TM mobility Air fluid levels Bubbles Amber or blue color

Absence of signs of acute inflammation

AOM vs OME
Bulging TM
TM red or yellow Pus, otorrhea, or bullae

Retracted or neutral TM TM amber or blue


Air fluid levels or bubbles

AOM vs OME

(c) B. Welleschik

Tympanometry
Quantifies pneumatic otoscopy Should be used with pneumatic otoscopy and is considered optional Measures:
Ear canal volume (cm3) Compliance (cm3) Pressures

Tympanometry
Type A curve
Normal compliance and pressures

Tympanometry
Type B curve
Decreased compliance MEE Stiff TM from scarring Tympanosclerosis Cholesteatoma tumor

Tympanometry
Type C curve
Negative pressures Retracted TM Eustachian tube dysfunction

Question #2
A 22 month old female patient is brought to you crying and in obvious acute distress from right ear pain. She has a fever of 103.6oF, has an immobile, bulging, erythematous right TM. The best treatment option would be:

A. Start antibiotics immediately B. Do not use antibiotics as this is probably a viral illness C. Have the parents observe for 24-48 hours and treat with antibiotics if the child does not improve D. Treat with anesthetic ear drops alone

Treatment of AOM
Pain control
Use ibuprofen or acetaminophen Anesthetic ear drops

Decongestants/Antihistamines
Not proven to help Increased side effects May relieve nasal congestion

Treatment of AOM
Three bacterial pathogens
S. pneumoniae H. influenzae M. catarralis

Antibiotics or Observation?
First studies out of Europe Concerns about over usage of antibiotics Several meta-analyses suggest most children do well without ATBs
61% resolve symptoms within 24 hours

Treatment of AOM
2013 AAP/AAFP Guideline

<6 months old


All get ATBs if AOM suspected

6 23 months
ATBs if diagnosis of AOM (unilateral or bilateral) and symptoms are severe ATBs if bilateral AOM and symptoms are non-severe Observation is an option if unilateral AOM and illness non-severe
http://pediatrics.aappublications.org/content/early/2013/02/20/peds.2012-3488 SORT A

Treatment of AOM
2013 AAP/AAFP Guideline

>24 months old


ATBs if child has severe symptoms Observation an option for AOM and non-severe symptoms

http://pediatrics.aappublications.org/content/early/2013/02/20/peds.2012-3488 SORT A

Observation of AOM
2013 AAP/AAFP Guideline

Observation based on shared decision-making with parents Is only appropriate if follow-up can be ensured and ATBs started if symptoms worsen or persist
Follow-up visit at 48-72 hours Phone call at 48-72 hours Prescription that can be filled if child not improved at 48-72 hours

Question #3
The same 22 month old female patient weighs 17kg (35LB.). What is the most appropriate antibiotic (assuming no drug allergies) and total daily dosage for this child?

A. B. C. D.

Amoxicillin-clavulanate 1200 mg/day Amoxicillin 750 mg/day Amoxicillin-clavulanate 750 mg/day Amoxicillin 1500 mg/day

Antibiotics for AOM


2013 AAP/AAFP Guideline

Amoxicillin 80-90 mg/Kg per day Amoxicillin-clavulanate for those


Treated with ATBs in last 30 days With concurrent conjunctivitis (H. influenzae) Taking prophylactic amoxicillin for recurrent AOM

Antibiotics for AOM


2013 AAP/AAFP Guideline

Penicillin allergy - No urticaria or anaphylaxis (Non-type 1) Cephalosporins


Cefdinir (Omnicef) 14 mg/Kg per day Cefpodoxime (Vantin) 10 mg/Kg per day Cefuroxime (Ceftin) 30 mg/Kg per day Ceftriaxone (Rocephin) 50 mg/Kg IM/IV

Antibiotics for AOM


2013 AAP/AAFP Guideline

Penicillin allergy with urticaria or anaphylaxis (Type 1)


Macrolides
Erythromycin + sulfisoxazole Azithromycin (30 mg/Kg single dose) Clarithromycin

Clindamycin

Not recommended due to resistance


Trimethoprim-sulfamethoxazole Levofloxacin

Antibiotics for AOM


2013 AAP/AAFP Guideline

Duration of treatment 10 day course of antibiotics


Children >6 years old may be treated for 5-7 days Poor quality studies on shorter courses of antibiotics in AOM

Question #4
Four weeks after antibiotic treatment, you see the child in follow-up. The child is asymptomatic but you determine she has a middle ear effusion. Your recommendation would be:

A. B. C. D. E.

Oral low dose steroids for 30 days Retreat with Amoxicillin-clavulanate Reassurance and reevaluate in 2 months ENT referral Oral antihistamine for 30 days

Follow-up for AOM


Monitor middle ear effusion (MEE) Does not mean treatment failure
70% had MEE after two weeks 40% after one month 20% after two months 10% after three months

Follow up recommended at 8-12 weeks Monitor for hearing, language and learning problems refer for ventilation tubes
http://www.uptodate.com/online/content/topic.do?topicKey=pedi_id/10593&sele ctedTitle=1~150&source=search_result SORT C

Persistent AOM
No improvement in 48-72 hours Usually caused by Penicillin resistant S. pneumoniae Must be reassessed to confirm diagnosis Switch to second line ATB assume resistant bacteria

Recurrent AOM
Antibiotic prophylaxis
NOT recommended Still widely used

Minimize risk factors


Exposure to cigarette smoke Pacifier, bottle feeding Daycare attendance

Treatment of Otitis Media with Effusion


Watchful waiting for three months Monitor for hearing loss at three months Tympanostomy tubes is preferred procedure Do not use antibiotics, antihistamines, or decongestants 2011 Cochrane Review suggests oral or nasal steroids with or without antibiotics hastens resolution of OME. (12 medium to high quality studies)
http://www.aafp.org/afp/2004/0615/p2929.html http://www.aafp.org/afp/2006/0915/p956.html http://onlinelibrary.wiley.com/doi/10.1002/14651858.CD001935.pub3/abstract

SORT B

Question #5
A 43 year old male presents to your office with five days of nasal congestion and headache. His temperature is 100.8oF, and he has purulent rhinorrhea, and minimal tenderness to palpation over the frontal and maxillary sinuses. Your recommendation would be:

A. B. C. D.

Get sinus x-rays Treat with Amoxicillin-clavulanate Treat with decongestants/mucolytics Transilluminate the sinuses

Rhinosinusitis
Classification
Acute - < 4 weeks Sub-acute 4-12 weeks Chronic - > 12 weeks Recurrent 4 or more episodes per year without symptoms between episodes

Acute Rhinosinusitis
Symptoms
Purulent nasal drainage Nasal Obstruction Facial pain, pressure or fullness

Etiology
Viral is most common etiology Bacterial rhinosinusitis 0.5-2%

Indicators of Bacterial Rhinosinusitis


Duration of seven or more days Worsening of symptoms Moderate to severe pain and fever >101oF Bimodal illness - worsening of symptoms after initial improvement
http://www.aafp.org/afp/2007/1201/p1718.html SORT C

Non-helpful Tests
Transillumination of sinuses Viral or bacterial cultures (except endoscopic) Sinus x-rays Sinus CT scans MRI or Ultrasound
http://www.aafp.org/afp/2007/1201/p1718.html SORT C

Treatment of Viral Rhinosinusitis


Self-limited disease treatment does not shorten the course Analgesics (NSAIDs, Acetaminophen) Saline nasal sprays (irrigation) Topical nasal steroids Topical decongestants Topical ipratropium (Atrovent) Mucolytics (guaifenesin) Anti-histamines may over dry and increase discomfort Zinc preparations show no benefit

Antibiotic Treatment of Bacterial Rhinosinusitis


To treat, or not to treat??? Observation OR antibiotics acceptable for patients with mild symptoms for 10 or more days High rate of spontaneous resolution
70% after seven days with or without ATBs

Observation should include supportive treatment for seven days


http://www.uptodate.com/online/content/topic.do?topicKey=pc_id/5943&selected Title=1~34&source=search_result SORT C

Antibiotics for Bacterial Rhinosinusitis


Start empiric antibiotics if
in moderate to severe pain and fever >101oF worsens or does not improve after seven days of observation Immunocompromised patient

Antibiotics for Bacterial Rhinosinusitis


Amoxicillin-clavulanate now recommended as empiric choice
Due to changing antibiotic resistance patterns in adults and children High dose (2 gm BID or 90mg/kg/day) if:
>10% penicillin resistant S. pneumonia Severe infections Immunocompromised Daycare attendance <2 y.o. or >65 y.o.

http://www.idsociety.org/uploadedFiles/IDSA/GuidelinesPatient_Care/PDF_Library/IDSA%20Clinical%20Practice%20Guideline%20for%20Acute%20Bacterial%20 Rhinosinusitis%20in%20Children%20and%20Adults.pdf SORT C

Antibiotics for Bacterial Rhinosinusitis


Fluoroquinolones and doxycycline are second line choices No longer recommended due to antimicorbial resistance:
Macrolides Trimethoprime-sulfamethozazole Cephalosporins

Treatment of Bacterial Rhinosinusitis


Same adjunctive therapy as for viral rhinosinusitis What about nasal steroids?
Reduces inflammation and swelling of nasal mucosa Meta-analysis showed increased symptom response, especially milder disease
http://onlinelibrary.wiley.com/o/cochrane/clsysrev/articles/CD005149/frame.html http://www.ncbi.nlm.nih.gov/pubmed?term=18056902 SORT B

Bacterial Rhinosinusitis in Children


New AAP guidelines in 2013 made several changes
Added worsening of symptoms after initial improvement to diagnostic criteria Added observation period of three days after 10 day illness Amoxicillin (80-90mg/Kg) is the drug of choice for children Recommend against any imaging studies
http://pediatrics.aappublications.org/content/early/2013/06/19/peds.2013-1071.full.pdf SORT C

Chronic Rhinosinusitis
12 weeks of symptoms
Nasal obstruction Facial pain Mucopurulent drainage Decreased sense of smell

Most patients cannot be cured control or reduce symptoms Complex inflammatory changes rather than persistent bacterial infection

Chronic Rhinosinusitis
Saline nasal sprays/irrigations Intranasal steroids sprays and instillations Oral steroids Topical and oral antimicrobials Leukotriene antagonists Referral to ENT specialist

Question #6
A 57 year old female patient of yours presents with dizziness and a sensation that she is spinning. It occurs when she turns in bed or lifts her head to look in an upper cabinet. Episodes are brief but are becoming more frequent. She has no tinnitus or hearing loss. The most likely diagnosis would be:

A. B. C. D.

Menieres Disease Benign paroxysmal positional vertigo Vestibular neuronitis Acoustic neuroma

Vertigo
Illusion of movement
Spinning tilting, swaying Subjective or objective (patient or environment)

Must be distinguished from presyncopal faintness and dysequilibrium Central and Peripheral causes

Peripheral Vertigo
Benign paroxysmal positional vertigo
Canalithiasis Brief spinning spells (seconds) when head moved Nausea, but rarely vomit No hearing loss, ear pain or tinnitus

Vestibular Neuronitis
Viral or post-viral inflammation of labyrinth Sudden onset of severe, persistent vertigo, nausea and vomiting, and gait instability With unilateral hearing loss, it is called labyrinthitis Last 1-2 days before resolution Must be distinguished from cerebellar hemorrhage/infarct

Peripheral Vertigo
Herpes Zoster Oticus (Ramsay-Hunt Syndrome)
Herpes infection of geniculate ganglion Acute vertigo, hearing loss, ipsilateral facial paralysis and zoster vesicles in canal and auricle Treat with anti-virals, ?steroids?

Meniers Disease
Endolymphatic Hydrops Spontaneous episodes of vertigo lasting minutes to hours Associated with tinnitus, hearing loss and ear fullness

Peripheral Vertigo
Labyrinthine concussion Perilymphatic fistula
Vertigo and/or hearing loss stimulated by sneezing, coughing, lifting, straining Tullio Phenomenon vertigo from loud noises

Acoustic neuroma
Vertigo is minor, tinnitus and hearing loss are main complaints

Aminoglycoside toxicity

Central Vertigo
Migrainous vertigo
Often associated with headache

Wallenbergs syndrome
Lateral medullary infarction Posterior inferior cerebellar artery from vertebral artery Vertigo, ipsilateral Horners syndrome, ipsilateral limb ataxia, hoarseness and dysphagia Loss of pain and temperature sensation on ipsilateral face and contralateral trunk

Central Vertigo
Cerebellar hemorrhage or infarction
Sudden intense vertigo and vomiting Markedly impaired gait falls to the side of the lesion Nystagmus away from the lesion Confused with vestibular neuronitis, but gait more disturbed

Chiari malformation
Congenital protrusion of cerebellar tonsils through the foramen magnum Positionally induced vertigo Headache, long tract signs and lower cranial nerve involvement

Central Vertigo
Cerebellopontine angle tumors Multiple sclerosis Drug-induced vertigo

Evaluation of Vertigo
Careful history Neurological exam Lab tests help < 1% of the time MRI if imaging necessary Dix-Hallpike Maneuver
http://www.aafp.org/afp/20050315/1115.html

Treatment of Vertigo
Medications
Most useful for vertigo that lasts hours or days not BPPV Lots of sedation as well as risk of falls and urinary retention in older patients Anticholinergics - scopolamine Antihistamines meclizine, dimenhydrinate Phenothiazines promethazine, metoclopramide Benzodiazipines diazepam, lorazepam

Treatment of Vertigo
Vestibular Rehabilitation (PT) CNS compensation for peripheral vestibular injury - ? Central ? When started early, balance and function are improved compared to controls Home exercises also effective
http://www.uptodate.com/online/content/topic.do?topicKey=genneuro/5875&selec tedTitle=1~150&source=search_result SORT B

Treatment of Vertigo
Benign Paroxysmal Positional Vertigo
Medications generally not helpful Canalith repositioning Epley Maneuver http://www.aafp.org/afp/20050315/1 115.html

Answer Key
1C 2A 3D 4C 5C 6B

Challenging Issues in Hematology


William J. Geiger, M.D. Associate Program Director Grant Family Medicine Residency OhioHealth Columbus, Ohio

Disclosure
Dr. Geiger has nothing to disclose

The AAFP has selected all faculty appearing in this program. It is the policy of the AAFP that all CME planning committees, faculty, authors, editors, and staff disclose relationships with commercial entities upon nomination or invitation of participation. Disclosure documents are reviewed for potential conflicts of interest and, if identified, they are resolved prior to confirmation of participation. Only those participants who had no conflict of interest or who agreed to an identified resolution process prior to their participation were involved in this CME activity.

Learning Objectives
Cite the assessment and plan of care for the patient with anemia Identify the diagnosis and clinical hallmarks of Sickle Cell Disease, TTP and Hemophilia. Recognize the evaluation and treatment of DVT and PE. Describe the unique challenges to managing the pregnant patient with a history of or current DVT or PE.

Venous Thromboembolism (VTE)


Manifested as deep vein thrombosis (DVT) or pulmonary embolus (PE) Significant cause of morbidity and mortality in hospitalized patients
16% of hospital deaths attributed to PE PE suspected <1/3 of the time prior to death

VTE Risk Factors


Virchows Triad
Stasis Endothelial injury Hypercoagulability

Other risk factors


Family history of VTE (thrombophilia) Obesity Prior episodes of VTE Malignancy Stroke Pregnancy/post-partum

Pre-test Probability for DVT


(Wells Criteria)
One point each for:
Active Cancer Treatment within last 6 months Palliative treatment Paralysis, paresis or immobilization of lower extremity Bedridden for >3 days or major surgery within 4 weeks Localized tenderness along deep veins Entire leg swollen Calf swelling >3 cm in symptomatic leg Pitting edema greater in symptomatic leg Collateral superficial veins

Alternative diagnosis more likely (-2 pts)

Pre-test Probability
(Wells Criteria)

Scoring
>3 = high probability 1-2 = moderate probability 0 = low probability

Question #1
A 38 year old female patient presents with pain in her R calf of 3 days duration. She had a lap cholecystectomy 8 weeks ago and a cervical conization for carcinoma-in-situ 6 years ago with no recurrence. No one in her family ever had a DVT. She has 2+ bilateral pitting ankle edema. Your choice of an initial diagnostic study would be:

A. B. C. D.

Proteins C and S Venous compression ultrasounds Venogram D-dimer

Diagnosing DVT
D-dimer (ELISA)
95% sensitive, 40-60% specific 95% negative predictive value Excellent test to rule out, not so good to rule in!

http://www.guideline.gov/content.aspx?id=35267&search=d-dimer SORT C

Diagnosing DVT
D-dimer <500 and low probability Wells Score makes DVT unlikely Moderate or high probability Wells score should get non-invasive testing Venous compression ultrasounds have 94% positive predictive value If negative and clinical suspicion is high, repeat US at 5-7 days
http://www.uptodate.com/online/content/topic.do?topicKey=coagulat/16240&selec tedTitle=2~150&source=search_result SORT C

Screening for Malignancy


Cancer risk = 1.3x expected Complete H&P (with rectal and pelvic exams) CBC, LFTs, CXR, stool guaiac Aggressive cancer work-up not necessary or cost-effective
Ca usually made its presence known prior to VTE Lung, Pancreas, Colon, Kidney, Prostate

Screening for Thrombophilia


Initial thrombosis prior to age 50 Family history of VTE Recurrent venous thrombosis Unusual vascular beds Warfarin induced skin necrosis Testing
Protein C, Protein S, fibrinogen, Anti-thrombin III, Factor V Leiden, Lupus anticoagulant, Anticardiolipin antibody, Prothrombin gene mutation

Question #2
A 63 year old man presents with a history of several days of shortness of breath with exertion and pleuritic chest pain. He suffered a DVT 5 years ago. His pO2 = 85% on RA and his HR is 110. He recently returned from vacationing in Japan. What is the best test for initial evaluation of this patients symptoms?

A. CT Angiography B. Compression US of the lower extremities C. VQ scanning D. D-dimer

Diagnosis of PE
Symptoms and physical finding are very non-specific Likewise, EKG and CXR findings not specific Arterial blood gases/pulse oximetry
90% with PO2 <80 25% with PO2 >80

Modified Wells Criteria for PE


Symptoms of DVT Other Dx less likely HR >100 Immobilization or surgery
(in last 4 weeks)

3.0 3.0 1.5 1.5 1.5 1.0 1.0

Previous DVT/PE Hemoptysis Malignancy

Assessment
High Moderate Low >6 2-6 <2

Diagnosis of PE
D-dimer (ELISA)
D-dimer <500 combined with low pretest probability (<2) rules out PE

Diagnostic Imaging
Pulmonary angiography is gold standard
Mortality 2% Morbidity 5%

Spiral CT-PA (Helical)


High sensitivity and specificity Low pre-test probability and neg CT rules out PE Use when PE unlikely and D-dimer >500 to rule out PE
http://www.aafp.org/afp/2007/1015/p1225.html SORT C

Diagnostic Imaging
V/Q Scanning
If normal, PE excluded (regardless of pretest probability) Low-probability scan + low clinical probability, PE excluded High-probability scan + high clinical probability, PE confirmed All other combinations require angiography or other imaging

Question #3
A 48 year old female presents with 3 days of left lower extremity swelling. Venous dopplers reveal a DVT. She is stable and has no other medical problems. What would be the best initial treatment for this patient?

A. Low Molecular Weight Heparin only as an outpatient B. Hospitalization for unfractionated heparin and concurrent warfarin therapy C. Low Molecular Weight Heparin and concurrent warfarin as an outpatient D. Inferior vena cava filter

2007 ACP/AAFP Guidelines Low molecular weight heparin is preferred to unfractionated heparin whenever possible.
Reduced mortality and major bleeds Greater bioavailability and duration of effect Anticoagulant effect correlates with body weight allowing fixed dose Lower risk of HIT Lab monitoring unnecessary
Snow V, Qaseem A, Barry P, et al. Management of venous thromboembolism: a clinical practice guideline from the American College of Physicians and the American Academy of Family Physicians. Ann Intern Med 2007; 146:204 SORT A

Treatment of VTE

Treatment of VTE
2007 ACP/AAFP Guidelines
Outpatient treatment with LMWH is safe and cost effective in selected patients:

Ambulatory and stable Low risk of bleeding No renal insufficiency Reliable patient and system to monitor blood work and complications
Massive DVT Symptomatic PE High risk for bleeding Co-morbid conditions

Criteria for inpatient treatment

Snow V, Qaseem A, Barry P, et al. Management of venous thromboembolism: a clinical practice guideline from the American College of Physicians and the American Academy of Family Physicians. Ann Intern Med 2007; 146:204 SORT A

Treatment of VTE
Use of unfractionated heparin is still acceptable
Must monitor the aPTT ratio (1.5 to 2.5) Monitor platelet count for HIT

Must bridge to warfarin for long term anticoagulation


Start warfarin on day 1 Maintain bridging until INR in range (2.0-3.0) on two consecutive days (24 hours)
http://www.uptodate.com/online/content/topic.do?topicKey=coagulat/16240&selecte dTitle=2~150&source=search_result SORT A

Treatment of DVTs
2007 ACP/AAFP Guidelines

Compression stockings should be started within one month and maintained for at least one year to prevent postthrombotic syndrome. Anticoagulation should be maintained for 3-6 months for VTE due to transient factors and >12 months for recurrent VTE
Snow V, Qaseem A, Barry P, et al. Management of venous thromboembolism: a clinical practice guideline from the American College of Physicians and the American Academy of Family Physicians. Ann Intern Med 2007; 146:204 SORT A

Duration of Anticoagulation
2012 ISTH Guidelines

Treat unprovoked calf DVT for 3 months Treat unprovoked proximal DVT or PE for 3-6 months Treat provoked DVT or PE for no more than 3 months

Definitions
Provoked clear clinical cause Unprovoked no clear cause Proximal Popliteal vein or above
Baglin T, Bauer K, Douketis J, et al. Duration of anticoagulant therapy after a first episode of an unprovoked pulmonary embolus or deep vein thrombosis: guidance from the SSC of the ISTH. J Thromb Haemost 2012; 10:698. SORT C

Treatment of PE
Hemodynamic stabilization and O2 Outpatient treatment is an option:
If patient is stable and doesnt require O2 Low risk of bleeding No renal insufficiency Reliable patient and system to monitor blood work and complications

IVC filter if too risky to anticoagulate Consider thrombolysis or embolectomy

Newer Anticoagulants
Direct thrombin inhibitors
Dabigatran (Pradaxa) 75-150mg BID orally Used in Atrial Fib to prevent embolic strokes

Factor Xa inhibitors
Rivaroxaban (Xarelto) Atrial Fib 20mg per day Post-op thromboprophylaxis 10mg per day New indication for treatment of DVT and PE Head to head study comparing rivaroxaban with enoxaparin and warfarin showed comparable effectiveness and fewer major bleeds 2012 ACCP Guidelines recommends either parenteral anticoagulation or rivaroxaban for DVT or PE
Kearon C, Akl EA, Comerota AJ, et al. Antithrombotic therapy for VTE disease: Antithrombotic Therapy and Prevention of Thrombosis, 9th ed: American College of Chest Physicians Evidence-Based Clinical Practice Guidelines. Chest 2012; 141:e419S. SORT C

Newer Anticoagulants
Drawbacks would be:
Higher cost No labs to monitor No antidote Only slightly less bleeding side-effects

May prove most useful when there are complications or contraindications of warfarin or heparin (HIT) Need more studies stay tuned

Question #4
A 28 year old pregnant female, G3P2, is diagnosed with a DVT in her right lower extremity. The treatment of choice for this patient would be:

A. B. C. D.

Vena cava filter Unfractionated heparin and warfarin Low molecular weight heparin alone Warfarin alone

VTE in Pregnancy
VTE risk increases 5 fold during pregnancy due to physiologic and anatomic factors Absolute risk is approximately 1/1,600 pregnancies Pulmonary embolism is one of the leading causes of pregnancy associated mortality and morbidity in the US 20-50% of pregnant women who experience a thrombotic event have an underlying thrombophilia Women with a history of thromboembolic events have a 3-4 fold increase in risk for recurrence during pregnancy

VTE in Pregnancy
Risk of VTE antepartum is as common as it is postpartum and occurs with equal frequency throughout all trimesters PE is more common in the postpartum period, especially the first week. Do not use D-dimer in pregnancy CXR is first study in pregnancy
If normal, do V/Q scan If abnormal, do CT scan
http://www.ncbi.nlm.nih.gov/pubmed?holding=mcwlib&term=22086989

Anticoagulation in Pregnancy
Heparin and LMWH do not cross placental barrier and are safe in pregnancy Unfractionated Heparin (UH)
Dosage requirements increase due to increases in heparin binding proteins, plasma volume, renal clearance and degradation by the placenta

Low Molecular Weight Heparin (LMWH)


Reduced risk for bleeding, osteoporosis and HIT in non-pregnant patients Longer half-life longer dosage intervals, but risk for neuro-axial anesthesia

Anticoagulation in Pregnancy
Warfarin
Crosses the placenta Associated with harmful fetal effects Use is generally only in the postpartum period or in selected patients with mechanical heart valves

All anticoagulants are safe for breast feeding mothers.


http://www.guideline.gov/content.aspx?id=34439&search=thromboembolism+in+pre gnancy SORT B

Anticoagulation in Pregnancy
Convert from LMWH to UH for last month of pregnancy. Compression stockings until anticoagulation resumed post-partum. Restart anticoagulation after delivery
4-6 hours post vaginal delivery 6-12 hours post c-section

May be bridged to warfarin post-partum for 6 weeks to 6 months.


http://www.guideline.gov/content.aspx?id=34439&search=thromboembolism+in+pre gnancy SORT C

Question #5
A 48 year old female with rheumatoid arthritis presents to your office complaining of two months of fatigue, and more recently dyspnea on exertion. Her Hgb=9.2 with normocytic, normochromic indices. You order iron studies which show low iron, low TIBC and high ferritin. The best treatment for this patient would be: A. B. C. D. Iron dextran Treat her Rheumatoid Arthritis Oral ferrous sulfate Oral B12 and Folic Acid

Kinetic Approach to Anemia


Three reasons for anemia:
Decreased RBC production
Lack of substrate Marrow disorders/suppression Decreased trophic hormones Chronic illness/inflammation

Increased RBC destruction


Hemolysis inherited or acquired

Blood loss
Occult Obvious

Morphologic Approach to Anemia


RBC indices reflect cell size
Mean Corpuscular Volume (MCV) Mean Corpuscular Hemoglobin (MCH) Mean Corpuscular Hemoglobin Concentration (MCHC) Red cell Distribution Width (RDW)

Use the RBC indices to direct your work-up of anemia


http://www.uptodate.com/online/content/topic.do?topicKey=red_cell/2950&selecte dTitle=1~150&source=search_result SORT C

Morphologic Approach to Anemia


Macrocytic (MCV >100)
Reticulocytosis (high RDW) B12 and Folate deficiencies Myelodysplasia Alcohol abuse, liver disease, hypothyroidism

Morphologic Approach to Anemia


Microcytic Anemia (MCV <80)
Iron deficiency Decreased heme synthesis (lead, sideroblastic anemia) Decreased globin synthesis (thalassemia states, hemoglobinopathies) Chronic illness/inflammation (uncommon)

Morphologic Approach to Anemia


Normocytic, normochromic anemia
Acute blood loss Acute hemolysis Hypersplenism Anemia of Chronic Illness (more common)

Iron Deficiency Anemia


Confirmed by low iron, high iron binding capacity and low ferritin (<10) MUST identify why the patient is iron deficient
Menstrual blood loss GI malignancy/blood loss

Reticulocyte response to iron therapy in 5-7 days

Iron Deficiency Anemia


Treat with FeSO4 325 mg TID Treat for three months after H/H has returned to normal to replete iron stores Acid and vitamin C enhance absorption Calcium, magnesium and tea inhibit absorption Warn patients about nausea, constipation and dark stools

Megaloblastic Anemia
Low B12, folate, and high methylmalonic acid levels Other sx glossitis, anorexia, diarrhea B12 paresthesias, ataxia, weakness, upgoing toes Other labs hypersegmented PMNs, elevated LDH, pancytopenia Treat with oral folate; B12 either oral or IM

Question #6
A 3 year old patient is brought to see you after moving to your community. He has Hgb SS disease. The parents ask you whether he needs to continue taking the penicillin he was prescribed by another physician. You should recommend:
A. B. C. D. Stop the penicillin to avoid antibiotic resistance Take penicillin V 125mg daily for the rest of his life Take penicillin V 250 mg daily until age 12 Take penicillin V 250mg BID until age 5 at least

Sickle Cell Disease


Most common single gene disorder in African-Americans Diagnosed by hemoglobin electrophoresis Due to homozygous abnormal hemoglobin S (SS disease)
Sickle SC disease Sickle-Thal

Hemoglobin S is poorly soluble when deoxygenated and forms polymers that deforms the cells

Sickle Cell Disease


Clinical Manifestations
Moderate anemia (7.9/22.9) with high retic count and normal or high MCV Vaso-occlusive crises muscular, CVA, renal, priapism, retinopathy Infections Pneumococcus, Hemophilus, Salmonella Aplastic Crisis - Parvovirus B19 marrow suppression Acute Chest Syndrome pneumonia + infarct Splenic Sequestration Crisis acute anemia

Sickle Cell Disease


Infection Prophylaxsis
Immunizations
Strep pneumoniae Neisseria meningitidis H. influenza, Type B Hepatitis B Influenza

Penicillin prophylaxis
Pen V p.o. 125mg BID from age 3 month to 3 years Pen V p.o. 250mg BID from age 3 to 5 years After age 5 is controversial

Hemophilia
Inherited bleeding disorders
Hemophilia A Factor VIII deficiency Hemophilia B Factor IX deficiency (Christmas Disease)

X-linked recessive predominantly males affected Become symptomatic within first two years of life
Only 50% have bleeding with circumcision 3-5% have bleeding in perinatal period

Hemophilia
Bleeding sites
Muscles Hematuria Gastrointestinal Epistaxis and oral cavity Joints
Late joint destruction Joints preserved by early initiation of factor concentrate treatments

Thrombotic Thrombocytopenic Purpura Pentad


Microangiopathic hemolytic anemia Thrombocytopenia usually with purpura Acute renal insuffiency (HUS) Neurological symptoms (TTP) Fever Actually rare for all five to be present

Thrombotic Thrombocytopenic Purpura


Causes
Usually idiopathic Shiga-toxin from E. coli 0157:H7 ADAMTS13 deficiency
Protease of VWF inhibitory auto-antibody Long polymers of VWF attract platelets

Medications
Ticlopidine, clopidogrel, quinine, mitomycin, tacrolimus

Treatment plasma exchange therapy curative

Answer Key
1D 2A 3C 4C 5B 6D

Fever and Infectious Diseases in Children

Disclosure Statement

It is the policy of the AAFP that all individuals in a position to control content disclose any relationships with commercial interests upon nomination/invitation of participation. Disclosure documents are reviewed for potential conflicts of interest. If conflicts are identified, they are resolved prior to confirmation of participation. Only participants who have no conflict of interest or who agree to an identified resolution process prior to their participation were involved in this CME activity.

Learning Objectives
1. Identify an approach to evaluate and manage infants/children with fever 2. Describe bacterial and viral illnesses of the respiratory tree with a focus on epidemiology, diagnosis, and treatment 3. List the characteristic features and describe the clinical courses of common exanthems

Fever
An abnormal condition of the body characterized by an undue rise in temperature, quickening of the pulse, and disturbance of various body functions Fever (> 100.4F or 38C)rectal most accurate
Usually does not indicate serious illness Can cause discomfort and seizures Does not cause brain damage Does help fight infection

1. 3-week-old infant brought to ED for fever of 102 F. Infant does not appear ill, but has not been feeding well. What should you do?
A. Sepsis work up and send home if negative B. Admit and do sepsis work up

C. Observe in ED and send home if fever resolves with acetaminophen D. Admit and observe until fever breaks

Fever in First Month


*Admit for temperature > 100.4F CBC with differential Urinalysis and urine culture Blood culture Lumbar puncture for CSF studies and culture Bugs: Group B strep, E. coli, Listeria IV antibiotics: amp/gent or amp/cefotaxime acyclovir
No ceftriaxone less than 1 month (kernicterus risk)

Fever in Infant 1-3 Months of Age Who Appears Toxic


What is toxic?
Cyanosis Decreased activity Inability to interact with surroundings Irritability Lethargy Poor tone Signs of poor perfusion Tachycardia Weak eye contact Inadequate feeding

Treat the same as infant < 1 month old

Outpatient Management of Fever in Infant 1-3 M/O


If non-toxic and low-risk, may be managed outpatient if:
WBC < 15,000 and < 1,500 bands, negative gram stain CSF < 8 WBC and negative gram stain Blood culture Urine culture (urethral catheterization) Ceftriaxone 50 mg/kg (max 1 g) IM Meets low-risk criteria
Term infant without chronic disease or hospitalizations Reliable caretakers Transportation and telephone available Willingness to return in 24 hr

Febrile Seizures
3-4% of children 9-20 mo most common age 30-40% will have a recurrence Family history or underlying neurological condition Not associated with brain damage No evaluation other than work up of fever is indicated for first febrile seizure

2. 3 y/o girl presents with 4 day history of fever without other symptoms. Throat and lung exams are normal, but she appears quite ill. What should be your next step? A. B. C. D. Order an abdominal sonogram Order UA with cultures Admit for IV fluids Reassure mother that it is viral and see her back the next day

UTI
Most common serious bacterial infection in children (look for it when there are no signs) 70-90% E. coli Newborns: males and premature infants more likely to have UTI Ages 1-5: girls 10-20 times more likely Urine culture needed: catheter or suprapubic tap

Treatment of UTI
Positive culture > 50,000 CFU/ml Cephalosporin 1st choice for 7-14 days Follow up renal ultrasound First febrile UTI Follow up VCUG If US reveals hydronephrosis/scarring Recurrent febrile UTIs

3. 4 y/o male seen in your office for sore throat. No fever, rash, or adenopathy. Tonsils are erythematous without exudates. Mother insists on antibiotics. What do you do? A. Treat with amoxicillin B. Treat with penicillin C. Do rapid strep screen and if negative do strep culture prior to treating D. Explain to mother that it is probably viral and that antibiotics are not indicated

Pharyngitis
Viral: most common Group A -hemolytic streptococcus: 15% Mod to severe tonsillar swelling with exudate Mod to severe anterior cervical adenopathy Absence of moderate to severe cough Fever Treatment with penicillin (erythromycin or clindamycin if pen allergic) prevents rheumatic fever but not glomerulonephritis Suppurative complications: peritonsillar abscess & retropharyngeal abscess

Peritonsillar Abscess & Retropharyngeal Abscess


Symptoms
Severe throat or neck pain Painful swallowing High fever Poor oral intake (dehydration)

Physical
Cervical adenopathy Uvular deviation Muffled voice with trismus

Elevated WBC with a left shift Treatment: surgical drainage, IV antibiotics

Epiglottitis
Rare since Haemophilus influenzae vaccine Can visualize swollen, cherry red epiglottis X-ray shows thumb print sign Airway management Antibiotic treatment
Cephalosporin +/- clindamycin

Steroids and racemic epinephrine are not used

4. A 2 y/o child is brought to the ED. She has a barking cough and some stridor that worsened tonight, and a temperature of 101 F. What is the treatment? A. B. C. D. Nebulized epinephrine and dexamethasone Albuterol Humidified oxygen Inhaled steroids

Croup (Laryngotracheobronchitis)
Viral illness causing edema of upper airways Etiology
Parainfluenza viruses 1, 2, 3, cause 75% Adenovirus Respiratory syncytial virus (RSV) Mycoplasma pneumoniae

Symptoms: URI symptoms, barky cough, hoarseness, tachypnea, mild stridorworse at night X-rays show subglottic narrowing (steeple sign) in 40-50%

Treatment of Croup
Cool, moist air widely used: no evidence Dexamethasone 0.6 mg/kg IM (or oral) reduces hospitalization rates and shortens ED stay
Single dose (multiple dose may lead to bacterial or fungal infections) Indicated for croup of any severity

Nebulized epinephrine for immediate relief: followed by admission or at least 3-hr observation and/or corticosteroids Beta-agonist bronchodilators not effective

5. 9-month-old child is admitted in Jan. for cough, wheezing, feeding, and fever of 38C. CXR shows mild peribronchial cuffing. Which treatment is indicated?

A. B. C. D.

Amoxicillin/clavulanate (Augmentin) Systemic corticosteroids Supplemental oxygen and fluids Nebulized ipratropium (Atrovent)

Respiratory Syncytial Virus (RSV)


Most common cause of bronchiolitis and pneumonia in infants under 1 year Rhinorrhea, pharyngitis, cough, wheezing, rhonchi, rales, CXR often normal, fever and WBC inconsistent Diagnosis: antigen detection assays **Treatment is supportive (oxygen, acetaminophen, fluids)/inhaled bronchodilator albuterol or epinephrine only if effective No evidence for steroids, antibiotics, or ribavirin

Pertussis
Symptoms
URI symptoms Paroxysms of coughing with whoops on inspiration 24 weeks Coughing to the point of vomiting Dyspnea Seizures 20-25%

Pertussis and bronchiolitis may present with apnea under 3 mo of age No specific physical findings, but high WBC Pneumonia most frequent

Treatment of Pertussis
Admission for most children < 3 mo Supportive: hydration, pulmonary toilet, oxygen Antibiotics: erythromycin, clarithromycin, azithromycin, TMP-SMX for 14 days Treat patient and all household contacts Adults should get Tdap as single booster dose

Impetigo
Caused by group A -hemolytic Streptococcus or Staphylococcus aureus Honey-colored crusts Treatment recommended because of contagiousness Mupirocin cream is treatment of choice Cephalexin (Keflex) if oral

MRSA
Consider even in neonates Any abscess is MRSA until proven otherwise Culture and treat by sensitivity
Sulfamethoxazole/ trimethoprim Clindamycin Tetracyclines

6. A 2 y/o boy has very red cheeks and a fine rash but does not appear ill. He had a fever a couple of days ago. When can he return to day care?

A. B. C. D.

Today Tomorrow 5 days After the rash is gone

Fifth Disease
Common disease, rarely clinically significant Caused by parvovirus B19 Rash is immune-mediated and occurs after acute infection, so children with rash may attend school or day care Phase I: facial flushing (slapped cheeks) Phase II: macular rash Phase III: reticular rash Arthritis rarely in adolescents Pregnant women: rare complication fetal hydrops and fetal demise; women develop arthritis

Guess the Exanthem

Human herpesvirus 6 (HHV-6) Presents with high fever, followed by transient rash

Roseola Infantum (Exanthem Subitum)


Sudden Rash herpes virus HHV-6 or HHV-7 Usually mild and sporadic Children 6 months to 4 years Natural course
Days 1-2: Anorexia/vomiting, occipital adenopathy Days 2-3: High fever (103-106F), possible seizures Day 4: Rash begins as fever abates Pink almond-shaped macules on trunk/neck Confluent then fade in hours to 2 days

Roseola
Pink, blanchable, discrete maculopapular rash mainly on trunk; resolves in 1-2 days Nearly 100% of children have antibodies to HHV-6 by 3 years of age

What Is This?

Henoch-Schnlein Purpura
Usually follows URI with low-grade fever, fatigue Triad of purpura, colicky abdominal pain, and arthritis Rash: pink maculopapules progressing to nonthrombocytopenic palpable purpura on buttocks and legs Hematuria and proteinuria ESR, WBCs, platelets, RBCs Can treat with prednisone in severe cases (renal complications)

Guess the Exanthem


Lesions on hands and feet Late summer and early fall

Hand, Foot, and Mouth (Coxsackie A)


Usually in children < 5 years Late summer, early fall Oral-oral, oral-fecal spread Incubation period 4-6 days Prodrome of fever, sore throat, and anorexia 1-2 days before the rash Small vesicles, erythematous base Hands (nail borders), feet (heel margins), buttocks Spontaneous resolution in a few days Sometimes magic mouthwash preparations needed

7. A 7 y/o male has 7 days of spiking high fevers, diffuse erythematous rash. His tongue and lips are red, dry, and cracked and he has large cervical lymph nodes. What next? A. Admit and treat with IV antibiotics B. Admit and treat with a single injection of immune globulin and high-dose aspirin C. Send home with frequent sips of rehydration solution D. Admit and treat with aspirin 40 mg/kg/d

Kawasaki Disease
Diffuse vasculitis of unknown etiology Leading cause of acquired cardiac disease in children in US Diagnostic criteria:
Fever > 5 days duration Bilateral conjunctival injection Oropharyngeal erythema, strawberry tongue, fissuring and crusting of the lips Induration of hands and feet, erythema of palms and soles; desquamation of fingertips and toes Erythematous rash (scarlatiniform or morbilliform) Enlarged lymph node mass (> 1.5 cm) May also be irritable and have perineal rash

Sequelae of Kawasaki Disease


Development of giant aneurysms (> 6 mm) Can lead to coronary thrombosis or stenosis Treat with warfarin and aspirin: INR 2.0-2.5 Long-term effects Persistent vascular changes Altered lipid metabolism Thickened coronary artery walls Abnormal reactivity with less flow during reactive hyperemia

Meningococcal Meningitis
Neisseria meningitidis: gram negative diplococcus. Clinical manifestations of meningococcemia or meningitis Abrupt onset: fever, chills, malaise, prostration Rash: maculopapular, macular, petechial Waterhouse-Friderichsen syndrome: purpura, DIC, shock, coma, death

Meningococcal Meningitis
Sporadic, usually type B in infants (no vaccine available yet) ***Rifampin is used for exposure prophylaxis (family/close contacts) Treatment:
Ceftriaxone (80-100 mg/kg/dose) bid on day 1 Cefotaxime (50-75 mg/kg q 6 h) Ceftazidime (75 mg/kg/dose q 8 h) No evidence that vancomycin increases survival Dexamethasone 0.6 mg/kg/d q 6 h for 4 days

Do Not Miss This One


Rash typically begins on day 3-5 of the illness, first appearing peripherally (hands, wrists, feet, ankles). Palm and sole involvement occurs in 50%-70% of individuals. However, the classic triad of fever, rash, and tick exposure only occurs in 20% at the time of presentation.

Rocky Mountain Spotted Fever


Rickettsia rickettsii, gram negative bacteria Most common fatal tick-borne disease in the U.S. The American dog tick is the vectorfound in tall grass Symptoms: sudden onset of fever, muscle pain, nausea, vomiting, then rash, abdominal and joint pain, hypotension, headache Lesions: begin as blanching macules and papules which become nonblanching, petechial, and hemorrhagic Usually starts on the legs, palms, and soles, spares the face

RMSF Management
Elevated serum creatinine associated with increased risk for fatality Antibiotics have reduced mortality from 30% to 40% Doxycycline 100 mg bid if > 45 lbs. (4 mg/kg/day if < 45 pounds)

8. Infants < 1 month old should be admitted for septic work-up if temperature is:

A. B. C. D.

> 100F > 100.4F > 102F No set temperature

9. RSV bronchiolitis treatment might include:

A. B. C. D.

Steroids Antibiotics Ribavirin Oxygen and fluids

10. The drug of choice for exposure prophylaxis of meningococcal meningitis is:

A. B. C. D.

Ceftriaxone Dexamethasone Rifampin Vancomycin

Take Home Pearls


Admit any infant < 1 mo. old with fever > 100.4 RSV treatment is supportive Rifampin for meningococcal post-exposure prophylaxis

Answers
1. 2. 3. 4. 5. 6. 7. B B D A C A B 8. B 9. D 10. C

Common Neurological Disorders


William J. Geiger, M.D. Associate Program Director Grant Family Medicine Residency OhioHealth Columbus, Ohio

Disclosure
Dr. Geiger has nothing to Disclose

The AAFP has selected all faculty appearing in this program. It is the policy of the AAFP that all CME planning committees, faculty, authors, editors, and staff disclose relationships with commercial entities upon nomination or invitation of participation. Disclosure documents are reviewed for potential conflicts of interest and, if identified, they are resolved prior to confirmation of participation. Only those participants who had no conflict of interest or who agreed to an identified resolution process prior to their participation were involved in this CME activity.

Learning Objectives
Describe the classification of seizure disorders. Identify the appropriate diagnostic tests in the evaluation of a seizure disorder. List the medications used to treat status epileticus. Select the common medications for epilepsy and appropriately monitor them. Describe the unique features of managing seizure disorders in a pregnant patient Diagnose and treat vascular headaches. Recognize the indications for neuroimaging in the evaluation of headaches.

Question #1
A 65 year old male patient of yours presents to the ED having had a seizure in his bedroom witnessed by his wife. She heard a cry and the fall and saw him stiffen and then shake all over, and become incontinent of urine. He was not arousable until he had been in the ED for several minutes. His seizure would be classified as:

A. B. C. D.

Complex Partial Generalized Tonic Clonic Grand Mal Myoclonic

Definition
A seizure is a transient occurrence of signs or symptoms due to abnormal excessive or synchronous neuronal activity in the brain, and can be either focal (partial) or generalized

Classification of Seizures
Focal Seizures (partial)
Local Ipsilateral propagation Contralateral propagation Secondarily generalized
Consciousness may or may not be impaired (simple vs complex)

Classification of Seizures
Generalized Seizures
Non-convulsive (absence)
Typical (3/sec spike and slow waves on EEG) Atypical (<3/sec spike and slow waves on EEG)

Convulsive
Myoclonic Clonic Tonic Tonic-clonic Atonic (drop attacks)

Question #2
The most likely cause of your patients seizure would be:
A. B. C. D. Cerebrovascular disease/stroke Idiopathic Metabolic derangement Brain tumor

Etiology of Seizures
Idiopathic 62% overall age groups Stroke 15% overall, 49% >age 60 Brain Tumor 6% overall, 11% >age 60 Head trauma Intracranial infection Cerebral degeneration Congenital brain malformations Inborn errors of metabolism

Provoked Seizures
Seizures that occur within a medical setting, that, if removed, presumably the seizures would not occur Metabolic derangements Hypo- and hyperglycemia Hyponatremia Hypocalcemia (usually neonates) Renal failure and uremia

Provoked Seizures
More rare metabolic causes
Hyperthyroidism Acute Intermittant Porphyria

Cerebral anoxia
Arrest, anesthesia, drowning, CO Syncope with brief hypoventilation

Drug toxicities/withdrawal
Alcohol Benzodiazepines

Epilepsy
Epilepsy is a chronic condition characterized by at least two unprovoked seizures at least 24 hours apart.

Seizure Imitators
Syncope Psychiatric disorders Sleep disorders Movement disorders Migraines TIAs Transient global amnesia

Question #3
Your patients complete history and physical is unremarkable for any provoked causes, signs of infection, drug toxicity, or neurological disease. The next step in the work-up for your patient would include:

A. B. C. D.

Lumbar puncture Toxicology screen Neuroimaging (CT/MRI) Prolactin level

Diagnostic Evaluation
Must start with a complete H&P
Witnessed description of the event Substance abuse Head Trauma Cerebrovascular events Cardiac history Sleep disorders Medication history (OTCs)

Diagnostic Evaluation
Blood studies
CBC Electrolytes Calcium, magnesium, phosphorus Glucose BUN/Creatinine ?ESR, LFTs, RPR Prolactin has limited utility low sensitivity

Diagnostic Evaluation
Lumbar puncture only if signs of infection EEG essential to diagnosis and classification Neuroimaging
MRI preferred over CT Better for identifying structural lesions Is the lesion the cause of the seizures?

Question #4
Your work up for this patient is completely negative. You diagnose an unprovoked seizure. Your first step in treatment would be:
A. B. C. D. Start IV phenytoin Start oral phenytoin Start oral valproic acid Start no medications at this time

Starting Anti-epileptic Drugs


(AEDs)
Not necessary in individuals with a single new onset seizure or infrequent seizures Treat if high risk of recurrence
Status epilepticus Hx of brain injury Brain lesion on neuroimaging Focal neurological abnormalities Mental retardation Abnormal EEG with epileptiform discharges

Beghi E. General conclusions and recommendations. Epilepsia 2008;49 Suppl 1:58-61. SORT C

Starting Anti-epileptic Drugs


(AEDs)

High risk seizure types


Any partial seizure Absence seizure Myoclonic seizure Atonic seizure

Anti-epileptic Drugs
Start with AED monotherapy Selection is individualized based on:
Seizure type Other medications (enzyme induction) Comorbid conditions Potential adverse effects Patient preferences and cost

Do not consider combination therapy until patient has failed two monotherapy trials Monitor patients closely for effectiveness, sideeffects, and compliance
http://www.uptodate.com/online/content/topic.do?topicKey=epil_eeg/4878&selectedTitle=1~150&source=se arch_result SORT B

Classification of AEDs
Broad Spectrum
All seizure types

Narrow Spectrum
Focal or secondarily generalized seizures

First Generation
Less expensive More side-effects and drug interactions Require monitoring of drug levels

Second Generation
More expensive Fewer side-effects and interactions Monitoring often unavailable
(See further slides at the end of the handout)

Toxicities of AEDs
Common toxicities with AEDs
Suicidality twice the risk with AEDs compared to placebos Neurotoxicities ataxia, dizziness, somnolence, fatigue, headache Rash wide-spectrum from simple maculo-papular rashes to StevensJohnson Syndrome Liver enzyme induction or inhibition

Question #5
A 46 year old male patient of your practice presents with fever, arthalgias, and erythroderma. He has no respiratory symptoms. His medications include lisinopril, vaproic acid, and atenolol. His fever is 102.2o and his skin reveals a very erythematous maculo-papular rtash and some tenderness. There are no oral lesions. You order blood work that reveals a WBC = 16,000 with 17% eosinophils, and AST and ALT about 2x normal. The most likely diagnosis is:

A. B. C. D.

Toxic Shock Syndrome DRESS Syndrome Stevens-Johnson Syndrome Erysipelas

DRESS Syndrome
Drug Reaction with Eosinophilia and Systemic Symptoms Maculopapular rash, fever, arthralgias and lymphadenopathy Persists weeks to months with 10-25% mortality Common offenders:
Carbamazepine, phenytoin, lamotrigine, valproate Allopurinol and sulfonamides
http://www.ncbi.nlm.nih.gov/pubmed?holding=mcwlib&term=21592453

Status Epilepticus
A single unremitting seizure lasting longer than 5-10 minutes, or recurrent seizures without interictal return to baseline Overall mortality rate of first episode is 20%
Metabolic stress of repeated muscular convulsions Neuronal death after 30-60 minutes of continuous seizure activity

Status Epilepticus
Evaluate causes while you treat Benzodiazepines
Diazepam 0.1-0.3 mg/Kg IV lasts only 20 min Lorazepam 0.02-0.03 mg/KG IV lasts 4-6 hours Midazolam 0.2 mg/Kg bolus, then 0.7510mcg/Kg per minute IV infusion Clonazepam longer duration
http://www.uptodate.com/online/content/topic.do?topicKey=epil_eeg/2440&selectedTitle=2~105&source=sear ch_result SORT A

Status Epilepticus
Phenytoins
Fosphenytoin has replaced phenytoin
Pro-drug dosed in phenytoin equivalents (PE) Less IV irritation

Barbiturates
Phenobarbital hypotension and respiratory suppression Thiopental high rate of cardiovascular adverse effects

Propofol
Limited experience in status Potential fatal adverse effects

Valproic Acid
Approved for slow IV infusion Give 10-20 mg/min IV

Seizures and Driving


0.01-0.1% of MVAs are attributable to seizures Fatal MVAs 8x greater from alcohol than seizures Seizure free interval is best indicator of driving safety
The longer the seizure free interval, the less likely an MVA The longer the patient is restricted from driving, the lower the compliance

1994 consensus statement of the American Academy of Neurology, American Epilepsy Society, and the Epilepsy Foundation of America recommends a three month seizure free interval before driving resumed.
Consensus statements, sample statutory provisions, and model regulations regarding driver licensing and epilepsy. American Academy of Neurology, American Epilepsy Society, and Epilepsy Foundation of America. Epilepsia 1994; 35:696

Seizures and Driving


States authorities make the ultimate decision about driving, not physicians. State laws vary regarding seizure free intervals. Only a few states require physicians to report patients with seizures. KNOW YOU OWN STATES LAWS!
http://www.epilepsyfoundation.org/resources/drivingandtrav el.cfm

Epilepsy and Pregnancy


Hormonal contraception failure on AEDs that stimulate cytochrome P450 system:
Carbamazepine Phenytoin Phenobarbital/Primadone Topiramate Oxcarbazepine

Epilepsy and Pregnancy


Consider withdrawing AEDs 6 months prior to conception if patient has been seizure free >2 years Do not change AEDs in pregnancy if patient well controlled Supplement Folic Acid 4 mg per day if patient on carbamazepine or valproic acid

Epilepsy and Pregnancy


Monitor both total and free drug levels of AEDs at 6 weeks, 10 weeks, then once a trimester, then first or second post partum week. Vitamin K supplementation 10-20 mg/day in last month of pregnancy if on phenobarbital, carbamazepine, phenytoin, topiramate, oxcarbazepine. May breast feed on all AEDs except lamotrigine

Question #6
A 36 year old female comes to you for a complaint of headache that she has had for years which occurs 8-10 times a year. They have not changed and start with flashing lights in her eyes, followed by left sided throbbing headaches associated with nausea and vomiting. She has taken OTC meds but they have not been helpful. The best initial therapy would be:

A. B. C. D.

Order a brain MRI Prescribe a sublingual ergotamine Prescribe propranolol Prescribe sumatriptan

Vascular Headaches
Trigeminal Autonomic Cephalgias
Primary, unilateral headache syndromes in the trigeminal nerve distribution Associated with ipsilateral autonomic symptoms Classed by duration and frequency of headaches
Cluster Headaches (minutes to hours duration, <10 attacks per day) Paroxysmal Hemicrania (minutes, 10-40 attacks per day) Short-lasting headaches (seconds, >200 attacks per day)

Migraine Headaches

Cluster Headaches
Prevalence <1% Males > females Orbital or temporal unilateral pain Associated ptosis, miosis, rhinorrhea, lacrimation or conjunctival injection Last 15 minutes to 3 hours Acute treatment oxygen and sumatripan SQ Preventive treatment
Verapamil drug of choice (240-320mg per day or more) Steroids, lithium, topiramate, methysergide

Migraine Headache
Affects 12% of the population, female predominance Pathophysiology - unclear
Cellular and blood-brain permeability changes leading to inflammation of meninges and vasodilation Genetic factors play a major role.

Recurrent attacks in four phases


Prodrome euphoria, depression, irritability, yawning Aura Headache Postdrome sudden head movement causes head pain

International Headache Society


Diagnostic Criteria for Migraine
Migraine without aura
Headaches last 4-72 hours Two of the following:
Unilateral Throbbing Moderate to severe intensity Worsened by usual physical activity

At least one of the following


Photophobia Phonophobia Nausea/vomiting

No evidence of other underlying disease


Headache Classification Subcommittee of the International Headache Society. The International Classification of Headache Disorders: 2nd edition. Cephalalgia 2004; 24 Suppl 1:9.

International Headache Society


Diagnostic Criteria for Migraine
Migraine with aura
Transient neurologic symptoms
Start gradually over 5-20 min Last less than 60 min Headache as above starts within 60 min

Types of aura
Visual flickering lights, spots or lines Sensory pins and needles, numbness Dysphasic speech Fully reversible symptoms At least two attacks of headache with aura
Headache Classification Subcommittee of the International Headache Society. The International Classification of Headache Disorders: 2nd edition. Cephalalgia 2004; 24 Suppl 1:9.

Migraine Variants
Menstrual migraine Basilar-type migraine
Tinnitus, vertigo, ataxia, diplopia

Familial hemiplegic migraine


Motor weakness that is fully reversible At least one first or second degree relative

Rare types
Aura without headache Ocular migraines Ophthalmoplegic migraines

Diagnosis of Migraine Headache


Clinical diagnosis primarily history Negative neurological examination Neuroimaging unnecessary, except with
Unexplained abnormal neuro findings Headaches that do not fit the strict definition

Headache red flags


Rapidly increasing headache frequency History of lack of coordination History of localized neurological signs Headaches that awaken from sleep

Acute Treatment of Migraine


Triptans serotonin 1b/1d agonists
All triptans have been found to be effective and well tolerated currently first-line Few head to head trials individualized choice Sumatriptan offers most delivery options Prompt treatment is best predictor of success Avoid triptans in
Familial hemiplegic migraine Basilar migraine Ischemic stroke Ischemic heart disease, Prinzmetals angina, uncontrolled hypertension Pregnancy

Acute Treatment of Migraine


Ergots
Sublingual ergotamine Dihydroergotamine (DHE 45)
Primary agents in ED setting IV, IM, SQ, IN delivery Contraindicated with hypertension, ischemic heart disease, MAO inhibitors, and the elderly

Anti-emetics
Used in combination with DHE 45 in emergency setting Phenothiazines chlorpromazine, prochlorperazine Metoclopramide usually used with DHE 45

Dexamethasone
10-25mg IV or IM signifcantly reduces the rate or early headache recurrance

Acute Treatment of Migraine


U.S. Headache Consortium (AAN, AAFP and five other organizations) recommendations:
Educate migraine patients about the condition and treatment Use migraine specific agents for those with more severe symptoms Non-oral route of administration for those with nausea and vomiting Use prophylactic medications for those overusing their medications
Silberstein SD, Rosenberg J. Multispecialty consensus on diagnosis and treatment of headache. Neurology 2000; 54:1553

Preventive Treatment of Migraine


Indications
Headaches significantly interfere with usual activities Overuse of acute therapies Adverse events with acute therapies Patient preference Certain migraine variants
Hemiplegic migraines Basilar migraines Prolonged aura

Preventive Treatment of Migraine


Antihypertensives
Beta-blockers Calcium-channel blockers ACEI/ARBs

Antidepressants
Amitriptyline Venlafaxine

Anticonvulsants
Valproate Gabapentin Topiramate

Answer Key
1B 2A 3C 4D 5B 6D

Classification of AEDs
Broad Spectrum
Carbamazepine (Tegretol) Phenytoin (Dilantin) Lamotrigine (Lamictal) Valproic Acid (Depakene) Levetiracetam (Keppra) Oxcarbazepine (Trileptal) Phenobarbital Topiramate (Topamax) * Zonisamide (Zonegran) *

Narrow Spectrum
Felbamate (Felbatol) Gabapentin (Neurontin) * Lacosamide (Vimpat) * Pregabalin (Lyrica) * Primidone (Mysoline) Tiagabine (Gabitril) * Vigabatrin (Sabril) * Rufinamide (Banzel) Ethosuximide (Zarontin)

First Generation AEDs


Drug
Phenytoin (Dilantin) Valproic Acid (Depekene) Carbamazepine (Tegretol) Ethosuximide (Zarontin) Phenobarbitol/ Primidone

Seizure type
Focal and Secondarily Generalized All seizure types Focal and Secondarily Generalized Absence only Focal and Secondarily Generalized

Side effects
Ataxia, diplopia, nystagmus, slurred speech Tremors, N/V, somnolence, weight gain Ataxia, N/V, rash, hyponatremia, aplastic anemia Rash, N/V, hyperactivity, headache Rash, sedation, hyperactivity, ataxia

Monitoring
Many drug/food interactions Blood levels Many drug interactions Blood levels Many drug interactions Blood levels Many drug interactions Many drug interactions Blood levels

Cost
$

$$

$$

http://www.ncbi.nlm.nih.gov/pubmed/21814641

Second Generation AEDs


Drug
Oxycarbazepine (Trileptal) Gabapentin (Neurontin) Lamotrigine (Lamictal) Topiramate (Topamax) Levetiracetam (Keppra)

Seizure type
Focal and Secondarily Generalized Focal and Secondarily Generalized All seizure types All seizure types Adjunctive Generalized

Side effects
Ataxia, rash, GI, hyponatremia, sedation, rash Fatigue, ataxia, weight gain, nystagmus Ataxia, tremor, Stevens-Johnson Syndrome, N/V Memory, wt.loss, depression, nephrolithiasis Somnolence, anxiety, wt loss, pancytopenia

Monitoring
Few drug interactions Few drug interactions, best safety profile Interacts with valproic acid Interacts with other AEDs Few drug interactions

Cost
$$$

$$

$$$

$$$

$$$

http://www.ncbi.nlm.nih.gov/pubmed/21814641

Guide for Exam Preparation


David G. Weismiller, MD, ScM, FAAFP
Department of Family Medicine The Brody School of Medicine at East Carolina University

Disclosure Statement
Dr. Weismiller has nothing to disclose.

The AAFP has selected all faculty appearing in this program. It is the policy of the AAFP that all CME planning committees, faculty, authors, editors, and staff disclose relationships with commercial entities upon nomination or invitation of participation. Disclosure documents are reviewed for potential conflicts of interest and, if identified, they are resolved prior to confirmation of participation. Only those participants who had no conflict of interest or who agreed to an identified resolution process prior to their participation were involved in this CME activity.

Learning Objectives
1. Manage anxiety in the anticipation of an important examination. 2. Develop an individual plan of study for the certification/recertification examination in Family Medicine. 3. Demonstrate specific test-taking techniques. 4. Explain the examination process for certification/recertification in Family Medicine.

THE EXAM

AAFP & ABFM

American Board of Family Medicine, Inc. 1648 McGrathiana Parkway Lexington, Kentucky 40505-4294 Phone: 859-269-5626 or 888-995-5700 Support Center: 877-233-7437

(Re)Certification Examination
ABFM
Exam Format
All multiple choice 4 answer
Stem and options (A, B, C, D)

Two general portions Two module choices


Selected on exam day during the exam

Understanding the Exam You Will Be Taking

(Re)Certification Examination
ABFM
Exam Format
All multiple choice 4 answer
Stem and options (A, B, C, D)

Two general portions Two module choices


Selected on exam day during the exam

Exam tutorial
www.theabfm.org Preview of the look and feel of a computerized examination

Computerized Testing

Exam Modules
Ambulatory Family Medicine Child and Adolescent Care Geriatrics Womens Health Maternity Care Emergent/Urgent Care Hospital Medicine Sports Medicine

Scoring
No penalty for incorrect answers
Leave no question unanswered Guess if you dont know

Setting a passing score


Angoff Method using a group of peers to estimate the percentage of family physicians who would answer each question correctly

Passing Threshold Angoff Technique

Pass Rates
(Combined Spring/Fall)
Year 2004 2005 Recerts 85% 94% Certs 86% 93% Score 380 330/380

2006
2007 2008 2009 2010 2011 2012

94%
82% 83% 81% 67% 66% 74%

95%
77% 80% 84% 82% 81% 83%

300
390 390 390 390 390 390

2013*

86%

86%

390

(*Spring only)

Manage Your Stress


Preparing Yourself Prior to the Examination

Summary
Steps to Creating an Effective Study Plan
Step
1

Action
Know what material is going to be on the exam in what proportions

Identify your level of need for study How did you perform on last standardized ABFM exam Do you practice broad-based family medicine
Identify how many hours you will need, would like, or will be able to study (Consider a minimum 3-month lead time if seeking serious score improvement [> 100 points, or if you are retaking the examination]) Evaluate how the designated amount of study will fit into your weekly schedule Identify and obtain the materials you plan to use Focus on relevant material with which you are last comfortable and familiar Divide up the material into your schedule as specifically as is reasonable
Adapted from The ABFM.org

4 5 6 7

Evidence Gathered to Date


Nearly any high-quality, comprehensive study material can be used effectively Key ingredient in exam success is scheduling regular time to study Performing well on an examination
General study strategies Test-taking strategies

Long-Term Strategies for Score Improvement


Broad population of practice
Studying is most likely to be effective (memorable) if you keep specific clinical practice cases in mind while studying material that applies to those cases Referring back to an article after seeing a complex patient aids retention

Narrowly defined population of practice


Systematic approach to reviewing the spectrum Medical knowledge fades without periodic re-exposure Include those areas not included in practice any longer, as well as common entities seen every day

Short-Term Study Tips


www.aafp.org/boardreview/questions https://www.theabfm.org/residency/ite.aspx

The best study information will be at the level of specificity found in textbook or journal readings Gaining points involves studying medical information and there is a strong dose-response relationship Use the in-training exams as a preand post-test

Sources of Questions
AAFP website Review questions
ABFM website In-training exams Prometrics practice

Pre- and Post-Test


Mark your starting point prior to studying and then to see if your study schedule is effective Try to take it according to the standardized timing and instructions Changes are likely to be noticeable within 1 month of regular studying, unless your initial performance was well above average Promotes focus on study
Allows one to actually SEE improvement

Creating an Effective Study Plan


Know what material is going to be on the exam
https://www.theabfm.org/moc/examcontents .aspx Identify your level/need for study Narrow-based practice Scored at < 50th percentile on previous exam

ABFM Certification/Recertification Examination Content


Cardiovascular Endocrine Gastrointestinal Heme/Immune Integumentary Musculoskeletal Nephrologic Neurologic Nonspecific Psychogenic ReprodFemale Reprod Male Respiratory 9% 6% 5% 2% 5% 9% 2% 2% 7% 5% 3% 1% 10% Special Sensory Population-based Care 2% 3%
Epidemiology, EBM, prevention, health policy & legal issues, bioterror, quality improvement, geographic/urban/rural issues

Patient-based Systems

3%

Clinical decision-making, communication and doctor-patient interaction, family & cultural issues, ethics, palliative care, end-of-life care

Module 1* Module 2*

13% 13%

Creating an Effective Study Plan


Identify how many hours you will need, would like, or would be able to study
If you are seeking serious score improvement (> 100 points, or if you are retaking the exam), then up to 10-14 hours per week, for 3 or more months may be needed

Evaluate how the designated amount of study will fit into your weekly schedule

Creating an Effective Study Plan


Identify and obtain the materials you plan to use; consider free resources
Already a participant in Maintenance of Certification for Family Physicians (MC-FP), Cochrane Library of evidence-based medicine is available through ABFM
Self-Assessment Modules (MOC Part II)

University or major hospital affiliation, check whether the library will allow staff online access to electronic journals Some material and journals are available for free online, eg, American Family Physician (http://www.aafp.org/afp.xml), and Journal of Family Practice (http://www.jfponline.com/)
AFP by Topic

Access to a medical library, then the need to spend money is cut further still. You are also likely to have useful textbooks and journals at your disposal already

Creating an Effective Study Plan


Focus on the relevant material with which you are least comfortable and familiar
Focus on established medicine 9-month development cycle for the examinations, so brand new information, no matter how reliable, is unlikely to appear as an item

Divide Up the Study Material into Your Schedule, as Specifically as Is Reasonable


Schedule regular time to study Start with those content areas that you find more challenging Serious score improvement needs 10-14 hours of study time per week (100 points) Keep a fact sheet for last-minute review Doing a lot of review questions is secondary (30 points)
www.aafp.org/boardreview/questions https://www.theabfm.org/residency/ite.aspx

Lectures/Number of days until exam = number of lectures reviewed each day

Divide Up the Study Material into Your Schedule, as Specifically as Is Reasonable


If you plan to study the last 5 years of 2 top family medicine publications = 120 journals, and you have 3 months to study, then that is roughly 10 journals per week Thus, summarizing the key findings in 2 journals per day is the pace that would be required
Unlikely that every article will be worth reading remember focus on the relevant material with which you are least comfortable and familiar

Proceed with Studying, Taking Weekly Inventory of Progress


Reevaluate at end of first week. If schedule is not as accommodating as you anticipated, consider changes to your schedule OR the ambitiousness of your study plan Consider using In-Training Exam items before beginning and after a month to see if performance seems to be improving

Exam Preparation
Board Review Preparation Courses
Listen loudly Actively participate

Study the material when you return home (primary)


Lectures/Number of days until exam = number of lectures reviewed each day
Schedule regular time to study Start with those content areas that you find more challenging Serious score improvement needs 10-14 hours of study time per week (100 points) Keep a fact sheet for last minute review Doing a lot of review questions is secondary (30 points)
www.aafp.org/boardreview/questions https://www.theabfm.org/residency/ite.aspx

How can we improve our response strategy in translating our knowledge into the exams response format

Test-Taking Strategies

ABFM Tips for Test-Taking


Read the stem carefully Read every response option (cross out the wrong ones) Consider the tables and images carefully Answer only after considering all options If you do not know guess! Go with the first thoughtful answer, unless you realize that you actually made a mistake

ABFM Tips for Test-Taking


Attempt to determine whether general knowledge or patient-specific knowledge is being assessed DO NOT try to out-guess the item number Dont read the answers first and try to find it in the questions. Knowledge is TOO vast Carefully manage exam time ALWAYS answer all items before exiting exam

TIP
Both questions and answers have catch words; look carefully for them
except least likely most likely all but never always all

Catch Words
Six-year-old Hillary DeLong is brought to the emergency room by her mother. Half an hour ago she was bitten on her right arm by a neighbors dog. The recommended first step in emergency treatment is to:
A.Thoroughly cleanse the area with soap and water B.Report the accident to the police C.Encourage free bleeding D.Cauterize the wound and suture it

Tip
Avoid unfamiliar choices These are made up by good test writers

Unfamiliar Choices
A pregnant woman at term with a 4-cm dilated cervix is found to have marginal placenta previa with mild bleeding. The appropriate management is:
A. B. C. D. Cesarean section Rupture of the membrane Internal podalic version Use of Willetts scalp traction forceps

Tip
Analyze similar answers carefully If one merely restates the other, both are wrong If one is the opposite of the other, one is correct If answers look similar, but have different numbers, one is correct

Similar Answers
The most appropriate treatment for erythrocytosis associated with polycythemia vera and a hemoglobin of 18.5 g/dL is:
A.Chlorambucil B.Phlebotomy to maintain hemoglobin at 14 g/dL in men and 12 g/dL in women C.Phlebotomy to maintain hemoglobin at 16 g/dL in men and 14 g/dL in women D.Radioactive phosphorus (P)
If answers look similar, but have different numbers, one is correct

Similar Answers
A 55-year-old woman has had pain and swelling of her left calf for 4 days while driving across the country. Physical examination shows slight swelling and tenderness of the left calf, but no other signs of deep venous thrombosis. She reports anaphylaxis after a previous injection of dye for intravenous pyelography. To promptly confirm the suspected deep venous thrombosis of the calf vein, you would now order: A.Impedance plethysmography B.Doppler ultrasound C.Radionuclide venography using 99m Tc macroaggregated albumin D.Radionuclide venography using 99m Tc-labeled erythrocytes

If one merely restates the other, both are wrong

Tip
For answers with numbers or percentages
Pick mid-range levels Pick values that look like the others

Numbers or Percentages
Implantation of the fertilized ovum usually occurs:
A.2 to 3 days following ovulation B.6 to 7 days following ovulation C.10 to 12 days following ovulation D.Exactly 14 days following ovulation

Numbers or Percentages
At the end of 28 weeks of gestation (7 lunar months), the fetus weighs (in grams):

A.15 B.600 C.1000 D.1800

TIP
Choose answers that are consistent with good family medicine values.

Values
Three times in the past month, a 32-year-old woman has arrived unexpectedly for consultation after hours at her family physicians office. She has also complained about the arrogance of the receptionist. The next time she arrives to see the doctor when he is working alone in the evening, he should:
A.Tell her family that there is nothing wrong with her and that she should see a psychiatrist B.Point out that she needs careful, thorough evaluation and give her the next available appointment during scheduled office hours C.Ask her to call the office the next morning D.Drive her out of the office

Values
In your urban family practice office, you encounter a Haitian patient with features of potential AIDS. Your best decision is to:
A.Immediately refer the patient to an AIDS clinic B.Send the patient to a hospital emergency room C.Provide concerned comprehensive and continuous care for the patient and the family D.Politely ask the patient to see the other family physician located on the next street

Study Notes/Charts/Fact Sheet Plan to stop studying 48 hours before the exam. Drive to the testing site before exam day. Plan exam day.

The Last Week Before the Exam

Plan Your Exam Day


Food
Can put in locker for break

Clothing
Comfort Look good, feel good, score good Lucky clothing

Identification 2 photo IDs, 1 governmentissued

The Exam Begins


Deep breaths; periodically pause and breathe Keep a positive attitude A? B? C? D? E? Read questions carefully

ABFP ABFM EXAMINEE Examinee

In the End
- Be Confident - Keep Calm - Plan to Study

Summary
Any high-quality, comprehensive study material can be used effectively Key ingredient in exam success is scheduling regular time to study Gaining points on the exam involves studying medical information, and there is a strong doseresponse relationship Use the In-Training Exams as a pre- and post-test Do not try to out-guess the item writers. Rely simply on your knowledge to respond to the selections

References
Rinaldo J. ABFM Guide for Exam Preparation. American Board of Family Medicine, 2006.
https://www.theabfm.org/cert/2009failsafe.pdf

Evaluate Todays Sessions

www.aafp.org/reportcme/boardrev/indianapolis

Board Review Express March 13-16, 2014 Indianapolis, Indiana


Day 2 - Friday, March 14
7:00 - 8:00 am 8:00 - 8:30 am 8:30 - 9:00 am 9:00 - 9:30 am 9:30 - 9:45 am 9:45 - 10:00 am 10:00 - 10:30 am 10:30 - 11:00 am 11:00 - 11:30 am 11:30 - 12:00 pm 12:00 - 12:15 pm 12:15 - 1:15 pm 1:15 - 1:45 pm 1:45 - 2:15 pm 2:15 - 2:45 pm 2:45 - 3:15 pm 3:15 - 3:30 pm 3:30 - 3:45 pm 3:45 - 4:15 pm 4:15 - 4:45 pm 4:45 - 5:15 pm 5:15 - 5:45 pm 5:45 - 6:00 pm 6:00 pm Breakfast Provided Pulmonary Infections & Pneumovax - Dana King, MD, FAAFP Musculoskeletal Medicine - Joseph Garry, MD, FACSM, FAAFP COPD, Lung Cancer, OSA - Dana King, MD, FAAFP Q&A Break Review of the Diseases of the Upper GI Tract - David Weismiller, MD Fracture Care in Family Medicine - Joseph Garry, MD, FACSM, FAAFP Asthma: Pediatrics & Adult - Dana King, MD, FAAFP Sports Medicine - Joseph Garry, MD, FACSM, FAAFP Q&A Lunch Review of the Diseases of the Lower GI Tract - David Weismiller, MD Pediatric Orthopedics - Joseph Garry, MD, FACSM, FAAFP Common Newborn Issues - David Weismiller, MD, ScM, FAAFP Management of Chronic Pain - Gary Levine, MD, FAAFP Q&A Break Managing Common Cutaneous Problems Part 1 - Gary Levine, MD, FAAFP Well Child Care and Adolescent Issues - David Weismiller, MD, ScM Managing Common Cutaneous Problems Part 2 - Gary Levine, MD, FAAFP Selected Issues in Women's Health - David Weismiller, MD, ScM, FAAFP Q&A Recess

Adult Pulmonary Infections


Dana E. King, MD, MS, FAAFP Professor and Chair Department of Family Medicine West Virginia University School of Medicine Morgantown, West Virginia

Disclosure Statement
Dr. King has nothing to disclose.

The AAFP has selected all faculty appearing in this program. It is the policy of the AAFP that all CME planning committees, faculty, authors, editors, and staff disclose relationships with commercial entities upon nomination or invitation of participation. Disclosure documents are reviewed for potential conflicts of interest and, if identified, they are resolved prior to confirmation of participation. Only those participants who had no conflict of interest or who agreed to an identified resolution process prior to their participation were involved in this CME activity.

Learning Objectives
1. Discuss the diagnostic and risk factor approaches to common types of pneumonia. 2. Be able to describe empiric therapy of pneumonia for outpatients and inpatients. 3. Explain the use and ACIP recommendations for pneumonia and influenza vaccines. 4. Manage a patient with positive test for TB.

Community Acquired Pneumonia (CAP)


8th leading cause of death in the United States (along with influenza), 6th leading cause in those > 65 yrs of age Over 55,000 deaths annually 1st globally in children 1.25 million cases of hospitalization for pneumonia 67% of those > 65 received pneumonia vaccine

Community Acquired Pneumonia (CAP)


5.6 million cases occur annually
18-20% require hospitalization 60% of these are > 65 yrs old Many now classified as having HCAP due to NH or frequent contacts with health care environments (dialysis, home infusions, repeated hospitalizations)

Mortality rate is 1-5% for those treated as outpatients


Up to 12-25% for those requiring hospitalization Up to 30-50% when ICU is needed

Pneumonia
Identification of specific pathogen is difficult
None is identified in one third to one half of patients, even with the most rigorous work-ups

Initial treatment is usually empiric Up to 50% may be viral

Pneumonia Guidelines
Implementation of CAP guidelines have shown several outcome improvements
Reduced costs Reduced length of stay Reduced in-hospital mortality Reduced number of days on mechanical ventilation

Pneumonia Studies
Study by McCabe et al (> 54,000 pts from 113 community-based hospitals and tertiary centers) 65% received initial guideline-concordant Rx; associated with decreased
In-hosp mortality (OR 0.7) Sepsis (OR 0.83) Renal failure (OR 0.79) Length of stay & parenteral Rx by 0.6 days (p < 0 001 for both)

These findings were linked to Rx with a fluoroquinolone or a macrolide

Pneumonia Guidelines
First in 1993 by the ATS and Canada Most recently in 2007 by the ATS/IDSA
Currently being updated in 2014

Medicare has developed performance measures for CAP that all hospitals must follow and that are publically reported

Hospital Admission Decision


Tools for identifying patients with CAP who may be candidates for hospitalization
Severity-of-illness score: CURB-65
Confusion Urea (BUN > 20) RR > 30 b/min BP sys < 90 mm Hg Age > 65

1 point each, 2 points or more is risky

CURB-65 Severity Scores for CAP


Clinical Factors Points Confusion 1 Blood urea nitrogen (BUN) > 19 mg/dL 1 Respiratory rate > 30 breaths/min 1 SBP < 90 mm Hg 1 DBP < 60 mg Hg 1 Age > 65 1 Total Points ?
If BUN is reported in mmol/L then the cut off is > 7 mmol/L

CURB-65 Severity Scores for CAP


CURB-65 Score Death %
0 1 2
_____

Recommendations
Low risk; consider home treatment

0.6 2.7 6.8

Short inpatient hosp or closely supervised outpatient treatment


Severe pneumonia; hospitalize, consider ICU

3 4 or 5

14 27.8

1. A 68-year-old woman presents to your office with cough, fever (T=102), and rales in the left base. Her BUN is 22, her BP is 130/96, RR=28, and her mentation is clear. Her CURB-65 score is?
A. B. C. D. E. 1 2 3 4 5

PSI
PSI (pneumonia severity index)risk calculator online, more complex http://pda.ahrq.gov/clinic/psi/psicalc.asp

ICU Admission Decision


Patients with either of 1 of the major criteria or 3 of the minor criteria
Major criteria
Minor criteria
Need for mechanical ventilation Septic shock PaO2/FiO2 ratio < 250 RR > 30/min Confusion Multilobar infiltrates SBP < 90 mm Hg despite fluid resuscitation BUN > 20 mg/dL Leukopenia (< 4000 cells/mm3) Thrombocytopenia (< 100,000 cells/mm3) Hypothermia (< 36 degrees Celsius) Hyponatremia (< 130 mEq/L) Arterial pH < 7.3

2. A 18 yo WM presents to the ED with a fever of 101.5 F, productive cough for the past 3 days. His parents smoke in the home and car.

What is the most likely pathogen causing his illness?

A. B. C. D.

Staphylococcus aureus Mycoplasma pneumoniae Streptococcus pneumoniae Chlamydia pneumoniae

Empiric Approach to Treatment


Empiric approach is used, treating for the epidemiologically most common organisms Rapid tests are not yet comprehensive enough and reliable enough to direct therapy to specific pathogens
Thorax, van der Eerden

Community Acquired Pneumonia (CAP)


Most common pathogens
Streptococcus pneumoniae 20-60% Mycoplasma pneumoniae 1-40% Chlamydia pneumoniae 4-10% Legionella 2-10% Haemophilus influenzae 3-10% Moraxella catarrhalis 1-5 % Virus & anaerobes 2-15%

Community Acquired Pneumonia (CAP)


Less common pathogens
Staphylococcus Gram negative bacilli (3-10%) Pneumocystis Mycobacterium tuberculosis Other causes Aspiration (6-10%)

Modifying Factors
Pediatric patients
Age 4 mo to 4 yrs
Most common pathogen
RSV

Peak incidence
2-7 mo of age

Age 5-15 yrs


Most common pathogen
Mycoplasma pneumoniae

Treat with a macrolide

Modifying Factors
Increased risk for drug-resistant streptococcus pneumoniae (DRSP)
Age > 65 (OR = 3.8) Beta-lactam Tx in last 3 mo (OR = 2.8) ETOH abuse (OR = 5.2) Immunosuppressive illness Multiple medical comorbidities
Chronic heart, lung, liver, or renal dz, DM, malignancies, asplenia, immunosuppressed conditions

Exposure to children in a day care center

Treatment Guidelines (2007 ATS)


OutpatientIf healthy, newer macrolide or doxycycline (no QT prolongation) Outpatient with co-morbidities respiratory quinolone, or a beta-lactam (amox, or with clavulanate) PLUS a newer macrolide

Treatment Guidelines (2007 ATS)


Inpatientnon-ICU, respiratory quinolone, OR a beta-lactam (ceftriaxone, amp-sulbactam, ertapenem) PLUS a newer macrolide ICUResp quinolone, plus anti-pneumobeta-lactam or aztreonam Or use anti-pseudomonal (pip-tazo or meropenem) plus quinolone plus vanc

DRSP
In the US, most penicillin resistance is intermediate and not highly resistant Definitions
Sensitive = MIC < 2 mg/L Intermediate = MIC of 4 mg/L Resistance = MIC > 8 mg/L

DRSP
Macrolide resistance is increasing in frequency
Upward of 40% Due to an efflux mechanism that may not be significant clinically because local concentrations at respiratory sites of infection may be adequate for effective therapy In Europe it is due to inability to bind at the ribosomal site of action, thus clinically significant

Quinolone resistance is uncommon Repeated doses of a given agent in the same patient increases risk for all classes of agents
Therefore pts should not receive the same agent if given in the past 3 months

CAP Diagnosis/Work-up
Thorough history and physical
Constellation of suggestive clinical features
Cough 90% Sputum production 66% Dyspnea 66% Pleuritic chest pain 50% Fever Malaise

Smoking history H/o comorbidities


Chronic heart, lung, liver, or renal disease DM Alcoholism Malignancies Asplenia Immunosuppression Previous antibiotics within 3 mo

CAP Diagnosis/Work-up
CXR
Demonstrable infiltrate False negative (repeat in 24-48 hrs if suspect clinically) Dehydration Elderly patients

CBC

CAP Diagnosis/Work-up
Sputum for gram stain and culture (SOR B)
Done prior to Rx only if good quality and rapidly processed in a microbiology lab Not sensitive but fairly specific May be useful to exclude Staph. aureus or gram negative rods

Blood cultures (x 2) (SOR A)


Most helpful in those with severe CAP
Collected prior to antibiotic Rx

Positive in 5-14% of cases S. pneumoniae accounts for 2/3 of positive cultures

CAP Diagnosis/Work-up
Those with severe illness and who have failed outpt Rx
Legionella, TB tests, and pneumococcal urinary antigen testing (UAT) should be done

If intubated
Endotracheal aspirate should be sent for culture

CAP Diagnosis/Work-up
Vital signs and mental status
Screening with pulse oximetry

Physical findings
Examination of lungs
Rales Bronchial breath sounds

Consider NP swab for influenza A & B (when in proper season)


Rapid testing is indicated if the Dx is uncertain

3. 55-yr-old WM with a 4 day history of productive cough, fever up to 102.3 F and a CXR showing a RLL infiltrate. PMH: smoker x 20 yrs, received a Z-pak 4 months ago for sinusitis. The appropriate choice for outpatient treatment is?

A. B. C. D.

Macrolide plus a beta-lactam Macrolide 3rd generation cephalosporin Beta-lactam if post influenza

4. 81-yr-old woman presents from the NH with a 2 day h/o increased confusion and poor po intake. She has had a productive cough for about 6 days and had been started on amoxicillin/clavulanate 3 days ago without improvement. VS: BP 85/50, RR 30, T 100.8, HR 120 and a CXR: bilateral LL infiltrates. True statements include all of the following except? A. The pt should be hospitalized for IV antibiotics B. She is at lower risk for drug resistant pneumonia C. She may have resistant organisms, so coverage could include pap-tazo plus cipro D. A combination of antibiotics is generally indicated

Principles to Guide Therapy


Give 1st dose rapidly and before leaving ED All pts should be treated for atypical pathogens and pneumococcus
Plus other pathogens based on risk factors

Monotherapy with macrolides should be limited to pt with no cardiopulmonary disease or recent antibiotics Anti-pseudomonal Rx should be used for pts with pseudomonal risk factors MRSA Rx should be used if increased risk No ICU admit should receive monotherapy

Treatment
Outpatient initial empiric therapy Previously healthy No risk for DRSP infection
Macrolides (SOR A)
Azithromycin Clarithromycin Erythromycin or

Doxycycline (SOR B)

Treatment
Outpatient initial empiric therapy Comorbidities present
Respiratory fluoroquinolone (SOR A)
Moxifloxacin, gemifloxacin, or levofloxacin

Beta-lactam plus a macrolide (SOR A)


High-dose amoxicillin (1gm tid) or Amox/clavulanate (2g bid) (preferred) Alternatives include ceftriaxone, cefpodoxime, cefuroxime (500 mg bid) Doxycycline can be an alternative to the macrolide

Treatment
Inpatient (Non-ICU) Respiratory fluoroquinolone (IV or PO) (SOR A) Or Beta-lactam (IV or IM)
Cefotaxime, ceftriaxone, and ampicillin/sulbactam Ertapenem for selected pts

Plus Macrolide (IV or PO) (erythro, clarithro, azithro)


Doxycycline maybe substituted (SOR C)

Treatment
Inpatient (ICU) Beta-lactam (IV)
Cefotaxime, ceftriaxone, and ampicillin/sulbactam

Plus Fluoroquinolone (SOR A) If PCN allergic Fluoroquinolone and aztreonam are recommended

Treatment
Inpatient (ICU) (SOR C)
**For Pseudomonas infections Anti-pneumococcal/Anti-pseudomonal betalactam (IV)
Piperacillin/tazobactam, cefepime, imipenem, or meropenem

Plus either a Fluoroquinolone (IV) (levofloxacin, ciprofloxacin)


or

An aminoglycoside
and

An antipneumococcal fluoroquinolone If PCN allergic substitute aztreonam for the betalactam

HCAP
(Health Care Associated Pneumonia)
Defined as a pt with prior hospitalization, NH residents, or immunocompromised state Treat differently from CAP More common pathogens:
MRSA S. pneumoniae P. aeruginosa MSSA (methicillin-sensitive S. aureus) H. influenzae

Hospital mortality is 12-23% BIRP criteria (pts at high risk for drug-resistant pneumonia)
B: broad spectrum antibiotics within past 3 mo I: ICU admission R: resident of a NH or poor functional status P: prior hospitalization within past 3 mo

HCAP
(Health Care Associated Pneumonia)
Nursing Home resident Antipseudomonal cephalosporin Cefepime, ceftazidime Or Antipseudomonal carbapenem Imipenem, meropenem Or Beta-lactam/Beta-lactamase inhibitor Piperacillin/tazobactam Plus Antipseudomonal fluoroquinolone Levo, cipro Or Aminoglycoside Gentamicin, tobramycin and Consider Anti-MRSA agents Vancomycin, linezolid

Treatment
Reduce time to first antibiotic dose (SOR B)
Admission through the ED
Given while in the ED

Switching from IV to PO (SOR B)


Hemodynamically stable Improving clinically Able to take PO and normal functioning GI tract

Treatment
Discharge (SOR B)
Clinically stable No other active medical problems Safe environment for continued care Inpatient observation is not necessary while receiving PO Rx

When to Stop Treatment


CAP pts should be treated
For a minimum of 5 days (SOR A) Until afebrile for 48-72 hrs Until no more than 1 CAP associated sign of clinical instability (SOR B)
Temp < 37.8 C HR < 100 BPM RR < 24 BrPM SBP > 90 mm hg O2 Sats > 90% pO2 > 60 mm Hg on Rm air Ability to maintain oral intake Normal mental status (Halm and Fine, JAMA)

Tx for Drug-Resistant S. Pneumoniae (DRSP)


Incidence of DRSP is stabilizing
Resistance to PCN and cephalosporins is decreasing Resistance to macrolides is increasing
> 25% in some areas

For CAP that is a MRSA infection


Add Vancomycin or linezolid (SOR C)

CAP That Is a MRSA Infection


Severe CAP, particularly after an episode of influenza, may need coverage for S. aureus including MRSA Linezolid with rifampin Vancomycin alone may not be adequate
It is not active against the PVL toxin that accompanies CA MRSA Add clindamycin to vancomycin or Linezolid

5. Which of the following does not need the pneumococcal vaccine?


A. A healthy 2-month-old infant

B. A 70-year-old woman who had pneumococcal vaccine 5 years ago


C. A 45-year-old female who smokes cigarettes and has not previously received the pneumococcal vaccine D. A 56-year-old male with chronic renal failure who received the pneumococcal vaccine at age 50 E. A 65-year-old male who is uncertain of his immunization status

Pneumococcal Vaccine
S. pneumoniae causes:
19,000 preventable deaths per year (pneumonia, bacteremia, meningitis) 7 million cases of otitis media per year

Polyvalent vaccine
23 serotypes that cause 80% of invasive pneumococcal disease in US B-cell response 96% drop in pneumonia caused by susceptible strains

PCV-13 (replaces PCV-7)


T-cell response

Pneumococcal Vaccine
Prevnar 13 (SOR A)
Decreases hospitalizations, costs, and invasive disease Primary series 2, 4, 6 months, booster 12-15 months Catch up: first dose at
7-11 months: give 2 doses and booster 8 wks later 12-23 months: 2 doses 8 wks apart 2-5 years: one dose for healthy, unvaccinated children 2-5 years: two doses 8 wks apart for at-risk children

If recd PCV-7 series, in children 14-59 months give one supplemental dose of PCV-13 http://www.aafp.org/dam/AAFP/documents/patient_care/im munizations/child-immunization-schedule.pdf

Invasive Pneumococcal Disease (IPD)


Children < 5 y continue to develop IPD Because patients who have received only the PCV 7 vaccine remain at risk for IPD caused by serotypes unique to PCV 13, CDC is urging health care professionals to
Review patients immunization records Provide patients ages 14-59 months who have completed the full PCV 7 series a supplemental dose of PCV 13 Use PCV 13 rather than PCV 7, even if office supplies of PCV 7 are not exhausted.

Polyvalent Vaccine (PPSV) 23


Single dose at age > 65 years Children at risk 2 yrs give at least 8 wks after last PCV 13 Indications for single dose for those 2-64 years of age:
Chronic cardiac disease (especially cyanotic congenital and failure) Cirrhosis, chronic liver disease, alcoholism Cochlear implants, cerebrospinal fluid leak Diabetes Chronic lung disease, asthma, smoker Residents of chronic care institutions

Indications for 2 doses 3-5 years apart ages 2-64


Chronic renal disease (renal failure and nephrotic syndrome) Asplenia, sickle cell* Immunocompromised (HIV, congenital, leukemia/lymphoma, multiple myeloma, drugs or radiation, organ transplant)

2nd dose = more local site reactions

Pneumococcal Vaccine
If > age 19 with
Immunocompromised patient Functional or anatomic asplenia CSF leaks Cochlear implants

Give PCV 13 one or more years after the last PPSV 23, OR If PCV 13- and PPSV 23-nave, receive single dose of PCV 13 followed by a dose of PPSV 23 at least 8 weeks later http://www.aafp.org/dam/AAFP/documents/patient_ca re/immunizations/adult-immunization-schedule.pdf

6. A 49-yo African American female with diabetes who is scheduled to start working in the ICU had a PPD placed 48 hours ago. She has never been tested previously, but her mother moved in with her a year ago and had a sister who died of TB as a child. What is considered a positive test in this patient?
A. B. C. D. Redness > 10 mm in diameter Induration and redness > 5 mm in diameter Induration > 10 mm in diameter Induration > 10 mm and redness > 15 mm

Testing for M. Tuberculosis Infection


Mantoux tuberculin skin test (TST)
Skin test that produces delayed-type hypersensitivity reaction in persons with M. tuberculosis infection

Interferon-Gamma Release Assays (IGRAs)


QuantiFERON Gold (QFT-G), T-SPOT, QFT-GIT (Gold In-Tube), TB Quant Blood test that measures and compares amount of interferon-gamma (IFN-) released by blood cells in response to antigens

IGRAs
CDC recommends that IGRAs can be used in all circumstances in which the TST is currently used, including contact investigations Can be used in place of the TST A positive test should prompt the same evaluation and management as a positive TST NO reason to follow a (+) IGRA with a TST Limitations
Must be processed within 8-16 hours Limited data on
Children < 5 yrs of age Recent exposed to TB Immunocompromised persons Those who will be tested repeatedly (serial testing)

Selecting a Test
IGRAs
Require a single visit Dont cause booster phenomenon Less subject to reader bias Unaffected by BCG and most environmental mycobacteria Preferred method for
Groups of people who have poor rates of returning to have a TST read Persons who have received BCG vaccine

TST
Preferred method for
Children < 5 yrs of age

Either may be used without preference for other groups that are tested for LTB

Reading a TST
Measure reaction in 48 to 72 hours Measure induration, not erythema Record reaction in millimeters, not negative or positive Ensure trained health care professional measures and interprets the TST
Positive TST reactions can be measured accurately for up to 7 days Negative reactions can be read accurately for only 72 hours

TB Skin Testing
> 5 mm is considered positive if:
HIV sero-positive Recent TB direct contact CXR shows prior inactive TB Immunosuppressed patients
Prednisone > 15mg/day Organ transplant recipients

TB Skin Testing
> 10 mm is considered positive if:
Diabetic Renal failure Cancer Immigrant < 5 yrs High-prevalence area
Long-term care facility
Resident or employee

Inmate IV drug user Children < 4 yrs of age Mycobacteriology lab personnel

TB Skin Testing
> 15 mm is considered positive if:
Any person with no known risk factors
even if prior BCG vaccination

Booster Phenomenon
Some people infected with M. tuberculosis may have a negative reaction to the TST if many years have passed since they became infected. They may have a (+) reaction to a subsequent TST because the initial test stimulates their ability to react to the test.
This may incorrectly be interpreted as a skin test conversion

The two-step test is indicated

Two-Step Skin Testing


Testing of persons who will periodically receive TSTs (eg, health-care workers or residents of long-term-care facilities) to reduce the likelihood of mistaking a boosted reaction for a new infection.

Two-Step Skin Testing


If an initial TST result is classified as negative, a second test is repeated 13 weeks later. If the first test is positive, consider them infected. If the second TST is also negative, the person is classified as not being infected. If the reaction to the second TST is positive, it probably represents a boosted reaction, indicating that the infection was most likely in the past and not recent.
If they have never been treated in the past, this patient should be evaluated and treated accordingly

PositiveTST or QFT-G
Check CXR for active disease Negative
INH Rx for 6-9 months (9 mo is preferred) Daily or intermittently (twice weekly) Use directly observed therapy (DOT) for intermittent regimen Rifampin Rx daily for 4 months Consider adding Pyridoxine Monthly exams for signs of hepatitis and medication adherence, check liver transaminases if indicated

PositiveTST or QFT-G
Check CXR for active disease
Positive
10 drugs are currently approved First-line agents form the core of Rx:
Isoniazid (INH) Rifampin (RIF) Ethambutol (EMB) Pyrazinamide (PZA)

Typically treated with several drugs for 6-12 months

Laboratory Testing
Routine baseline transaminases are not necessary Recommended for pts with any of the following
Liver disorders H/o liver disease (Hep B or C, alcohol hepatitis or cirrhosis) Regular use of alcohol Risks for chronic liver disease HIV infection Pregnancy or immediate postpartum period (3 months)

Consider if on meds for chronic conditions Periodic retesting only for those with abnormal initial results or those at risk for hepatic disease, or anytime pts develop Sxs of hepatitis

PositiveTST or QFT-G
Treatment continued:
Initial phase of 2 mo Continuation phase of either 4 or 7 mo

Treatment completion is determined by the number of doses ingested over a given period of time.
Basic TB regimens are broadly applicable; modifications should be made for:
HIV infection Drug resistance Pregnancy Children

Active TB Preferred Rx Regimen


Initial phase
INH, RIF, PZA, EMB x 8 wks (56 doses)
EMB can be discontinued if TB is susceptible to first-line drugs

Continuation phase
INH and RIF for 16 wks (126 doses) Or Twice wkly for 18 wks (36 doses)

Post-Treatment Follow-Up
Patient should receive documentation of
TST or IGRA results Radiograph results Dosage and duration of medication

Present this document any time future testing is required Re-educate pts about signs and Sxs of TB Regardless of whether Rx for LTBI was completed, serial or repeat CXRs are not indicated unless signs or Sxs of TB develop

Pneumonia Guidelines Refs


Infectious Diseases Society of America/American Thoracic Society Consensus Guidelines on the Management of CommunityAcquired Pneumonia in Adults
Clinical Infectious Diseases. 2007;44:S27-S72.

Evidence-based guidelines Graded scientific evidence Strength of recommendations

Answers
1. 2. 3. 4. 5. 6. B B A B B C

Musculoskeletal Medicine: Common Orthopedic Problems


Joseph Garry, MD, FAAFP, FACSM University of Minnesota

Disclosures
Dr Garry discloses that he holds stock in Pfizer and Merck pharmaceuticals.

It is the policy of the AAFP that all individuals in a position to control content disclose any relationships with commercial interests upon nomination/invitation of participation. Disclosure documents are reviewed for potential conflicts of interest. If conflicts are identified, they are resolved prior to confirmation of participation. Only participants who have no conflict of interest or who agree to an identified resolution process prior to their participation were involved in this CME activity.

Learning Objectives
1. Cite strong indicators for immediate imaging of the lumbar spine. 2. Know the evidence-based management for low back pain. 3. Cite the risk factors and management of overuse injuries (tendinopathy, shoulder pain, carpal
tunnel syndrome).

4. State management of 3 common foot problems


(plantar fasciitis, Mortons neuroma, tarsal tunnel syndrome).

Most Common Causes of LBP in Adults

Strain or sprain
Degenerative

70%
10%

80%

Discogenic
Osteoporotic fracture

4%
4%

Spinal stenosis
Spondylolisthesis

3%
2%

Other includes neoplasia, pelvic etiology, abdominal etiology, etc.

Question 1.

All of the following are red flags suggestive of early imaging of the lumbar spine except

A. Urinary retention B. Renal cancer

C. Back pain at rest


D. Urinary incontinence

LBP Imaging Red Flags


History Findings
Cancer metastatic to bone
(breast, lung, thyroid, renal, prostate)

Exam Findings
Major motor weakness or sensory loss Saddle anesthesia Loss of anal sphincter tone Fever, urinary tract infection, or wound in spine region

Urinary or fecal incontinence Urinary retention Progressive lower extremity motor or sensory loss Significant trauma related to age Severe pain and lumbar spine surgery in the prior 12 months

These are all STRONG findings and the presence of a STRONG finding indicates need for imaging.

Summary of the American College of Physicians Best Practice Advice:

Diagnostic Imaging for Low Back Pain


Indications for Diagnostic Imaging
Immediate imaging recommended for acute LBP with major risk factors for or signs of cauda equina syndrome or severe progressive neurological deficits. Imaging after trial of therapy for those with minor risk factors for cancer, risk factors for inflammatory back disease, risk factors for VCF, signs or symptoms of radiculopathy, or risk factors for or symptoms of symptomatic spinal stenosis. Repeated imaging is only recommended in patients with new or changed low back symptoms.
Ann Intern Med. 2011;154(3).

Summary of the American College of Physicians Best Practice Advice:

Diagnostic Imaging for Low Back Pain


Evidence that expanding imaging to those without indications does not improve outcomes
RCTs of routine imaging vs usual care without routine imaging in patients without indications for diagnostic imaging suggest no clinically meaningful benefits on outcomes related to pain, function, quality of life, or mental health. Other supporting evidence includes the weak correlation between most imaging findings and symptoms, the favorable natural history of acute LBP with or without imaging, the low prevalence of serious or specific underlying conditions, and unclear effects of imaging on treatment decisions.
Ann Intern Med. 2011;154(3).

LBP Imaging
Adult with acute LBP without red flags Suspect degenerative changes/sprain/strain 4-6 weeks of treatment/therapy
Symptoms improve Stop imaging Symptoms continue (XR)
Neuro deficit No neuro deficit

MRI

Stop imaging
Humphreys et al. Am Fam Physician. 2002.

Question 2.

A 43-yo male was moving a dresser 2 days ago and felt pain in his low back. Pain persists and radiates to left buttock. No radiation of symptoms below the level of the knee. Which of the following recommendations have been shown to be helpful for recovery?

A. Bed rest until pain resolves B. Use of an NSAID C. Referral to back school D. Early imaging
BMJ www.clinicalevidence.com Griffin G. J Fam Pract. (POEMS), 2000.

Acute LBP
Beneficial Therapies
NSAIDs (A) Advice to remain active (A)

Likely Beneficial Therapies


Muscle relaxants weigh side effects (B) Epidural steroids (diskogenic pain) Physical Therapy directed home exercise program

Choi et al. Coch Database of System Rev 2010; Gelhorn et al. Spine 2010; Neurology 2007, 68(10); Cochrane Database of System Rev 2004, 2003; Griffin G., J Fam Pract (POEMS), 2000

Question 3.

All of the following are beneficial in the treatment of chronic LBP to reduce pain and improve function except
A. Tramadol B. Oxycontin C. Exercise D. Massage

Chronic LBP
Beneficial Therapies

Pain -

Function

Pain
Antidepressants (B) NSAIDs (B) Analgesics* (B) (narcotic analgesics) Back school (B) Manipulation (B) Harpagophytum procumbens (devil's claw) Salix alba (white willow bark)

Intensive multidisciplinary treatment program (B) Tramadol* (B) Exercise (B) Massage (A) Behavioral therapy (B)

AFP 2009, 79(12); Malanga et al. Spine 2008; Cochrane Database of System Rev 2007, 2005; Gagnier et al. Cochrane Database of System Rev 2006

A Comparison of the Effects of 2 Types of Massage and Usual Care on Chronic Low Back Pain: A Randomized, Controlled Trial, Cherkin et al. Ann Intern Med. 2011;155(1):1-9.

RCT comparing massage (structural n = 132, relaxation n = 136) to usual care (n = 133) for chronic LBP in 20-65 yo patients; weekly massage (50-70 min) x 10 weeks; benefits of massage persisted for 26 wks.

Mean Roland Disability Questionnaire (top) and symptom bothersomeness (bottom) scores.

Copyright The American College of Physicians. All rights reserved.

Psychosocial Factors Predicting

Long-Term Disability in Chronic LBP


Affect
Anxiety, depression, feeling of uselessness

Behavior
Adverse coping strategies, impaired sleep, passive

Beliefs
Pain is harmful and must be eliminated

Social
History of abuse (physical/sexual/drug), lack of support

Work
Expect pain will increase with work, pending litigation
AFP. 2009, 79(12).

Question 4.

A 33-yo male presents with acute LBP after reaching down to load the dishwasher. The following day noted right leg pain and lateral foot paresthesias. At presentation, you find plantar flexor weakness in addition to positive dural tension signs. Your recommendation is

A. Go to bed for rest and return in 3 days. B. Start high dose prednisone as moderate to strong evidence supports this for symptom relief. C. Prescribe an upper lumbar ESI and if symptom relief then obtain an MRI. D. Consider surgical intervention despite outcomes which are similar to non-surgical care at 2 years.

Lumbar HNP

Conservative Treatment
80% of all adult LBP resolves within 90 days
(AHCPR Clinical Practice Guideline No. 14, 1994)

90% lumbar HNP starts to improve at 6 wks and resolve by 12 wks (Saal et al. Spine 1989) Avoid bedrest (Hagen et al. Coch Data Syst Rev 2004) Oral corticosteroids and NSAIDs have weak evidence for benefit (Holve et al. JABFM 2008; Roelors et al.
Coch Data Syst Rev 2008)

LESI provides benefit in terms of pain relief and preference for non-surgical treatment; mean=3.6 inj/yr
(Manchikanti et al. Spine 2011; Benyamin et al. Pain Physician 2012)

Lumbar HNP

Surgical Treatment
Indications
Severe progressive motor deficits, cauda equina syndrome, unremitting radiculopathy x 6 weeks

Spine Patient Outcomes Research Trial


Surgery provides greater improvements in pain and disability in the first two years vs nonsurgical treatment. Treatment effects are similar at >2 yrs Microdiskectomy as effective as open diskectomy
Gibson et al. Cochrane 2007 Weinstein et al. JAMA 2006 Pearson et al. Spine 2012

Evidence-Based Practice
Recommendations for Lumbar HNP
1. 2. 3. 4. Advice to remain active (A) Oral steroids and NSAIDs have limited benefit (B) ESI provide (short term) symptom improvement (A) If no red flags, then radiculopathy may be managed conservatively, without imaging, for up to 6 wks (A) 5. Radiculopathy not improving after 6 wks of conservative management may benefit from diskectomy for more rapid clinical relief (A) 6. Diskectomy has similar long-term outcomes as nonsurgical treatment (A)

Characteristics of Overuse Injury


Low contact activities/sports that involve long training sessions with repetitive motion Gradual onset and increase in symptoms make these difficult to identify early Consequences include loss of time in activity, reduced function, pain, and psychological exhaustion
Female athletes at greater risk
Yang et al. J Athletic Tng 2012

Question 5.

Which of the following statements is most correct regarding overuse injury?


A. Menstrual dysfunction is related to overuse injury B. Prior injury is not important in the etiology of overuse injury C. Chronic tendinopathies are characterized by inflammatory infiltrates D. Overuse injury is most commonly associated with a single traumatic event

Overuse Injury

Contributing Factors
Pediatric & Adolescent Susceptibility of growth cartilage Adolescent growth spurt Developmental level
Intrinsic Factors Prior injury Inadequate conditioning

Anatomic factors
Extrinsic Factors Training errors Technique

Menstrual dysfunction
Equipment Psychological factors

Overuse Injuries
Tendinosis

Non-inflammatory degenerative condition


Supraspinatus, epicondylosis, patellar, achilles

Etiopathogenic theory suggesting insufficient tensile strength of the tendon exposed to external loads which cause progressive damage

Question 6.

Which of the following statements regarding chronic tendinopathy (lateral epicondylosis, patellar tendinosis, achilles tendinosis) is most correct?
A. Corticosteroid injections provide long term relief for patellar tendinosis B. Corticosteroid injections provide long term relief for lateral epicondylosis C. Eccentric strengthening is beneficial for lateral epicondylosis and patellar tendinosis D. Concentric strengthening is more efficacious then eccentric strengthening in achilles tendinosis

Eccentric Strengthening
Lengthening of a contracting muscle Initial exposure to EC can result in muscle damage & delayed onset muscle soreness

Repeated EC results in protective adaptations in muscle


EC leads to greater muscle mass, strength, power Tendons demonstrate increased stiffness, greater force needed for failure, and improved ability to absorb force at the myotendinous junction

Eccentric Strengthening
References
Lateral epicondylosis Patellar tendinosis

9 wks of treatment, 3x per week resulted in tendon thickness, pain and disability, strength Meta-analysis of RCTs found EccS to be beneficial in treatment; step program or use of 25 decline board may have further benefit in treatment EccS demonstrates benefit over usual care, and concentric strengthening; 70% improve with EccS

Crosier et al. Brit J Sports Med 2007

Visnes et al. Br J Sports Med 2007; Young et al. Br J Sports Med 2005

Achilles tendinosis

Mafi et al. Knee Surg Sports Traumatol Arthrosc 2001; Silbernagel et al. Scand J Med Sci Sports 2001

Corticosteroid Injection
References
Lateral epicondylosis Patellar tendinosis

Beneficial for pain reduction for up to 8 weeks but inferior to other treatments for longer term Beneficial for pain reduction for up to 4-26 weeks with predictable relapse; failure to demonstrate tendon healing on imaging Beneficial for short term pain reduction which is lost in the intermediate term

Coombes et al. Lancet 2010

Van Ark et al. Brit J Sports Med 2011; Fredberg et al. Arthritis Rheum 1998

Achilles tendinosis

Fredberg et al. Scand J Rheumatol 2004

Posterior Tibialis Tendon


Figure from orthoinfo.aaos.org Acute or overuse injury Risk factors; female, > 40 years, obesity, diabetes, hypertension Pain with passive dorsiflexion and heel rise

(posterior tibialis muscle is a plantarflexor and invertor of the foot)

Treatment may extend 3-6 months Rest, ice, NSAIDs, may boot for 3-6 weeks, orthotics, PT, and eccentric strengthening

Shoulder Pain
Impingement
Age > 35 Overuse injury with an insidious onset of pain AC spurring & tendon degeneration Pain with overhead activity Supraspinatus tendon most commonly involved followed by infraspinatus

RC Tendonitis
Age < 35 Typically acute onset Pain with manual muscle testing Partial tears may be associated with night pain

Shoulder Pain
Management Therapies
Physical therapy
Beneficial for pain and function

Subacromial Injection
Improves abduction, pain

NSAIDs
Likely beneficial vs. placebo (B)

Extracorporeal shockwave therapy


ECSWT helpful in chronic shoulder pain with calcific tendonitis
BMJ. www.clinicalevidence.com

Cochrane 2009, 2008

Rotator Cuff Tears


Supraspinatus weakness Weakness in external rotation Positive impingement signs (Neers, Hawkins)
Tears more likely with increased age (> 60)

3 positive signs (any age), or 2 positive signs and age > 60 = 98% probability of RC tear
Murrell et al. Lancet 2001

Biceps Tendon Rupture


Long head of the biceps tendon most common Risk factors include: age, heavy overhead activity, shoulder overuse injury, smoking, corticosteroids Audible pop, bruising, weakness (supination) Surgical treatment debatable but consider for athletes, manual laborers, and those who require maximal supination strength

Question 7.

64-yo male with a 4-mo history of shoulder pain and LROM. No history of trauma or arthritis. Had pain for 2 mo and then noted onset of LROM. The best initial treatment for short-term pain relief & functional improvement includes which of the following?

A. Acetaminophen 650 mg po qid B. 3-4 weeks or oral prednisone C. Referral to physical therapy D. Reassurance and follow up in 2 months

64-yo male with 4-mo history of shoulder pain and LROM. No history of trauma or arthritis. Had pain for 2 mo and then noted onset of LROM. The best initial treatment for short-term pain relief & functional improvement includes which of the following? Adhesive capsulitis 3-4 weeks or oral prednisone (20 mg/d) was superior to PT or acetaminophen in improved function and reduced pain in the short term (1-2 months) Cochrane 2006
Ann Rheum Dis 2004

Adhesive capsulitis is associated with

diabetes, cardiac disease, thyroid


dysfunction, and Parkinsons disease.

Adhesive Capsulitis
PT/analgesics = mainstay of treatment despite limited evidence PT + IA steroids better than PT alone at 3 mo Oral steroids pain & function at 1 mo Efficacy of capsular distension is equal to manipulation
Arth Rheum 2003: 48(3), 829 Cochrane 2006, 2008

Stage I: painful" stage, 6 wks to 9 months, slow onset of pain. As pain worsens, shoulder loses motion. Stage II: "freezing" stage, marked by a slow improvement in pain, but persistent LROM, generally lasts 4-9 months. Stage III: The final stage is the "thawing", during which shoulder motion slowly returns toward normal. This generally lasts 5-26 months.

J Should Elbow Surg 2009

Enter question text... The diagnostic test of choice for carpal


tunnel syndrome is
A. Patient history

Question 8.

B. Paresthesias in the radial 2 digits


C. Nerve conduction velocities D. Ultrasound demonstrating compression of the median nerve

Carpal Tunnel Syndrome


Classic vs. Dynamic Associated with thyroid, diabetes, pregnancy Compression of the median nerve
Sensory radial 3 digits Motor intrinsic thumb, radial lumbricals

Flick test (most sensitive finding) Phalens test Monofilament testing 2-point discrimination Weak thumb abduction
DArcy et al. JAMA 2000 LeBlanc, AFP 2011

Carpal Tunnel Syndrome


Treatment Options
Wrist splints, ultrasound, yoga, carpal bone mobilization possibly beneficial in short term Corticosteroid injection beneficial at 1 & 4 mo, superior to oral corticosteroids Surgical release beneficial over splinting at 6 mo

Surgical release to prevent progression of symptoms as opposed to return to normal


Cochrane 2009

AFP 2011: 83(8)

Dupuytrens Contracture
Figure courtesy of orthoinfo.aaos.org Males > 40 years 4th finger most common Associated with northern European/Scandinavian ancestry, hereditary, smoking, alcohol, and diabetes Development of nodules to progressive fibrosis of the palmar fascia causing shortening and thickening with subsequent flexion contracture of the digit

CSI for painful nodules; splinting not helpful Collagenase injections Surgical management

Purulent Tenosynovitis
Flexor tendons most common
Kanavels signs
Slight digital flexion

Uniform volar swelling


Flexor tendon tenderness Pain with passive extension

S. aureus or Streptococcus
Antibiotics and surgical consultation

Question 9.

Risk factors for plantar fasciitis include all of the following except
A. Pes cavus foot structure B. Obesity C. Pes planus foot structure D. Normal ankle dorsiflexion

Plantar Fasciitis Is More Likely to Occur in Persons Who


Are obese Spend most of the day on their feet

Have limited ankle dorsiflexion, tight achilles


Have pes cavus or pes planus foot structure Participate in excessive running

JBJS (Am) 2003; 85-A, 872-77 J Foot Ankle Surg, 2010

Plantar Fasciitis
Heel pain
Worse with first steps after prolonged rest or first steps in the morning Tender at medial calcaneal tubercle

Cochrane 2009 JAAOS 2008

Foot & Ankle Intl 19(1): 1998

Plantar Fasciitis Management


1. Initiate patient directed therapies
Relative rest, ice massage, analgesics, stretching, weight loss

2. Initiate physician directed therapies


Physical therapy, stretching, deep myofascial massage
Orthotics (B) Night splint (B) CS injection (B) Autologous blood injection (B) ECSWT is an option for chronic recalcitrant cases (B)

Mortons Neuroma
Irritation, trauma or excessive force on the intermetatarsal plantar nerve 3-4 web space most common (8-10x) >> Burning pain in foot with toe numbness Palpable mass or click between metatarsals
(Mulders sign)

Treatment: shoes with wide toe box and low heels, orthotics, injection [80% improve with these measures]

Tarsal Tunnel Syndrome


Entrapment of the posterior tibial n. just posterior to the medial malleolus Provoked by subtalar pronation

Can be mistaken for plantar fasciitis


Nontender medial calcaneal tubercle

Electrodiagnostics may be useful in diagnosis (C)


Illustration courtesy of Steven Oh

Problems with false negatives

* Correct biomechanics and consider injection

DARE 2006

Answers
1. 2. 3. 4. 5. 6. 7. 8. 9. C B B D A C B C D

COPD, Lung Cancer, OSA


Dana E. King, MD, MS, FAAFP Professor and Chair Department of Family Medicine West Virginia University School of Medicine Morgantown, West Virginia

Disclosure Statement
Dr. King has nothing to disclose.

The AAFP has selected all faculty appearing in this program. It is the policy of the AAFP that all CME planning committees, faculty, authors, editors, and staff disclose relationships with commercial entities upon nomination or invitation of participation. Disclosure documents are reviewed for potential conflicts of interest and, if identified, they are resolved prior to confirmation of participation. Only those participants who had no conflict of interest or who agreed to an identified resolution process prior to their participation were involved in this CME activity.

Learning Objectives
1. Explain the common approach(es) to diagnosis and treatment of COPD. 2. Summarize risk stratification and treatment in patients with a COPD exacerbation. 3. State the USPSTF recommendation on screening for lung cancer. 4. Describe the diagnosis and treatment of obstructive sleep apnea (OSA). 5. Summarize the risks associated with untreated OSA.

COPDWhat Is It?
Key elements of COPD
Toxin exposure (primarily tobacco) Airway inflammation Airflow obstruction (not fully reversible) 3rd leading cause of death in the US
133,965 deaths in last reported year
Amer Lung Fact Sheet

COPD
Costs
Economic costs approx $49.9 billion, $29.5 billion direct medical costs

715,000 hospital discharges each year Prevalence


Estimated at 10.1% (BOLD study) 18.9% (Tinkelman et al) in high risk patients 24 million people have COPD

1. Which of the following is NOT associated with COPD?


A. It is the 3rd leading cause of death in the US B. Airway inflammation C. Second-hand smoke D. Fully reversible airflow obstruction

COPD
Hallmark Symptoms Less Commonly Reported Symptoms

Cough (85%) Increased sputum production (45%) Dyspnea, exertional (70%) Wheezing (40%) Exercise intolerance

Fatigue Edema Chest tightness Weight loss Increased nocturnal awakenings

Decreased quality of life

COPD Symptoms
Relationship between airflow obstruction and patient perception of symptoms is highly variable NHANES survey
Only 60% of pts with moderately reduced FEV1 (50-85% of predicted) complained of symptoms

Multicenter trial suggests that dyspnea may be a better predictor of mortality than spirometry

2. Which of the following are true of COPD?


A. B. C. D. Common symptom is dyspnea Weight gain is common Most frequent cause of death in men Alpha1-antitrypsin harms the lungs

Medical Research Council (MRC) Dyspnea Index


Grade
1 2

Level of Dyspnea
Not bothered by dyspnea, except during strenuous activity Shortness of breath when walking up a short hill

3 4
5

Walks more slowly than others because of breathlessness; stops to catch breath when walking at own pace Stops to catch breath after walking 100 m (328 ft) on level ground
Too short of breath to leave the house; breathless with activities of daily living, such as dressing and undressing

Stephens, et al. Diagnosis of Chronic Obstructive Pulmonary Disease, Am Fam Physician. 2008;78(1):87-92.

BODE Index
Variable 0 Points 1 2 3

FEV1 Distance walked in 6 min

>65 >350

Grade on the MRC 0-1 4-5 dyspnea scale BMI <21 >21 Note: BODE index scores range from 0-10 pts. The higher scores indicate a greater risk of death.
Stephens et al. Diagnosis of Chronic Obstructive Pulmonary Disease, Am Fam Physician. 2008;78(1):87-92.

50-64 250349 2

36-49 150249 3

<35 <149

Differential Diagnosis includes:


Asthma CHF Bronchiectasis Lung cancer Interstitial lung disease/pulmonary fibrosis Sarcoidosis Tuberculosis Bronchopulmonary dysplasia

3. Risk factors for developing COPD include all of the following except?
A. B. C. D. Coal mining for 25 yrs Family history of COPD Family history of asthma Alpha-1-antitrypsin deficiency

COPD
Primary Risk Factor Other Risk Factors

Cigarette Smoking

Advancing age

80% of lung cancer deaths are Secondhand smoke exposure directly attributable to smoking Family h/o COPD 12-13 times more likely to die Chronic exposure to from COPD than non-smokers environmental or occupational pollutants Absolute risk of COPD among Alpha-1-antitrypsin deficiency active continuous smokers is Infections at least 25% Infections Childhood history of recurrent respiratory infections

COPD Pathophysiology
Chronic airway irritation inflammation increased mucus production decreased mucociliary function increased coughing and sputum production smokers cough

Continued airway irritation scarring within the airways progressive airway obstruction dyspnea prompting medical attention This also predisposes patients to respiratory infections
Another reason to seek medical attention

Physical Findings
Normal in many patients Abnormal findings
Lung hyperinflation
Widened anteroposterior chest diameter Hyperresonance on percussion Diminished breath sounds

Persistent pulmonary damage may lead to increased right sided heart pressure cor pulmonale
Accentuated second heart sound Peripheral edema JVD Hepatomegaly

Predicting Airflow Obstruction


Single best variable for identifying adults is a hx of > 40 pack yrs of smoking Combination of findings is more helpful for diagnosing it than any individual sign, sxs, or hx Combination of all 3 of the following almost assures the presence of obstruction
> 55 pack yr hx Wheezing on auscultation Patient self-reporting of wheezing

Absence of all 3 practically rules it out

Physical Findings
Abnormal findings
Increased work of breathing
Use of accessory respiratory muscles Paradoxical abdominal movement Increased expiratory time Pursed lip breathing Wheezing (variable)

Cachexia Cyanosis

Physical Findings
Abnormal findings
Chronic weight loss (independent predictor of mortality)
BMI should be monitored

Clubbing (rare) should prompt search for other causes including:


Cancer Pulmonary fibrosis Bronchiectasis

4. A 62 yo wf presents with progressive cough, increased sputum production and SOB over the past 510 yrs, now to the point that she can not walk the length of her living room without stopping to rest. She has smoked 1-2 ppd for nearly 40 yrs, has a BMI of 36, always feels fatigued to the point that she rarely leaves the house. You suspect that she has severe COPD.
Which of the following results would indicate severe COPD? A. B. C. D. FEV1/FVC > 0.7, FEV1 50-79% predicted FEV1/FVC > 0.7, FEV1 30-49% predicted FEV1/FVC < 0.7, FEV1 30-49% predicted FEV1/FVC < 0.7, FEV1 < 30% predicted

Diagnostic Testing
Spirometry is the key test
Recommended for all symptomatic adults
NOT as a screen for asymptomatic pts regardless of COPD risk factors Asymptomatic is not precisely defined in the literature

Key features
FEV1
Volume of air expired in one second after a full inspiration

FVC
Maximum volume of air exhaled after a full inspiration

Not recommended to track disease progression or modify Rx after it has been initiated

Diagnostic Testing
A post-bronchodilator FEV1/FVC < 0.7 associated with an FEV1 < 80% of predicted value is diagnostic of airflow limitation and confirms COPD

Diagnostic Testing
CXR to evaluate for Lung nodules Masses Fibrotic changes **Annual lung CT scans in those with a > 30 pack yr, age 55-80 now recommended** USPSTF Dec 2013 AAFP statement is neutral

Diagnostic Testing
ECG and echocardiography
In patients with signs of cor pulmonale

Pulse oximetry at rest, with exertion, and during sleep


Evaluate the need for supplemental oxygen

COPD Staging (Based on Spirometry)


Stage 0 (at risk) I (mild) II (moderate) III (severe) GOLD --FEV1/FVC < 0.7 FEV1 > 80% predicted FEV1/FVC < 0.7 FEV1 50-79% predicted FEV1/FVC < 0.7 FEV1 30-49% predicted FEV1/FVC < 0.7 FEV1 < 30% predicted or < 50% with chronic sxs

IV (very severe)

Stephens, et al. Am Fam Physician. 2008;78(1):87-92.

COPD Staging (Based on Spirometry)


Stage 0 (at risk) ATS/ERS Smokers with sxs FEV1/FVC > 0.7 FEV1 > 80% predicted FEV1/FVC < 0.7 FEV1 > 80% predicted FEV1/FVC < 0.7 FEV1 50-79% predicted FEV1/FVC < 0.7 FEV1 30-49% predicted FEV1/FVC < 0.7 FEV1 < 30% predicted

I (mild) II (moderate) III (severe) IV (very severe)

Stephens et al. Am Fam Physician. 2008;78(1):87-92.

Goals of COPD Management


To reduce long-term function decline Prevent and treat exacerbations Reduce hospitalizations and mortality Relieve disabling dyspnea Improve exercise tolerance Improve health-related quality of life

COPD Management
Modify risk factors if possible
Stop smoking!
ASK about tobacco use at every visit ADVISE all users to stop ASSESS users' willingness to make an attempt to quit ASSIST users' efforts to quit ARRANGE follow-up

* Tobacco cessation and O2 Rx are the only interventions proven to prolong survival of patients with COPD*

COPD Management
Establish severity (spirometry) Assess need for pharm and non-pharm Rx Education plan based on pts specific needs Encourage exercise Immunization status monitoring
Pneumococcal Influenza (yearly)

Other behavioral changes

COPD Management
Symptomatic pts with FEV1 < 60% predicted, benefit from inhaled Rx
Anticholinergics LABA ICS

Asymptomatic pts
Evidence doesnt support treating regardless of the presence or absence of airflow obstruction or risk factors

COPD Management
Mild disease (FEV1 > 80%)
SABA q 2-6 hrs prn (SOR B)

Moderate disease (FEV1 50-79%)


LABA or tiotropium (SOR B) If not effective, trial of ICS, continue only if effective (SOR B)

COPD Management
Severe disease (FEV1 30-49%)
Oxygen Pulmonary rehab All previously mentioned meds
Theophylline in severe disease (SOR C)

Lung volume reduction surgery Transplant

Inhaled Rx on Long-Term Decline in Lung Function


Pooled results for 9 long-term trials demonstrated Inhaled Rxs reduced the annual decline in mean FEV1 more than placebo
LABA, ICS, or combination LABA and ICS

Inadequate evidence to predict which pts will have the best response, or whether mono or combination will work best for them

Inhaled Rx on Long-Term Decline in Lung Function


Monotherapy trials report no significant change in the annual rate of FEV1 decline assoc with use of Tiotropium, ICS, and LABA

5. A 75 yo white male with a h/o COPD is complaining of a 3-day h/o rhinorrhea, increased coughing productive of clear sputum, wheezing, and O2 sats on RA of 91%. Which of the medications will be the least effective for him?

A. B. C. D.

Systemic steroids Antibiotics Short-acting bronchodilators Inhaled steroids

Classification of COPD Exacerbations by Severity


Severity Mild Description Can be controlled with an increase in dosage of regular medications Requires treatment with systemic corticosteroids or antibiotics

Moderate

Severe

Requires hospitalization or evaluation in the emergency department

Adapted from: Evensen AE. Management of COPD Exacerbations. Am Fam Physician. 2010;81(5):607-613.

Acute COPD Exacerbations


Oximetric evaluation is mandatory
O2 sats 80-90% on RA
Titration with O2 to 90% with little concern for hypercarbia

ABG evaluation
O2 sats < 80% on presentation Admission if pH < 7.32
Secondary to risk of respiratory failure

Home O2 may be needed if sats < 90% with ambulation and outpatient management decided

Acute COPD Exacerbations


Antibiotics should be given to: Exacerbation of COPD with all three symptoms (increased dyspnea, increased sputum volume, and purulence) (SOR B) Exacerbations of COPD with only two of the symptoms, if one of them is increased purulence of sputum If hospitalized for COPD and mechanical ventilation (invasive or non-invasive) is required

Acute COPD Exacerbations


Antibiotics
Not needed if cough is clearly post viral Warranted if a prolonged illness with purulent sputum, even in mild cases
of patients have high concentrations of bacteria in the lower airways
S. pneumoniae, H. influenzae, M. catarrhalis, M. pneumoniae

Use of antibiotics in moderate or severe exacerbations reduces the risk of Rx failure and death

Acute COPD Exacerbations


Therapeutic Options
Which Antibiotics?
Choice and duration of use are unclear First-line agents are often effective
Amoxicillin TMP/SMX, macrolides Doxycycline

Second-line agents may be preferable due to increasing microbial resistance in many areas
Cephalosporins (2nd or 3rd generation) Amoxicillin/clavulanate Quinolones

Acute COPD Exacerbations


Therapeutic Options
Bronchodilators (inhaled)
Albuterol is preferred (rapid onset) Inhaled anticholinergics produce additive bronchodilation (allows lower doses of albuterol) next choice
Tiotropium (long-acting) is more effective than ipratropium (short-acting)

Steroids (systemic)
30-60 mg daily for 10-14 days Longer duration needs tapering No need to stop inhaled steroids (may minimize systemic dose needed)

COPD Exacerbations
Note** LABA, ICS, tiotropium mono therapy is superior to placebo or SABA in reducing exacerbations

Acute COPD Exacerbations


Therapeutic Options
Methylxanthines (parenteral)
Theophylline
Not routinely recommended Less effective More potential adverse effects

Others lack adequate evidence for routine use


Mucolytics Nitric oxide Chest physiotherapy Antitussives Morphine

Joint Guidelines from ACP, ACCP, ATS, and ERS (GOLD 2011)
Published in the Annals of Internal Medicine 8/2/11 (updates the 2007 guidelines)

Recommendation 1: Spirometry should be obtained to diagnose airflow obstruction in pts with resp Sxs and NOT in those without (Grade: strong, moderatequality evidence) Recommendation 2: Stable COPD pts with resp Sxs and FEV1 btwn 60-80% predicted, suggest that Tx with inhaled bronchodilators may be used (Grade: weak recommendation, low-quality evidence)

Joint Guidelines from ACP, ACCP, ATS, and ERS


Published in the Annals of Internal Medicine 8/2/11 (updates the 2007 guidelines)

Recommendation 3: Stable COPD pts with resp Sxs and FEV1 < 60% predicted, recommend Tx with inhaled bronchodilators (Grade: strong, moderate-quality evidence) Recommendation 4: Clinicians prescribe monotherapy using either longacting inhaled anticholinergics or LABA for symptomatic pts with COPD and FEV1 < 60% predicted. (Grade: strong, moderate-quality evidence). Base choice on pt preference, cost, and adverse effect profile.

Joint Guidelines from ACP, ACCP, ATS, and ERS


Published in the Annals of Internal Medicine 8/2/11 (updates the 2007 guidelines)

Recommendation 5: Suggest that clinicians may administer combination inhaled Tx (long-acting inhaled anticholinergics, LABA, or ICS) for symptomatic pts with stable COPD and FEV1 < 60% predicted. (Grade: weak; moderate evidence) Recommendation 6: Clinicians should prescribe pulmonary rehab for symptomatic pts with an FEV1 < 50% predicted. (Grade: strong, moderate-quality evidence). Consider roflumilast for symptomatic pts with frequent exacerbations and FEV1 < 50% predicted (Grade: Weak)

Joint Guidelines from ACP, ACCP, ATS, and ERS


Published in the Annals of Internal Medicine 8/2/11 (updates the 2007 guidelines)

Recommendation 7: Clinicians should prescribe continuous O2 Tx in pts with COPD who have severe resting hypoxemia (PaO2 < 55 mm Hg or SpO2 < 88%). (Grade: strong recommendation, moderate-quality evidence)

Joint Guidelines from ACP, ACCP, ATS, and ERS


Treat co-morbidities: CVD, osteoporosis, and depression Treat exacerbations with short-acting betaagonists, systemic steroids, and antibiotics, O2 as needed

Lung Cancer
#1 cancer in men and women 160,000 people, 28% of cancer deaths; 374,000 people in the US are living with cancer 81% are over 60 years of age The 5-year survival rate (16.3%) is lower than many other cancers: colon (65.2%), breast (90.0%), prostate (99.9%).
Amer. Lung Assoc. 2012

Lung Cancer
Number of cases
175 150 125 100 75 50 25 0 Lung Cancer Prostate Breast Pancreas

Number of cases

Lung Cancer Causes


90% due to smoking Radon and asbestos 5-10% Air pollution 1-2%

Lung Cancer
The five-year survival rate for lung cancer is 52.6 percent for cases detected when the disease is still localized within the lungs. Only 15 percent of lung cancer cases are diagnosed at an early stage. Later stage five-year survival rate is only 3.5 percent. Over half of people with lung cancer die within one year of being diagnosed.

Lung Cancer Screening


USPSTF draft recommendationlow-dose lung CT, annual, ages 55-79; accompanied by stop-smoking advice If 30 pack-year history of smoking or more AAFP recommendationneutral Need ability to follow up and investigate nodules if found Best done at experienced centers Based on results of the National Lung Screening Trial 2012

6. A 59-year-old African-American female with diabetes comes in for follow up. You note that she has been smoking a pack a day since age 15. No cough or dyspnea. She has not had a chest x-ray in 5 years. Her mother died of colon cancer a year ago at age 83. What would you recommend for this patient?
A. B. C. D. Chest x-ray No testing at this time Low-dose lung CT Low-dose abdominal CT

Sleep Apnea (OSA)


OSA is caused by a blockage of the airway, soft tissue in the rear of the throat collapses during sleep, leading to poor sleep, poor oxygenation, and apneic periods. It affects more than 18 million Americans, according to the National Sleep Foundation. Risk factors include being male, HTN, overweight, snoring, and over the age of 40, but sleep apnea can strike anyone at any age.

Sleep Apnea Consequences


High blood pressure CVD Sleep disturbance Memory problems Weight gain, impotence, headaches. May lead to daytime sleepiness, job impairment, and motor vehicle crashes

Sleep Apnea Diagnosis


Epworth Sleepiness Scale, 0-3 for each factor Sitting, in a car, watching TV, etc. Scoring:
0-10: Normal 10-12: Borderline 13-24: Abnormal

Sleep Apnea Diagnosis


Sleep study, whole night or a split-night study. If obstructive sleep apnea is found, the patient is awakened and fitted with a PAP device and retested. The apnea-hypopnea index, or AHI. An apnea is not breathing for > 10 seconds. Hypopnea is a constricted breath that lasts > 10 seconds. The AHI is the number of apneas and hypopneas per hour. An AHI of 5 to 15 is mild; 15 to 30 is moderate OSA; > 30 is severe

Sleep Apnea Treatment


Weight loss (a factor in 70% of cases) Nasal decongestant Positional therapy (tennis ball behind the head) Surgerymultiple options, individual; uvulopalatopharyngoplasty, or UPPP Sleeping appliance Positive airway pressure device

Sleep Apnea Treatment


CPAP or BiPAP PAP is the most effective treatment, per AHRQ 2011 national report Reduces AHI Reduces blood pressure Improved daytime alertness

COPD (Websites)
American Thoracic Society website:

http://www.thoracic.org/clinical/copd-guidelines/
American Lung Association website: http://www.lung.org/lung-disease/lungcancer/resources/facts-figures/lung-cancer-factsheet.html

Answers
1. 2. 3. 4. 5. 6. D A C C B C

Review of the Diseases of the Upper GI Tract


David G Weismiller, MD, ScM, FAAFP
Department of Family Medicine
The Brody School of Medicine at East Carolina University
weismillerd@ecu.edu

Disclosure Statement
Dr Weismiller has nothing to disclose.

The AAFP has selected all faculty appearing in this program. It is the policy of the AAFP that all CME planning committees, faculty, authors, editors, and staff disclose relationships with commercial entities upon nomination or invitation of participation. Disclosure documents are reviewed for potential conflicts of interest and, if identified, they are resolved prior to confirmation of participation. Only those participants who had no conflict of interest or who agreed to an identified resolution process prior to their participation were involved in this CME activity.

Learning Objectives
1. Determine the approach to the patient with dyspepsia. 2. Discuss the common disorders of the esophagus: motility and GERD. 3. Describe the diagnosis and treatment of PUD and Helicobacter pylori.

Disorders of motility Gastro-esophageal Reflux Disease Inflammatory and Infectious Diseases Tumors of the Esophagus

Esophageal Disorders

Symptoms from the Esophagus History


Swallowing difficulties dysphagia Pain heartburn, odynophagia, chest pain Regurgitation effortless appearance of gastric or esophageal contents in the oral cavity

Esophageal Motility Disorders


Achalasia Spasm
Diffuse Localized nutcracker esophagus

Scleroderma

1. Which of the following is an indicated treatment for achalasia?


A. B. C. D. Beta blockers Alpha blockers Calcium channel blockers H2 blockers

Motility Disorders
Disorder
Achalasia
(Absence of peristaltic progression)

Clinical

Diagnosis

Treatment
Long-acting nitrates, Ca channel blockers, pneumatic dilatation of LES Long-acting nitrates, Ca channel blockers

Dysphagia solids Barium swallow, and liquids, manometry increased risk of SCC Heartburn, chest Barium swallow pain, or dysphagia; often swallowinduced always exclude CAD. None to severe reflux; often with strictures, motility abnormalities

Diffuse Esophageal Spasm


(Spastic Motor Disorder)

Scleroderma Esophagus
(90% of patients with scleroderma have esophagus involved.)

Barium swallow, Manage reflux; treat manometry esophagitis with H2 blockers, PPIs, prokinetic drugs.

GERD
Reflux of acid or gastric juice into the lower esophagus with some combination of symptoms, inflammation, and/or complications Common disorder Weekly Sx: 10%-20% of US population Within past month: 30%-40%

GERD Pathophysiology
Motor components of LES, esophagus, and stomach Noxious esophageal contents Mucosal barrier, saliva Mechanical factors gravity, hiatus hernia, obesity, etc Sensory components appreciation of pain

2. Which one of the following is a true statement regarding GERD?


A. One of the more common complications is obstructive sleep apnea. B. Dietary modifications are not part of the treatment according to the current American College of Gastroenterology guidelines. C. Endoscopic surveillance for dysplasia is indicated in Barretts esophagus (squamous metaplasia). D. Corrective laparoscopic reflux surgery is not indicated when there are persistent reflux symptoms despite acid suppression.

GERD Diagnosis
No gold standard; EGD is to assess complications (SOR: A).
Erosive esophagitis Stricture Barretts esophagus Cancer

GERD Diagnosis
No gold standard; EGD is to assess complications (SOR: A). EGD lacks adequate sensitivity in determining pathologic reflux (SOR B). pH probe is accepted as standard (SOR B); still false positives and false negatives. Sensitivity: 85% Specificity: 95% Barium radiology: Limited usefulness; not recommended (SOR B)

GERD Diagnosis
An empiric trial of acid suppression therapy for 4-8 weeks can identify patients with GERD who do not have alarm symptoms (SOR A). Alarm symptoms Black or bloody stools Choking Chronic cough Dysphagia Early satiety Hematemesis Hoarseness Iron deficiency anemia Odynophagia Weight loss

American College of Gastroenterology 2005 Volume 135, Issue 4; 1383-1391.e5, October 2008

Treatment Guidelines
Step 3 Severe symptoms Erosive disease

Continue with measures. Step 2 GI workup (+/) endoscopy Non-responders Non-erosive disease High-dose H2 antagonists Higher dose PPI Continue lifestyle/dietary modification. H2 antagonists (SOR A) Step 1 Proton pump inhibitor (PPI)* (SOR A) Mild symptoms Pro-motility agent (SOR A) 8-12 weeks of therapy Dietary modifications Lifestyle modification (SOR C) * PPI should be taken 30-60 minutes Trial of patient-directed therapy with prior to a meal (the first meal of the day) to optimize effectiveness (SOR B). antacids or OTC H2 antagonists

Barretts Esophagus
Risk of esophageal metaplasia (Barretts) and adenocarcinoma increases with GERD symptom severity, duration, and frequency. Endoscopic surveillance for dysplasia is indicated in Barretts esophagus - ??? Surveillance of known Barretts esophagus is controversial, because adenocarcinoma of the esophagus is rare in the US (6000-7000 cases/yr) and GERD/Barretts occurs in 0.4%0.8% of the population.
From: Google Images.

AGA Position Statement on Screening for Barretts Esophagus 2011


Whom to screen?
Long-standing (> 5 years) heartburn symptoms Long-standing (> 5 years) need for medication

Recommend against screening the general population Support is stronger for screening those patients with multiple risk factors.

Risk Factors for Barretts Esophagus


Men
Screening women for Barretts is like screening men for breast cancer.

Caucasian Age > 50 years Hiatal hernia Increasing BMI Abdominal fat distribution (abdominal obesity)

2008 ACG Guidelines for Surveillance of Barretts Esophagus


EGD Barretts Esophagus
No dysplasia
Second EGD with biopsies within year to confirm there is no dysplasia If both EGDs with biopsies () for dysplasia, repeat EGD with biopsy q 3 years.

Low-grade dysplasia (LGD)


Review by expert pathologist to rule out HGD. Repeat EGD with biopsies within 6 months to reassess for dysplasia. If () for dysplasia on repeat EGD, yearly EGD with biopsy recommended until 2 years with EGDs showing no dysplasia.

High-grade dysplasia (HGD)


Expert pathologist confirm HGD. Mucosal irregularity remove with endoscopic mucosal resection (EMR). EGD with biopsies repeated in 3 months to look for HGD and small cancers. Possible interventions for HGD: espohagectomy, EMR, photodynamic therapy, radiofrequency ablation, ablation using cryotherapy.

At the present time, only specialized intestinal metaplasia of the esophagus is classified as Barrett's esophagus. Currently, it is recommended that only patients with this diagnosis undergo periodic cancer surveillance.

GERD Surgical Treatment


Who Complications of reflux Non-responding esophagitis Stricture Barretts metaplasia Inability to tolerate medications including noncompliance Persistent reflux symptoms despite acid suppression, ie, chronic reflux with recalcitrant symptoms (SOR A) Asthma

GERD Surgery
Laparoscopic anti-reflux Complications rate surgery Splenic injury Treatment of choice at Esophageal or gastric many centers perforation Less perioperative M&M Dysphagia Shortens the Inability to belch or postoperative stay vomit Needs experienced Vagal denervation surgeon

GERD Follow-up and Surveillance


If symptoms remain unchanged in a patient with a prior normal EGD, repeating EGD is not recommended (SOR C). Chronic reflux has been suspected to play a major role in the development of Barretts esophagus, yet it is unknown if outcomes can be improved through surveillance and medical treatment (SOR C). Anti-secretory therapy has not been shown to reduce the need for recurrent dilation from esophageal stricture formation (SOR A).

Inflammatory Disorders Esophagitis


Disorder Pill-induced Infective* Viral Fungal Corrosive Offending Agents Doxycycline, NSAIDs, steroids HSV, CMV Candida Alkalis or acids

Eosinophilic:
pronounced eosinophilic infiltration

Allergic or idiopathic; Tx steroids, diet, anti-allergy medications


*Mostly in immunosuppressed patients

3. Which of the following statements regarding esophageal tumors is true? A. 90% are cancer. B. Adenocarcinoma is the most common histologic type. C. They are more common in women. D. The incidence of squamous cell carcinomas is increasing.

Esophageal Tumors
90% are cancer. Much more common in males; 10% 5-yr survival rate overall (treatment improving) Dx endoscopy and radiography Squamous cell carcinoma Most common, declining incidence Predominant esophageal cancer in African Americans More common with heavy alcohol and tobacco use Adenocarcinomas Arise from columnar epithelium in cardia or from Barretts Recall that the lower esophagus is lined by specialized intestinal epithelium. GERD is a risk factor.

Acid Peptic Disorders of the Stomach and Duodenum Infections Motor Disorders Cancer

Diseases of the Stomach

Acid Peptic Disorders Stomach and Duodenum


Common Problem 5%-10% of population will have PUD in their lifetimes; 50% recurrence in 5 years DU/GU 4:1 90% DU in duodenal bulb GU most common on lesser curve

Mortality Rates
Ulcer Type Men Women

Duodenal Ulcer
Gastric Ulcer

1/100,000
1.5/100,000

0.5/100,000
1.2/100,000

4. Which of the following is a clear predisposing or exacerbating factor for acid peptic disorders?
A. B. C. D. Caffeine Stress Tobacco Chili peppers

Why Do Acid Peptic Disorders Develop?


Current theory PUD is an imbalance between protective and aggressive factors.

Protective factors
Surface epithelial cells with mucus and bicarbonate secretions Apical surface membrane of gastric mucosal cells Prostaglandins E1 and E2

Aggressive factors
Production of gastric acid NSAIDs Corticosteroids Smoking Alcohol consumption ? Psychological stress Probably not diet

Predisposing Factors
H. pylori infection NSAIDs Double the annual chance of complicated PUD from 1%2% to 3%-4% Worse with alcohol Longer-acting NSAIDs are worse. Dose, duration important variables Milk: May slow healing of DU Caffeine: No clear evidence of worsening Peppers: No slowing of DU healing Alcohol: Worse with NSAIDs; unclear otherwise Tobacco: Much higher rates of ulcer and slower healing Stress: Remains controversial

Diagnosis PUD
History Persistent pain relieved by food and antacids Pain in upper abdomen or back Hematemesis, melena, or hematochezia Cannot usually separate GU from DU by history On exam Mid-epigastric tenderness Laboratory Limited usefulness, except H. pylori tests Consider serum gastrin (especially if recurrent ulcer disease) Hematocrit Stool guaiac Endoscopy (SOR A) 90% sensitivity and specificity

Diagnostic Tests for Helicobacter pylori


Test Usefulness Sensitivity (%) Specificity (%)

Invasive
Endoscopy with biopsy Histology Diagnostic strategy of choice in children with persistent or severe upper abdominal symptoms Sensitivity reduced by PPIs, antibiotics, bismuth-containing compounds. > 95 100

Urease activity

Test of choice when endoscopy 93-97 indicated; rapid results (20 min); () results may need confirmation by histology or other test; sensitivity reduced by PPIs, antibiotics, bismuthcontaining compounds, and active bleeding. Technically demanding; only use for resistant organism or refractory disease. 70-80

> 95

Culture

100

Diagnostic Tests for Helicobacter pylori


Test Usefulness
Sensitivity and specificity vary widely; assist with initial diagnosis; not as helpful in following patients due to prolonged presence of antibody after eradication.

Sensitivity (%) 85

Specificity (%) 79

Noninvasive
serology for immunoglobulin G

Urea breath test

Reliable test for cure can > 95 document eradication as early as 4 weeks post-treatment; requires separate appointments; sensitivity reduced by PPIs, antibiotics, and bismuth-containing compounds. Detects active infection; test for 93-97 cure seven days after therapy is accurate; sensitivity reduced by PPIs, antibiotics, and bismuth-containing compounds.

100

H. pylori stool antigen

> 95

PUD and H. Pylori


Disorder H. pylori (+) Associated Treatment H. pylori gastritis Duodenal 90% 70% Eradication of ulcer infection markedly decreases Gastric 70% 60%-80% recurrences of DU. ulcer A 1-2 week course of H. pylori eradication therapy is an effective treatment for H. pylori (+) PUD.

Who Should Be Tested for H. Pylori?


All newly diagnosed (by radiography or endoscopy) complicated or non-complicated DU or GU Hx of PUD receiving maintenance anti-secretory therapy or documented past ulcer Mucosa-associated lymphoid tissue lymphoma (MALT) Some decision and cost-benefit analysis support non-endoscopic diagnostic testing in patients < 55 with symptoms of ulcer-like dyspepsia and no alarm symptoms. Deciding on what test to use in which situation relies upon whether a patient requires evaluation with upper endoscopy and an understanding of the strengths, weaknesses, and costs of the individual test.
ACG Guideline on the Management of H. pylori infection. Am J Gastroenterol. 2007;102:1808.

5. When treating H. pylori, which of the following statements is true? A. Most therapies approach 100% effectiveness. B. Use of anti-secretory agents with antimicrobials increases eradication rate. C. PPIs have no intrinsic in vivo activity against H. pylori. D. An increased gastric pH decreases the efficacy of some antibiotics.

Specific H. Pylori Therapy


Treatment
PPI, amoxicillin 1g, clarithromycin 500 mg
bismuth 525 mg, metronidazole 500 mg, tetracycline 500 mg PPI, amoxicillin 1 g, metronidazole 500 mg PPI, levofloxacin 250-500 mg, amoxicillin 1 g PPI, rifabutin 150 mg, amoxicillin 1 g PPI plus amoxicillin 1 g

Dosing/Comments
All BID for 7-14 days First line (up to date 1A recommendation)
All QID with PPI BID for 7-14 days First line or retreatment All BID for 14 days First line in macrolide allergic or retreatment All BID for 14 days Rescue for two prior treatments All BID for 14 days Rescue PPI BID, amoxicillin TID for 14 days Rescue

6. Regarding anti-secretory therapy for PUD, which of the following is true? A. H2 blockers lead to faster healing than proton pump inhibitors. B. Therapy is usually longer for duodenal ulcers versus gastric ulcers. C. The addition of sucralfate to the antisecretory medications hastens healing. D. It is the mainstay of therapy in uninfected patients.

Specific Ulcer Treatment Anti-secretory


Anti-secretory therapy Mainstay of therapy in uninfected patients PPI, H2 blocker Appropriate for maintenance therapy in selected cases Usually 4-6 weeks for DU Generally longer for GU 12 weeks PPIs lead to faster healing than H2 blockers.

Treatment for H. Pylori Infection Summary


Most important therapy in affected individuals No therapy 100% effective Triple or quadruple therapy most effective Use of anti-secretory agents with antimicrobials increases eradication rate. Increased gastric pH increases efficacy of some antibiotics. PPIs have intrinsic in vivo activity against H. pylori. Anti-secretory therapy hastens relief of ulcer symptoms. Compliance is essential for eradication.

7. In considering NSAID-induced ulcers, which of the following is true regarding prevention? A. H2 blockers are superior to PPIs in preventing ulcers. B. H. pylori should not be treated if present. C. PPIs superior to misoprostol (200 mg QID) in preventing ulcer relapses. D. Sucralfate is contraindicated in preventing NSAID-induced ulcers.

NSAID Ulcers
Risk factors Prior adverse GI event (ulcer, hemorrhage) Age > 60 High-dose NSAID (> twice normal) Glucocorticoid use Anticoagulant use Risk for NSAID-induced GI toxicity 9% at 6 months with multiple risk factors present In nave NSAID users, H. pylori is significant risk factor for complicated ulcer disease screening may be indicated.

NSAID Users Medical Treatment of Peptic Ulcer Practice Guidelines


Treatment of NSAID ulcers D/C NSAIDs Treat H. pylori if present. PPIs superior to H2 receptor antagonists Prevention of NSAID ulcers PPIs superior to H2 blockers in preventing ulcers PPIs superior to misoprostol (200 mg QID) in preventing ulcer relapses NEJM. 1998;338:727. Treat H. pylori if present. Primary Care. September, 2001:28(3):487-503.
Am J Gastrolenterol. 1998;93:2037.

Treatment NonH. Pylori PUD


Withdrawal of potential offending or contributing agents NSAIDs, cigarettes, excess ETOH No firm dietary recommendations avoid foods that precipitate dyspepsia. Address psychosocial issues and comorbidities no firm evidence, but may have deleterious health consequences.

Controversy: Treatment of H. Pylori in Non-ulcer Dyspepsia


Efficacy of treatment is controversial. Recent review of RCTs: Eradication provides small but significant benefit for dyspeptic symptoms.* Eradication may be cost-effective intervention for non-ulcer dyspepsia. And H. pylori appears to have a net suppressive effect on acid production, so treating may make GERD worse.
*Practice Guideline. Moayyedi P, Soo S, et al. Eradication of Helicobacter pylori for non-ulcer dyspepsia. The Cochrane Database of Systematic Reviews. 2005. http://www.cochrane.org/cochrane/revabstr/ab002096.htm

An Approach to Dyspepsia
55 years old No alarm features Dyspepsia

H. Pylori prevalence < 10%


Fails

H. pylori prevalence 10%


Fails

Trial of PPl
Fails

Test and treat for H. pylori.


Fails

> 55 years old or presence of alarm symptoms, family or personal Hx GI CA or PUD, wt. loss, GI bleeding, anemia, or dysphagia

Test and treat for H. pylori.

Trial of PPl
Upper endoscopy

Consider upper endoscopy.

Consider upper Endoscopy.

Adapted from American College Gastroenterology, 2005

8. Risk factors for ulcer complications include all of the following except: A. B. C. D. Previous history of complications Prior refractory or protracted course Small ulcers (< 2 cm) Deformed ulcer bed or dense fibrosis

Risk Factors for Ulcer Complications


Previous history of complications Prior refractory or protracted course Big ulcers (> 2 cm) Deformed ulcer bed or dense fibrosis

Complications of PUD
Bleeding Most common complication and leading cause of death (4%-9% mortality rate) Occurs in 10%-20% Patients with 1 episode of bleeding more likely to rebleed 90% stop without specific treatment. Increased morbidity with associated portal hypertension*
* Am J Gastro 1998;93:336

Complications of PUD
Gastric outlet obstruction Usually mechanical obstruction due to edema or scar Most due to DU Rare: 2% of ulcer patients Perforation and penetration 2% of ulcers perforate. Average duration of Sx prior to perforation: 5 years NOTE: Complicated ulcer disease less likely to involve H. pylori

PUD
Surgical Treatment/Management
Dramatically declined over past two decades Indications Hemorrhage not responsive to medical therapy Gastric outlet obstruction not reversed by medical treatment Perforation Malignancy

SORT: Key Recommendations for Practice


Ables AZ, Simon I, and Melton ER. Update on Helicobacter pylori Treatment. Am Fam Physician. 2007;75:351-8.

Clinical recommendation A test-and-treat strategy is recommended in patients with symptoms of dyspepsia. Helicobacter pylori eradication therapy is recommended to prevent recurrence and re-bleeding in patients with peptic ulcer. Short-course drug therapy is an option for H. pylori eradication in adult patients.

Evidence Comment rating A Test-and-treat strategy reduces endoscopies and use of anti-secretory medications. It is unnecessary to continue antisecretory maintenance therapy in patients after H. pylori eradication. Eradication rates using short-course therapy are similar to those of traditional treatment with the potential for greater compliance. Urea breath test is more reliable in children older than six years; monoclonal antibodybased stool antigen is an alternative.

The urea breath test is the most C reliable noninvasive diagnostic test in children with suspected H. pylori infection.

Gastric Dysmotility

Gastric Dysmotility Slow or Delayed Emptying


Etiology Mechanical or outlet obstruction PUD, bezoar, etc. Functional obstruction (gastroparesis) Drugs opiates, anticholinergics, beta and Ca channel blockers Diabetes, Parkinsons, hypothyroidism, hypoparathyroidism Pregnancy Post-vagotomy

Gastric Dysmotility Slow or Delayed Emptying


Diagnosis Nausea, vomiting, dysphagia, post-prandial abdominal pain, GERD Tests: Scintigraphy, electrogastrogram (evaluate gastric myoelectrical utility), ultrasonography Treatment Remove causes. Low-fat diet; avoid large meals. Metoclopramide, erythromycin, prokinetics

Gastric Dysmotility
Rapid Gastric Emptying
Dumping Syndrome Most commonly seen post-operatively from gastric surgery or a truncal vagotomy Symptoms 15-30 minutes after eating nausea, nonproductive vomiting, sweating, flushing, abdominal cramping, diarrhea Treatment 6-8 small, low-CHO meals/day; avoid excessive liquids; use of opiates and anticholinergic drugs; fiber products; possibly surgery

9. A 65-yo male smoker complains of dyspepsia, weight loss, early satiety, and occasional nausea and vomiting. Which one of the following would be the initial diagnostic method of choice?

A. Upper GI endoscopy B. CT of the upper abdomen C. Single-contrast upper GI barium swallow D. Endoscopic ultrasonography

Cancer of the Stomach


One of the most common internal malignancies in the world 95% are adenocarcinomas. Chronic GERD is the leading cause of esophageal adenocarcinoma (68%-90%). Only 10%-20% of US GI tumors probably because of lower rates of H. pylori in US, due to cleaner food and water 2x as common in as in 2x as common in African Americans and Hispanics as in Caucasians Dx: Endoscopic biopsy in patients with upper GI symptoms or high-risk or double-contrast barium swallow Tx: Surgical excision; 5-yr survival rate < 10%

Pancreatic Cancer
Fourth leading cause of cancer-related death; second only to CRC as cause of GI cancer-related death Higher incidence: and African Americans Risk factors: Smoking, chronic pancreatitis, diabetes, hereditary predisposition History/PE Abd pain, weight loss, jaundice, pancreatitis Jaundice, abdominal mass, ascites

Pancreatic Cancer
Diagnosis
U/S, EUS CT, MRI ERCP, FNA, CA19-9 All sensitive and specific

Treatment
Surgical resection only potential curable treatment

Prognosis 5-yr survival


Node () Node (+) 25%-30% 10%

Answers
1. 2. 3. 4. 5. 6. 7. 8. 9. C C A C B D C C A

Sticks and Stones


Fracture Care in Family Medicine

Joseph Garry, MD, FAAFP, FACSM University of Minnesota

Disclosures
Dr. Garry discloses that he holds stock in Pfizer and Merck pharmaceuticals.

It is the policy of the AAFP that all individuals in a position to control content disclose any relationships with commercial interests upon nomination/invitation of participation. Disclosure documents are reviewed for potential conflicts of interest. If conflicts are identified, they are resolved prior to confirmation of participation. Only participants who have no conflict of interest or who agree to an identified resolution process prior to their participation were involved in this CME activity.

Learning Objectives
1. Know the Salter-Harris classification, most common types of pediatric fractures, and reduction technique for nursemaids elbow. 2. Cite the causes and management considerations for common adult fractures. 3. Cite the frequency and management of stress fractures, including metatarsal, navicular, and femoral neck stress fractures.

1. Which of the following statements is most correct regarding pediatric fractures?

A. Approximately 25% of pediatric fractures involve the growth plate. B. Salter-Harris type II fractures are the least common type fracture. C. Approximately 45% of pediatric fractures involve the growth plate. D. Salter-Harris type III fractures are most commonly treated non-surgically.

Pediatric Fractures
15%-30% of fractures involve the growth plate.
> 30% occur in the long bones of the fingers. 1%-10% of physis fractures result in growth deformity.

Boys >> Girls


33% of physis fractures occur in sports.
20% occur as the result of recreational activities

If suspected, radiographs are imperative.

Pediatric Fractures
Type I Type II Type III Type IV Type V
Typically treated with immobilization Most common type, immobilization Older children, surgical evaluation for

ORIF
Arrest growth, surgical treatment with ORIF Arrest growth, casting or surgery, refer

Figure from www.orthoinfo.aaos.org

Pediatric Fractures
Following acute injury with normal radiographs and refusal to use extremity consider advanced imaging.
25 patients with above criteria underwent MR imaging: Salter II (2), Salter III (1), Salter IV (3), epiphyseal (18), diaphyseal (1).

Naranja et al. Clin Orthoped Rel Res 1997

2. A 10-yo child presents to your office with left elbow pain after a fall at school earlier today. He is able to fully flex the elbow but unable to fully extend it. You conclude that

A. B. C. D.

The child has an elbow contusion. The child is malingering. The child has a fracture. Munchausen by proxy syndrome is likely.

In acute pediatric elbow injuries, the inability to fully extend the elbow was associated with an elbow fracture in all cases.
Darracq et al. Am J Emer Med. 2008.

3. The best initial treatment for nursemaids elbow is A. Traction to the arm

B. Hyperpronation of the forearm


C. Flexion and supination of the forearm D. A sling

Nursemaids Elbow
2-3 yo most common
Traction injury Elbow held in extension and pronation, or at side

Radiographs recommended if concern for fracture or if initial reduction(s) fail.


Hyperpronation required fewer attempts, was more successful initially, and often successful when supination/flexion failed (A).
Macias et al. Pediatrics 1998

4. An 11-yo female presents with the following radiograph and you recommend

A. An MRI of the shoulder B. A sling C. Surgical referral D. A shoulder abduction brace

Clavicle Fractures
Most commonly fractured bone 85% occur in middle third. Surgical referral for skin
tenting, NV compromise, significant displacement, or overriding fragments by > 2 cm

Distal clavicle fractures Surgical repair if ligaments


disrupted

XR = AP and 45 cephalic tilt views Treat with sling or figure 8.

5. A 25-yo male falls onto his shoulder and presents with this radiograph. The most appropriate management at this time includes

A. Immediate surgical referral B. Obtain stress radiographs. C. Obtain a shoulder MRI. D. Placement in a sling

AC Joint Injury

Illustration courtesy of Steve Oh

AC Joint Injury
Fall directly on shoulder Stress radiographs not helpful Grades I-III managed nonoperatively
Sling for comfort Grade III nonoperative treatment = outcomes for surgical treatment (B)

Grades IV-VI require immediate surgical consultation


Phillips et al. Clin Ortho Rel Res, 1998

6. All of the following are true regarding acute compartment syndrome, except
A. Females are at higher risk B. Patients on warfarin therapy are at higher risk C. Can occur in either the upper or lower extremities D. Decreased sensation in the limb is a symptom of acute compartment syndrome

Compartment Syndrome
Can occur in any extremity Causes include fractures, hemorrhage, casting, rhabdomyolysis Anti-coagulation and hemophilia increase risk of CS Symptoms
sensation Pain out of proportion Paresthesias, paleness, weakness

Diagnose with intra-compartmental pressures Treatment with fasciotomy

Distal Radial Fracture


16% of all fractures Figure from orthoinfo.aaos.org Older age and osteoporosis are risks. FOOSH injury Universal classification system Confirm median nerve function via thenar sensation. Non-displaced fractures treated with casting. 1-2 mm of articular incongruity raise risk of OA.

7. A 36-yo female falls on an outstretched hand and presents with wrist pain and this radiograph. What is the best treatment option?

A. Wrist splint for 2 weeks B. Ulnar gutter splint and repeat xrays in 2 weeks C. Recommend bone density testing. D. Thumb spica cast for 8 weeks

Scaphoid Fracture
FOOSH injury Tenderness in the anatomical snuff box Trauma + tenderness + normal x-rays = splint and repeat x-rays in 2 weeks. Bone scan (72 hrs), CT, or MRI

Complication = avascular necrosis

Scaphoid Fracture
Operative vs nonoperative treatment

There is not a clear benefit to operative treatment of an acute nondisplaced scaphoid fracture (B).
Dias et al. JBJS, 2005

Question 8.

A 17 yo football player comes to clinic the morning after a game in which he injured the middle finger of his right hand. During the game he grabbed the jersey of an opposing player as he attempted to tackle him and the patient immediately felt pain in the distal aspect of the affected finger. On examination there is mild swelling of the finger and he cannot flex at the DIP joint. Radiographs show a small bony fragment at the volar surface of the proximal distal phalanx. Which

one of the following would be the most appropriate management? A. Referral to a hand surgeon B. Splinting the DIP joint in flexion for 8 weeks C. Splinting the DIP joint in extension for 8 weeks D. Splinting the PIP joint in extension for 6 weeks followed by night splinting x 4-6 weeks

Jersey Finger
Avulsion of the flexor digitorum profundus from the distal phalanx [ring finger most common] Palmar digit swelling or pain with ecchymosis, which may also occur more proximally in palm if tendon has retracted Cannot actively flex the distal phalanx [FDP]
Cannot actively flex at the PIPJ [flexor superficialis]

Surgical correction within 10-12 days

Boutonniere Deformity
Central slip injury to PIP joint Figure from orthoinfo.aaos.org Causes: Jammed finger, dorsal laceration, or rheumatoid arthritis Presentation 7-21 days post-injury with PIP joint in flexion and DIP joint in hyperextension Manage with splinting of PIP joint in extension for 6 weeks followed by night splinting x 4-6 weeks. Surgical correction for persistent deformity and functional impairment

Mallet Finger
Figure from orthoinfo.aaos.org

Forced flexion of an extended DIP joint results in avulsion of the extensor digitorum tendon. Visible deformity Radiographs to evaluate for fracture Splint DIPJ in full extension continuously for 8 weeks. Surgical intervention considered if > 30% of the articular surface is involved.

8. A 74-yo female patient with osteoporosis and arthritis is seen for routine follow-up. She notes insidious onset of back pain about 2 months ago. She denies any trauma or associated symptoms. You obtain this radiograph. Which statement is most accurate?
A. This is an unusual finding, and she will need to be evaluated for multiple myeloma. B. 20% of these go undiagnosed. C. She has an increased mortality rate of 15% over those without this condition. D. Alendronate can be used to treat her pain.

Vertebral Compression Fracture


Prevalence
25% of postmenopausal women 40% of women > 80 years
L1

With VCF have a 15% higher mortality rate Risks include: osteoporosis, female,
age, caucasian, dementia, risk of falls, body weight, Ca++ or vit D, smoking

Most common sites are T8-L1 and L4.


Important to image entire spine

2/3 of fractures not diagnosed.

Kado et al. Arch Intern Med. 1999;159.

Vertebral Compression Fracture


Treatment

Vertebroplasty
Injection of cement into fractured vertebra to pain

Kyphoplasty
Use of a balloon to restore height to fractured vertebra prior to injection of cement to pain and restore height Best if performed within 3 months of fracture

Calcitonin (Miacalcin)
Modest pain reduction in VCF Nasal spray: 1 spray in alternating nostril qd

Avascular Necrosis of the Hip


Males > females 30-60 years of age Imaging
#1 Radiographs (crescent sign) #2 MRI without contrast

Risk Factors
Joint trauma (dislocation) Excessive alcohol Hyperlipidemia Steroids Organ transplant Diabetes, lupus, HIV, sickle cell Radiation therapy Gauchers disease Bisphosphonates Hemodialysis Familial thrombophilia

Hip Fracture
258,000 hospital admissions for hip fractures among people aged 65 and older (2010) More than 95% of hip fractures are caused by falling, most often by falling sideways onto the hip. Affected leg shortened and externally rotated
Intracapsular Intertrochanteric Subtrochanteric
Natl Hospital Discharge Survey (NHDS), National Center for Health Statistics Figure courtesy of orthoinfo.aaos.org

Hip Fracture
Post-surgical anticoagulation recommended for 35 days (Amer Thoracic Soc). A large proportion of fall deaths are due to complications following a hip fracture; 20% one year mortality (Farahmand et al. Osteoporosis Inter. 2005). 33% of adults who lived independently before their hip fracture remain in a nursing home for at least a year after their injury (Leibson et al. J Amer Geriat Soc 2002).

Proximal 5th Metatarsal Fractures


Tuberosity fracture
Most common type Can be treated with splinting/casting and progressive WB as tolerated

Jones fracture and proximal diaphyseal stress fracture


Occurs at the metaphyseal-diaphyseal junction High rate of nonunion 6-8 wks of protected NWB 8-12 wks of protected WB Surgical fixation can be considered.
Hatch et al. Am Fam Phys 2007;76(6)

Stress Fractures of the Foot


2nd and 3rd metatarsals most common Navicular and calcaneus not uncommon Pain with activity and relieved by rest Radiographs Metatarsal shaft fractures can be treated with activity modification, firm soled footwear, and following symptoms for 6-8 weeks.

9. A 39-yo female runner presents with a stress fracture and requests management. Which of the following best describes a stress fracture?
A. A medial tibial stress fracture is a high-risk stress fracture. B. Use of aromatase inhibitors and anticonvulsants are associated with risk of fracture. C. Femoral stress fractures are one of the most common lower-extremity stress fractures. D. Untreated osteoporosis confers no added risk for the development of a stress fracture.

Stress Fractures
RISK FACTORS
Excessive exercise Runners > 25 mpw Female Athlete Triad

SYMPTOMS and SIGNS


activity and limited rest
Pain with ambulation (81%) Focal tenderness (65%100%)

Low vitamin D
Runners, soccer, dance Osteoporosis

Medications
Aromatase inhibitors GnRH agonists Depot medroxyprogesterone Anticonvulsants SSRI

Edema (18%-44%)

Browles SK, PSAP-VII, ACCP

Stress Fractures
Overuse Injuries

Stress Reaction

Stress Fracture

Fracture

Bone fatigue-failure continuum Insidious onset interference with performance daily pain X-ray preferred if 2 weeks of symptoms. MRI is secondary imaging test.

Frequency of Stress Fractures


ADULTS
Tibia (50%) Metatarsal (25%) Fibula (10%) Femur Navicular 1st metatarsal sesamoid Upper extremities

Pelvis

PEDIATRICS
Tibia (50%)
Metatarsal (25%) Fibula (20%)
Synder et al. Clin Sports Med. 2006.

Bennell et al. Clin Sports Med. 1997.

Stress Fractures
Overuse Injuries
Low-risk stress fracture
Titrate activity to pain-free level for 4-8 wks. Bracing or crutches Modify risk factors.

High-risk stress fracture


Typically NWB x 4-6 wks Protected pain-free WB x 4-6 wks Rehabilitation prior to RTP

Tarsal Navicular
Stress Fracture

Central third has poor vascularity yet subject to compressive forces during foot-strike phase. Presentation
Insidious onset, dorsal foot pain, exacerbated with activity Soreness or cramping of the dorsomedial foot

Examination
TTP over the tarsal navicular (+) hop test

Tarsal Navicular
Stress Fracture NWB in cast 6 weeks followed by protected WB for 2-6 weeks until pain free Surgical consideration for failures NWB success (96%) vs surgical success (82%)
No difference between NWB management and surgery in terms of outcome or RTA Torg et al. Am J Sports Med 2010

Femoral Neck
Stress Fracture
FNSF up to 11% of stress fractures in athletes Runners most common Pain with foot strike

Compression side FNSF can be treated nonoperatively.


Early diagnosis and management imperative due to risk of malunion/nonunion/displacement. Young athletes poor prognosis: 20%-86% develop AVN.

Tension

Compression

DeFranco et al. Clin Sports Med. 2006.

Femoral Neck
Stress Fracture
Compression side FNSF
3

weeks NWB (crutches) 3 weeks PWB WB as tolerated RTA when full ROM and 90% strength

Tension

Compression

Tension side FNSF referred to orthopedics, crutches, and NWB


If antalgic gait or pain at rest and consideration of femoral neck stress fracture then NWB (crutches) until diagnosis confirmed or excluded

Prevention and Rehabilitation of Stress Fractures


Use of shock-absorbing inserts in footwear reduces the incidence in military personnel (B).
Rehabilitation of a tibial stress fracture may be shortened by use of an air stirrup brace (B).

Cochrane 2005

Answers
1. 2. 3. 4. 5. 6. 7. 8. 9. A C B B D A D C B

Asthma: Pediatric and Adult


Dana E. King, MD, MS, FAAFP Professor, Family Medicine West Virginia University School of Medicine Morgantown, WV

Disclosure Statement
Dr. King has nothing to disclose

The AAFP has selected all faculty appearing in this program. It is the policy of the AAFP that all CME planning committees, faculty, authors, editors, and staff disclose relationships with commercial entities upon nomination or invitation of participation. Disclosure documents are reviewed for potential conflicts of interest and, if identified, they are resolved prior to confirmation of participation. Only those participants who had no conflict of interest or who agreed to an identified resolution process prior to their participation were involved in this CME activity.

Learning Objectives
1. Identify a treatment rationale for the patient who presents with a acute status asthmaticus. 2. Describe pharmacologic therapy in the treatment of asthma. 3. State the current NIH guidelines for the treatment of chronic asthma. 4. Discuss the approach to the pregnant patient with asthma.

Asthma
National Heart, Lung, and Blood Institute (NHLBI) practice guidelines National Asthma Education and Prevention Program (NAEPP) Third Expert Panel Report (EPR-3)
Initial report in 1991 Second report in 1997

Updated in 2002
Most comprehensive EB guidance for Dx and Rx to date

Third report in 2007

EPR-3: What Changed in 2007?


Recommend assessing asthma severity before starting Rx and assessing asthma control to guide adjustments in Rx (SOR B, C) Address both severity and control in terms of impairment and risk (SOR A) Feature 3 age breakdowns (0-4 yrs, 5-11 yrs, 12 yrs) and a 6-step approach to management (SOR C) Make it easier to individualize and adjust Rx (SOR B)

1. & 2. A 33-year-old female with no chronic illnesses arrives in your office relating a history of several episodes of shortness of breath associated with a hacky cough and chest tightness. She recently developed nasal congestion, sinus pressure, and muscle aches and thinks she has a cold. She has occasionally before been short of breath, does not smoke, and has no family history of respiratory disease. She has taken over-the-counter decongestants and cough suppressants with little relief of the coughing. She takes no prescription drugs. VS: BP 108/65, HR 80, RR 14, T 37.2 degrees C, O2 sats 95% on RA. PE: is remarkable for clear rhinorrhea, mild scattered expiratory wheezes, a normal cardiac exam, and non-tender maxillary and frontal sinuses.

1. What is the most likely trigger for her current symptoms?


A. B. C. D. Viral upper respiratory tract infection Allergic rhinitis Acute bacterial bronchitis Gastroesophageal reflux

2. What initial treatment would you prescribe for her at this time?
A. Montelukast B. A steroid nasal spray C. Corticosteroid inhaler D. Albuterol inhaler

What Is Asthma?
Clinical symptoms (recurrent)
Intermittent Sxs Exacerbations Cough Perennial/seasonal Wheeze Episodic/continual SOB/breathlessness Diurnal Chest pain Rescue med use Diurnal variation Varying triggers

What Is Asthma?
Biological indicators, pathophysiology
Chronic airway inflammation Bronchial hyperresponsiveness (BHR) Airflow limitation
Airway smooth muscle bronchoconstriction Airway edema Mucus plug formation Bronchiolar obstruction Airway remodeling

Asthma Differential Dx
Viral pneumonitis/bronchitis COPD GERD Pneumothorax Pulmonary embolism Vocal cord dysfunction syndrome Pulmonary edema Endobronchial obstruction (tumor or FB) Acute hypersensitivity pneumonitis Epiglottitis

COPD vs Asthma
Symptom Chronic cough & sputum Breathlessness on exertion or poor lung function Onset prior to 40 yrs Tobacco use Airway hyperresponsiveness Progression of Sxs Identifiable triggers Bronchodilator response COPD Common Persistent; slowly progressive Less common Almost always Common Slowly; little variability Uncommon Modest Asthma Variable Variable, intermittent, largely reversible Common Sometimes Always Episodic and variable Common Often marked

3. Which of the following tests would you use to better categorize a wheezing patients condition?
A. B. C. D. Spirometry CXR ABG Methacholine challenge

Establishing a Diagnosis
Based on a patients
Medical history Physical exam Pulmonary function tests (PFTs) Laboratory tests

Spirometry (PFTs) is recommended in order to make the Dx Level of severity based on


Impairment Risk

Clinical Testing
Spirometry
Recommended for every pt 5 yrs of age
If pt < 5 yrs of age, a therapeutic trial of medication is recommended

Studies specific to individual patients


Allergy testing CXR Bronchial provocation testing Sinus x-rays or CT scan GERD evaluation CBC with eosinophils, total IgE, sputum exam

4. Which one of the following PFT results is most likely to be below the normal predicted range in a patient with asthma?
A. B. C. D. FVC (forced vital capacity) FEV1 (forced expiratory volume in 1 second) TLC (total lung capacity) FRC (functional residual capacity)

5. What percentage of airway reversibility and change in FEV1 do you need in order to confirm the diagnosis of asthma?
A. 12% and 500 mL B. 25% and 200 mL C. 12% and 200 mL D. 18% and 100 mL

Spirometry
Measurements pre and post a short-acting beta2-agonist (SABA)
FEV1 FVC FEV1/FVC Reduced FEV1 and FEV1/FVC values relative to predicted values (FEV1 < 80% predicted) Increase by 12% and 200 mL in FEV1

Airflow obstruction Significant reversibility after inhaling a SABA


Pulmonary Function Tests


Asthmatic patient
FEV1 is decreased to < 80% predicted FVC may fall, but FEV1 is much more commonly below predicted TLC is normal to elevated FRC is usually elevated

Asthma Triggers
URI Viral Environmental Allergens Irritants -Perfume -Tobacco smoke -Wood burning stoves Temperature Humidity Exercise Occupational or Recreational Allergens Irritants Drug ASA NSAIDs CHF Cardiac asthma GERD Trigger for bronchospasm Consider in both pediatric and adult patients

Beta blockers Sulfites (food)

6. In considering the atopic patient and asthma. Which of the following is true?
A. Atopy is a strong predisposing factor for the patient with asthma B. A peripheral eosinophil count is always elevated in patients with atopy and asthma C. The CXR is usually abnormal in asthma D. Skin allergy tests are always positive in the atopic patient

The Atopic Patient


Atopy is one of the strongest predisposing factors for the patient with asthma
Genetic predisposition for the development of an IgE mediated response

Investigation into the role of allergy


Complete history in every patient is indicated
There is a high prevalence of positive skin tests among individuals with asthma And benefits of limiting exposure to known allergens

History may help to distinguish seasonal allergies but may be inadequate for perennial allergies

The Atopic Patient


Eosinophil count and IgE may be elevated in asthma
Neither test has sufficient specificity or sensitivity to be used alone in a diagnosis

The chest x-ray and electrocardiogram are usually normal in asthma


May be useful to exclude other pulmonary or cardiac conditions

Sputum examination may be helpful if sputum eosinophilia or infection are suspected

Allergy Treatment Can Improve Asthma Sxs


Key to control
Avoidance of allergens or environmental control

Intranasal corticosteroids
Reduce both allergic rhinitis and asthma sxs in pts with mild asthma

Antihistamines alone or combined with a decongestant


May reduce asthma and rhinitis sxs

Leukotriene modulators
Treat sxs of asthma and allergic rhinitis at the same time

Immunotherapy
May reduce development of asthma in pts with seasonal rhinoconjunctivitis

Control of House Dust Mites


Recommended actions (controversial and expensive)
Encase the mattress in an allergen impermeable cover Encase the pillow in an allergen impermeable cover or wash it weekly Wash the sheets and blankets on the patient's bed weekly in hot water A temperature of 130F is necessary for killing house-dust mites Cochrane Review 2011 Sums up the results of 55 randomized trials of mite control. There was no difference in peak flow (a measure of lung function), asthma symptoms and medication scores, or the number of patients reporting an improvement in their asthma symptoms.

Severity and Control


Severity
A measure of the intrinsic intensity of the disease process established ideally before initiating treatment

Control
Monitored over time to guide adjustments to therapy

Assess Severity and Control Within the Domains of:


Impairment:
Symptoms (pt or caregiver recall of the past 2-4 wks)
Day sxs Nighttime awakenings Rescue SABA use
Quality of life Functional limitations

Risk:
Exacerbations Progressive decline in pulmonary function (or reduced lung growth in children) Adverse events Adverse effects of medication

Low lung function


Assess Severity and Control


Predictors of increased risk for exacerbations or death:
Persistent and/or severe airflow obstruction At least 2 visits to the ED or hospitalizations for asthma within the past year And a history of intubation or admission to the ICU, especially within the past 5 years

CLASSIFYING ASTHMA SEVERITY AND INITIATING TREATMENT IN CHILDREN 04 YEARS OF AGE

CLASSIFYING ASTHMA SEVERITY AND INITIATING TREATMENT IN CHILDREN 511 YEARS OF AGE

CLASSIFYING ASTHMA SEVERITY AND INITIATING TREATMENT IN YOUTHS 12 YEARS OF AGE AND ADULTS

ASSESSING ASTHMA CONTROL AND ADJUSTING THERAPY IN CHILDREN 04 YEARS OF AGE

ASSESSING ASTHMA CONTROL AND ADJUSTING THERAPY IN CHILDREN 511 YEARS OF AGE

ASSESSING ASTHMA CONTROL AND ADJUSTING THERAPY IN YOUTHS 12 YEARS OF AGE AND ADULTS

STEPWISE APPROACH FOR MANAGING ASTHMA IN CHILDREN 04 YEARS OF AGE

STEPWISE APPROACH FOR MANAGING ASTHMA IN CHILDREN 511 YEARS OF AGE

STEPWISE APPROACH FOR MANAGING ASTHMA IN YOUTHS 12 YEARS OF AGE AND ADULTS

Asthma Classification
Mild Intermittent Sxs < 2x/wk Mild Persistent Sxs > 2x/wk, but < 1x/d exacerbations may affect activity Nocturnal sxs > 2x/mo FEV1 or PEF > 80% predicted PEF variability 20%-30% Moderate Persistent Daily sxs, daily use of inhaled SABA, exacerbations affect activity, exacerbations > 2x/wk; may last days Nocturnal sxs > 1x/wk FEV1 or PEF 60%80% predicted PEF variability > 30% Severe Persistent Continual sxs, limited physical activity, frequent exacerbations Frequent nocturnal sxs FEV1 or PEF < 60% predicted PEF variability > 30%

Nocturnal sxs < 2x/mo FEV1 or PEF > 80% predicted

Am Fam Physician. 2011 Jul 1;84(1):40-47

7. In classifying the asthmatic patient, an individual with daily symptoms, daily use of a SABA > 2 times/wk but not daily, and with nocturnal symptoms < 1 time/wk would be classified as:
A. B. C. D. Mild intermittent Mild persistent Moderate persistent Severe persistent

Stepwise Preferred Treatment 0-4 Years of Age


Most recommendations are based on limited data.
Step 1: Step 2: Step 3: Step 4: SABA PRN Low-dose ICS Medium-dose ICS Medium-dose ICS and LABA or montelukast (Singulair) Step 5: High-dose ICS and LABA or montelukast Step 6: High-dose ICS and oral corticosteroid and LABA or montelukast

Stepwise Preferred Treatment 5-11 Years of Age


Step 1: SABA PRN Step 2: Low-dose ICS Step 3: Low-dose ICS and LABA, LTRA or theophylline, OR medium dose ICS Step 4: Medium-dose ICS and LABA Step 5: High-dose ICS and LABA Step 6: High-dose ICS and LABA and oral corticosteroids

Stepwise Preferred Treatment 12 Years of Age


Step 1: SABA PRN Step 2: Low-dose ICS Step 3: Low-dose ICS and LABA OR medium dose ICS Step 4: Medium-dose ICS and LABA Step 5: High-dose ICS and LABA; consider omalizumab (Xolair) if allergies Step 6: High-dose ICS and LABA and oral corticosteroid; consider omalizumab if allergies

8. Which of the following is NOT true?


A. Long-acting beta-agonists like salmeterol can increase mortality if used alone B. ICS use is indicated if PRN use of albuterol exceeds 2 times/wk C. Oral corticosteroids are as effective as IV D. Codeine is an excellent recommended cough suppressant in acute asthma patients

Treatment
Albuterol
The most appropriate treatment for acute bronchospasm is an inhaled SABA
Can use a spacer

Codeine or other cough suppressants are basically ineffective. Nasal and inhaled steroids have an onset that is slow.

Treatment
Inhaled corticosteroids (ICS)
Most potent and effective long-term controller therapy Foundation of therapy for patients of all ages who have persistent asthma (SOR A) ICS improve long-term outcomes in children with mild to moderate persistent asthma (SOR A) Studies are limited comparing them to other alternative treatments

The Expert Panel Recommends


For patients 5 years of age with mild or moderate persistent asthma, the preferred therapy is inhaled corticosteroids (low-dose) (SOR A) Alternative therapies (listed alphabetically due to insufficient data to rank)
Cromolyn (only nebulized form available) LTRAs (leukotriene receptor antagonists) Sustained-release theophylline

Clinical Trials Following Children for Up to 6 Years (Strong Evidence)


Inhaled corticosteroids at recommended doses
Do not have long-term, clinically significant, or irreversible effects on any of the outcomes reviewed

Vertical growth Bone mineral density (BMD) Ocular toxicity Suppression of adrenal/pituitary axis
For children with mild or moderate persistent asthma The potential, but small, risk of delayed growth is well balanced by their effectiveness

Do improve health outcomes (SOR A, B)

Early Intervention with ICS and the Progression of Asthma


Evidence is insufficient to draw conclusions
Early intervention with inhaled steroids likely will improve overall asthma management, but its effect on preventing irreversible airway injury remains to be determined (SOR A, B)

Treatment: Key Points


Long-acting inhaled Beta2-agonists (LABA):
Used concomitantly with low- to mediumdose ICS are the preferred combination therapy for long-term control and prevention of symptoms in moderate and severe persistent asthma (SOR A, B)

Treatment: Key Points


Cromolyn
Used as alternative (not preferred) medications for the treatment of mild persistent asthma (SOR A, B)

Leukotriene modifiers
Again as an alternative (not preferred) medication for the treatment of mild persistent asthma (SOR B)

Treatment: Key Points


Safety is a key consideration Weigh the benefits and risks of therapy
Systemic effects of higher doses of ICS Rare but potential risk of life-threatening or fatal exacerbations with daily LABA Rx Oral corticosteroid use regardless of age

Reassessment

Follow-up
Every 2-6 wk intervals for starting Rx or those that require a step up to regain control Once controlled, reassess at least every 1-6 mo
Measures of control are the same as those to assess severity plus use of, Validated multidimensional questionnaires like the asthma control test (ACT)
http://www.asthmacontrol.com/pdf/ACT_AdultEng.pdf

A step down is recommended for patients whose asthma is well controlled for 3 months or more Decrease dose of ICS gradually
25% to 50% q 3 mo Deterioration in asthma control is highly variable

9. In considering the use of an asthma action plan, which of the following is true?
A. Data is sufficient to support the benefits of verbal action plans. B. A PEF of 80% or more defines the Yellow Zone. C. The use of written action plans is recommended. D. It is recommended that home peak expiratory monitoring be used in patients with mild intermittent asthma.

Asthma Action Plans


Data are insufficient to support or refute the benefits of written action plans (SOR B) Expert Panel opinion
Use of written action plans is recommended (SOR B, C)
Especially for patients with moderate or severe persistent asthma Patients with a history of severe exacerbations

Action plans should


Address individual sxs, and/or PEF measurements Include self-management instructions

Asthma Action Plan


Green Zone:
Usual activity PEF 80% or more of personal best

Yellow Zone:
Some of usual activity PEF 50%-80% of personal best

Red Zone:
Cannot do usual activities PEF less than 50% of personal best

Home Peak Expiratory Flow (PEF) Monitoring


Evidence neither supports nor refutes the benefits of peak flow monitoring
Expert Panel opinion (SOR B)

PEF monitoring should be considered for


Patients with moderate or severe persistent asthma Difficulty recognizing signs of exacerbations History of severe exacerbations

It may enhance clinician-patient communication It may increase patient and caregiver awareness of the disease status and control

Patient Education
Control is enhanced
Ensuring access to education about asthma Skills necessary to manage it Self-monitoring Correct use of inhalers Following a plan for managing asthma longterm Promptly handling signs of worsening asthma

10. A patient who comes to the emergency department in acute respiratory distress caused by a severe attack of asthma should be treated with all of the following EXCEPT:
A. B. C. D. IV fluids Humidified, high-flow-rate oxygen IV corticosteroids IV antibiotics

Acute Asthma Exacerbation


Symptoms
Progressive breathlessness Cough Wheezing Chest tightness

Severity assessment is critical


Using objective measures Focused H&P Measurement of airflow
FEV1 PEF

Acute Asthma Exacerbation


Characterized by
Decreased PEF (< 50% predicted normal)
FEV1 may be more useful in predicting exacerbations

Failure to respond to a beta2-agonist Extreme anxiety due to breathlessness Gasping for air, sweaty, or cyanotic Rapid deterioration over a few hours Severe retractions and nasal flaring Hunched forward

Acute Asthma Exacerbation


Degree of Severity Symptoms & Signs
Mild Dyspnea only with activity Dyspnea interferes with or limits activity Dyspnea at rest interferes with conversation

Initial PEF or FEV1 Clinical Course


PEF > 70% predicted or PB (personal best) Rx at home, prompt relief with SABA, possible oral steroids

Moderate

PEF 40-69% predicted Office or ED visit; or PB relief with frequent use of SABA, oral steroids PEF < 40% predicted or PB ED visit and likely admission, partial relief from frequent SABA, oral steroids, and adjunctive RX ED visit, possible ICU admission, little to no relief from frequent SABA, IV steroids, adjunctive Rx

Severe

Life Threatening

Too dyspneic to speak, sweating

PEF < 25% predicted or PB

Am Fam Physician. 2011 Jul 1;84(1):40-47

Acute Asthma Exacerbation


Risk factors for death from asthma
History of sudden severe exacerbations Prior intubation for asthma Prior admission to ICU for asthma 3 emergency care visits for asthma in the past year Hospitalization or an emergency care visit for asthma within past month > 2 canisters per month of inhaled shortacting beta2-agonist

Acute Asthma Exacerbation


Risk factors for death from asthma (cont)
Current use or recent withdrawal from systemic corticosteroids Difficulty perceiving airflow obstruction or its severity Serious psychiatric disease or psychosocial problems Low SES and urban residence Illicit drug use Sensitivity to alternaria (ascomycete fungi)

Exacerbation Management
SABA by MDI or nebulizer Corticosteroids oral or IV
Strongly consider systemic use
Aids symptom resolution Prevents asthma relapse

Exacerbation Management
Alternatives
Epinephrine (1:1000) Ipratropium added to nebulized SABA
Improves lung function and decreases hospitalizations in children 1-18 yrs of age with mild moderate or severe exacerbations

Levalbuterol Corticosteroids
Initiate or increase anti-inflammatory medication
ICS Cromolyn Consider leukotriene modifiers

Status asthmaticus
ED or inpatient management
Most children who require hospitalization can be identified by a repeat assessment 1 hr after initial treatment. If a patient meets criteria for:
Severe exacerbation, > 86% chance of hospitalization Moderate exacerbation, 84% chance of hospitalization Mild exacerbation, 18% chance of hospitalization

Status Asthmaticus
Poor response to treatment
Early intervention with BiPAP may prevent mechanical intubations Heliox may be a secondary therapy if no response to first-line Rx KetamineConsider for use only in severe exacerbations Magnesium sulfate IV for severe exacerbations
Decreases hospitalization in children 1-18 yrs of age, not adults

Reassess pt shortly after inpatient admission Decision when to D/C from ED or admit must be individualized and depends on response to treatment, pulmonary function, and socioeconomic factors

Exacerbations and Antibiotics


Benefit from antibiotic therapy for asthma exacerbations has not been demonstrated:
Whether administered routinely When suspicion of bacterial infection is low (SOR B)

Exercise-Induced Bronchospasm (EIB)


Occurs in
90% of pts with asthma > 10% of general population

Often indicates poorly controlled asthma


Use long-term control therapy if appropriate

Pretreatment prior to exercise


SABAs, LABAs are effective in up to 80% of pts LTRAs are effective in up to 50% of pts

Encourage pts to warm up prior to exercise and consider wearing a mask or scarf in cold weather

Goals of Therapy Asthma Control Summary


Minimal or no chronic symptoms day or night Minimal or no exacerbations No limitations on activities; no school or work missed Maintain (near) normal pulmonary function Minimal use of short-acting inhaled beta2agonist Minimal or no adverse effects from medications

Websites
Expert Panel Report 3 (EPR3): Guidelines for the Diagnosis and Management of Asthma (440 pgs)
http://www.nhlbi.nih.gov/guidelines/asthma/asthgdln.htm

ICSI Heath Care Guidelines: Diagnosis and Management of Asthma (70 pgs)
https://www.icsi.org/_asset/rsjvnd/Asthma.pdf

The Journal of Family Practice: Help Patients Gain Better Asthma Control (10 pgs)
http://www.jfponline.com/Pages.asp?AID=6634

AsthAsthma During Pregnancy During Pregnancy


Increased risk
Perinatal mortality Preeclampsia Preterm birth LBW infants
LBW infants at greater risk for: DM, hypertension, heart disease as adults
http://www.nhlbi.nih.gov/health/prof/lung/asthma/astpreg/astpreg_qr.pdf

Asthma During Pregnancy


Cohort study (140,299 pregnancies Tennessee Medicaid program from 1995-2003) 23% of white pts, 40% of black pts had a hospitalization or ED visit during their pregnancy
Black pts were 1.6 times more likely than white pts to receive care for an exacerbation (95% CI 1.5-1.7)

77% of pts did not use control medications Dose response trend (P < 0.001) between LBW and increasing use of oral corticosteroids

Asthma During Pregnancy


Monthly evaluation of asthma history and pulmonary function Albuterol is the preferred SABA ICSs are preferred controller medication
Budesonide has the most reliable safety profile Cromolyn, LTRAs, and LABAs may be alternatives, but have lower efficacy or less safety data available LABA should not be used as monotherapy

Comorbid allergic rhinitis can be managed with intranasal steroids

Asthma During Pregnancy


Extremely variablesymptom severity may improve, worsen, or remain unchanged in approximately equal portions as compared with the pregravid state Uncontrolled asthmamaternal and fetal complications
Hyperemesis HDP Vaginal bleeding Complicated labor IUGR Preterm birth Neonatal hypoxemia

Management
Treatment is organized around 4 components of management
1. Assessment and monitoring of asthma, including objective measurement of pulmonary function 2. Control of factors contributing to asthma severity Spirometry Ultrasound

Allergens Irritants 3. Patient education Access Skills 4. A stepwise approach to pharmacologic Medications therapy Frequency

Quick Relief
Short-acting bronchodilator
2-4 puffs as needed for symptoms

Intensity of treatment will depend on severity of exacerbation


Up to 3 treatments at 20-minute intervals or a single nebulizer treatment as needed Course of systemic corticosteroids may be needed

Use of SABA > 2 times/week in intermittent asthma, or daily use in persistent asthma, may indicate the need to initiate (or increase) long-term control therapy

Theophylline
Studies and clinical experience confirm the safety of theophylline at recommended doses during pregnancy
Serum concentration 5-12 mcg/mL

What one seesincreased side effects and discontinuation of medication

Stepwise Treatment Approach


Severity
Mild Intermittent Mild Persistent Moderate Persistent Symptoms/Day

Preferred Treatment
No daily medication Low-dose inhaled corticosteroid* (FDA Category C) Low-dose inhaled or medium-dose inhaled corticosteroid; +/- long acting inhaled beta2-agonist (FDA Category C) High-dose inhaled corticosteroid AND long-acting inhaled beta2agonist, AND, if needed, systemic corticosteroid (FDA Category C)

Symptoms/Night
2d/week 2n/month > 2d/w, but < daily > 2n/month Daily > 1n/week Severe Persistent Continual Frequent
* More data on using budesonide during pregnancy than on using other inhaled corticosteroids

Answers
1. A 2. D 3. A 4. B 5. C 6. A 7. B 8. D 9. C 10. D

What Winners Know in Sports Medicine


Joseph Garry, MD, FACSM, FAAFP

Associate Professor University of Minnesota Minneapolis, Minnesota

Disclosures
Dr. Garry discloses that he holds stock in Pfizer and Merck pharmaceuticals.

It is the policy of the AAFP that all individuals in a position to control content disclose any relationships with commercial interests upon nomination/invitation of participation. Disclosure documents are reviewed for potential conflicts of interest. If conflicts are identified, they are resolved prior to confirmation of participation. Only participants who have no conflict of interest or who agree to an identified resolution process prior to their participation were involved in this CME activity.

Learning Objectives
1. Cite recent updates to concussion management 2. Review key factors related to medical conditions in athletes (AMS, IM, EIB, cardiac, Marfans, FAT, PPE) 3. Cite management and care for acute shoulder dislocation 4. Review lower extremity conditions in athletes (FAI, acute ligamentous knee injury, patellofemoral pain, chronic exertional compartment syndrome, ankle sprains)

Question 1.

You are a team physician for your local high school soccer team and you diagnose a player with a concussion during the game. Which statement is most correct regarding your next action?
A. The athlete may return to the game if asymptomatic B. You recommend a change in helmets and use of a mouthguard to prevent a concussion C. You expect progressive resolution over 7-10 days D. You have no special concerns if this adolescent athlete had a previous concussion last week

Concussion
Complex pathophysiological process affecting the brain, induced by traumatic biomechanical forces.
Simple vs Complex classification abandoned
Most concussions fall into the simple category with progressive resolution w/o complication over 710 d, and may be longer for children/adolescents

Evolving injury
Consensus Statement, Clin J Sport Med 2013

Concussion
Management

Physical & cognitive rest until symptoms resolve Monitor for deterioration post-injury and medical evaluation post-injury Graded program of exertion prior to medical clearance and return to play No same day RTP for athletes < 18 yo No distinction in management between elite and non-elite athletes
Consensus Statement, Clin J Sport Med 2013

Concussion
Most Current Update

Consensus statements address concussion in those > 13 years Child-SCAT3 is developed for ages 5-12 years
No return to sport is recommended prior to successful return to school No evidence that currently available protective equipment will prevent concussion
Consensus Statement, Clin J Sports Med 2013

Concussion
Return-to-Play Guidelines Rest until asymptomatic Day 1 Light aerobic activity Day 2 Sport-specific exercise Day 3 Noncontact training drills Day 4 Full contact after medical clearance Day 5 Game play
If the athlete becomes symptomatic at any time, then drop back to previous activity after 24 hr rest and restart progression

Concussion
Second Impact Syndrome

Children & adolescents only


Occurs when second concussion occurs

prior to resolution of first

Rapid brain swelling

No reliable treatment
High rate of morbidity and mortality

Question 2.

Which one of the following is best for preventing acute mountain sickness?
A. B. C. D. Acetazolamide (Diamox) started the day before arriving at altitude Prednisone started the day before arriving at altitude Moderate alcohol consumption on the first day at altitude Ascending quickly, then resting to acclimatize before beginning planned activities

Acute Mountain Sickness


Altitude > 8000 feet Symptoms include HA, nausea or vomiting, anorexia, dizziness, fatigue, weakness, difficulty sleeping Slow ascent is best way to prevent AMS Acetazolamide or dexamethasone can be use to prevent or treat Alcohol on first day can exacerbate AMS

Altitude Sickness
HAPE
Prevention can include acclimation to altitude, nifedipine or sildenafil

HACE
Prevention is acclimation to altitude Treatment = immediate

Treatment = descent, O2,


nifedipine, sildenafil or dexamethasone

descent, O2,
dexamethasone

Question 3.

Which of the following statements is most correct?

A. The athlete with infectious mono has the greatest risk for splenic rupture in the first week of symptoms B. A primary objective of the PPE is to screen for lifethreatening conditions C. Medical conditions would be an uncommon reason for restrictions resulting from the PPE D. EIB is diagnosed by a change in FVC from baseline to the lowest level within 30 min post exercise of > 10%

Infectious Mononucleosis
Hoaglands criteria
50% lymphocytes (10% atypical) in presence of fever, pharyngitis, adenopathy and confirmed by serological testing

Risk of splenic rupture is 0.1-0.5% and highest in first 3 weeks Physical examination for splenomegaly has poor sensitivity and specificity

Infectious Mononucleosis
Return to play
Based on clinical judgment Generally no RTP within 21 days, if symptomatic with fever or fatigue, or in the presence of organomegaly At 21 days may resume light aerobic activity and

progress as tolerated
Full RTP when fully asymptomatic and post 21 days
Putukian et al. Clin J Sport Med 2008

Exercise-Induced Bronchospasm
Change in FEV1 from baseline to lowest level within 30 min post exercise of > 10%
FEV1 10-24% (mild) FEV1 25-50% (moderate) FEV1 > 50% (severe) Manage with warm-up exercise, SABA 15 min prior to exercise; if SABA needed daily then add daily ICS, leukotriene receptor antagonist, or mast cell stabilizer LABA not recommended
Parsons et al. Am J Respir Crit Care Med 2013

Objectives of the PPE


Primary Objectives
1. Screen for conditions that may be life-threatening or disabling 2. Screen for conditions that may predispose to injury or illness

Secondary Objectives
1. Determine general health 2. Serve as an entry point to the healthcare system for adolescents 3. Provide an opportunity to initiate discussion on healthrelated topics

PPE Screening
Elevated BP is the most common CV abnormality identified during the PPE
Children and adolescents:

Average SBP or DBP > 95th percentile for sex, age, and height on at least 3 separate occasions
National High Blood Pressure Education Program Working Group on High Blood Pressure in Children and Adolescents. Pediatrics 2004

Adults:

Two or more BP readings on each of 2 or more office visits


Chobanian et al. JAMA 2003

Conditions Resulting in Restriction


PPE
60 50 Percentage 40

30
20 10 0

53

47 20

Medical MSK CV Percent based on 26,899 evaluations (10 studies)

Question 4.

Which of the following conditions is the most frequent cause of sudden cardiac death in athletes under the age of 28 years?

A. B. C. D.

Coronary artery anomalies Hypertrophic cardiomyopathy Myocarditis Coronary artery disease

Distribution of Cardiovascular Causes of Sudden Death in 1435 Young Competitive Athletes (Age < 35 Years)

Copyright American Heart Association, Inc. All rights reserved.

Maron B J et al. Circulation. 2007;115

Hypertrophic Cardiomyopathy
Inherited as an autosomal dominant trait with variable penetrance
ECG 75-95% will have abnormal 12-lead ECG No characteristic pattern Echocardiography Diagnosis is best made with echocardiogram even in the absence of symptoms

Marfan Syndrome
Autosomal-dominant connective tissue disease

Stigmata: arachnodactyly, high arched palate,


hyperextensible joints, anterior chest deformity, scoliosis, arm span > height, > 70 in, > 72 in, myopia

Complications: Cardiac (ascending aortic


dilatation, aortic dissection, MVP, dysrhythmias), Pulmonary (sp. pneumothorax), Occular (lens dislocation) High suspicion physical exam, then echocardiogram (95% abnormalities), SLE (60% lens subluxation)

Question 5.

A 17-yo thin female ballet dancer presents with her 2nd stress fracture in the past 18 months. She is otherwise well, and takes daily MVI since her first stress fracture. She does admit to some intermittent restrictive eating habits. Menarche at age 14 and she has had 2 periods in the past 5 months. Her uHCG is negative. You counsel her that

A. She should be considered for a DXA scan B. She should start an OCP to help with bone density, as this will be more beneficial than weight gain C. Disordered eating is no more common in ballet dancers than other activities D. Functional hypothalamic amenorrhea is diagnosed and should be treated

Female Athlete Triad


Spectrum Disorder

Low energy availability Menstrual irregularity Osteoporosis

Multidisciplinary treatment team (MD/DO, RD, PhD)


Nattiv et al. Med Sci Sports Exer 2007

Female Athlete Triad


Spectrum Disorder

A statements
Menstrual irregularity and low BMD increase risk of stress fracture Disordered eating, ED, amenorrhea occur more frequently in sports that emphasize leanness

B statements
Functional hypothalamic amenorrhea is a diagnosis of exclusion
Nattiv et al. Med Sci Sports Exer 2007

Female Athlete Triad


Spectrum Disorder

C statements
Screening should occur at PPE or annual exam Athletes with 1 component should be assessed for others Athletes with disordered eating should be referred to MH DXA if stress fracture & 6 months of amenorrhea/ oligomenorrhea/disordered eating or ED In functional hypothalamic amenorrhea, BMD increases with weight increase moreso than with Nattiv et al. Med Sci Sports Exer 2007 OCP/HRT supplementation

Question 6.

A17-yo football player presents with his first anterior shoulder dislocation. You recommend which of the following to reduce his risk of recurrence?
A. Sling and swathe for 6 weeks B. Surgical intervention

C. NSAIDs and sling until no pain, followed by physical therapy D. Reassurance since all interventions have similar outcomes regarding recurrence

Anterior Shoulder Dislocation


For younger (< 25 yo) patients with a first anterior shoulder dislocation, surgical treatment results in fewer subsequent dislocations and decreased likelihood for an unstable shoulder.
Cochrane 2003 Bottoni et al. Am J Sports Med 2002 Arciero et al. Am J Sports Med 1994

Adolescent Athlete

Anterior Shoulder Dislocation


70% are anterior
90% of ASD are traumatic Shoulder instability in the young athlete is likely result
Dynamic muscular control is often lost

Early reduction on the field ease of reduction

pain reduction and

Femoral Acetabular Impingement


C sign in 85%

C sign

Insidious onset (FAI) as intermittent discomfort with repetitive hip motion progresses to more constant or intense pain, worse with activity & sitting ROM with pain in HF and IR most common (+) FAdIR (most sensitive test for FAI) X-ray should include the modified Dunn view
MacFarlane et al. Ann R Coll Surg Engl 2010; Clohisy et al. Clin Orthop Relat Res 2009; Phillipon et al. Knee Surg Sports Traumatol Arthrosc 2007

Cam lesion

Flexion Adduction Internal Rotation

Modified Dunn View - Hip in 90 flexion and 20 abduction

Femoral Acetabular Impingement


MR arthrogram is test of choice
Intra-articular anesthetic if MRa normal and diagnosis remains in question

PT and activity modification may alleviate symptoms


Surgical consults for refractory symptoms
Early surgical consult due to risk of OA Treatment includes laparoscopic repair or resection (if labral tear), or osteoplasty or labral rim trimming (if FAI) Surgical treatment improves symptoms, and those with more significant arthritic changes are less likely to benefit
Toomayan et al. AJR 2006 Ng et al. Am J Sports Med 2010

Question 7.

Which statement is true regarding high school athletes and injury?

A.

The knee is the most commonly injured body part in high school athletics Most injuries occur in practice as opposed to competition The ankle is the most commonly injured body part in high school athletics Football has the highest injury rate

B.

C.
D.

National High School


Sports-Related Injury Surveillance Study 2011-2012
400

378

Injury Rate/100,000 Athletic Exposures

350
300

250
200

150
100 82 99

145 158
112

176

198 200 210

50 0

National High School


Sports-Related Injury Surveillance Study 2011-2012
Injury Rate/1000 Athlete Exposures

15

10

Competition

Practice

Most Commonly Injured Body Region (%)


National High School Sports-Related Injury Surveillance Survey 2011-2012

All Head/Face Ankle Knee Hip/Thigh Hand/Wrist Shoulder Trunk 25.1 16.1 13.4 9.8 8.6 6.6 5.0

Low Leg
Arm/Elbow Foot

4.5
4.0 3.4

Question 8.

Which of the following is the most commonly injured knee structure in high school athletics?

A. B. C. D.

Meniscus Medial collateral ligament (MCL) Anterior cruciate ligament (ACL) Patella or patellar tendon

Most Commonly Injured Knee Structure


Percent of all Injuries
National High School Sports-Related Injury Surveillance Survey 2011-2012

Male
MCL 31.8

Female
17.4

Total
26.6

ACL
Patella Patellar tendon Meniscus

20.6
14.8 17.4

29.8
32.8 12.5

23.9
21.3 15.6

LCL
PCL

8.5
3.7

8.8
2.5

8.6
3.3

Question 9.

The following is true regarding ACL injuries.

A. Most injuries occur in practice B. Prevention programs are ineffective C. Most occur as noncontact injuries D. Most occur as isolated injuries

ACL Disruption

Isolated ACL disruption occurs < 10% of injuries

Meniscal tears (60-75%) Articular cartilage injury (46%) Complete collateral ligament tears (5-24%)

Key historical points


Noncontact mechanism Audible pop Early onset knee effusion Inability to continue play

MRI for confirmation (sens 86%, spec 95%)


Spindler et al, NEJM 2008

Crawford et al, Br Med Bull 2007

ACL Tears
70% are noncontact injuries
Twisting with rotatory force or hyperextension Female athletes at higher risk (LOE 2) Neuromuscular training reduces risk of injury in female and male athletes Surgical repair is treatment of choice for instability
Sadoghi et al. JBJS(A) 2012; FPIN 2011; www.fpin.org; Agel et al. Am J Sports Med, 2005; Scholten et al. JFP, 2003

Lachman Testing
Most sensitive test for ACL integrity A negative Lachman is the best test for ruling out an ACL rupture

Jackson et al. Ann Int Med 2003; 139 Scholten et al. J Fam Pract 2003; 9 Benjaminse et al. J Orthop Sports Phy Ther 2006; 36(5)

Question 10.

A 22-yo female runner presents with peri-patellar knee pain x 5 weeks with painful giveway of the knee without swelling or locking. What is your next best course of action?

A. Obtain knee MRI B. Stop all running until improved C. Evaluate pelvic mechanics and strength D. Recommend orthotics

Anterior Knee Pain


Women >> Men
Peripatellar and anterior knee pain Pain with prolonged knee flexion

Pain with stairs


(+) Patellar findings on exam Tight ITB (Obers test) Pelvifemoral dysfunction and quad weakness
From: Wikipedia

gluteal

Anterior Knee Pain


Management

Short-term pain relief


NSAIDs

Quadriceps strengthening Physical therapy Pelvic-femoral rehab

Patellar taping (pain relief)


Bracing (benefit in runners)
Cochrane Database of Systematic Reviews, 2009, 2008, 2004, 2003, 2002 Wilson et al. J Ortho Spt Phy Ther, 2003

Extensor Mechanism Rupture


Quadriceps Tendon Most common form Age > 40 Acute injury Extensor lag Indentation at superior patella and low-riding patella on radiographs Surgical correction Patellar Tendon Least common form Age < 40 Acute injury Extensor lag High riding patella on exam or radiographs Surgical correction

Chronic Exertional

Compartment Syndrome
Median age is 20 years Recreational runners, elite athletes, military recruits May account for 27% of cases of chronic anterior leg pain
95% involve the anterior and lateral compartments Risks include anabolic steroids, creatine use, eccentric exercise, biomechanics
Figure courtesy of Orthoinfo.aaos.org

Chronic Exertional

Compartment Syndrome
Symptoms: pain onset occurs at same distance and intensity of exercise; burning, aching, pressure sensation; bilateral in 70-80%
Symptoms resolve with 20-30 min rest

Measurement of intra-compartmental pressures at rest and post-exercise Definitive treatment = fasciotomy

Question 11.

A 26-yo male suffers an inversion ankle injury and ambulates to the ED for evaluation. You find lateral ankle swelling and tenderness over the anterior distal fibula. The next best course of action includes A. Recommendation for early range of motion for the ankle B. Radiographs of the ankle C. Strict non-weight-bearing for 3-5 days D. Avoidance of use of NSAIDs

Ankle Sprain
Lateral sprains most common (80%)
Initial treatment with rest, ice, elevation, protection

Early range of motion improves recovery (A)

BMJ. www.clinicalevidence.com Karlsson et al. Scand J Med Sci Sports, 1999 Dettori et al. Mil Med, 1994 Thacker et al. Am J Sports Med, 1999 Wolf Michael et al, Am Fam Physician. 2001 Jan

Ankle Sprains
Clinical Recommendations

NSAIDs reduce pain post-injury and may reduce time for return to play
Semi-rigid ankle supports are recommended as functional treatment for ankle injuries Graded exercise programs and proprioceptive training are recommended to reduce the risk of ankle sprains
AFP 2006; 74(10)

Ottawa Ankle Rules


X-ray if
Malleolar pain or pain in region of navicular or proximal 5th metatarsal

AND
Inability to weight bear 4 steps at exam OR Discrete bone tenderness at any site noted

Stiell et al. JAMA, 1994 (A) Ivins, Am Fam Physician.2006 Nov 15

Answers
1. 2. 3. 4. 5. 6. 7. C A B B A B D 8. B 9. C 10. C 11. A

Review of the Diseases of the Lower GI Tract


David G Weismiller, MD, ScM, FAAFP Department of Family Medicine
The Brody School of Medicine at East Carolina University
weismillerd@ecu.edu

Disclosure Statement
Dr. Weismiller has nothing to disclose.

The AAFP has selected all faculty appearing in this program. It is the policy of the AAFP that all CME planning committees, faculty, authors, editors, and staff disclose relationships with commercial entities upon nomination or invitation of participation. Disclosure documents are reviewed for potential conflicts of interest and, if identified, they are resolved prior to confirmation of participation. Only those participants who had no conflict of interest or who agreed to an identified resolution process prior to their participation were involved in this CME activity.

Learning Objectives
1. Discuss the diagnosis and treatment of irritable bowel syndrome. 2. Recognize the common forms of malabsorption. 3. Review the diagnosis and treatment of cancer of the small intestine, large intestine, and anus. 4. Summarize other clinical entities of the lower GI tract, including constipation and infections.

1. A 20-year-old college student comes to student health services to discuss her multi-year history of abdominal pain and constipation. It has gotten worse since she returned to school this fall. She describes crampy pain and bloating that eases after defecation. Her bowel movements are firm and difficult to pass, and occur about every 4 days on average. She denies vomiting, weight loss, blood in the stool, or melena. Her menses are regular and she is otherwise healthy. Her family history is negative for any gastrointestinal or genitourinary diseases. Her abdominal examination is normal. You diagnose her with irritable bowel syndrome. Which ONE of the following would be most appropriate at this time?

A. B. C. D. E.

A therapeutic trial of increased soluble fiber intake A therapeutic trial of lubiprostone (Amitiza) Abdominal ultrasonography Abdominal CT Colonoscopy

Irritable Bowel Syndrome


Defined
GI syndrome characterized by altered bowel habits and abdominal pain, in the absence of detectable structural abnormalities Prevalence
10%-15% in North America

Irritable Bowel Syndrome Clinical Features


Patterns
80% diarrhea + constipation + pain 20% painless diarrhea only

Altered bowel habits


Alternation of diarrhea with constipation
Usually beginning in adult life, one usually predominant Usually, constipation begins as episodic, later becomes constant Evacuation usually feels incomplete No nocturnal diarrhea Worse with stress

Additional Clinical Features


Abdominal pain
Episodic and cramping, highly variable; rarely interferes with sleep

Gas and flatulence


C/o distention, increased belching, increased flatulence (objective measurements are normal)

Upper GI symptoms
25%-50% c/o dyspepsia, heartburn, nausea, vomiting

Irritable Bowel Syndrome Pathophysiology


Remains uncertain Heredity and environmental factors play a potential role. Abnormal myoelectric activity
Basal rhythm 3 cycles/min in IBS (6 cycles/min is normal)

Visceral afferent hypersensitivity Microscopic mucosal inflammation Psychosocial dysfunction

Irritable Bowel Syndrome Diagnosis


Not purely a diagnosis of exclusion Careful H&P
Timing and pattern of pain Targeted testing when no alarm symptoms

All patients
CBC, chemistry panel

If diarrhea predominant
TSH, stool for O&P, 24 h stool occasionally helpful

If > 40 at onset
Consider flex sig, BE, or colonoscopy to r/o cancer If younger, consider endoscopy to rule out inflammatory bowel disease

If mostly diarrhea
R/o lactase deficiency, sorbitol excess, celiac disease

If mainly upper GI symptoms


Consider EGD

In our Case
Patients symptoms are No red flags consistent with irritable Unintentional and bowel syndrome (IBS). unexplained weight loss History, physical Rectal bleeding examination, and Family history of bowel laboratory evaluation did or ovarian cancer not show any evidence of Change in bowel habits peptic ulcer disease, persisting MORE than 6 celiac disease, thyroid disease, or inflammatory weeks in a person > 60 bowel disease. The patient does not have any of these findings
and therefore does not require any additional testing to confirm the diagnosis of IBS.

Irritable Bowel Syndrome


Treatment
Self-help, patient education, cognitive behavioral treatment Dietary changes, when indicated, insoluble fiber may be helpful, powerful GI tract stimulant
High-fiber diet, bran

Soluble fiber (eg, psyllium) NOT EFFECTIVE Lubiprostone is effective for constipation-predominant IBS.
Works by increasing the amount of fluid in intestine, making it easier for stool to pass

Antispasmodics
Dicyclomine, tincture of belladonna, hyoscyamine

Antidepressants
Low dose: Analgesia, mood, colonic transit slowing

Antidiarrheal agents
Diphenoxylate, loperamide; no help with pain
Wilkins T, Pepitone C, Alex B, Schade RR: Diagnosis and management of IBS in adults. Am Fam Physician. 2012;86(5):419-426; and Ruepert L, Quartero AO, de Wit NJ, et al. Bulking agents, antispasmodic and antidepressants for the treatment of irritable bowel syndrome. Cochrane Database Syst Rev. 2011;(8):CD003460.

IBS Treatments
Evidence-Based Recommendations
Lubiprostone is effective for constipationpredominant IBS (SOR B). Antidepressant treatment of pain-predominant IBS
TCAs (SOR B) SSRIs (insufficient data)

Psychologic treatment
Hypnotherapy (SOR B) Cognitive-behavioral therapy (SOR B)
essentialevidenceplus.com

Malabsorption
Defined
Impaired absorption of nutrients

Clinical features
Symptoms (consistent across types)
Weight loss Diarrhea Bloating and flatulence Malnutrition

Classification of Malabsorptive Disorders


Luminal disorders
Hepatobiliary disease Pancreatic insufficiency Altered bacterial flora
Jejunal diverticulosis Blind-loop syndrome Scleroderma

Wall disease
Regional enteritis Amyloid disease

Vascular disease
Mesenteric artery disease Cardiac failure

Mucosal abnormalities
Structural
Celiac disease (sprue) Tropical sprue Radiation Enteritis Agammaglobulinemia

Metabolic disorders
Pancreatic adenoma Carcinoid Hypopituitarism Carcinoma of the bronchus

Biochemical
Abetalipoproteinemia Amino acid absorptive disorders

Lymphatic abnormalities
Whipples disease Lymphangiectasia Lymphoma

Iatrogenic causes

Surgery and drugs Rigas B, Spiro H. Clinical Gastroenterology, 4th ed. McGraw Hill, 1995.

Malabsorption Physical Findings


Pallor, anemia Muscle wasting Hair loss Edema Pagophagia (ice pica) Amenorrhea Paresthesias Glossitis Cheilitis Bruising

Malabsorption Diagnostic Tests


Decrease in serum carotene, cholesterol, albumin Quantitative or qualitative fecal fat Carbon 14 triolein breath test
Decrease CO2 fat malabsorption

Small intestine films and biopsy Cultures from small intestine Breath hydrogen tests (for bacterial overgrowth) General therapy for malabsorption is nutritional.

Common Syndromes Maldigestive and Malabsorptive


Exocrine pancreatic insufficiency
Dx: Steatorrhea + normal D-xylose (no proximal small bowel dysfunction) Tx: Pancreatic enzyme replacement Giardiasis Cryptosporidium

Bile acid deficiency short gut Parasitic diseases Immunodeficiency states Lactase deficiency

Dx: Breath hydrogen lactose challenge Tx: Diet

2. A 19-year-old patient presents to your office for follow-up. She reports continued intermittent abdominal cramping and bloating, diarrhea, fatigue, and a 4.5-kg weight loss. She initially was diagnosed with irritable bowel syndrome, but you suspect celiac disease. Which one of the following should be used to establish the diagnosis? A. Colonoscopy B. Serologic testing C. Serologic testing initially, followed by endoscopy if test results are positive D. Serologic testing initially, followed by colonoscopy E. Clinical impression is sufficient.

Celiac Sprue
Chronic intestinal malabsorption disorder
Intolerance to the gliadin fraction of gluten in wheat

Features
Flat jejunal mucosa with clinical and/or histologic improvement following withdrawal of dietary gluten

Celiac Sprue
Symptoms and signs
No typical presentation May be asymptomatic Steatorrhea common Failure to thrive Anemia Symptoms of various deficiency states Bone loss Arthritis Neuropsychiatric disease

Laboratory
Fe deficiency anemia in children Adults
Iron deficiency Folate deficiency Vitamin D deficiency

Celiac Disease
Diagnosis
Several serologic tests are readily available for diagnosis.
Endomysial antibody* (sensitivity 81%, specificity 97%) IgA anti-tissue transglutaminase antibody* (sensitivity 79%-90%, specificity 98%) Deamidated gliadin peptide antibody (IgG/IgA) (sensitivity 95%98%)

No one test has been demonstrated in clinical studies to be more accurate than another. Test while consuming a diet of gluten-containing foods. Gold standard: Histologic assessment of multiple proximal small intestinal mucosal biopsies villous atrophy
* AGA recommends these tests as initial serologic testing in adults.

Celiac Disease Treatment


Gluten avoidance

3. Inflammatory bowel disease is characterized by which of the following?


A. Men are affected more often than women. B. Older children tend to be most often affected. C. An association with an infection with Giardia. D. An immune role as mediator of tissue injury in the disorder

Inflammatory Bowel Disease


Onset
Usually young adults Affects men and women equally More common among Caucasians in N. America and N. Europe

Ulcerative colitis Crohns disease

Inflammatory Bowel Disease


Spectrum
Crohns, indeterminate, ulcerative colitis 600,000 people in the US have some form of inflammatory bowel disease.

Pathogenesis
Some genetic role No identified infectious agent Strong evidence for immune role as mediator of tissue injury unknown trigger Serologic markers (ASCA, G0F/G2F, etc)

Ulcerative Colitis
Incidence: 10/100,000, 25% family history Pathology
CONFINED TO MUCOSA Starts in rectum, moves proximally WITHOUT skips

Clinical features
Mild to severe at start Abrupt onset Rectal bleeding, diarrhea, fever, cramping pain, weight loss

Differential Diagnosis of Ulcerative Colitis and Recommended Testing


Disease Amebic dysentery Bacterial colitis Findings that suggest diagnosis Travel to endemic areas or exposure to illness Should be routinely considered; exposure history may increase suspicion Recent antibiotic use Increased suspicion with disease not limited to colon Risk factors for vascular disease Nonbloody stools Immunocompromised Evaluation Anti-amebiasis antibodies, microscopy (O&P) Stool culture, including testing for E. coli 0157:H7 Stool studies for C. difficile toxin Endoscopic biopsy Endoscopic biopsy Endoscopic biopsy Endoscopic biopsy

Clostridium difficile infection Crohns disease Ischemic colitis Microscopic colitis Viral or parasiteinduced colitis

Medical Therapies for UC


Medication
Sulfasalazine 5-Aminosalicylic acid Oral Suppository Enema Hydrocortisone Enema Foam Prednisone

Dosage for active disease


4-6 g/day divided QID

Maintenance dosage
2-4 g/day

Adverse effects
HA, interstitial nephritis, N/V Interstitial nephritis Anal irritation Difficulty retaining, rectal irritation Rectal irritation Rectal irritation Adrenal suppression, impaired wound healing, PUD, etc. Increased risk infection, lymphoma Allergic reactions, BM suppression, infection Infection, nephrotoxicity, seizures

2-4.8 g/d, divided TID 1000 mg/d 1-4g/d

1.2-2.4 g/d 500 mg 1-2x/d 2-4g daily to every third day Not recommended Not recommended Not recommended

100 mg 90 mg 1-2X/d 40-60 mg/d until clinical improvement, then taper 5-10 mg/w 40-60 mg/d 5-10 mg/kg weeks 0, 2, and 6 Not recommended 2-4 mg/kg/d

Methylprednisolone Infliximab (Remicade) Azathioprine (Imuran) Cyclosporine

Not recommended 5-10 mg/kg q 4-8 weeks 1.5-2.5 mg/kg/d Not recommended

Ulcerative Colitis SORT:


Key Recommendations for Practice
Clinical recommendation
5-aminosalicylic acid is highly effective for inducing remission and preventing relapse. Oral corticosteroids are effective for inducing remission. Remicade is effective for inducing remission in corticosteroid-refractory UC. Imuran is effective for preventing relapse. Probiotics Lactobacillus GG and E. coli Nissle 1917 (Mutaflor) are as effective as 5-ASA in maintaining remission.
Am Fam Physician. 2013; 87(10):699-705.

Evidence rating
A B A B B

Ulcerative Colitis Link with Colon Cancer


2.8-15x as likely to develop colon cancer Colonoscopy q 1-2 years* [SOR: C]
Initiated 8-10 years after UC is diagnosed

* Kornbluth A, Sachar DB. Practice Parameters Committee of the ACG. Ulcerative colitis practice guidelines in adults. Am J Gastroenterol. 2010;105(3):501-523.

Crohns Disease
Pathology
FULL WALL THICKNESS Any part of GI tract may be affected. Terminal ileum most common site SKIPS

Clinical features
Insidious onset Commonly with mild diarrhea and pain

Diagnosis
Cultures O&P Biopsy Colonoscopy Radiographs

4. Possible treatments for Crohns disease include all of the following EXCEPT: A. B. C. D. Steroids Aspirin Cyclosporine Sulfasalazine

Crohns Disease
Treatment*
Patient education and support Sulfasalazine and 5-ASA Similar cancer risk as UC after Steroids long-standing disease Immunosuppressants
6-MP Azathioprine Methotrexate Cyclosporine

similar screening recommendation and controversy*

Antibiotics

Anti-tumor necrosis factor agents (biologics), eg, Infliximab Steroids, eg, Budesonide Surgery * Winawer S, et. al. Colorectal cancer screening and
Surveillance: Clinical Guidelines and Rationale Update Based on New Evidence. Gastroenterol. 2003;124:544-560.

Metronidazole for rectal fistulas

Crohns - SORT: Key Recommendations for Practice

Am Fam Physician. 2011; 84(12):1365-1375.

GI Infections

Common GI Infections
Goals
Replace the fluid lost through diarrhea and vomiting. Identify and eradicate the causative agent.

Diagnosis
History: Work, travel, eating, others ill, recent antibiotics, immunocompromised, HIV Vomiting implies gastric involvement with preformed toxin. Pain implies distention and inflammation. Lab: Stool for fecal blood and leukocytes, cultures, O&P* Sigmoidoscopy
* Most acute diarrhea is viral and self-limited. Order O&P if history of travel or diarrhea is chronic.

Rehydration
Recommendations
Oral rehydration therapy (ORT)
Recommended by the AAP as the preferred treatment of fluid and electrolyte losses caused by diarrhea in children with mild to moderate dehydration. WHO reduced-osmolarity oral rehydration solution (ORS) now recommended for all rehydration in children, both cholera-endemic areas and non-cholera endemic. Pedialyte, Rehydrolyte, Ceralyte, Infalyte Sports drinks, diluted fruit juices, watery soups (adults)

Intravenous therapy
Parenteral saline and electrolytes Ringers lactate

Dietary adjustments (following rehydration)


Boiled vegetables, starches, soups, yogurt Gavin N, et al. Pediatrics. 1996;98(1):45-51. Avoid high fat. Simple sugars as opposed to complex carbohydrates

5. Which of the following is the most common cause of infectious diarrhea in children both in developed and developing countries? A. Campylobacter B. Rotavirus C. Shigella D. Norovirus

Common GI Infections
Rotavirus Norovirus Campylobacter Salmonellosis Shigella Amebiasis Giardiasis Cryptosporidium Pseudomembranous enterocolitis Travelers diarrhea Vibrio cholerae

(Viral) Gastroenteritis
Majority of Gastroenteritis Is Viral
Rotavirus Norovirus Adenovirus Astrovirus Rotavirus is the most common cause in children and produces similar incidence rates in both the developed and developing world. Norovirus is the leading cause of gastroenteritis among adults in America, causing greater than 90% of outbreaks.

Rotavirus
Most important viral cause of severe gastroenteritis in children worldwide
Most common cause in US Large-volume diarrhea without blood or leukocytes in stool Daycare centers fecal oral spread

Dx: Immune-based assays of stool, PCR Complications: Necrotizing enteritis, biliary atresia, intussusception, chronic diarrhea

Rotavirus
Treatment is generally supportive. Immunization (SOR A)
CDC/AAP recommend universal immunization in US with oral PRV vaccine.
2 months
First dose minimum 6 weeks, maximum < 15 weeks

4 months 6 months
Must complete by 8 months

Norovirus
Norovirus is the leading cause of gastroenteritis among adults in America, causing greater than 90% of outbreaks.
Top pathogen contributing to domestically acquired foodborne illness Second most common foodborne illness resulting in hospitalization Fourth in terms of domestically acquired foodborne illness resulting in death

6. Which of the following organisms is the most common cause of bacterial diarrhea in adults in the USA?
A. Salmonella B. Shigella C. E. coli 0157:H7 D. Campylobacter

Bacterial Gastroenteritis
Adults Campylobacter Children E. coli Salmonella Shigella

Campylobacter
10%-15% of US acute diarrhea Animal reservoir
Most human cases are contaminated poultry (~50%)

Severe cramps, bloody diarrhea, anorexia, malaise; rarely, Guillain-Barre, reactive arthritis Diagnosis: Culture Treatment
Spontaneous clearing without antibiotics or Erythromycin if culture proven FQ > resistance

Shigella
Bloody diarrhea Fecal oral spread

Highly contagious

Principal effect on colon mucosa


Low-volume diarrhea Blood Mucus Fever Tenesmus

Diagnosis
Culture

Shigella
Clinical
Self-limiting Rarely
Rectal prolapse, proctitis, toxic megacolon, perforation, obstruction, seizures in children, HUS

Treatment
TMP-SMX (children) Quinolone Fluids No vaccine

Salmonellosis
10%-15% of US acute diarrhea
Second leading cause of acquired foodborne illness in US Top pathogen of domestically acquired foodborne illness resulting in hospitalization and in death

Non-typhoidal starts 6-48 hours after exposure, resolves spontaneously


Vomiting Nausea Pain Diarrhea

Sources
Eggs and poultry Pet reptiles

Salmonellosis
Clinical
Self-limited Can cause bacteremia
Endocarditis Osteomyelitis Mycotic aneurysm

Typhoid via fecal-oral contamination, rare in US


Constipation and rash early, then diarrhea and pain Diagnosis
Culture from blood or stool

Treat
Fluoroquinolone Vaccine available

E. coli 0157:H7 (EHEC)


Sporadic and large outbreaks
Produces Shiga toxin Diagnose with culture. Clinical
Dysentery Striking abdominal pain Usually no fever

E. coli 0157:H7 (EHEC)


Complication
Hemolytic uremic syndrome

Source
Contaminated meat

Treatment
Supportive Antibiotic NOT indicated

Foodborne Illness Summary


Pathogen (top 5 domestically acquired foodborne illnesses) Norovirus Salmonella (nontyphoidal) Clostridium perfringens Campylobacter spp Staphylococcus aureus Percent

58 11 10 9 3

CDC 2011 Estimates: www.cdc.gov/foodborneburden/2011-foodborne-estimates.html Last updated June 21, 2013

Amebiasis
5% reported US carriage rate E. histolytica mostly
Fecal-oral spread Abdominal cramps Chills, fever Liquid BMs with bloody mucus Sigmoidoscopy O&P Stool/serum antigen Serology

Diagnosis

Treatment

Metronidazole etc.

Giardiasis
Common

Symptoms

Mostly from contaminated water Abdominal cramps Malabsorption Nausea and vomiting Watery diarrhea

Diagnosis

Treatment

O&P Giardia antigen assay


Metronidazole

7. Of the following antibiotics, which is the agent recommended for treatment of pseudomembranous enterocolitis?
A. B. C. D. Amoxicillin Ciprofloxacin Metronidazole Doxycycline

Pseudomembranous Enterocolitis
Common in post-antibiotic setting
Etiology
Amoxicillin is most common offending antibiotic. Occurs with most antibiotics Toxin from Clostridium difficile Cytotoxin assays Immunoassays to toxin Stop antibiotic use metronidazole or vancomycin Fluids
No reported resistance

Diagnosis

Treatment

Relapse and carrier rates are possible (10%-25%).

C.difficile in the Elderly


Common due to frequent exposure to antibiotics Toxin titer correlates with illness severity. 2007 Treatment Guidelines: Vancomycin (125 mg po QID) if 2 or more factors:
Age > 60 Temp > 100.9F (38.3C) Cr increased 1.5x baseline Albumin < 2.5 WBC > 15,000

Traveler's Diarrhea
Multiple causes
Enterotoxigenic E. coli most common Campylobacter jejuni Shigella spp Salmonella spp

Prophylaxis
NOT recommended by CDC Routine prophylaxis increases the travelers risk for adverse reactions and for infections with resistant organisms.

Strict adherence to preventive measures Usually self-limited

Travelers Diarrhea (TD) If Treatment Is Needed


Travelers who develop > 3 loose stools in an 8-hour period especially if associated with nausea, vomiting, abdominal cramps, fever, or blood in stools Antibiotics*
Fluoroquinolones are drugs of choice when needed single dose or 1 day (increasing resistance) Alternative: Azithromycin (500 mg q day for 1-3 days) NO trimethoprim-sulfamethoxazole or doxycycline because of high levels of resistance

Bismuth subsalicylate may also be used for treatment.


One fluid oz or two 262 mg tablets q 30 minutes for up to eight doses in a 24hour period The anti-motility agent loperamide is a well-established antidiarrheal agent.
Its effective and safe use as an adjunct to antibiotics in the treatment of TD has been demonstrated in several studies. No other non-antibiotic treatment for TD has significant guideline or clinical trial support.

* CDC Health Information for International Travel 2012. New York: Oxford University Press; 2012.

Vibrio Cholerae
In the developing world, especially sub-Saharan Africa and Asia, cholera is a common cause of gastroenteritis.
Contaminated water or food

In US, generally associated with travel


Asia China South/Central America

OR consumption of contaminated seafood OR recent consumption of contaminated imported foods

Treatment
Vigorous rehydration Doxycycline

Key Learning Points: Diarrhea


Rotavirus is the leading cause of infectious diarrhea in children in the US. Norovirus is the leading cause of foodborne disease in US and the leading cause of gastroenteritis in US adults. Campylobacter is the most common cause of bacterial diarrhea in adults in the US. Antibiotics are not indicated for treatment of E. coli 0157:H7. Amoxicillin is most common offending antibiotic for pseudomembranous enterocolitis. Prophylaxis for travelers diarrhea is NOT recommended.

8. Which of the following statements is true regarding diverticular disease?


A. MRI is the test of choice in patients with suspected diverticulitis. B. Outpatient management with rest and fluids is effective for patients with mild diverticulitis. C. There is good evidence that avoiding nuts, corn, or popcorn decreases the risk of diverticulitis. D. Colonoscopy is typically necessary in patients with uncomplicated diverticulitis.

Diverticular Disease
Diverticulosis
Presence of diverticula Actually pseudo-diverticula: Formed as herniations of mucosa through the muscularis at the site of penetrating arteries

Diverticulitis
Inflammation of diverticula

Increase with age


30% at age 60 80% at age 80 90% of people will remain asymptomatic.

Diverticulosis
Symptoms
90% asymptomatic Intermittent LLQ abdominal pain Irregular defecation
From: Wikipedia

Exam
Tender LLQ () rectal exam without occult blood

Diagnosis
Flexible sigmoidoscope and barium enema

Treatment
High-fiber diet (unprocessed bran, hydrophilic bulk laxatives)

Diverticulitis
Clinical presentation
Acute lower abdominal pain Fever (usually below 102 F) Tachycardia Physical findings
Tender lower abdomen, possibly with rebound
Tenderness only in the LLQ significantly increases the likelihood of diagnosis ([+] LR = 10.4).

Acute abdomen is possible

Diverticulitis
Diagnostic Studies
CBC: Leukocytosis with left shift

55% will have leukocytosis.


Assess electrolytes and renal function.

BMP Consider C-reactive protein


LLQ tenderness AND a CRP > 50 mg/L, in the absence of vomiting, likelihood of acute diverticulitis significantly increased ([+] LR = 18).

UA: Sometimes WBC and RBC Plain films: Sometimes free air US: Sometimes abscess CT: Evolving as preferred method
Quite a bit of controversy with CT scan regarding the use of contrast agents: Intravenous, oral, or rectal contrast agents versus giving no contrast agents at all, CT KUB. Colonoscopy 4-6 weeks post-resolution of symptoms in patients with complicated disease

AVOID endoscopy and BE in acute setting.

Diverticulitis Treatment
General
Depends on severity May require only clear liquids and oral antibiotics OR MAY NEED (hospitalization)
NPO NG suction IV fluids IV antibiotics Can be done outpatient with follow-up in 2-3 days Ciprofloxacin and/or metronidazole

Ampicillin + aminoglycoside + metronidazole OR Imipenem/cilastatin OR Piperacillin/tazobactam

Diverticulitis Treatment
Surgery
15%-30% admitted for acute diverticulitis will need surgical intervention during the admission. For peritonitis, perforation, unresolved obstruction, and colovesical fistula Avoid for uncomplicated diverticulitis From: Wikipedia

Bleeding

15%-40% Profuse, painless Generally self-limited

SORT: Key Recommendations for Practice


Clinical Recommendation Antibiotics may not be necessary in patients with uncomplicated diverticulitis who are being treated in the outpatient setting. Evidence Rating B

There is NO clear evidence that avoiding nuts, corn, or B popcorn decreases the risk of diverticulosis or diverticular complications, such as diverticulitis.

Wilkins T, Embry K, and George R. Diagnosis and Management of Acute Diverticulitis. Am Fam Physician. 2013;87(9):612-20.

Cancer of Small Intestine


Rare
Most commonly seen in Crohns

Adenocarcinoma: 46%
Others: Lymphoma, carcinoid

From: Wikipedia

Diagnosis
Usually imaging study

Treatment
Usually surgery

Colonic Polyps
Types
Adenomas
Tubular Villous Tubulovillous

Clinical presentation
Asymptomatic or Bleeding Rarely
Obstruction or intussusception

Hamartomas Hyperplastic Inflammatory

From: Wikipedia

Colonic Polyps
Diagnostic studies
Endoscopy superior to barium enema

Treatment
Remove during colonoscopy If found during flexible sigmoidoscopic exam: Biopsy AND have patient undergo colonoscopy.

9. All of the following are true regarding colon cancer EXCEPT:


A. 95% of large intestine neoplasms are squamous cell carcinomas. B. Colon cancer is the second most frequent cause of cancer death in the United States. C. Colon cancer is equally frequent in men and women. D. Inflammatory bowel disease predisposes an individual to colon cancer.

Cancer of Large Intestine


Most frequent internal neoplasm in the US
Second most frequent cause of cancer death AFTER lung cancer 5%-6% lifetime risk (1 in 17) More common in Western nations Equal frequency in men and women African Americans and Caucasians equally affected
African Americans have a higher mortality

Cancer of Large Intestine


Histology
95% Adenocarcinoma
Progression from adenoma (adenomatous polyp) to carcinoma May take 10 years

Polyps
< 1 cm: < 1% chance of CA 1-2 cm: 10%20% chance of CA > 2 cm: 30%50% likelihood Detecting and removing polyps early CAN PREVENT much colon cancer.
From: Emmanuelm@en.wikipedia

Conditions Predisposing to Colon Cancer


Increasing age Family history of colon or rectal cancer or polyps Low-fiber (controversial), high-fat diet Inflammatory bowel disease Genital tract cancer in women
Adenomatous polyps Familial polyposis, HNPCC, Gardners syndrome, Turcot syndrome, Peutz-Jeghers syndrome, family cancer syndrome Diabetes Acromegaly Cholecystectomy Streptococcus bovis endocarditis

Diagnosis of Cancer of the Large Intestine


Symptoms: Variable and nonspecific
Rectal bleeding Lower abdominal pain Change in bowel habits

Endoscopy
Flexible sigmoidoscopy Colonoscopy

Imaging
Barium enema CT Rectal ultrasound

Physical findings
Abdominal mass Enlarged liver

Lab
Stool for occult blood

10. According to the 2008 ACS Colorectal Cancer Screening Guidelines, colon cancer screening should begin at which age? A. B. C. D. Age 40 Age 45 Age 50 Age 55

11. Which of the following is NOT a recommended method of screening for colorectal cancer per the 2008 ACS Colorectal Cancer Screening Guidelines? A. B. C. D. Annual FOBT with flex sig q 5 years Colonoscopy q 10 years Double-contrast BE q 5-10 years Flex sig q 3 years

Six Guidelines on Screening for Colorectal Cancer


Joint guideline: American Cancer Society, US Multi-Society Task Force on Colorectal Cancer and the American College of Radiology* (2008) AGA* (2008) Institute for Clinical Systems Improvement USPSTF* (2008) American College of Radiology American College of Physicians* (2012)

Summary of the 2008


Recommendation of the USPSTF on Screening for Colorectal Cancer

For all populations, evidence is insufficient to assess the benefits and harms of screening with computerized tomography colonography (CTC) and fecal DNA testing.

Screening tests
High sensitivity FOBT, sigmoidoscopy with FOBT, and colonoscopy are effective in decreasing colorectal cancer mortality. The risks and benefits of these screening methods vary: Colonoscopy and flexible sigmoidoscopy entail serious complications.

Summary of Principle
Recommendations in National CRC Screening Guidelines

Screening should begin at age 50. Recommended methods

Colonoscopy q 10 years (preferred) Double-contrast BE q 5 years FOBT and flex sig combined, annually and q 5 years, respectively Flex sig every 5 years gFOBT or FIT (fecal immunochemical test for blood) annually CT colonography q 5 years sDNA: Interval uncertain, manufacturer rec q 5 years

2008 ACS, ACR, US Joint Colorectal Cancer Screening Guidelines http://onlinelibrary.wiley.com/store/10.3322/CA.2007.0018/asset/130_ftp.pdf?v=1&t=hrwdlbj0&s=a4 ee2ea5fc73e9057321f7a6389b9e83fad454b3

Algorithm for CRC Screening AGA, 2008


*HNPCC=hereditary nonpolyposis colorectal cancer & FAP=familial adenomatous polyposis **either colorectal cancer or adenomatous polyp

Men and women Asymptomatic

Symptomatic

Volume 134, Issue 5; 1570-1595, May 2008

Diagnostic work-up

Age < 50 years

Age 50 Years

Negative family history

Positive family history

Negative family history

No screening

Avg. risk screening

HNPCC* or FAP

2 or more first-degree relatives affected* Or 1 first-degree relative affected at age < 60 years Colonoscopy beginning age 40 years or 10 years earlier than the youngest diagnosis** in the family, whichever comes first

1 first-degree relative affected at age 60 years

Genetic counseling And special screening

Average-risk screening, but beginning at age 40 years

CRC Screening Guidelines


American College of Physicians, 2012
Individualized CRC risk evaluation: Older age, black race, personal history of polyps, inflammatory bowel disease, CRC, family history of CRC Begin at age 50; adults at high risk age 40 or at 10 years before sentinel family case diagnosed Screening test based on risk stratification
Average: Stool-based test (annually), flexible sigmoidoscopy (5 years), or optical colonoscopy (10 years) High: Optical colonoscopy

Stop screening > 75 years or life expectancy of less than 10 years (potential harms of screening outweigh the potential benefits).
Qaseem A, et al. Screening for Colorectal Cancer: A Guidance Statement from the American College of Physicians. Ann Intern Med. 2012;156:378-386.

Cancer Screening 2010


Data from the 2010 National Health Interview Survey
Colorectal cancer screening rate 58.6% (Healthy People 2020 target: 70.5%) Other colorectal cancer screening rates
No usual source of health care: 20.8% No health insurance: 20.7%

Significant upward trends were seen in the proportion of adults up-to-date with colorectal cancer screening from 2000-2010 using any colorectal cancer screening regimen. CDC. MMWR. January 27, 2012;61(3).

Treatment of Cancer of the Large Intestine


Surgical excision with 5 cm margin Clearing colonoscopy at time of diagnosis; thereafter, q 3-5 years 40%-50% of patients have long-term survival after resection. Chemotherapy with 5-FU produces partial tumor remission in 15%-20% New agents: Irinotecan and oxaliplatin Radiation therapy useful for symptomatic metastases

Answers
1. B 2. C 3. D 4. B 5. B 6. D 7. C 8. B 9. A 10. C 11. D 12. A

12. Which of the following statements regarding constipation is true? A. It is the most common digestive complaint in the general population. B. Hyperthyroidism is a common etiology.
C. Acute treatment consists of bowel training. D. Empiric treatment without diagnostic testing should uncommonly be used.

Constipation
A symptom, not a disease
Unsatisfactory defecation: Difficult, infrequent, incomplete 95% of people have at least three BMs per week.
2% of people report chronic constipation (M:F 1:3).

Most common digestive complaint in general population

Etiologies

Constipation
Etiologies*
Diet Drugs Lack of adequate fluid intake Lack of exercise Irritable bowel syndrome Depression Hypothyroidism Pelvic floor dysfunction Spinal cord injury

History
Beware of constipation of recent origin. Stool pattern changes or alarm signs/symptoms: Rule out CANCER.

* Best Practice 1993;3(1)

Constipation
Physical findings
Rectal exam: Fissures, hemorrhoids, sphincter abnormalities, anal/rectal prolapse, impaction Abdominal exam: Check for abdominal mass. Check for signs of hypothyroidism.

Diagnostic studies
Insufficient evidence to support routine use of blood tests, radiography, or endoscopy in workup without alarm signs/symptoms (CBC, FOBT, TSH) Endoscopy: Flexible sigmoidoscopy or colonoscopy Imaging: Barium enema, ultrasound, CT, colonic transit studies, rectal manometry

Constipation Treatment
Depends on etiology
Empiric treatment without diagnostic testing can be considered when alarm features are absent. Acute
Enemas, suppositories, osmotic laxatives

Patient education: Lifestyle, exercise, hydration, bowel training Diet: High fiber (psyllium, methylcellulose, bran, polycarbophil) Laxatives: Mineral oil, lactulose, polyethylene glycol Rule out fecal impaction. Treat depression, if present.

Cancer of the Anus


Uncommon
Only 2%-3% as frequent as colon CA 50%-60% squamous cell CA
Associated with chronic inflammation of anus, especially inflammatory bowel disease

Diagnosis: Biopsy Treatment: Excision and radiation

Dr Noorali Bharwani, nbharwani.com

Hemorrhoids
Pathogenesis
Dilated sinusoids within anal canal and distal rectum External vs internal
Determined by origin: Above/below the dentate line

History
Usually bleeding is the symptom; can experience anal itching (external) or mild pain (internal); severe pain only with thrombosed hemorrhoids.

Physical exam
Visual exam and digital exam Anoscopy and sigmoidoscopy
From: Wikipedia

Hemorrhoids
Treatment
High-fiber diet Stool softeners Hot sitz baths Topical agents Ligation Infrared ablation Surgery

Other Common Anal Problems


Pruritus ani Anal fissure

Other Common Anal Problems


Pruritus ani
Severe itching around anus Many causes
Worse with anxiety

Diagnosis: Rule out causes above. Treatment


Improve anal hygiene. Treat constipation and diarrhea. 0.25% hydrocortisone cream

Fissure, fistula, hemorrhoid, dermatitis Diabetes, STDs, premalignant lesions Parasites, functional

Anal Fissure
Tear in anal mucosa Severe pain with defecation Diagnosis
Anoscopy

From: Wikipedia

Treatment

Treatment
Botulinum toxin Warm sitz baths injection Anesthetic ointments Surgery Soften stool. Disrupting to internal Sclerotherapy sphincter Topical nitrates or 30% have problem controlling flatus. CCBs
20% with minor fecal incontinence

References
Longstreth GF, Thompson G, Chey WD, et al. Functional bowel disorders. Gastroenterology. 2006;130:1480. American Gastroenterological Association position statement: Guidelines on constipation. Gastroenterology. 119(6):1761-1778. Bapat et al. Cost comparison of predictive genetic testing versus conventional clinical screening for familial adenomatous polyposis. Gut. 1999;44:698-703. Byers T et al. American Cancer Society guidelines for screening and surveillance for early detection of colorectal polyps and cancer: Update 1997. CA. 1997;47:154-160. Cromwell et al. Cost analysis of alternative approaches to colorectal screening in familial adenomatous polyposis. Gastroenterology. 1998;114:893-901. Friel CM and Matthews JB. Diverticular disease of the colon. Clinical Perspectives. Gastroenterology. 2000;3:187. Gavin N, Merrick, et al. Efficacy of glucose-based oral rehydration therapy. Pediatrics. 1996;98(1):45-51.

References
Ghosh S et al. Ulcerative colitis. BMJ. 2000;320(7242):1119-1123. Infliximab (Remicade) for Crohns disease. The Medical Letter. 2-2699,41:19-20. Imperiale et al. Risk of advanced proximal neoplasm in asymptomatic adults for colorectal cancer. N Engl J Med. 2000;343:169-174. Johnson S et al. Epidemics of diarrhea caused by a clindamycinresistant strain of Clostridium difficile in four hospitals. N Engl J Med. 1999;341:1645-1651. Kornbluth A and Sachar DB. Ulcerative colitis practice guidelines in adults. AJG. 1997;92(2):204-211. Leiberman et al. Use of colonoscopy to screen asymptomatic adults for colorectal cancer. N Engl J Med. 2000;343:162-168. Rex DK et al. Colorectal cancer prevention 2000: Screening recommendations of the American College of Gastroenterology. Am J Gastroenterol. 2000;95:868-877.

References
Rockey DC et al. Relative frequency of upper gastrointestinal and colonic lesions in patients with positive fecal occult-blood test. N Engl J Med. 1998;339:153-159. Sharma VK and Howden CW. Colorectal cancer screening: How are we doing and how can we improve? Oncology Spectrums. 2001;2(1):25-31. Rex DK, Johnson DA, Anderson JC, et. al. American College of Gastroenterology Guidelines for Colorectal Cancer Screening 2008. Am J Gastroenterol online: http://www.medicine.nevada.edu/residency/lasvegas/internalmed/document s/coloncaGuideline.pdf Wilkins T, Jarvis K, and Patel J. Diagnosis and Management of Crohns Disease. Am Fam Physician. 2011;84(12):1365-1375. Wilkins T, Embry K, and George R. Diagnosis and Management of Acute Diverticulitis. Am Fam Physician. 2013;87(9):612-20.

Pediatric Orthopedics: Small Skeletons, Big Problems


Joseph Garry, MD, FAAFP, FACSM University of Minnesota

Disclosures
Dr. Garry discloses that he holds stock in Pfizer and Merck pharmaceuticals.

It is the policy of the AAFP that all individuals in a position to control content disclose any relationships with commercial interests upon nomination/invitation of participation. Disclosure documents are reviewed for potential conflicts of interest. If conflicts are identified, they are resolved prior to confirmation of participation. Only participants who have no conflict of interest or who agree to an identified resolution process prior to their participation were involved in this CME activity.

Learning Objectives
1. Know the key factors of common pediatric orthopedic problems 2. Cite the causes of intoeing 3. Cite the common hip conditions in the pediatric and adolescent population 4. Cite key factors associated with osteochondroses 5. Know the common apophysitides affecting children 6. Describe benign nocturnal limb pains of childhood

1. Which statement is most accurate regarding Duchennes muscular dystrophy?


A. B. C. D. Peak onset of symptoms occur at age 8 Girls and boys are equally affected It is due to dysregulation of relaxin It is the most common fatal disease affecting children

Duchennes Muscular Dystrophy


Most common fatal disease affecting children 1:3500 live male births Dystrophin abnormality Age 6-7 years boys start falling, early fatigue, and will develop calf pseudohypertrophy Muscle weakness starts in lower extremities Late pulmonary and cardiac involvement

2. You examine an infant at a routine 4 weeks check-up. Which statement is most accurate?

A. If the infant had Erbs palsy, you would now discontinue daily physical therapy B. The infant with club foot would be placed in a Pavlik harness C. The infant with hip dysplasia would be treated with the Ponseti method of serial bracing D. If the infant was female and breech you would consider screening for hip dysplasia

Erbs Palsy
Brachial plexus injury
Neuropraxia; resolution at 3 months Neuroma; partial recovery due to scarring Rupture; donor graft splicing Avulsion; most severe

Photo courtesy of orthoinfo.aaos.org

Symptoms of weakness, loss of sensation, partial or complete paralysis Daily physical therapy Serial exams and if no improvement then consider surgical consult at 3-6 months

Club Foot
Hereditary predisposition 1:1000 live births; boys > girls
Photo courtesy of orthoinfo.aaos.org

Management includes serial casting (Ponseti method) followed by heel cord release and night time bracing x 2 years Surgical correction occurs at 9-12 months, if needed

Developmental Dysplasia of the Hip


Left hip most common Risk higher among girls; first-born child; breech position; FHx; oligohydramnios Evaluate using Ortolanis and Barlows maneuvers
Clunk is positive (not click)

Refer early or for persistent findings at 2 months Pavlik harness (hips in flexion and abduction) for Dx from birth to 6 months Spica casting for Dx from 6 mo to 2 years
Courtesy of orthoinfo.aaos.org

USPSTF Recommendation
The USPSTF concludes that evidence is insufficient to recommend routine screening for developmental dysplasia of the hip in infants as a means to prevent adverse outcomes.
Rating: "I" statement.

> 90% of those identified as newborns resolve spontaneously Poor evidence of the effectiveness of surgical or non-surgical treatments

3. A 3-year-old female is brought to your office for evaluation of mild intoeing. The childs patellae face forward, and her feet point slightly inward.
Which one of the following would be most appropriate?
A. B. C. D. E. Continued observation Foot stretching exercises Orthotics Night splints Surgery

Intoeing
Metatarsus adductus
Most common congenital foot deformity Flexible type improves at 4-6 mo Rigid type may require serial casting at 6-9 mo 85-90% resolve by one year of age

Internal tibial torsion


Most common cause of intoeing Walks with patella facing forward and feet intoeing 90% resolve by age 8 PT, orthotics, splints are not effective
Figures courtesy of orthoinfo.aaos.org

Intoeing
Femoral anteversion
Both knees and feet point inward Tendency to sit in a W position Most noticeable at age 4-6 yrs 80% improve by age 9-10 yrs Surgery at age 9-10 if more severe
Figure courtesy of orthoinfo.aaos.org

4. What is the most likely diagnosis of a new limp in a 7-year-old male?


A. Legg-Calv-Perthes osteochondrosis B. Transient synovitis C. Slipped capital femoral epiphysis D. Femoral neck stress fracture

Transient Synovitis Transient Synovitis vs Septic Hip


Differentiate from Septic Hip

Fever and hip pain Limping Radiographs, CBC, ESR, CRP, temperature Ultrasound, and if effusion present, then aspiration and culture IBU shortens course of transient synovitis

T > 38.1 CRP > 2.0 ESR > 40 WBC > 12

NWB

As more factors are (+) the likelihood of septic hip rises


Kocher et al. JBJS 1999

Transient Synovitis*
Age 4-11 years

Slipped Capital Femoral Epiphysis


11-16 years

Legg-CalvPerthes
4-10 years

Risks

Male, AA, obesity

Perinatal HIV infection, LBW, +FH, low SES


Yes, and may be intermittent abduction and IR Yes, for diagnosis and may need to repeat Insidious onset, pain can radiate to thigh and knee, poor prognosis if age > 6 Early pediatric orthopedic referral

Antalgic Gait ROM X-rays Caution

Yes Limited and painful Yes, to rule out bony process Need to consider septic hip (T > 38.1, CRP
> 2.0, ESR > 40, WBC > 12, NWB)

Yes Limited and painful Yes, for diagnosis Acute SCFE risk of AVN is 30%

Management
*most common

Ibuprofen to shorten course

Surgical referral immediate if acute

Slipped Capital Femoral Epiphysis

Legg-Calv-Perthes

Flattening and early fragmentation of the femoral epiphysis

Khlers Osteochondroses
Ages 28 years Boys are 5x more commonly affected than girls Presents with limp and midfoot pain dorsal foot swelling Tender over the navicular, swelling/warmth
Treat in a short leg cast for 8 weeks

Sclerosis and flattening of the navicular

Freibergs Osteochondroses
Disordered ossification of the 2nd MT head
Most common in adolescent girls in dance or ballet Causes: repetitive stress, trauma, disruption of blood supply Bilateral in < 10% of cases

Pain with WBA, dorsal swelling; tenderness over the metatarsal head Treat with activity modification, analgesics, ice, metatarsal pads, well-padded shoes

Osteochondritis Dissecans of the Knee


M > F, average age is 10-20 years, bilateral in 30-40% 85% in medial epicondyle & 70% at posterolateral aspect (LAME) Radiographs as initial test MRI can demonstrate degree of involvement Girls < 11 & boys < 13 usually do well without surgery if fragment intact

5. You are asked by your local school board to provide a talk to parents regarding scoliosis and the new decision by the school nurses to discontinue screening exams for scoliosis. Which statement below would you NOT include in your talk?

A. Scoliosis has a relatively low prevalence rate B. The USPSTF recommends against screening for adolescent idiopathic scoliosis C. Juvenile idiopathic scoliosis is the most common form of scoliosis D. Infantile idiopathic scoliosis can resolve spontaneously

Idiopathic Scoliosis
Females more commonly affected Prevalence of 0.5-3% Infantile idiopathic scoliosis is most likely to spontaneously resolve Juvenile idiopathic scoliosis may be considered the most malignant form of scoliosis due to high rates of progression Adolescent idiopathic scoliosis is most common

Adolescent Idiopathic Scoliosis


Observe for Cobb angle < 30 and perform serial radiographs to follow
Brace for angles of 30-40 Surgery for angles > 40
Cobb angle measured off of PA radiograph

USPSTF recommends against screening Grade D

6. Which statement about traction apophysitis is most correct?


A. It is most commonly an acute injury

B. Physical activity is not a prerequisite


C. It often occurs at the time of the preadolescent or adolescent growth spurt D. The physis of long bones is the most common site of involvement

Traction Apophysitis
Insidious onset that results from injury, or overuse of the [tendon and] surrounding ossification centers Boys are more commonly affected and symptoms generally appear between 10-14 years of age Often occurs at or after growth spurt Physical activity required

Principles of rehabilitation include ice (pain), stretching (developmental inflexibility), and modification to activity (relative rest)

Little League Elbow

Baseball players and Their Shoulder Injuries; Houston Clinic Health Alert

Progressive medial elbow pain, diminished throwing effectiveness, decreased throwing distance Refrain from throwing for 3-6 weeks until pain-free & nontender then progressive return-to-throwing program

7. An 11-year-old male is brought to your office for evaluation of bilateral posterior heel pain that has occurred for the past few months. He plays basketball and soccer several times a week and the pain begins several minutes into each of these activities. There is no pain at rest or with walking. He has not noticed any numbness, tingling, or weakness. On examination you find no swelling or tenderness of the heel or Achilles tendon. Reflexes, strength, and range of motion at the ankle are intact, but he does have bilateral posterior heel pain when you passively dorsiflex the ankles.

Which one of the following is the most likely diagnosis?

A. B. C. D.

Achilles tendonopathy Severs disease Plantar fasciitis Heel pad syndrome

Apophysitides
Site ASIS Name Attachment Sartorius Age 10-14 Symptoms Anterior hip pain Anterior hip pain Posterior thigh pain 10-14 Anterior knee pain Posterior heel pain Prox 5th MT pain Add quad and hip flexor stretching Add heel lift Add heel lift, contralateral XR Management Rest, ice, stretch, activity modification

AIIS Ischial tuberosity Tibial tuberosity Calcaneus OsgoodSchlatters Severs

Rectus femoris Hamstring Patellar tendon Achilles Peroneus brevis

10-14

8-12

5th metatarsal Iselins

Severs
Calcaneal Apophysitis

Normal radiograph in symptomatic child

Fragmentation demonstrated late in the symptomatic child

Iselins Apophysitis

8. A mother brings in her 6-year-old son to be evaluated for leg pain that has been present for the past 2 weeks. He plays soccer 4 times per week and has no history of injury. Pain occurs at night and wakes him from sleep, and resolves by the morning. There is no limp and it does not interfere with his activity or soccer. No fevers, no illness, and immunizations are UTD. You advise her that which of the following will help alleviate his pain?

A. Warm baths prior to bed B. Increased exercise C. Ice application to the affected area after exercise D. The Nike Jr. Mercurial Vapor IX FG cleats

Benign Nocturnal Limb Pains of Childhood


Growing Pains

Affects 35% of 4-6 yo (can persist to age 19) Bilateral and affects lower extremities Deep cramping pain that often awakens from sleep Resolves by morning NO LIMP Aggravated by heavy exercise during the day Alleviated by heat, massage, PT Management: Reassurance, heat, warm baths, exercise to non-aggravating levels, mild analgesics
Uziel et al. Pediatr Rheumatol 2007 Evans et al. J Pediatr 2004

Answers
1. 2. 3. 4. 5. 6. 7. D D A B C C B

8. A

Common Newborn Issues

Disclosure Statement

It is the policy of the AAFP that all individuals in a position to control content disclose any relationships with commercial interests upon nomination/invitation of participation. Disclosure documents are reviewed for potential conflicts of interest. If conflicts are identified, they are resolved prior to confirmation of participation. Only participants who have no conflict of interest or who agree to an identified resolution process prior to their participation were involved in this CME activity.

Learning Objectives
Discuss common newborn assessment, and management of the following issues: Initial newborn evaluation Nutrition Respiratory and cardiac Hip dysplasia Hyperbilirubinemia Gastrointestinal Infectious disease Sudden infant death syndrome Failure to thrive Lacrimal duct obstruction

Evaluation of the Newborn


Apgar scores Height/weight/head circumference/vital signs Skin: rashes HEENT: red reflex, palate Heart/lungs: sounds, murmurs, pulses, color Abdomen: cord, hernias, organomegaly Genitalia: palpable testicles, penis, labia, anus Hips: Ortolani and Barlow maneuvers Neurologic: sutures, fontanelles, reflexes

Newborn Hearing
2008 the USPSTF recommends screening for hearing loss in all newborn infants (B rec) Risks: Family history of sensorineural hearing loss, NICU admission for 2+ days, craniofacial abnormalities, and certain congenital syndromes and infections 50% of newborns with permanent hearing loss have no risk factors

USPSTF Recommendations
Grade A
Screening for congenital hypothyroidism, PKU, and sickle cell disease Ocular prophylaxis for gonococcal infection Universal HIV screening for pregnant women

1. If infants less than 4 months of age only receive breast milk, what supplement should they receive?
A. B. C. D. Vitamin D Multivitamins Iron Calcium

Vitamin D
Breast milk contains small amounts of vitamin D (< 25 IU/L) AAP (2008)
*Breastfed infants should have 400 IU vitamin D/day starting in first few days of life Poly-Vi-Sol and most infant vitamins contain 400 IU (although some do have 200 IU) Need 1 L or 33 oz. formula

Feeding and Nutrition


Breastfeeding: best food, recommended for 1 yr Infant benefits

Protection from infectious disease Reduction in atopic diseases Protection from SIDS Preterm infant: protection from NEC and sepsis
Breast and ovarian cancer Cardiovascular disease Emotional health Postpartum weight loss

Maternal improved health risks

Few contraindications
Galactosemia, maternal conditions (HIV, TB), medications/chemo

Storage and Handling of Breast Milk


May be safely stored up to 4-10 hours at room temperature May be stored up to 4-8 days in the refrigerator May be frozen for up to 3-6 months After freezing, should be thawed slowly and not refrozen

Feeding
Bottle-feed with iron-containing formula Formula at room temp, not in microwave
Cows milk: not until after 1 year Soy: not as allergenic, vegetarian

Starting foods
Cereals (4-6 months) Vegetables and fruits, then table foods

No honey for first year Fluoride supplementation if water is deficient (6+ months) B recommendation

Other Nutrition Pearls


Wet diapers 6/day Stools 1-3/day Regain birth weight by 14 days Acceptable weight loss in first 2 weeks of life is < 10%

Respiratory Distress
Transient tachypnea of the newborn (TTN) resp rate > 60/min
CXRdiffuse parenchymal infiltrates and fluid in the pulmonary fissures 93% resolve in 2 hours; may last for 2 days Most common cause of respiratory distress (> 40%) Diagnosis of exclusion Treatment is supportive (oxygen)

Other common causes:


Respiratory distress syndrome in premature infants Meconium aspiration Pneumonia

2. The most common form of congenital heart disease is:


A. B. C. D. Ventricular septal defect Patent ductus arteriosus Pulmonary stenosis Atrial septal defect

Cyanotic Congenital Heart Defects


Transposition of the great vessels
Most common cardiac cause of cyanosis in neonates

Tetralogy of Fallot
Most common cardiac cause of cyanosis in toddlers

Total anomalous pulmonary venous return Truncus arteriosus Tricuspid valve abnormalities Pulse ox congenital disease guidelines

Heart Murmurs
Most are transitional and benign Usually not associated with other signs or symptoms Pathologic murmurs
Grade III or louder Harsh Continuous Abnormal second heart sound Diastolic Other signs or symptoms

Developmental Dysplasia of the Hip


Risk factors
Female Breech 1st degree relative Possible association with foot deformities and IUGR

Evaluate using Ortolani and Barlow maneuvers Clunk is positive (not click) Orthopedic consult more reliable than ultrasound (not accurate for 6 weeks) Refer early or for persistent findings at 2 months Pavlik harness (hips in flexion and abduction)

USPSTF Recommendation
The USPSTF concludes that evidence is insufficient to recommend routine screening for developmental dysplasia of the hip in infants as a means to prevent adverse outcomes.
Rating: "I" statement.

> 90% of those identified in the newborn resolve spontaneously Poor evidence of the effectiveness of surgical or non-surgical treatments

3. You were planning to send home a well, term infant, but his bilirubin returns at 11.0 mg/dL at 30 hours. Mother is breastfeeding. What should you do?
A. Reassure mother and send infant home B. Send infant home, but return tomorrow for repeat bilirubin C. Stop breastfeeding and switch to formula D. Start phototherapy

Treatment of Hyperbilirubinemia
Increase feeds (may continue to breastfeed) Phototherapy (see bilitool.org)
Total bilirubin > 11.7 at 24 hours of age Total bilirubin 12.7 at 30 hours of age

Exchange transfusion
Signs of encephalopathy at any bilirubin level Total bilirubin level > 20-25 at < 48 hours and failure of phototherapy

Hyperbilirubinemia
Physiologic jaundice
Newborns conjugate bilirubin slower, have higher RBC turnover, and have decreased excretion Peaks at 3-5 days Usually 3-12 mg/dL (up to 17 mg/dL)

**Breastfeeding Jaundice
Early onset2-5 days of age Transientup to 10 days Exaggeration of physiologic jaundice Most common cause of unconjugated hyperbilirubinemia

Insufficient feeds
Stools infrequent

**Breast Milk Jaundice


Later onset5-10 days of age Persists > 1 month Prolongation of physiologic jaundice caused by a factor in human milk Frequent feeds Normal stooling Serum bili eventually returns to normal without stopping breastfeeding

Hyperbilirubinemia Risk Factors


Breastfeeding Hemolytic disease Asian ethnicity Prematurity Maternal DM Infection

Failure to Stool
70% of infants pass meconium in first 12 hours of life After 24 hours consider secondary causes
Hirschsprungs Imperforate anus Cystic fibrosis

Necrotizing enterocolitis
Most common cause of acute intestinal obstruction and septic abdomen in neonates

Constipation Treatments
Rectal stimulation with thermometer Glycerin suppositories 100% fruit juice Miralax (polyethylene glycol) if > 6 months High-fiber foods if older (cereals, fruits, vegetables)

Pyloric Stenosis
Over first two weeks of life
Nonbilious and projectile vomiting No other systemic symptoms Infant appears hungry and feeds often

Other causes of vomiting


Overfeeding GERD small amounts of vomiting; weight gain Midgut volvulus: bilious vomiting with double-bubble sign Gastroenteritis: fever, diarrhea CNS injury: other neurological signs

Infantile Colic
Uncontrolled crying especially at night Frequent spitting up Appropriate weight gain and normal exam Emerging evidence for probiotics The 5 Ss
Swaddling Side/stomach position Shushing sounds Swinging Sucking

http://www.happiestbaby.com/

4. You delivered a 36-week gestation female to a 16 y/o first-time mother with poor prenatal care. The infant is still not feeding well after nearly 48 hours. Weight loss is 10%, infant is moderately jaundiced, temperature 37.8 C. What should you do? A. Send the child home with the mother with careful precautions B. Observe the child for the next 12-24 hours C. Order a CBC D. Do a complete septic work up

Signs of Sepsis in the Newborn


Feeding problems Temperature instability Respiratory distress Vomiting/diarrhea Abdominal distension Jaundice Pallor Skin rash/petechiae Hypotension Tachycardia Apnea and bradycardia Irritability High-pitched cry Lethargy Weak suck Convulsions Bulging or full fontanelle

Torch Syndrome
Agents
Toxoplasmosis (Treat: pyrimethamine and sulfadiazine 21 days) Other infections (i.e. syphilis) Rubella (congenital cataracts and patent ductus) Cytomegalovirus (CMVrisk is greatest in 1st half of pregnancy) Herpes simplex

Common symptoms

Lymphadenopathy Hepatosplenomegaly Hemolytic anemia and thrombocytopenia with jaundice Stillbirth and neonatal death
CBC, blood, urine, CSF cultures (viral, fungal, bacterial) CXR CT head Lumbar puncture

Evaluation

Herpes Simplex
Manifestations: 1/3 each
Disseminated disease involving multiple organs, primarily liver and lungs Localized CNS disease Localized to skin, eyes, and mouth

Treatment
IV acyclovir for 2-3 weeks

Picture courtesy of CDC/Dr. Millar

Hepatitis B
HBsAg+ mother
HBIG within 12 hours of birth Hepatitis B vaccine within 12 hours

Mothers status unknown


Hepatitis B vaccine given within 12 hours of age Mother tested for Hepatitis B, and if positive HBIG given as soon as possible (prior to 1 week)

5. Six weeks after leaving the hospital, an African-American male is rushed into the office after mother found him not breathing. He aroused with stimulation. Infant has not been ill, examination is normal. What should you tell the parents to help prevent another episode? A. Have the infant sleep with mother for the next few weeks B. Place baby in the prone position for sleep C. Avoid exposure of the infant to second-hand smoke D. Discourage the use of a pacifier

SIDS
Sudden infant death syndrome (SIDS) has a peak incidence at 2-3 months of age African-Americans, Native and Alaskan Americans have rates 2-3x the national average Infants should be kept in close proximity in a separate bed Infants should be placed on their backs to sleep Exposure to cigarette smoke increases risk Pacifier use may be beneficial Fans may be helpful Breastfeeding is protective

Failure to Thrive
Inadequate physical growth diagnosed by observation of growth over time using standard growth chart Occurs in 5-10% primary care practices Weight falls below the 5th percentile, or crosses two major percentile lines Weight for length less than 75% of the median Growth charts at www.cdc.gov or www.who.int/childgrowth Neglect is most common form of child abuse (60%)

Etiology
***Inadequate caloric intake Inadequate or inappropriate feeding by parents Poverty, neglect Mechanical feeding problems Inadequate absorption Celiac disease or milk allergy Cystic fibrosis Vitamin or mineral deficiencies Biliary atresia or liver disease Increased metabolism Hyperthyroidism Chronic infection (HIV) Hypoxemia/congenital cardiac disease Defective utilization Genetic abnormalities (Downs) Metabolic disorder (storage diseases) Congenital infections Psychosocial

Treatment
High-calorie diet (150% of recommended) Feeding behaviors Hospitalization Referralmultidisciplinary interventions with home nursing visits (SOR A) Parents may need treatment

Lacrimal Duct Obstruction


Persistent tearing, usually unilateral Can try milking the duct Antibiotics or steroids are not indicated Usually resolves by 6 months Treat if not resolved in 1 year

6. Infants and children that do not receive 33 ounces of formula or milk each day should receive vitamin D in the following dose:
A. B. C. D. 200 IU 400 IU 600 IU 800 IU

7. The following are all true about breastfeeding jaundice except:


A. B. C. D. Early onset of 2-5 days of age Infrequent stools Sufficient feeds Transient up to 10 days

8. The majority of failure to thrive cases are due to:


A. B. C. D. Cystic fibrosis Inadequate caloric intake Hyperthyroidism Celiac disease

Take Home Pearls


Vitamin D 400 IU for all infants and children Know difference between breastfeeding jaundice and breast milk jaundice #1 cause of FTT is inadequate caloric intake

Answers
1. 2. 3. 4. 5. 6. A A B D C B 7. C 8. B

Management of Chronic Pain


Gary I. Levine, MD, FAAFP
Associate Professor Department of Family Medicine The Brody School of Medicine at Eastern Carolina University Greenville, North Carolina

Learning Objectives
1. 2. 3. 4. Identify pain as the 5th vital sign. List several illnesses and conditions associated with chronic pain. Identify mechanisms for quantifying pain. Recognize potential challenges in the management of chronic pain. Describe pharmacologic therapy of chronic pain, including the rational use of opioids. Describe opiate side effects and withdrawal. Identify the differences between tolerance, physical dependence, addiction, and pseudoaddiction. Identify key facts regarding urine drug testing. List elements of a pain management (informed consent & treatment) agreement as recommended by the Federation of State Medical Boards

5.
6. 7.

8. 9.

Faculty Disclosure
Gary Levine, MD, returned disclosures indicating that indicating that he has no affiliation or financial interest in any organization(s).

The AAFP has selected all faculty appearing in this program. It is the policy of the AAFP that all CME planning committees, faculty, authors, editors, and staff disclose relationships with commercial entities upon nomination or invitation of participation. Disclosure documents are reviewed for potential conflicts of interest and, if identified, they are resolved prior to confirmation of participation. Only those participants who had no conflict of interest or who agreed to an identified resolution process prior to their participation were involved in this CME activity.

TAXES

Management of Chronic Pain

Acknowledgements
Charles Argoff, MD, Assistant Professor of Neurology, New York University School of Medicine Maged Hamza, MD, Director, Interventional Pain Management, Virginia Commonwealth University Spine Center Robert Newman, MD, Professor, Dept. of Family Medicine, Eastern Virginia Medical School John Purvis MD, Tallahassee Memorial Hospital Family Practice Residency Program

References
Berland DA, Rogers P. Rational Use of Opioids for Management of Chronic Nonterminal Pain. Am Fam Physician. 2012;86(3):252-258. Lambert M. ICSI (Institute for Clinical Systems Improvement) Releases Guideline on Chronic Pain Assessment and Management. Am Fam Physician. 2010;82(4):434-439.

References
AAFP Position Paper on Pain Management and Opioid Abuse, 2012. Elder NC, Simons T, Regan S, Gerrety E. Care for Patients with Chronic Nonmalignant Pain with and without Chronic Opioid Prescriptions: A Report from the Cincinnati Area Research Group Network. J Am Board Fam Med. 2012; 25(5):652-660.

References
FDA Blueprint for Prescriber Education for Extended Release & Long Acting Opioids. Aug, 2012. Federation of State Medical Boards. Model Policy on the Use of Opioid Analgesics in the Treatment of Chronic Pain. July 2013.

1. Which of the following statements regarding pain is true:


A. Pain is always associated with tissue damage B. Pain is the 5th vital sign C. Pain is the 3rd most common reason why patients seek medical attention D. Noncardiac chest pain is primarily neuropathic E. Chronic pain affects over 50% of the US population each year

Pain
An unpleasant sensory and emotional experience associated with actual or potential tissue damage, or described in terms of such damage.
International Association for the Study of Pain (IASP)

Chronic Pain
Persistent pain, either continuous or recurrent, that affects patients well-being, level of function, and quality of life Lasts > 12 weeks No functional benefit Doesnt resolve with tissue healing Can be a primary diagnosis May result from overtreatment

Classification of Pain
Pain
Acute
Injury Postoperative Flare

Chronic

Neuropathic

Mixed

Nociceptive

Visceral

Diabetic neuropathy (DN) Post-herpetic neuralgia (PHN) Radiculopathy (RADIC)

Cancer pain Low back pain Fibromyalgia

Osteoarthritis Rheumatoid arthritis Fibromyalgia

IBS Pancreatitis Bladder pain Noncardiac chest pain Abdominal pain syndrome

Epidemiology of Pain
Each year, between 15% and 20% of the US population experiences acute pain Chronic pain affects approximately 30% of the US population annually Pain is the most common reason patients seek medical attention

2. Which of the following illnesses has the highest prevalence of chronic pain the USA:
A. Back pain B. Arthritis C. Headache D. Diabetes E. Fibromyalgia

Prevalence of Pain in Selected Chronic Illnesses


Disease Back Pain*** Arthritis (DJD)*** USA Prevalence (Millions) 54.6 43

Chronic Headache***
Osteoporosis Diabetes Rheumatoid Arthritis Fibromyalgia

40
25 15.7 2.5 2

Arthritis Foundation, American Diabetes Association, National Institute of Neurologic Diseases and Stroke, 1997-1999

3. Which of the following statements is correct:


A. Depression is not commonly associated with chronic pain B. On a numerical pain scale, 10 usually represents true happiness C. Inadequate training is not a barrier to effective pain management D. JCAHO is unconcerned with identifying patients with pain within a hospital environment E. Patients from ethnic minorities and cultures different from the health care professionals treating them receive inadequate pain management

Chronic PainAssociations
Insomnia Anxiety Depression Weight loss Decreased quality of life

Barriers to Effective Pain Management


Inadequate training in pain management Poor methods of pain assessment Conflicting clinical guidelines Provider concerns
Regulation of controlled substances Fear of causing addiction or being deceived*** Management of side effects

JCAHOPain Management Standards


Patient Rights
Appropriate assessment and management of pain Care decisions

Assessment of Patients
Identification of patients with pain

Care of Patients
Safe medication prescribing

JCAHOPain Management Standards


Education
Making sure patients and families understand pain and the importance of pain management

Continuum of Care
Provides for continuing pain management needs in discharge planning

Improving Organization Performance

Assessment of Pain Intensity Verbal


No pain Mild pain Moderate pain Severe pain Very severe pain Worst possible pain

Assessment of Pain Intensity Numerical Scale


0 ---- No pain 1 2 3 4 5 ---- Moderate pain 6 7 8 9 10 ---- Worst possible pain

Assessment of Pain Intensity Visual Analog Scale


Worst possible pain

No pain

Assessment of Pain Intensity Faces Scale

0
1 2

4 5

Cultural Aspects of Pain


Patients from ethnic minorities and cultures different from the health care professionals treating them receive inadequate pain management Physicians from a stoical culture are likely to be more attentive to the patient who is stoical The culture of pain in mainstream American culture tends to teach the hurting person to be stoical and the attending person to honor that stoicism.

Cultural Aspects of Pain


It is important to understand how his or her own upbringing affects attitudes about pain. It is important to overcome the belief that one's own reaction to pain is "normal" and that other reactions are "abnormal." Even subtle cultural and individual differences between patient and physician, particularly in nonverbal, spoken, and written language, can affect care.

Federation of State Medical Boards Pain Treatment Guidelines


The Federation of State Medical Boards has written the Model Policy on the Use of Opioid Analgesics for the Treatment of Chronic Pain.
Updated July, 2013 www.fsmb.org

These guidelines are for use by all State Medical Boards, with specific recommendations for practicing physicians

Federation of State Medical Boards Pain Treatment Guidelines


Documentation of a relevant history, physical examination, and laboratory evaluation Assessment of risk of alcohol or medication misuse Screening for depression

Federation of State Medical Boards Pain Treatment Guidelines


Documentation of the patients treatment plan, including ways to measure treatment response Documentation of monitoring effectiveness (5 As) at each visit

Patient Assessment/Reevaluation
The 5 As of Chronic Pain Management
Analgesia Activities of daily living Adverse effects Aberrant drug-taking behaviors Affect

Federation of State Medical Boards Pain Treatment Guidelines


Documentation of signed written informed consent and treatment agreement

Federation of State Medical Boards Pain Treatment Guidelines


Periodic drug testing An accurate record of all treatments and medications prescribed Having the patient return at appropriate intervals for evaluation
More frequently during initiation of opioid trial

Federation of State Medical Boards Pain Treatment Guidelines


Seek consultation/referral as needed
Pain, psychiatry, addiction, mental health specialist

The physician must be in compliance with applicable federal and state regulations

4. Which of the following statements regarding pharmacotherapy in chronic pain is true:


A. Combination drug therapy is preferred to monotherapy B. Agents effective against pain should be continued despite intolerable side effects C. TCA antidepressants have demonstrated efficacy in the treatment of diabetic neuropathy D. SSRIs are effective in treating the pain of post-herpetic neuralgia E. Cessation of chronic carisoprodol (Soma) use is not associated with withdrawal symptoms

Chronic PainManagement
Physical modalities
Physical/occupational therapy

Nonpharmacologic therapies
Cognitive-behavioral

Pharmacologic therapies Neural blockade Implantable devices

Pharmacotherapy Guidelines
Monotherapy if possible
Titrate only 1 drug at a time

Improvement in pain Improvement in function Limited side effects Recognition that each patient is different
Individual variation in response

Pharmacologic Agents
Acetaminophen (eg, Tylenol) NSAIDs
Traditional Cox-2 agents

Opioids

Pharmacologic Agents
Adjuvant agents
TCA SNRI Anticonvulsants Neuroleptics Centrally acting alpha-adrenergic agents Muscle relaxants Topical anesthetics

Antidepressants
TCA
Amitriptyline (Elavil)

Others
Duloxetine (Cymbalta) Milnacipran (Savella) Venlafaxine (Effexor)

Antidepressants (TCA & SNRI)


Demonstrated benefit in randomized studies
Illness Diabetic neuropathy Postherpetic neuralgia Effect size 1.7 1.4

Tension type headache


Migraine Atypical facial pain Fibromyalgia

1.1
0.8 0.8 0.6

Anticonvulsants
FDA indication for chronic pain
Carbamazepine (eg, Tegretol) Divalproex sodium (eg, Depakote) Gabapentin (eg, Neurontin) Pregabalin (Lyrica) Topiramate (eg, Topamax)

Anticonvulsants
Post-herpetic neuralgia
Gabapentin (eg, Neurontin) Pregabalin (Lyrica)

Diabetic neuropathy
Carbamazepine (eg, Tegretol) Phenytoin (eg, Dilantin) Gabapentin (eg, Neurontin) Pregabalin (Lyrica) Lamotrigine (eg, Lamictal)

Anticonvulsants
HIV-associated neuropathy
Lamotrigine (eg, Lamictal)

Trigeminal neuralgia
Carbamazepine (eg, Tegretol) Lamotrigine (eg, Lamictal) Oxcarbazepine (eg, Trileptal)

Centrally Acting Alpha-Adrenergic Agents


Clonidine (eg, Catapres)

Centrally Acting Alpha-Adrenergic Agents


Trigeminal neuralgia Chronic back pain Chronic headache
Cluster headache Tension type headache

Spastic torticollis Neuropathic pain

Muscle Relaxants
Cyclobenzaprine (eg, Flexeril) Carisoprodol (eg, Soma) Methocarbamol (eg, Robaxin) Metaxalone (eg, Skelaxin) Orphenadrine (eg, Norflex) Tizanidine (eg, Zanaflex)

Muscle Relaxants
Cyclobenzaprine (eg, Flexeril)
Centrally acting Nocturnal muscle spasms Anticholinergic side effects

Carisoprodol (eg, Soma)


Centrally acting Muscle spasms Withdrawal side effects
Agitation, anorexia, hallucinations, seizures

5. Which of the following statements is true:


A. Topical analgesics are not typically associated with systemic side effects B. Topical analgesics are typically associated with multiple drug interactions C. Topical diclofenac is indicated for treating CABG perioperative pain D. EMLA is FDA approved for Rx of pain E. Lidocaine 5% patch (Lidoderm) is effective when applied anywhere on the body, not just over the site of pain

Topical Analgesics
Active within the skin, peripheral nerves No significant serum levels of drug
No significant systemic side effects Relatively free of drug interactions

Topical Analgesics
ASA Diclofenac topical (Voltaren gel) Capsaicin
Neuropathic pain & DJD Effectiveness limited by side effects & compliance

Local anesthetics
EMLA
Eutectic mixture of lidocaine & prilocaine Not FDA approved for pain

Lidocaine patch 5% (Lidoderm)

Diclofenac Topical (Voltaren 1%Gel)


Osteoarthritis pain 2 g QID for upper extremity joints 4 g QID for lower extremity joints
Comes with dosing card

Can see NSAID side effects Black box warning for cardiovascular thrombotic events & GI bleeding Contraindicated for perioperative CABG pain

Topical Lidocaine Patch 5% (Lidoderm)


Up to 3 patches at a time over painful site
12 hrs duration

Side effects
Application site sensitivity Systemic side effects rare

Mechanical barrier may decrease allodynia Efficacy demonstrated for post-herpetic neuralgia

Opioids

Prof. Dr. Otto Wilhelm Thom ''Flora von Deutschland, sterreich und der Schweiz'' 1885, Gera, Germany

6. Which of the following statements regarding opioids is true:


A. Opioids have been shown to be effective for cancer-related pain, but not for neuropathic pain B. A realistic goal of opioid therapy is complete pain relief C. The CAGE questionnaire is a good screen for EtOH abuse, but not substance abuse D. It is unnecessary to identify individuals at risk for substance abuse prior to beginning opioid therapy E. Treatment should begin with short-acting agents, then titrated to optimal effect

Opioid Epidemic
Sales of opioids increased 4x from 19972006
Oxycodone inc 8x Methadone inc 9x

ED visits for nonmedical use of opioids increased 3x from 2004-2009

Opioid Epidemic
124% inc in unintentional deaths due to Rx med use 1999-2007
14,000 deaths in 2009 #2 (MVA = #1) Mostly opioids
Methadone- 30% OD deaths, only 2% of analgesic Rxs

Highest risk in doses > 50 mg morphine equiv/day High risk in Iraq/Afghan vets with PTSD/MH Dx

Opioid Epidemic
Pain often goes undetected in primary care, and when detected, is often undertreated

Increased opioid prescribing has resulted in increased opioid misuse, including fatal overdose, diversion, and addiction

Opioid Epidemic
FDA
Risk Evaluation & Mitigation Strategy (REMS) FDA Blueprint for Prescriber Education for Extended Release & Long-Acting Opioids
http://www.fda.gov/downloads/Drugs/DrugSafety/PostmarketDrugSafetyInformati onforPatientsandProviders/UCM311290.pdf

Voluntary CME Drug company sponsored On-line Targets


Extended release/long-acting opioids Transmucosal immediate release fentanyl

Efficacy of Opioids for Chronic Pain


Good evidence for effectiveness in pain from cancer Cochrane review
45% of pts have 50% reduction in pain scores per Cochrane

Osteoarthritis Neuropathic pain Back pain

Efficacy of Opioids for Chronic Pain


Patients with chronic diffuse centralized pain
HA, fibromyalgia, abd pain, pelvic pain Less likely to benefit from opioids More likely to be harmed from opioids

Opioid TherapyRealistic Goals


Reach agreement with patient on shared goals of treatment Complete pain relief rarely achieved Common goals include
Pain reduction Improvement in selected areas of function Improved mood Improved work

Opioid TherapyRealistic Goals


Treatment with opioids should result in functional improvement Not merely reduction in pain score

Prior to Opioid Use


Focal or treatable pain generators should be identified and treated Use physical and psychological modalities Attempt restoration of sleep Treat mood disorders Use non-opioid/adjuvant analgesics

Opioid Therapy
Informed consent & treatment agreement Opioid therapeutic trial < 90 days
Specific evaluation points

Begin with short-acting agents (morphine) Titrate to optimal effect Aggressively manage side effects Document exit strategies

Selecting Patients for Narcotic Therapy


What is the diagnosis? Assessment for addiction Check criminal record: www.doc.state.nc.us/offenders Check for controlled substance Rxs
Risk mitigation tools

Informed consent & treatment agreement Drug screening at least every 12 months Reassessment
Is the medication improving function? If not, implement exit strategy.

Screening for Substance Use Disorders


Does patient identify drug(s) of choice for emotional symptoms?
Determine quantity, frequency, and duration of use for each drug of choice

Has patient had formal treatment for substance use disorders? Is there a family history of substance use and/or other psychiatric disorders?

Opioid Risk Tool


Female Family Hx EtOH Family Hx Illicit Drugs Family Hx Rx Drugs Patient Hx EtOH Patient Hx Illicit Drugs Patient Hx Rx Drugs Age 16-45 Hx preadolescent sex abuse ADHD/OCD/Bipolar/Schizophrenia Depression 0-3 = low risk, 4-7 = mod risk, > 8 = high risk 1 2 4 3 4 5 1 3 2 1 Male 3 3 4 3 4 5 1 0 2 1

Pain Assessment: Substance Abuse History Using the CAGE Questionnaire as a Screening Tool
Cut down
(Ever felt you needed to cut down your use of ____?)

Annoyed
(With others comments about your drug use?)

Guilty
(About your use of ____?)

Eye opener
(Need to use in AM to function?)

7. Which of the following statements regarding opioids is true:


A. There are 3 opioid receptors: phi, beta, & kappa B. Morphine is available in both short- and long-acting preparations C. Oxycodone is roughly 3x as potent as morphine D. When pain is not controlled, a longacting narcotic dose should be increased by 10% E. Transdermal fentanyl is indicated for acute and post-op pain management

Opioid Receptors

Three opioid receptor classes


Mu
Morphine

Kappa
Butorphanol

Delta
Enkephalins

Short-Acting Opioids
Morphine sulfate (eg, Roxanol) Codeine Hydrocodone (eg, Zydone, Vicodin, Lortab, Lorcet, Norco, Vicoprofen) Oxycodone (eg, Percocet, Tylox, Percodan) Hydromorphone (eg, Dilaudid) Oxymorphone (eg, Opana) Fentanyl (eg, Actiq)

Long-Acting Opioids
Methadone (eg, Dolophine, Methadose) Sustained-release morphine (eg, MS Contin, Avinza, Kadian, Oramorph) Sustained-release oxycodone (eg, Oxycontin) Sustained-release hydromorphone (Exalgo) Sustained-release oxymorphone (eg, Opana) Buprenorphine/naloxone (eg, Suboxone) Transdermal buprenorphine (Butrans) Transdermal fentanyl (eg, Duragesic)

Long-Acting Opioids
Only use the following if patient is opioid tolerant:
Transdermal fentanyl ER hydromorphone High doses of most long-acting opioids

Opioid tolerant
Taking 60 mg morphine, 25 mcg fentanyl, 30 mg oxycodone, 8 mg hydromorphone, 25 mg oxymorphone per day Duration > 1 week

Long-Acting Opioids
ER tablets should be swallowed whole
Do Not cut, crush, break

Some ER morphine capsules can be opened and sprinkled on applesauce Transdermal patches should be used intact
Do NOT cut or tear or expose to heat

Improper administration can lead to rapid absorption of high doses of opioids, overdose, respiratory depression, and death

NarcoticsEquianalgesic Doses
Drug Proprietary PO IM Name (mg) (mg) Roxanol, MSIR [off market] MS Contin Avinza life (hrs) Duration (hrs)

Morphine

20-30 10

2-3

2-4

Morphine CR Morphine SR

20-30 10 20-30 10

2-3 2-3

8-12 24

NarcoticsEquianalgesic Doses
Drug Proprietary PO IM Name (mg) (mg) life Duration (hrs) (hrs)

Oxycodone

Percocet

20

2-3

3-4

Oxycodone CR Oxycontin

20
1

2-3
2-3

12
2-4

Oxymorphone Numorphan 10

Management of cancer pain: Pharmacotherapy. Pain management: The online series, module 11. American Medical Association, 2006

NarcoticsEquianalgesic Doses
Drug Brand Name
Subutex, Butrans PO (mg) IM (mg) life Duration (hrs) (hrs) 0.4 180 0.3 2-7 3-4 250 mcg IV = 4 7-12 mg/hr 16-242 48-72 30 3-4

Buprenorphine Codeine

Fentanyl Fentanyl TTS


Hydrocodone

Sublimaze Duragesic Lortab, Vicodin

NarcoticsEquianalgesic Doses
Drug Proprietary Name
PO IM (mg) (mg) life (hrs) Duration (hrs)

Hydromorphone Dilaudid Levorphanol Meperidine Methadone Tapentadol Tapentadol ER

7.5

1.5 2

2-3 12-15

2-4 4-6 6-8

Levo-dromoran 4 Demerol Dolophine Nucynta

300 100 20 100 10

12-190 4-12 6

Nucynta

100

12

Conversion to Long-Acting Opioid


With chronic pain, conversion from a short-acting to a long-acting opioid can be a reasonable choice for overall pain management Long-acting opioids may provide a more stable blood level

Treatment of Breakthrough Pain


If the patients baseline pain is uncontrolled, most experts would recommend increasing the long-acting opioid by:
20-25% for mild to moderate pain 50-100% for severe pain

Rationale for Opioid Rotation


Ineffectiveness of initial opioid Adverse effects/toxicity of initial opioid Inter-patient variability of response Incomplete cross-tolerance

Fentanyl Patch (Duragesic) Advantages


25 mcg/hr of the fentanyl patch (Duragesic) is equivalent to 60-135 mg per day of orally administered morphine Can provide pain control in patients who are unable to swallow medications Can be applied by the patient, a family member, or other caregiver

Fentanyl Patch (Duragesic)


Contraindications
Not opioid tolerant Acute pain or pain of short duration Post-op pain Mild pain Intermittent pain

Buprenorphine Patch (Butrans)


Recently FDA approved for chronic pain 5 mcg/hr in patients who were receiving a total daily morphine dose or equivalent of less than 30 mg 10 mcg/hr in patients who were receiving a total daily morphine dose or equivalent of 30-80 mg Titrate to pain control at min 72 hrs Maintenance dose = 20 mcg/hrchange patch every 7 days

8. Which of the following is correct:


A. Constipation is rarely seen with opioid use
B. Respiratory depression is a concerning side effect of opioid use C. Withdrawal symptoms are not seen with cessation of long-acting narcotics, ie, methadone D. Miosis is noted during opioid withdrawal E. Opioid withdrawal precipitated by narcotic antagonists is typically mild, and not life threatening

Opioid Side Effects


Nausea/vomiting Sedation Respiratory depression Constipation Dizziness/orthostatic hypotension Mental confusion Itching Edema Decreased libido

Opioid Withdrawal
After cessation of opioid Use of partial agonists
Buprenorphine (Buprenex)

Use of agonist-antagonists
Pentazocine (Talwin), nalbuphine

Use of antagonist
Naloxone (Narcan)

Opioid Withdrawal
May begin 6 to 12 hours after the last dose of a short-acting opioid
Persist for several days

24 to 48 hours after cessation of methadone


Persist for several weeks

May begin immediately after reversal with antagonist


Potentially life-threatening

Peak within 24 to 48 hours of onset

Opioid Withdrawal
Symptoms
Nausea, vomiting Abd pain Diarrhea Goose bumps Excessive yawning Tremors Myalgias & arthralgias Rhinorrhea Lacrimation Anxiety & restlessness

Opioid Withdrawal
Physical findings
Mydriasis Yawning Hyperactive bowel sounds Piloerection Tachycardia Hypertension or hypotension Tachypnea

Opioid Withdrawal
Treatment
Naturally occurring withdrawal
Either opioid or nonopioid adjunctive medication Methadone 10 mg IM or 20 mg PO
Usually sufficient to relieve symptoms of withdrawal without producing intoxication

Clonidine (Catapres)
0.1-0.3 mg PO every hour

Benzodiazepines Antiemetics Antidiarrheals

Opioid Withdrawal
Treatment
Iatrogenic withdrawal (from an opioid antagonist)
Adjunctive medications should be used rather than opioids

9. Continued exposure to a drug induces changes that result in a decrease in that drugs effect over time describes which of the following:
A. Addiction B. Pseudo-addiction C. Physical dependence D. Tolerance E. Litigation

Opioid-Associated States
Tolerance Physical dependence Addiction Pseudo-addiction

Tolerance
Continued exposure to a drug induces changes that result in a decrease in drug effects over time

Physical Dependence
State of adaptation manifested by drug classspecific withdrawal syndrome Precipitated by
Abrupt cessation Dose reduction Decreased blood level Administration of antagonist

Addiction (ASAM 2001)


Primary, chronic, neurobiologic disease with genetic, psychosocial, and environmental factors influencing its development and manifestations Characterized by one or more of following behaviors
Compulsive use Continued use despite harm Craving

Addiction (ASAM 2011)


Addiction is a primary, chronic disease of brain reward, motivation, memory and related circuitry. Dysfunction in these circuits leads to characteristic biological, psychological, social and spiritual manifestations. This is reflected in an individual pathologically pursuing reward and/or relief by substance use and other behaviors. Addiction is characterized by inability to consistently abstain, impairment in behavioral control, craving, diminished recognition of significant problems with ones behaviors and interpersonal relationships, and a dysfunctional emotional response. Like other chronic diseases, addiction often involves cycles of relapse and remission. Without treatment or engagement in recovery activities, addiction is progressive and can result in disability or premature death.

Pseudo-Addiction
Iatrogenic syndrome of behaviors developing as a result of inadequate pain control Behaviors disappear when pain is adequately controlled, unlike addiction Controversial
Not recognized in Federation of State Medical Boards definition list

Aberrant Drug-Related Behaviors Addiction


Rx forgery Concurrent use of illicit drugs Recurrent Rx losses Selling Rx drugs Stealing or borrowing drugs Getting meds from nonmedical sources Frequent unsanctioned dose escalations

Aberrant Drug-Related Behaviors (Addiction Less Likely)


Drug hoarding Aggressive complaining about need for higher doses Unapproved use to treat other symptoms Requesting specific drugs Acquisition of meds from medical sources Infrequent dose escalations

10. Which of the following statements are true:


A. Pain management (informed consent & treatment) agreements have been proven to dramatically decrease the incidence of addiction B. Pain management (informed consent & treatment) agreements are recommended for all chronic opioidreceiving patients by the Federation of State Medical Boards C. Pain management (informed consent & treatment) agreements typically mandate regularly scheduled drug testing D. Synthetic and semisynthetic opioids like oxycodone, fentanyl, and methadone are usually detected in immunoassay-based urine drug testing E. Opioid use should generally be stopped after there is evidence of effective pain relief and improved ADLs

Pain Management Agreements


(Informed Consent & Treatment Agreements) Frequently recommended by experts in chronic pain management Recommended by Federation of State Medical Boards Discussed extensively in related literature Used by many physicians No studies have demonstrated a reduction in the incidence of addiction or abuse when contracts are used

Pain Management Agreements


(Informed Consent & Treatment Agreements)

Sample PMA
www.aapainmanage.org/literature/ Articles/OpioidAgreements.pdf

Pain Management Agreements


(Informed Consent & Treatment Agreements) Potential risks & benefits of opioid Rx Potential side effects Likelihood of tolerance/physical dependence Risk of opioid misuse, addiction, overdose Risk of drug interactions, oversedation, and impaired motor skills Providers prescribing policies and expectations Reasons why therapy may be changed/stopped

Pain Management Agreements


(Informed Consent & Treatment Agreements) Goals of treatment Patient responsibility for safe medication use Patient responsibility to obtain opioids from only one source Patient agreement to periodic drug testing Providers responsibility to be available or have coverage for unforeseen problems/scheduled refills

Urine Drug Testing


Enzyme-linked immunoassay testing of antibodies is usually the first urine drug test done because it is quick and may be used to test for multiple drugs at once Immunoassay testing is not always accurate for detecting all drugs and there is sometimes cross reactivity with others Use gas chromatography/mass spectrometry for specific drug testing

Urine Drug Testing


Morphine and codeine usually are detected accurately Synthetic and semisynthetic opioids like oxycodone, fentanyl, and methadone are not always detected with immunoasssay
Can be specifically tested with GC/MS

Urine Drug Testing


Cocaine testing has a low degree of cross reactivity and is generally a true positive A negative test for benzodiazepines may not be accurate because it sometimes does not detect recent use Tests for amphetamine/methamphetamine have a high degree of cross reactivity to other sympathomimetics like ephedrine or pseudoephedrine

Continue Opioid Therapy


If there is
Improvement in ADLs and psychosocial functioning Effective pain relief Management or countering of side effects

Opioid Exit Strategy


Upon initiating opioid therapy, agree with patient on criteria for stopping opioid Rx Common failure criteria include: Lack of significant pain reduction Lack of improvement in function Persistent side effects Persistent noncompliance

Opioid Exit Strategy


Discontinue opioids
Test positive for illicit drug use Threatening behavior to provider/staff Forgery/diversion Drug OD/intoxication

Tapering off opioids Gradual


10% dose reduction every 1-4 weeks

Rapid
25% dose reduction every 3-7 days

When to Refer to Pain Specialist


> 100 mg/day of morphine equivalent History of abuse or drug-related offenses Failure to improve with multiple trials of medications Selected patients who could benefit from invasive therapy, ie, steroid epidural injection Consider psychological evaluation in all patients

Summary
Chronic pain management is a major health care issue/challenge Successful management requires provider knowledge, and both interpersonal and organizational skills

Summary
Opioids are an essential tool in the overall treatment and management of chronic pain. Appropriate use of opioids depends upon differentiating among dependence, tolerance, and addiction, and identifying aberrant drug-taking behaviors.

Answers
1. 2. 3. 4. 5. 6. 7. 8. 9. 10. B A E C A E B B D B

Management of Chronic Pain


Gary I. Levine, MD, FAAFP
Associate Professor Department of Family Medicine The Brody School of Medicine at Eastern Carolina University Greenville, North Carolina

Managing Common Cutaneous Problems (Part 1)


Gary Levine, MD, FAAFP Associate Professor Brody School of Medicine at East Carolina University Greenville, NC

Faculty Disclaimer
Gary Levine, MD, returned disclosures indicating that indicating that he has no affiliation or financial interest in any organization(s). Gary Levine, MD, returned disclosures indicating that he is not a dermatologist
The AAFP has selected all faculty appearing in this program. It is the policy of the AAFP that all CME planning committees, faculty, authors, editors, and staff disclose relationships with commercial entities upon nomination or invitation of participation. Disclosure documents are reviewed for potential conflicts of interest and, if identified, they are resolved prior to confirmation of participation. Only those participants who had no conflict of interest or who agreed to an identified resolution process prior to their participation were involved in this CME activity.

Learning Objectives
1. 2. Summarize the AAFP Core Educational Guidelines Conditions of the Skin. Recognize many of the dermatologic entities included in the curriculum guide, including but not limited to: psoriasis, tinea corporis, ichthyosis, lichen planus, bullous pemphigoid, neurodermatitis, tinea versicolor, herpes zoster, rhus dermatitis, contact dermatitis, scabies, atopic dermatitis, folliculitis, furunculosis, pseudofolliculitis barbae, acne vulgaris, rosacea, seborrheic dermatitis, perioral dermatitis, discoid lupus, pityriasis alba, vitiligo, erythrasma, melasma.

References
Charles M. Phillips, MD
Dept. of Medicine, Brody School of Medicine

Richard P. Usatine, MD
Dept. of Family Medicine, UTHSCSA

Clinical Dermatology
Thomas Habif
5th Edition , 2009

Centers for Disease Control & Prevention


Sexually Transmitted Diseases Division

Jefferson Medical College Image Bank

AAFP Core Educational Guidelines Conditions of the Skin


American Family Physician. Vol 60 #4 Sept. 15, 1999 Curriculum guide for dermatologic entities that family physicians should be familiar with

Primary Lesion Types


Macules

Papules

Nodules
Dr. Richard P. Usatine

Primary Lesion Types


Plaques

Pustules

Vesicles/Bullae
Dr. Richard P. Usatine

Primary Lesion Types


Wheals

Scales

Crusts
Dr. Richard P. Usatine

Primary Lesion Types


Erosions/Ulcers

Fissures/Atrophy

Scars
Dr. Richard P. Usatine

Case # 1

Dr. Richard P. Usatine

Case # 1
A 45 y/o male presents with a chronic rash that is present over his knees and elbows.

Dr. Richard P. Usatine

1. Which of the following represents the most likely diagnosis?

A. Tinea corporis B. Psoriasis C. Discoid lupus erythematosus D. Bowens disease

Psoriasis

Dr. Richard P. Usatine

Plaques & Scales


Psoriasis Chronic cutaneous (discoid) lupus Tinea corporis Pagets disease Lichen planus Cutaneous T-cell lymphoma Eczema Pityriasis rosea Secondary syphilis Bowens disease Ichthyosis

Discoid Lupus

Discoid lupus

Images Dr. Richard P. Usatine

Tinea Corporis

Dr. Richard P. Usatine

Pityriasis Rosea

Dr. Richard P. Usatine

Bowens Disease

Dr. Richard P. Usatine

Ichthyosis

Dr. Richard P. Usatine

Lichen Planus

Jefferson Clinical Images Database. http://jeffline.jefferson.edu/JCI/.

Lichen Simplex Chronicus

Jefferson Clinical Images Database. http://jeffline.jefferson.edu/JCI/.

Neurodermatitis

Images Dr. Richard P. Usatine

Psoriasis

Dr. Richard P. Usatine

Psoriasis
Oval, erythematous, plaque-like lesions Can develop at sites of trauma Often involves extensor surfaces
Elbows, knees, & scalp

Pitting fingernails Associated with asymmetric polyarthritis

Psoriasis

Google Images

PsoriasisTypes
Chronic plaque psoriasis Guttate psoriasis Pustular psoriasis Erythrodermic psoriasis

PsoriasisTreatment
< 20% of body involved
Topical corticosteroids Calcipotriene (Dovonex)
Vitamin D3 analog

Tazarotene (Tazorac) Anthralin (Anthraderm) Tar UVB Intralesional steroids

PsoriasisTreatment
> 20% of body involved (phototherapy)
UVB
Broad band, narrow band (II-III, B-C) +/- topical, systemic, biologic agents

PUVA
Ultraviolet + psoralen (IA) +/- topical, systemic, UVB (II-III, B-C)

Excimer laser (IIB)

PsoriasisTreatment
Severe recalcitrant disabling (FDA approved)
Methotrexate (eg, Rheumatrex) (IIB)
Gold standard

Acitretin (eg, Soriatane) (IIB)


Plaque type

Cyclosporine (eg, Sandimmune) (IIB)

Anti-TNF agents
Infliximab, etanercept, adalimumab

Case # 2

Images Dr. Richard P. Usatine

Case # 2
A 65 y/o woman presented with this painful rash on her face

Images Dr. Richard P. Usatine

2. Which of the following therapeutic modalities would be most beneficial?


A. Prednisone B. Acyclovir (eg, Zovirax) C. Silver sulfadiazine (eg, Silvadene) D. Gabapentin (Neurontin)

Herpes Zoster

Images Dr. Richard P. Usatine

Bullous Lesions
Herpes simplex Erythema multiforme Herpes zoster Porphyria cutanea Impetigo tarda Dermatitis Fixed drug herpetiformis eruptions Burns Bullous pemphigoid Pemphigus vulgaris

Bullous Pemphigoid

Images Dr. Richard P. Usatine

Burn2nd Degree

Images Dr. Richard P. Usatine

Erythema Multiforme

Jefferson Clinical Images Database. http://jeffline.jefferson.edu/JCI/.

Herpes Zoster (Shingles)


Varicella-zoster virus Reactivation of latent infection 10-20% lifetime incidence Generally involves skin of a single dermatome Pre-eruptive pain, itching, burning (4-5 days) Fever, headache, malaise

Herpes Zoster (Shingles)


Vesicles, of varying size, on erythematous base Successive crops over 7 days Crust lasts 2-3 weeks Postherpetic neuralgia
Increases with age of patient Increases with pain during eruptive phase Incidence = 20% at 1 month

Herpes ZosterTreatment
Analgesics Wet compresses (Burows Solution) Antiviral therapy
Acyclovir (eg, Zovirax)
800 mg QID x 7 days

Famciclovir (eg, Famvir)


500 mg q 8 hrs x 7 days

Valacyclovir (eg, Valtrex)


1 g TID x 7 days

Herpes ZosterTreatment
Oral corticosteroids (eg, Prednisone)
May decrease pain initially during acute phase Does not reduce subsequent postherpetic neuralgia

Sympathetic nerve blocks


Bupivacaine Must be given within 2 months of onset to be effective

Postherpetic NeuralgiaTreatment
Narcotic analgesics Anticonvulsants
Pregabalin (Lyrica) Gabapentin (eg, Neurontin)

Tricyclics Capsaicin

Herpes Zoster Vaccine (Zostavax)


Contains the same live attenuated varicella virus as Varivax but at a much higher titer of vaccine virus Approved by FDA for persons 50 years of age and older Recommended by CDC for > 60 y/o Regardless of past Hx of zoster Do not use if immunosuppressed, immunodeficient, pregnant, TB, or allergic to neomycin/gelatin Administered by the subcutaneous route

Herpes Zoster Vaccine Efficacy


Compared to the placebo group, the vaccine group had:
51% fewer episodes of zoster Less severe disease 66% less postherpetic neuralgia

No significant safety issues were identified

NEJM 2005;352(22):2271-84.

Case # 3

Dr. Richard P. Usatine

Case #3
A 23 y/o man presents with an itchy rash on his arm

Dr. Richard P. Usatine

3. What is the most likely etiology of this rash?


A. Exposure to poison ivy B. Staphylococcus aureus (MRSA) C. Sarcoptes scabiei D. Thorn from a rose (sporotrichosis)

Rhus Dermatitis

Dr. Richard P. Usatine

Rhus Dermatitis

Images Dr. Richard P. Usatine

Contact Dermatitis

Contact dermatitis - nickel

Dr. Richard P. Usatine

Rhus Dermatitis
Caused by contact with urushiol (from sap) Found in plants from Anacardiaceae family, Rhus (Toxicodendron) genus
Poison ivy Poison oak Poison sumac Cashew Mango Ginkgo Japanese lacquer tree

Rhus Dermatitis
Linear lesions Vesicles
Fluid does not contain resin and wont spread rash

Erythema May occur within 8 hrs or up to a week after exposure

Rhus DermatitisTreatment
Decontamination within 10 min after exposure
Not helpful after 1 hr

Wet compresses Topical corticosteroids Systemic corticosteroids


Prednisone 20 mg BID x 1 week Prednisone 40-60 mg single dose

IM steroids
Triamcinalone acetonide (Kenalog) 40 mg IM

Case # 4

Images Dr. Richard P. Usatine

Case # 4
A 25 y/o male with known HIV infection, who lives in substandard housing, presents with a generalized pruritic rash

4. Which of the following represents the best treatment for this condition?
A. Prednisone B. Hydroxyzine C. Permethrin 5% (Elimite) D. Abacavir (Ziagen)

Scabies

Images Dr. Richard P. Usatine

Papules (Pruritic, Erythematous)


Miliaria rubra Atopic dermatitis Urticaria Insect bites Scabies Pruritic papular eruption (HIV) Pruritic urticarial papules and plaques of pregnancy (PUPPP)

Atopic Dermatitis

Dr. Richard P. Usatine

Atopic Dermatitis

Google Images

Atopic Dermatitis
Chronic inflammatory skin condition
Involves a genetic defect in the proteins supporting the epidermal barrier

Atopic dermatitis and eczema are frequently used interchangeably Prevalence =11% Onset before 5 y/o Staph. aureus colonization is common

Atopic Dermatitis
Infantile
Pruritic, red, scaly, and crusted lesions Extensor surfaces, cheeks, scalp Diaper area usually spared

Late childhood & adult


Lichenified plaques Flexural distribution
Antecubital and popliteal fossae, volar aspect of the wrists, ankles, and neck

Atopic DermatitisRx
Avoid trigger factors
Heat, perspiration, low humidity

Treat skin infections if present


Staph. aureus, herpes simplex

Antihistamines for sedation/itching Treat stress and anxiety Moisturizers to maintain skin hydration

Atopic DermatitisRx
Low-potency steroid (class V or VI) on face
Desonide 0.05% ointment daily x 3 wks

Topical calcineurin inhibitor


If Rx required on face/flex creases > 3 wks
Tacrolimus 0.03% BID Pimecrolimus

Adults with mod-severe disease


Medium to high potency topical steroids

Urticaria

Dr. Richard P. Usatine

Scabies

Dr. Richard P. Usatine

Scabies
Hypersensitivity reaction to Sarcoptes scabiei
Eggs, fecal pellets (scybala)

Nocturnal pruritus
Scratching spreads mites to other areas

Curved or linear burrows Vesicles or small papules Pustules indicate secondary infection

Scabies
Location of lesions
Finger webs Wrists Elbows Knees Buttocks Axilla Waist Breasts Genitals

Scabies Burrow

Images Dr. Richard P. Usatine

ScabiesDiagnosis
Locate burrow with felt-tip pen ink Scrape with #15 curved scalpel blade View under mineral oil or KOH Look for mites, eggs, feces (scybala)

Sarcoptes Scabiei

Dr. Richard P. Usatine

Norwegian (Crusted) Scabies


Overwhelming infestation Crusted lesions Not particularly pruritic Seen mostly in immunocompromised patients

ScabiesTreatment
Launder all bedding and clothes worn within 48 hrs in hot water, or dry clean Treat patient, intimate contacts, and family members in same household

ScabiesTreatment
5% Permethrin cream (Elimite)
Drug of choice Apply below the neck, may repeat in 1 week

Lindane
More toxic, especially in children/pregnancy

Ivermectin (Stromectol)
Effective for Norwegian scabies

Crotamiton (Eurax) Benzyl benzoate 25% lotion 6% precipitated sulfur in petrolatum

Case # 5

Images Dr. Richard P. Usatine

Case # 5
A 55 y/o congressman c/o new rash on his chest after returning from a business trip with a female lobbyist. He admitted to spending an inordinate amount of time having meaningful conversations with her in a hot tub.

Images Dr. Richard P. Usatine

5. Which one of the following would you recommend for the management of this condition?
A. Oral ciprofloxacin B. No pharmacologic therapy, but stay out of hot tubs C. Oral TMP-SMX D. Topical clotrimazole

Images Dr. Richard P. Usatine

Folliculitis

Images Dr. Richard P. Usatine

Folliculitis

Google Images

Folliculitis
Folliculitis
Superficial bacterial infection of the hair follicles Purulent material in the epidermis

Carbuncle
Coalescence of several inflamed follicles into a single inflammatory mass Purulent drainage from multiple follicles

Furuncle (boil)
Infection of the hair follicle in which purulent material extends through the dermis into the subcutaneous tissue Small abscess forms
Collection of pus within the dermis and deeper skin tissues

Furunculosis

Google Images

Sycosis Barbae

Google Images

Folliculitis
Clusters of multiple small, raised, pruritic, erythematous lesions less than 5 mm in diameter Sycosis barbae
Folliculitis seen in area of repeated shaving

Folliculitis
Etiologic agents
Usually attributable to Staph. aureus Pseudomonas & aeromonas
Hot tubs, whirlpools, swimming pools

Candida species
Broad-spectrum antibiotics Immunocompromised Steroid Rx

Non-TB mycobacteria

FolliculitisRx
Lesions usually resolve spontaneously
With or without drainage

Warm compresses may be applied three times daily Shaving should be avoided in involved areas Hot tub folliculitis
Avoid exposure to the source of contaminated water

Mupirocin
Persistent lesions with suspected Staph etiology

Topical antifungals (azoles)


Suspected fungal etiology

Pseudofolliculitis Barbae

Google Images

Pseudofolliculitis Barbae
Razor bumps, shave bumps, ingrown hairs 45-83% black men, 3% white men who shave facial hair Results from an inflammatory response to the cutaneous entrapment of recently cut, short hairs Firm papules and pustules in the beard area Complications
Postinflammatory hyperpigmentation, secondary bacterial infection, scarring, keloid formation

Pseudofolliculitis BarbaePrevention
Cessation of shaving Alternative shaving methods
Electric shaver adjusted to leave hair long Single-blade razor Highly lubricating shaving cream Circular washing method of beard daily

Alternative hair removal techniques

Pseudofolliculitis BarbaeRx
Medical therapies
Topical retinoids Low-potency topical corticosteroids Topical antimicrobials Eflornithine hydrochloride (Vaniqa) Alpha hydroxy acids
Glycolic acid

Case # 6

Images Dr. Richard P. Usatine

Case # 6
A 16 y/o female would like to be treated for this facial rash, which has not responded to OTC cleansing preparations. Her high school prom is 3 months away. She is not sexually active.

Images Dr. Richard P. Usatine

6. Which one of the following would you recommend for the management of this condition?
A. Topical retinoid alone B. Topical retinoid + topical benzoyl peroxide C. Topical retinoid + topical benzoyl peroxide + oral doxycycline D. Isotretinoin
Images Dr. Richard P. Usatine

Acne Vulgaris

Dr. Richard P. Usatine

Acne Vulgaris

Google Images

Acne Vulgaris
Disorder of the pilosebaceous units Chronic inflammatory dermatosis Notable for open/closed comedones, papules, pustules, or nodules.

Acne Vulgaris
Androgen-mediated disorder of pilosebaceous units
Androgens stimulate sebum production and proliferation of keratinocytes Keratin plug obstructs follicle os Propionibacterium acnes proliferates in plugged follicle P. acnes growth produces inflammation

Acne VulgarisEpidemiology
Predominant age: early to late puberty, may persist into 4th decade 8095% of adolescents affected. 8% of adults aged 2534; 3% at 3544 years Predominant sex:
Male > Female (adolescence) Female > Male (adult)

Familial association in 50%


If a family history exists, the acne may be more severe and occur earlier.

Acne VulgarisRisk Factors


Increased endogenous androgenic effect Oily cosmetics Rubbing or occluding skin surface
Sports equipment such as helmets and shoulder pads, telephone, or hands against the skin

Polyvinyl chloride, chlorinated hydrocarbons, cutting oil, tars Numerous drugs


Androgenic steroids Some birth control pills

Acne VulgarisRisk Factors


Endocrine disorders
Polycystic ovary syndrome Cushing syndrome Congenital adrenal hyperplasia Androgen-secreting tumors Acromegaly

Stress High glycemicload diets and milk Smoking


May worsen severe acne

Acne VulgarisLesions
Closed comedones (whiteheads) Open comedones (blackheads) Nodules or papules Pustules (cysts) Scars: ice pick, rolling, boxcar, atrophic macules, hypertrophic, depressed, sinus tracts

Acne Vulgaris

Google Images

Acne Vulgaris

Google Images

Acne Vulgaris

Google Images

Acne VulgarisLesions
Most common areas affected:
Areas of greatest concentration of sebaceous glands

Face Chest Back Upper arms

Acne VulgarisLesions
Grading system
American Academy of Dermatology, 1990

Mild: Few papules/pustules; no nodules Moderate: Some papules/pustules; few nodules Severe: Numerous papules/pustules; many nodules Very severe: Acne conglobata, acne fulminans, acne inversa.

Acne VulgarisTreatment
Recommended vehicle type
Dry or sensitive skin
Cream or ointment

Oily skin, humid weather


Gel, solution, pledget, or wash

Hair-bearing areas
Lotion, hydrogel, or foam

Apply topical agents to entire affected area


Not just visible lesions

Mild soap daily to control oiliness


Avoid abrasives

Acne VulgarisTreatment
Avoid drying agents with keratinolytic agents Use of a gentle cleanser and noncomedogenic moisturizer
Helps decrease irritation from keratinolytic agents

Oil-free, noncomedogenic sunscreens Stress management


If acne flares with stress

Acne VulgarisTreatment
Comedonal acne
Keratinolytic agent

Mild inflammatory acne


Keratinolytic agent +/- BP/topical antibiotic

Moderate inflammatory acne


BP/topical antibiotic +/- systemic antibiotic

Severe (nodulocystic) acne


Isotretinoin

Acne VulgarisTreatment
Topical keratinolytics
Tretinoin (eg, Retin-A)
Start with lowest concentration of cream and advance as tolerated Apply hs after washing May cause an initial flare of lesions Side effects include erythema, dryness, scaling

Adapalene (eg, Differin)


Apply hs after washing Better tolerated than tretinoin and equally effective

Acne VulgarisTreatment
Topical keratinolytics
Tazarotene (Tazorac)
Binds to nuclear retinoic acid receptors Side effects similar to tretinoin, not as well tolerated Teratogenic

Azelaic acid (Azelex, Finacea)


Apply BID Keratinolytic, anti-inflammatory, and antibacterial Can cause hypopigmentation

Acne VulgarisTreatment
Topical antibacterials
Benzoyl peroxide
Apply thin film 1-2x/day, preferably hs No known resistance May cause skin irritation and bleach clothes

Benzoyl peroxide/erythromycin
Apply once daily Must be refrigerated

Benzoyl peroxide/clindamycin

Acne VulgarisTreatment
Topical antibacterials
Erythromycin (eg, A/T/S, Emgel)
Increasing P. acnes resistance

Clindamycin (eg, Cleocin T, Dalacin T)


Apply BID

Tetracycline (Topicycline) 0.22% solution


Apply BID May cause skin to fluoresce in black lights

Metronidazole (Metrogel, Metrocream)


Apply once daily

Dapsone (Aczone) 5% gel, apply BID

Acne VulgarisTreatment
Systemic antibiotics
Tetracycline
500-1,000 mg daily Begin at high dose, and taper in 2-4 weeks if patient responds Do not use in children < 8 years old May cause photosensitivity

Acne VulgarisTreatment
Systemic antibiotics
Minocycline (Minocin)
50-200 mg daily Most effective and expensive of tetracycline group Less photosensitivity than tetracycline, but other side effects include vertigo, autoimmune hepatitis, and lupuslike syndrome

Doxycycline (Vibramycin)
50-200 mg daily Can take with food, but higher incidence of photosensitivity

Woods Lamp

Google Images

Acne VulgarisTreatment
Systemic antibiotics
Erythromycin
500-1000 mg daily GI side effects are commonly seen Resistance in P. acnes may limit effectiveness

Azithromycin
500 mg initially then 250 mg x 4 days

Sulfamethoxazole-trimethoprim (Bactrim, Septra)


1 DS tablet once or twice daily Used for gram-negative folliculitis or resistance to tetracycline and erythromycin

Acne VulgarisTreatment
Use oral or topical antibiotics for 3 months
Stop if inflammatory lesions resolve

Can switch abruptly from oral to topical without taper Do not use topical and oral together

Acne VulgarisTreatment
Oral contraceptives
Ethinyl estradiol +
Norgestimate (eg, Ortho Tri-cyclen) Norethindrone (eg, Estrostep) Levonorgestrel (eg, Alesse) Drospirenone (eg, Yaz, Yasmin)

Spironolactone (eg, Aldactone)


Only use in women

Acne VulgarisTreatment
Isotretinoin
60-90% cure rate Given for 12-20 weeks Side effects
Cheilitis, hyperlipidemia, pseudotumor

Highly teratogenic
Must register with iPLEDGE program
www.ipledgeprogram.com

Acne VulgarisTreatment
Isotretinoin (Accutane)
Side effects
Highly teratogenic Pancreatitis Hypertriglyceridemia Hepatitis Blood dyscrasias Hyperostosis

Acne VulgarisTreatment
Isotretinoin (Accutane)
Side effects
Premature epiphyseal closure Night blindness Erythema multiforme Stevens-Johnson syndrome Suicidal ideation Psychosis

Acne VulgarisTreatment
Monitoring on isotretinoin
Pretreatment and monthly
Lipids Liver function tests Pregnancy tests

Acne VulgarisTreatment
Light-based treatments
Ultraviolet A/Ultraviolet B (UVA/UVB), blue or blue/red light, pulse dye laser, KTP laser, infrared laser Photodynamic therapy for 3060 minutes with 5-aminolevulinic acid 3 sessions is effective for inflammatory lesions Greatest utility when used as adjunct to medications or in patient who cant tolerate medications

Acne VulgarisSurgical Treatment


Comedo extraction after incising the layer of epithelium over closed comedo Incision and drainage for abscesses Inject large cystic lesions
0.050.3 mL triamcinolone (Kenalog 25 mg/mL) Use 30-g needle to inject and slightly distend cyst

Acne VulgarisSurgical Treatment


Acne scar treatment
Retinoids Steroid injections Cryosurgery Electrodessication Micro/dermabrasion Chemical peels Laser resurfacing

Acne VulgarisTreatment
Topical tea tree oil
Is effective, slow onset

Google Images

Acne VulgarisReferral
Consider referral/consultation to dermatologist
Refractory lesions despite appropriate therapy Consideration of isotretinoin therapy Management of acne scars

Case # 7

Images Dr. Richard P. Usatine

Case # 7
A 45 y/o woman c/o progressive facial rash. It has not improved with topical benzoyl peroxide.

Images Dr. Richard P. Usatine

7. The most likely diagnosis in this case is:


A. Acne vulgaris B. Rosacea C. Lupus erythematosus D. Seborrheic dermatitis

Rosacea

Images Dr. Richard P. Usatine

Rosacea

Images Dr. Richard P. Usatine

Rosacea
Prevalence = 15 million in US Most common in Celtic ethnicity Most common after age 30 More common in women Unknown etiology Chronic, intermittent Involves forehead, cheeks, nose, ocular area

Rosacea
Primary features
Erythema
Transient symmetric flushing
Accentuated by hot liquids and alcohol

Non-transient

Papules and pustules Telangiectasia

Rosacea

Google Images

Rosacea
Secondary features
Burning or stinging Plaque Dry appearance Edema Nasal hypertrophy/scarring (rhinophyma) Ocular manifestations

Rhinophyma

Google Images

RosaceaTriggers for Flushing


Extremes of temperature Sunlight Spicy foods Alcohol Exercise Acute psychological stressors Medications Menopausal hot flashes

Rosacea

Google Images

RosaceaTreatment
Topical antibiotics
0.75% metronidazole (Metrogel) BID 1% metronidazole (Noritate) daily Azelaic acid 15% (Azelex) BID Benzoyl peroxide 5% - BID
+/- erythromycin or clindamycin

Clindamycin cream less effective

Other topical agents


Pimecrolimus (Elidel) 1% cream Adapalene 0.1% gel, tretinoin 0.025% cream Permethrin

RosaceaTreatment
Oral antibiotics
Doxycycline (Vibramycin) 100-200 mg/day Tetracycline 1 gram/day Erythromycin 1 gram/day Minocycline (Minocin) 100-200 mg/day Metronidazole (Flagyl) 250 mg BID Azithromycin (Zithromax) 500 mg, then 250 mg x 4 days

Retinoids
Isotretinoin (Accutane) 0.5 mg/kg/day x 20 weeks for severe resistant cases

RosaceaTreatment
Vascular laser
Useful for resistant telangiectasia & persistent erythema

Rhinophyma Rx
Mechanical dermabrasion CO2 laser peel Surgical excision Electrocautery

Seborrheic Dermatitis

Images Dr. Richard P. Usatine

Seborrheic Dermatitis
Chronic, relapsing, and usually mild form of dermatitis
Occurs in infants and in adults

Cause is unknown
Not a disease of the sebaceous glands Rate of sebum excretion is not increased in patients with seborrheic dermatitis

Sebaceous glands are necessary for development of dermatitis Occurs in body sites with higher amounts of sebaceous glands
Face, scalp, upper trunk, ano-genital area

Indirect evidence for a role of Malassezia


Most of the effective therapeutic agents have antifungal activity

Seborrheic Dermatitis

Google Images

Seborrheic Dermatitis
Biphasic incidence
Infants between the ages of 2 weeks and 12 months During adolescence and adulthood

Prevalence
3% Men > women

Increased among individuals with HIV


Prevalence = 35% with early HIV infection 85% among patients with AIDS

Increased in Parkinsons disease


May improve with L-dopa therapy

Seborrheic Dermatitis
Gets worse with
Stress Cold and dry winter months

Gets better
Summer months
Probably from sun exposure

Seborrheic DermatitisTreatment
Face initial Rx
Low-potency topical corticosteroid cream Topical antifungal BID until better
Ketoconazole 2% Ciclopirox 0.77% cream/gel (Loprox)

Mustaches and beards


Ketoconazole 2% shampoo - daily

Face long-term control


Ketoconazole 2% cream or shampoo Ciclopirox 1% cream Once per week

Seborrheic DermatitisTreatment
Scalp
Antifungal shampoos
Selenium sulfide 2.5% Ketoconazole 2% Ciclopirox 1%

Alternative medicated shampoos


Coal; tar Salicylic acid Sulfur 2-3x/wk for several weeks

Perioral Dermatitis

Google Images

Perioral Dermatitis
Multiple small inflammatory papules
Around mouth, nose, eyes

Acneiform or rosacea-like eruption


With or without associated features of a mild eczematous dermatitis

Epidemiology
Women between 16 and 45
Majority of cases

Perioral DermatitisDx
Sparing of the skin immediately adjacent to the vermilion border of the lip Coexisting features of eczematous dermatitis Burning or stinging sensations Recent use of topical, nasal, or inhaled corticosteroids History of disease flares after topical corticosteroid withdrawal Absence of comedones

Perioral DermatitisRx
Severe involvement
Tetracycline
250-500 mg BID

Doxycycline
50-100 mg BID

Minocycline
50-100 mg BID

Mild-moderate involvement
Topical pimecrolimus 1% BID Topical erythromycin 2% gel BID Topical metronidazole 0.75-1% gel or cream daily

Lupus Erythematosus (Systemic)

Google Images

Lupus Erythematosus
Most patients have mucocutaneous lesions at some time during the course of the illness Butterfly rash
Most common lesion Erythema over the cheeks and nose Spares the nasolabial folds Appears after sun exposure

Oral and/or nasal ulcers


Usually painless 12 to 45% of patients

Lupus ErythematosusRx
Hydrocortisone
Superficial/mild involvement

Fluorinated topical corticosteroids


Thicker involvement Used very cautiously No more than 2 weeks

Ointments are more effective than creams

Discoid Lupus Erythematosus

Google Images

Discoid Lupus Erythematosus


Variants of cutaneous lupus May occur
Independently As manifestation of systemic lupus erythematosus

Most commonly occurs on the head Characterized by well-defined inflammatory plaques


Evolve into atrophic, disfiguring scars

Discoid Lupus Erythematosus Treatment


Photoprotection Avoidance of aggravating drugs Smoking cessation Topical corticosteroid (group 1 or 2) BID
Clobetasol (Temovate)

Topical calcineurin inhibitors


Pimecrolimus 1% cream (Elidel) Tacrolimus 0.03% cream or 0.1% ointment (Protopic)

Discoid Lupus Erythematosus Treatment


Intralesional corticosteroids Anti-malarials last resort
Hydroxychloroquine Quinacrine Chloroquine

Case # 8

Images Dr. Richard P. Usatine

Case # 8
A 21 y/o male c/o pale spots on his back and upper chest

Images Dr. Richard P. Usatine

8. Despite treatment with a topical agent, they are essentially unchanged. These areas should now be treated with which of the following?
A. Benzathine penicillin G B. Pimecrolimus (Elidel) C. Oral fluconazole (Diflucan) D. Prednisone

Tinea Versicolor

Images Dr. Richard P. Usatine

Acquired Hypopigmented Lesions


Pityriasis alba Vitiligo Tinea versicolor Postinflammatory hypopigmentation Leprosy Halo nevus Chemical induced Phytophotodermatitis

Pityriasis Alba

This is not an adult


Images Dr. Richard P. Usatine

Pityriasis Alba

Google Images

Pityriasis Alba
Occurs predominantly in children
3-16 years

Nonspecific dermatitis with residual postinflammatory hypopigmentation Hypopigmented patches on face, neck, upper trunk, proximal extremities
Well-defined borders Do not tan More obvious after sun exposure

Usually asymptomatic
Occasional itching

Pityriasis AlbaRx
Sun protection Mild, topical glucocorticoids Calcineurin inhibitors Emollients Repigmentation typically takes months to years

Vitiligo

Images Dr. Richard P. Usatine

Tinea Versicolor

Images Dr. Richard P. Usatine

The Tinea Family


Tinea
Means fungal infection

By site
Tinea capitis Tinea corporis Tinea pedis

Other
Tinea versicolor Tinea gladiatorum Tinea incognito

Tinea Corporis

Dr. Richard P. Usatine

Tinea Capitis

Dr. Richard P. Usatine

Tinea Incognito

Dr. Richard P. Usatine

Erythrasma (J)

Jefferson Clinical Images Database. http://jeffline.jefferson.edu/JCI/.

Tinea Versicolor
Caused by Malassezia (pityrosporum) species Lesions result from conversion from budding to mycelial form Occurs with heat, humidity, pregnancy, steroids, immunosuppression More common at ages associated with high sebaceous activity

Tinea Versicolor
Lesions begin as circular macules that enlarge Occur as tan, dark brown, or hypopigmented lesions Have a powdery scale that is noted with scraping Usually occur on upper trunk, neck, abdomen Pale yellow fluorescence with Woods lamp

Tinea VersicolorTreatment
Topical for limited disease
Ketoconazole 2% shampoo (Nizoral) x 3 days Selenium sulfide 2.5% x 7 days or q week x 4 Terbinafine 1% solution (Lamisil) BID x 1 week Traditional topical antifungals BID x 2-4 weeks

Tinea VersicolorTreatment
Oral for extensive disease or nonresponders
Itraconazole (Sporanox) 200 mg daily x 5-7 days Fluconazole (Diflucan) 300 mg single dose, repeat in 1 week Avoid oral ketoconazole
Concern about hepatotoxicity

Tinea VersicolorTreatment
Prophylaxis
Ketoconazole 2% shampoo (Nizoral) q week Itraconazole (Sporanox) 200 mg PO BID q month x 6 months

Melasma

Google Images

Melasma
Acquired hyperpigmentation of the skin
Sun-exposed areas of the face

Women of reproductive age


With darker complexions Live in areas of intense ultraviolet radiation

Occurs during pregnancy


15-50%

Melasma
Centrofacial
Cheeks, forehead, upper lip, nose, and chin

Malar
Cheeks and nose

Mandibular
Ramus of the mandible

MelasmaRx
Sunscreen Triple combination cream
Hydroquinone 4% (Lustra) Tretinoin 0.05% Fluocinolone acetonide 0.01%

Azelaic acid (Azelex) Refractory cases


Chemical peels, laser, light-based therapies

Answers
1. 2. 3. 4. 5. 6. 7. 8. B B A C B C B C

Managing Common Cutaneous Problems (Part 1)


Gary Levine, MD, FAAFP Associate Professor Brody School of Medicine at East Carolina University Greenville, NC

Well Child Care and Adolescent Health Issues

Disclosure Statement

It is the policy of the AAFP that all individuals in a position to control content disclose any relationships with commercial interests upon nomination/invitation of participation. Disclosure documents are reviewed for potential conflicts of interest. If conflicts are identified, they are resolved prior to confirmation of participation. Only participants who have no conflict of interest or who agree to an identified resolution process prior to their participation were involved in this CME activity.

Objectives
1. Recognize the key predictors of normal pediatric development 2. Discuss issues and screening in the care of the routine well child 3. Recognize the demographic of the adolescent age group and the unique medical and psychosocial health problems seen in this group 4. Identify the diagnostic and therapeutic approaches that are appropriate for disorders in adolescents

The Well-Child Visit


History from parents Parental guidance: sleeping, feeding, behavior, safety, health, parenting Growth and development (milestones) Physical exam Other health screenings (vision, fluoride, lead) Immunizations

1. Mother brings her 9-month-old daughter in for routine care. Which of the following should be addressed at this visit?
A. B. C. D. Starting table food Sleeping on back Hearing test Toilet training

Parental Guidance
2-4 wk:
Issues: sleep (position), feeding, crying, response, growth Safety: car seats, exposure to smoking, shaking baby

4 mo
Issues: introducing food, sleep, talking to baby Safety: falling, car seats

6 mo
Issues: food Safety: child-proofing house, poisons, walkers, car seats

2 mo

Issues: sleep (position), feeding, growth 9 mo Safety: burns, sun exposure, Issues: table food, using cup, smoking, car seats, shaking teeth baby Safety: drowning, burns, car seats

Parental Guidance
12 mo
Issues: weaning, brushing teeth, lead screening, 24 mo playing with baby Issues: talking with child, Safety: child-proofing house, toilet training, TV, games, choking, car seats language Safety: guns, lifejackets, 15 mo traffic, poisons, matches Issues: nutrition, feeding self, development Safety: falls, car seat Safety: guns, lifejackets, traffic, poisons, matches

36 mo

18 mo
Issues: nutrition, TV, toilettraining, language development

Issues: nutrition, handwashing, talking, TV, peers, dentist Safety: home safety, car seat, helmets

Sleep
6 mo: majority sleep through the night Nightmares during the second half of the night Sleepwalking, night terrors during the first half Benign nocturnal limb pain within hours of falling asleep
Ages 4-6 in knees, shins, calves (sometimes thighs)

Sleeplessness in a child can be a major indication of stressors Lack of data for insomnia meds in children

Accident Prevention
MVA: car seats
Backward facing until 2 years and 20 lb

Burns: curling irons, stoves, hot liquids Bikes, roller sports: helmets Poisoning: no ipecac Drowning: #1 cause of accidental death < 5 yr Guns

2. 6-month-old infant is brought to your clinic. Which of the following would be a cause for alarm?
A. B. C. D. Does not sit without support Cannot form any words Keeps hands continually clenched Does not cruise

*Development
1 mo:
Looks at face Responds to voice Moves extremities equally Lifts head

4 mo:
Holds head at 90 Laughs Follows past midline No persistent fist clenching*
No head lag Bears weight on legs Rolls over Turns toward voice Transfers hand to hand

2 mo:
Vocalizes Smiles Follows to midline Responds to sounds

6 mo

*Development
9 mo:
Sits without support Stands holding on Cruises Imitates speech Thumb finger grasp Dada, mama Peek a boo

1 yr:
Stands alone Walks with help Specific dada mama Responds to no Pincer grasp Waves bye-bye Bangs 2 blocks together

*Development
18 mo

2 yr
Kicks ball Takes off clothes 2 words together Knows 50 words Understands 2-part command Uses own name Stacks 4 blocks 6 body parts

Walks backward 2 body parts Drinks from cup Imitates household chores Uses 20 words Scribbles Stacks 2 blocks Understands simple commands

*Development
3 yr
Washes hands Draws vertical line Understands tired, hungry Throws ball Pedal tricycle Asks Why?

4 yr
Dresses self Plays games (tag) Says what to do when tired, hungry, cold First and last name Up and down stairs alternating feet Balances each foot 2 sec Draws a circle

Vision
The USPSTF recommends vision screening for all children at least once between the ages of 3 and 5 years, to detect the presence of amblyopia or its risk factors. Grade: B Recommendation Refer for:
Visual acuity < 20/40 age 3, < 20/30 age 5 Asymmetry at any age Strabismus > 3-6 mo Any abnormal red reflex

Amblyopia due to strabismus is the leading cause of monocular vision loss in children

Dental
Tooth eruption:
12 mo: 4 teeth (then one tooth a month until 20 teeth at 28 months)

Teething: Does not cause fever, diarrhea, or other infections Treat with acetaminophen, teething ring USPSTF recommends:
Primary care clinicians prescribe oral fluoride at currently recommended doses for preschool children older than 6 mo. old whose primary water source is deficient in fluoride (B rec.)
http://www.uspreventiveservicestaskforce.org/3rduspstf/dentalchil d/dentchrs.htm

Dietary Fluoride Supplementation Schedule


American Academy of Pediatric Dentistry Age Birth - 6 months 6 months 3 years 3-6 years 6 years up to at least 16 years < 0.3 ppm F 0 0.25 mg 0.50 mg 1.00 mg 0.3-0.6 ppm F 0 0 0.25 mg 0.50 mg > 0.6ppm F 0 0 0 0

Lead Screening
Lead levels
5-9 g/dL: repeat and follow >10 g/dL: refer to health dept. to check house > 45 g/dL: drug chelation (> 70 g/dL: 2 drugs)

USPSTF recommends AGAINST screening asymptomatic children at average risk (D recommendation) USPSTF concludes that evidence is insufficient to recommend for or against screening in asymptomatic children who are at increased risk (I recommendation)

Adolescent Health Care

Health Care in Adolescents


Screening in adolescents
Yearly exams may not be cost-effective

BIHEADS screening
Body Image Home/Health Education/Employment Activities Drugs/Depression Safety/Sexuality

Bullyingwork with school officials

Vaccinations
**Age 11/12HPV, Tdap, Meningococcal (MCV4) Influenzayearly Age 16-18Meningococcal booster

USPSTF Recommendations
Pap smears:

A Recommendations (Strongly Recommended)

Beginning at age 21

Sexually active females younger than 25 years for chlamydia infection Folic acid supplementation for females HIV screening for adolescents > 15 or younger if at increased risk for HIV infection Syphilis screening for those at increased risk

B Recommendations (Recommended)
Gonorrhea infection if at increased risk Counseling on sexually transmitted infections (STIs) for all sexually active adolescents at increased risk Screening of adolescents (12-18 years of age) for major depressive disorder (MDD) Obesity screening for children 6 years and over

B Recommendations (Recommended)
Counseling (ages 10-24 with fair skin) about minimizing UV exposure Intimate partner violence screening for females of childbearing age Interventions to prevent initiation of tobacco use in school aged and adolescents

I Recommendations (Insufficient Evidence)


Lipid disorders (ages 1-20) Speech and language delay Physical abuse or neglect of children Proper use of motor vehicle occupant restraints Counseling re: driving under the influence or with alcohol-impaired drivers STI counseling for non-sexually active adolescents Alcohol misuse or illicit drug use by adolescents

I Recommendations (Insufficient Evidence)


Children (7-11 years of age) for depression Physical activity counseling Nutrition Primary HTN in asymptomatic children and adolescents Iron deficiency anemia in asymptomatic children Gonorrhea screening for males at increased risk

D Recommendations (Not Recommended)


Testicular cancer screening in asymptomatic adolescent males Asymptomatic adolescents for idiopathic scoliosis Pap smears in females less than 21 Screening for herpes or syphilis in asymptomatic adolescents Screening for gonorrhea in males/females at low risk Ovarian cancer screening

3. Which of the following is true regarding suicide?


A. Women have a higher rate of completion than men B. Men attempt suicide more frequently than women C. Among men, African Americans have the highest suicide rate D. 2/3 of suicide deaths occur on the first attempt

Suicide in Adolescents
The USPSTF recommends screening of adolescents (12-18 years of age) for major depressive disorder (MDD) when systems are in place to ensure accurate diagnosis, psychotherapy (cognitive-behavioral or interpersonal), and follow-upB recommendation 75% of completed suicides are by white males Adolescents and the elderly are high risk

Suicide Risk Factors


Psychiatric disorders
Mood disorders Substance abuse Eating disorders Personality disorders Psychoses

Social and environment


Family crisis Physical abuse Suicide in community Trouble in school or with the law

Family history and genetics

Often unexpected

4. A 17-year-old female presents to the emergency department. Which condition can you treat without the consent of her parents?
A. A broken arm B. An exposure to a sexually transmitted disease C. A 3-cm laceration on her leg D. A headache

Consent
Age of consent is 18-21 y.o. and state-dependent Either parent can consent No parental consent usually required for:
Contraception, STIs, rape, incest Drug and alcohol treatment Emancipation, married, parent, living independently Emergency where delay in treatment could cause harm

Always encourage discussion with parents

5. You are seeing a young girl for her pre-high school physical. The form asks for Tanner stage. She has small but developed breasts and a small amount of dark, straight pubic hair. She is Tanner stage:

A. B. C. D. E.

I II III IV V

Tanner Staging
Allows physicians to give anticipatory guidance Females
Development begins age 8-13 with breast buds Growth spurt (~ 4 in) during stage 2 Menarche ~ age 12 Acne common during stages 3 and 4

Males
Development begins ages 9-13 with scrotal enlargement Growth spurt (~ 5-7 in) between stages 3 and 4 Ejaculations begin during stage 3 Strength peaks between stages 4 and 5

6. A 16-year-old is brought to you by her mother with concerns that she may have a body image problem. She denies any problems. She is normal weight but you note that the enamel on her teeth is thinned. What is the probable diagnosis?

A. B. C. D.

Addisons disease Bulimia nervosa Anorexia nervosa Hyperthyroidism

Bulimia Nervosa
Prevalence 1-19% Clinical features:
Chronic vomiting, dehydration, electrolyte abnormalities, bloating, diarrhea, swelling of hands and feet, loss of tooth enamel Chronic laxative use leads to hypokalemia (weakness and tingling), acute acidosis, chronic alkalosis, dehydration, constipation

Diagnostic criteria:
Recurrent episodes of binge eating Recurrent inappropriate compensatory behavior in order to prevent weight gain. Binge eating and compensatory behaviors occur at least twice weekly for 3 months Self-evaluation unduly influenced by body shape and weight

Anorexia Nervosa
Prevalence 0.3-3% 90% white; 75% started in adolescence Most are middle to upper class High-risk sports
Ballet, wrestling, swimming, gymnastics, skating

Anorexia: Diagnosis
Clinical features:
Amenorrhea Fluid/electrolyte imbalance Metabolic alkalosis Hypothermia Bradycardia Hypotension Lanugo-type hair (face, shoulders, back) Lethargy

Diagnostic criteria (DSM V):


Restriction of food intake leading to weight loss or failure to gain weight Intense fear of gaining weight Distorted body image Restricting vs. binge/purging type

Anorexia: Labs
Elevated BUN Leukopenia Normal or low LH and FSH Normal or low T4 and T3 Elevated liver transaminases EKG: bradycardia, low-voltage changes Most common cause of sudden death is prolonged QT interval resulting in arrhythmia Criteria for admission: heart rate < 40 beats/min, blood pressure < 80/50 mm Hg, temperature < 36 C

Obesity
***Defined by BMI percentile for age and gender
Overweight 85-94th% Obese > 95th% The USPSTF recommends that clinicians screen children aged 6 years and older for obesity and offer them or refer them to comprehensive, intensive behavioral interventions to promote improvement in weight status.

(B Recommendation)

7. Which of the following milestones would an 18 mo. old not be expected to reach yet?

A. B. C. D.

Walks Speaks some words Understands simple commands Draws circle

8. All of the following are routine immunizations offered at 11 years old, except:
A. B. C. D. HPV Tdap Meningococcal Pneumococcal

9. The CDC definition of obesity for ages 2-19 is:

A. B. C. D.

BMI 85-94% BMI > 95% BMI 25-29 BMI > 30

Take Home Pearls


Know developmental milestones Immunizations at age 11 BMI %s for overweight/obesity

Answers
1. 2. 3. 4. 5. 6. A C D B C B 7. D 8. D 9. B

Managing Common Cutaneous Problems (Part 2)


Gary Levine, MD, FAAFP Associate Professor Brody School of Medicine at East Carolina University Greenville, NC

Faculty Disclaimer
Gary Levine, MD, returned disclosures indicating that indicating that he has no affiliation or financial interest in any organization(s). Gary Levine, MD, returned disclosures indicating that he is not a dermatologist

The AAFP has selected all faculty appearing in this program. It is the policy of the AAFP that all CME planning committees, faculty, authors, editors, and staff disclose relationships with commercial entities upon nomination or invitation of participation. Disclosure documents are reviewed for potential conflicts of interest and, if identified, they are resolved prior to confirmation of participation. Only those participants who had no conflict of interest or who agreed to an identified resolution process prior to their participation were involved in this CME activity.

Learning Objectives
1.

2.

Summarize the AAFP Core Educational Guidelines Conditions of the Skin. Recognize many of the dermatologic entities included in the curriculum guide, including but not limited to: molluscum contagiosum, genital warts, common warts, genital herpes, basal cell carcinoma, sebaceous hyperplasia, keratoacanthoma, seborrheic keratosis, malignant melanoma, actinic keratosis, squamous cell carcinoma, dermatosis papulosa nigra, acrochordon, Bowens disease, alopecia areata, tinea capitis, pediculosis capitis, erythema migrans, impetigo, erysipelas, herpes labialis, perleche, angioedema, aphthous ulcer, epidermal cyst, keloid, and xanthelasma.

Case # 9

Picture courtesy of the CDC

Case # 9
This 26 y/o male dermatology resident presented to your office concerned about a new patch of warts on his penis.

Picture courtesy of the CDC

Case # 9
Funny that you should ask, my girlfriend has some of these umbilicated papules on her bottom, as well.

Picture courtesy of the CDC

9. The most likely cause of his lesions is:

A. Treponema pallidum B. Herpes simplex virus C. Human papilloma virus D. Molluscum contagiosum virus

Molluscum Contagiosum

Picture courtesy of the CDC

Courtesy of G Levine, MD

Genital Warts

Courtesy of the CDC

Wart
Due to infection with HPV (Human Papillomavirus)
150 types HPV type 1 plantar warts HPV types 6 & 11 genital warts

Infection with HPV occurs by skin-to-skin contact

Genital WartTreatment
Patient administered
Podofilox - 0.5%
BID x 3 days, off 4 days, repeat cycle x 4

Imiquimod -5% cream (Aldara)


HS, 3x/week, x 16 wks

Sinecatechins 15% ointment (Veregen)


TID for maximum of 16 wks

Genital WartTreatment
Provider-administered
Cryotherapy BCA/TCA Podophyllin resin 10-25% Surgical or laser removal Interferon alfa-2B (Intron-A)

Wart

Google Images

Common Warts

Images Dr. Richard P. Usatine

Plantar Wart

Jefferson Clinical Images Database. http://jeffline.jefferson.edu/JCI/.

Wart
Lesions may occur singly, in groups, or as coalescing lesions forming plaques Warts obscure normal skin markings

Callus

Jefferson Clinical Images Database. http://jeffline.jefferson.edu/JCI/.

WartRx
Spontaneous regression occurs in 66% of warts within two years Skin may look normal after treatment
Virus is often still present in the remaining tissue Recurrence rate around 33%

Least painful methods should be used initially


Especially in young children

More aggressive or destructive therapies


Reserved for areas where scarring is not a consideration Numerous, large, recalcitrant, or highly symptomatic lesions

Duct Tape on Wart

Google Images

WartRx
Common, plantar, and palmar warts
Salicylic acid Duct tape with salicylic acid
Controversial

Liquid nitrogen Bichloracetic acid Trichloroacetic acid Cantharidin Imiquimod Topical and intralesional immunotherapy Surgical excision Laser

Genital Herpes

Courtesy of Centers for Disease Control & Prevention

Genital HerpesTreatment
Acyclovir (Zovirax)
Primary = 400 mg TID x 7 days, 200 mg 5x/day x 7 days Recurrent = 400 mg TID x 5 days, 800 mg TID x 2 days Suppression = 400 mg BID

Famciclovir (Famvir)
Primary = 250 mg TID x 7 days Recurrent = 125 mg BID x 5 days, 1 g BID x 1 day Suppression = 250 mg BID

Valacyclovir (Valtrex)
Primary = 1 g BID x 7 days Recurrent = 1 g daily x 5 days, 500 mg BID x 3 days Suppression = 500 mg or 1 g daily

Molluscum Contagiosum
Caused by double-stranded DNA poxvirus Spread by skin-to-skin contact and autoinoculation Umbilicated, firm, flesh-colored, domeshaped papules Children
Lesions anywhere except palms & soles

Adults
Lesions mostly in genital area

Molluscum Contagiosum

Google Images

Molluscum Contagiosum

Images Dr. Richard P. Usatine

Molluscum Contagiosum Treatment


Curettage
May cause scarring

Cryosurgery Imiquimod 5% cream (Aldara)


TID x 5 days/wk x 1 month

Cantharidin Cimetidine (Tagamet) 40 mg/kg/day x 2 months (children) Laser TCA peel q 2 weeks KOH 5%

Case # 10

Images Dr. Richard P. Usatine

Case # 10
A 68 y/o male was sent in by his daughter who was concerned about a small growth on her father s scalp.

Images Dr. Richard P. Usatine

10. Which of the following is the most likely diagnosis?

A. Keratoacanthoma B. Dermatofibroma C. Sebaceous hyperplasia D. Basal cell carcinoma

Basal Cell Carcinoma

Images Dr. Richard P. Usatine

Nodular Lesions
Basal cell carcinoma Squamous cell carcinoma Keratoacanthoma Sebaceous hyperplasia Melanoma Neurofibroma Hemangioma Prurigo nodularis

Sebaceous Hyperplasia

Images Dr. Richard P. Usatine

Sebaceous Hyperplasia

Google Images

Sebaceous Hyperplasia
Hypertrophy of sebaceous glands Occur mostly on the central face of adults Isolated, yellow papule
No history of recent change

Must be differentiated from basal cell carcinoma Treatment


Electrosurgery Shave excision TCA CO2 laser Dermabrasion

Keratoacanthoma

Dr. Richard P. Usatine

Basal Cell Carcinoma


Most common skin cancer Male > female Mostly in age > 40 85% occur in head/neck Clinical course is unpredictable
Can remain small for years or develop in growth spurts

Diagnosis by biopsy

Basal Cell CarcinomaSubtypes


Nodular
Most common Less aggressive

Superficial
Plaque-like

Sclerosing
Rare

Pigmented

Basal Cell Carcinoma (Nodular)

Images Dr. Richard P. Usatine

Basal Cell Carcinoma

Google Images

Basal Cell Carcinoma

Google Images

Basal Cell Carcinoma

Google Images

Basal Cell Carcinoma (Superficial)

Images Dr. Richard P. Usatine

Basal Cell Carcinoma (Sclerosing)

Images Dr. Richard P. Usatine

Basal Cell CarcinomaTreatment


Excisional biopsy
Often adequate for small lesions

Electrodesiccation/curettage, cryotherapy
Nodular & superficial
< 6 mm in size, < 3 mm in depth

Basal Cell CarcinomaTreatment


Mohs micrographic surgery
Sclerosing Other BCCs with poorly defined margins High-recurrence areas
Nose, eyelid

Large (> 2 cm) primary or recurrent BCCs Lesions where conservation of tissue is important

Basal Cell CarcinomaTreatment


Radiation
Non-surgical candidates

Imiquimod 5% cream (Aldara) 5-Fluorouracil 5% (Efudex)


BID x 12 weeks

Case # 11

Images Dr. Richard P. Usatine

Case # 11
A 55 y/o male is found to have these skin lesions at the time of a routine exam.

Images Dr. Richard P. Usatine

11. Which of the following would be the most reasonable course of action?
A. Intralesional corticosteroid injection B. Observation of lesions C. Excisional biopsy with wide 4 cm margins D. Referral for Mohs micrographic surgery

Seborrheic Keratoses

Images Dr. Richard P. Usatine

Pigmented Lesions
Intradermal nevus Melanoma Seborrheic keratosis Kaposis sarcoma Cherry angioma Pigmented basal cell carcinoma

Cherry Angiomas

Dr. Richard P. Usatine

Cherry Angioma

Google Images

Hemangioma

Jefferson Clinical Images Database. http://jeffline.jefferson.edu/JCI/.

Pyogenic Granuloma

Jefferson Clinical Images Database. http://jeffline.jefferson.edu/JCI/.

Atypical (Dysplastic) Nevus

Google Images

Atypical (Dysplastic) Nevus

Images Dr. Richard P. Usatine

Malignant Melanoma

Google Images

Malignant Melanoma

Google Images

Malignant Melanoma
Mostly found in non-Hispanic Caucasians Median age = 53 Men 1.5x > women Metastasize widely

Malignant MelanomaRisk Factors


Large number of atypical nevi Other skin cancers Congenital giant nevus Family history of melanoma Immunosuppression UV radiation exposure

Malignant MelanomaSubtypes
Superficial spreading
Most common Upper back & legs in 40-50 y/o

Nodular
Mostly men in 50-60 y/o

Lentigo maligna
Facial location in 60-70 y/o

Acral lentiginous
Digits & mucous membranes

ABCDEs of Melanoma
A Asymmetry B Border irregularity C Color variegation D Diameter greater than 6 mm E Evolving (changing)

Nodular Malignant Melanoma

Images Dr. Richard P. Usatine

Malignant Melanoma

Dr. Richard P. Usatine

Lentigo Maligna Melanoma

Images Dr. Richard P. Usatine

Lentigo Maligna

Google Images

Superficial Spreading Melanoma

Images Dr. Richard P. Usatine

Acral Lentiginous Melanoma

Images Dr. Richard P. Usatine

Malignant Melanoma
Thickness determines prognosis
Breslow microstage (mm)
Measured depth of tumor invasion Most accurate

Clark level
Histologic layer of dermis involvement

Seborrheic Keratosis

Images Dr. Richard P. Usatine

Seborrheic Keratosis vs. MelanomaDiagnosis


Melanoma suspected
Full-thickness biopsy
Excision preferred Very wide excision not necessary
Generally 2-3 mm is sufficient

Punch if small enough

Seborrheic KeratosisTreatment
Certain seborrheic keratosis
Destructive treatment
Curettage +/- electrodesiccation Cryosurgery

Observation

Dermatosis Papulosa Nigra

Google Images

Dermatosis Papulosa Nigra


Variant of seborrheic keratosis Commonly found on the faces of African American patients. Small, darkly pigmented papules
May be pedunculated

Dermatosis Papulosa NigraRx


No treatment generally is indicated unless lesions are cosmetically undesirable Treatment modalities
Abrasive curettage Liquid nitrogen cryotherapy Electrodesiccation/curettage Laser therapy

Aggressive therapeutic complications


Hyperpigmentation, hypopigmentation, scarring, keloids

Acrochordon (Skin Tags)

Google Images

Acrochordon (Skin Tags)


Outgrowth of normal skin Occur in approximately 25% of adults
Increases with age

At sites of friction
Axilla, neck, inframammary, inguinal regions

May be associated with insulin resistance

Acrochordon (Skin Tags)Rx


Excision
Forceps or fine-grade scissors

Cryosurgery with liquid nitrogen Electrodesiccation Lesions often bleed freely


Aluminum chloride (Drysol) AgNO3 Electrocautery Larger lesions may require suturing

Case #12

Courtesy of Charles Phillips, MD

Case #12
A 68 y/o male notes an increasing number of rough/scaly areas on both of his hands.

12. Which of the following would be most appropriate course of action?


A. Observation of lesions with annual photographs on your smart phone B. Surgical excision C. Application of topical 5-fluorouracil D. Application of topical high-potency corticosteroid gel

Actinic Keratosis

Courtesy of Charles Phillips, MD

Actinic Keratosis

Google Images

Actinic Keratosis

Google Images

Actinic Keratosis
Prevalence
Increases with age 20 to 29 y/o
< 10%

> 40 y/o
60%

80-90 y/o
75%

Actinic Keratosis
Risk factors
Men > women Fair-skinned, blue-eyed Sun exposure

Location
Epidermis on sun-exposed areas of the body Head, neck, forearms, and hands

Actinic Keratosis
Rough scaly patches
Normal skin tone to reddish brown Circumscribed 1 mm to 2.5 cm

Often multiple Generally asymptomatic


Can see pruritus or a burning sensation

Actinic Keratosis
Most common premalignant lesions seen by dermatologists Potential to progress to squamous cell carcinomas
Most actinic keratoses do not progress to cancer 26% regress spontaneously 60% of cutaneous squamous cell carcinomas arise from AKs

Actinic KeratosisRx
Treatment
Cosmetic reasons Relief of associated symptoms Prevent squamous cell carcinomas

Actinic KeratosisRx
Cryotherapy
20-sec freeze is 80% effective

Curettage
Best for hyperkeratotic lesions

Photodynamic therapy
Aminolevulinic acid Methyl aminolevulinate

Actinic KeratosisRx
Topical
Fluorouracil 5% cream BID x 2-4 wks 0.5% cream may have fewer side effects Imiquimod 5% cream daily 2-3x/wk x 16 wks 50% response, min progression to SCC Diclofenac 3% gel daily x 3 months

Bowen Disease

Google Images

Bowen Disease
Bowen disease (squamous cell carcinoma in situ)
Slowly growing, scaly, red plaque Typically appears on sun-exposed skin

Treatment
Surgical excision Electrodesiccation and curettage Cryotherapy Topical fluorouracil (5-FU)

Keratoacanthoma

Google Images

Keratoacanthoma
Mostly occurs in
Middle-aged and elderly individuals with fair skin.

Firm, rapidly growing, erythematous papule


Central keratotic plug

Lesions are usually 1 to 2 cm in diameter


Most commonly found on sun-exposed skin

Keratoacanthoma
Controversy
Whether KA represents a distinct disease entity
Or a variant of cutaneous squamous cell carcinoma

Complete removal of the lesion via surgical excision is the preferred biopsy procedure

KeratoacanthomaRx
Potential for spontaneous resolution Surgical excision is first-line treatment
Mohs surgery
Lesions in areas where tissue-sparing is desired
Central face

Electrodesiccation and curettage (ED&C) Intralesional pharmacologic therapy


5-FU Methotrexate

Radiation therapy Topical therapy daily x 8 wks


5-FU Imiquimod 5% (Aldara)

Squamous Cell Carcinoma

Google Images

Cutaneous Squamous Cell Carcinoma


Incidence
100 to 150 per 100,000 persons/year

2:1 male-to-female ratio Increased risk


Advancing age Proximity to the equator

Cutaneous Squamous Cell Carcinoma


Precursors
Actinic keratosis
Same disease process as SSC but at different stage of evolution

Risk factors
Chronically diseased or injured skin
Ulcers, sinus tracts

Exposure to iodizing or UV radiation


Use of any tanning device may increase the risk of squamous cell carcinoma by 2.5-fold

Immunosuppression Xeroderma pigmentosa

Cutaneous Squamous Cell Carcinoma


Spreads by local infiltration and expansion
May follow tissue planes, nerves, vessels

Distant metastasis in 5%
Tumors > 2 cm are three times more likely to metastasize than smaller tumors. Higher rate of metastasis = 40%
Location on lip, ear Chronically diseased or injured skin Immunosuppressed

Cutaneous Squamous Cell CarcinomaDx


Shave technique
If the lesion is raised

2- to 4-mm punch biopsy


Of the most abnormal-appearing skin

Cutaneous Squamous Cell CarcinomaRx


2010 Cochrane review
Little evidence of comparative effectiveness of different treatments for primary, nonmetastatic squamous cell carcinoma

Surgical excision
Typically recommended as first-line treatment
Mohs micrographic surgery

Electrodesiccation/curettage or cryotherapy
Smaller, low-risk lesions

Radiotherapy
Tumors in high-risk, surgically difficult areas Usually is not considered for patients younger than 55 years

Cutaneous Squamous Cell


Risk of subsequent skin cancer after treatment
35 percent at three years 50 percent at five years Increased in smokers

Case # 13

Images Dr. Richard P. Usatine

Case # 13
A 36 y/o female cosmetologist c/o recent onset of hair loss, and is afraid she will become bald like her father and older brothers.

13. Which of the following represents the most likely diagnosis?


A. Tinea capitis B. Traction alopecia C. Alopecia areata D. Androgenic alopecia

Alopecia Areata

Images Dr. Richard P. Usatine

Alopecia
Scaring alopecia
Lymphocytic
Discoid lupus

Neutrophilic
Dissecting cellulitis

Mixed
Acne keloidalis nuchae

Alopecia
Non-scaring alopecia
Focal
Alopecia areata Traction alopecia Post-operative alopecia Secondary syphilis

Patterned
Androgenic Female pattern Trichotillomania

Diffuse
Anagen effluvium (chemotherapy) Telogen effluvium

Traction Alopecia

Jefferson Clinical Images Database. http://jeffline.jefferson.edu/JCI/.

Androgenic Alopecia

Jefferson Clinical Images Database. http://jeffline.jefferson.edu/JCI/.

Alopecia Areata

Jefferson Clinical Images Database. http://jeffline.jefferson.edu/JCI/.

Alopecia Areata
Prevalence = 1 in 1,000 Usually < 30 y/o Autoimmune etiology
Association with thyroiditis and vitiligo

Alopecia Areata
Smooth, circular, discrete areas of complete hair loss
Develops over a few weeks

Affected skin may be slightly reddened


No other changes

Exclamation point hairs


At edges of patches Short broken hairs Proximal end is narrower than the distal end Can be extracted with minimal traction

Alopecia Areata
Can see simultaneous
Persistent areas of alopecia Patches exhibiting regrowth of hair

Rarely may see


Complete loss of scalp hair
Alopecia totalis

Complete loss of all scalp and body hair


Alopecia universalis

Alopecia AreataRx
Corticosteroids
Intralesional injections
Q 4-6 wks

Topical Systemic (short-term)

Topical minoxidil 5% Topical immunotherapy


Alopecia totalis

Photochemotherapy Wigs

Alopecia AreataPrognosis
50% with limited patchy hair loss
Recover within a year

May persist for several years Occasionally hair never regrows 10% progress to total hair loss

Tinea Capitis

Google Images

Tinea Capitis
Dermatophyte infection of the scalp, almost always occurs in small children
Infection due to Trichophyton species

Erythematous, scaling, well-demarcated patch on the scalp that spreads centrifugally Hairs within the patch break off a millimeter or two above the level of the scalp
Short, broken hairs take on a frosted appearance

Kerion

Google Images

Kerion
Boggy, elevated, tender nodules Surface of these lesions
Devoid of hair Covered with viscid exudate

Immune response to fungus Often see secondary staph infection Regional/posterior cervical adenopathy

Tinea CapitisDx
Spores identifiable by KOH examination of manually epilated hair Bright green fluorescence with Woods lamp Culture of hair with Sabourauds medium

Tinea CapitisRx
Griseofulvin
20 to 25 mg/kg/day (microsize formulation) for 6 to 12 weeks 10 to 15 mg/kg/day (ultramicrosize formulation) for 6 to 12 weeks Terbinafine 10 to 20 kg: 62.5 mg daily for two to four weeks 20 to 40 kg: 125 mg daily for two to four weeks Above 40 kg: 250 mg daily for two to four weeks

Tinea CapitisRx
Itraconazole
3 to 5 mg/kg daily for four to six weeks

Fluconazole
6 mg/kg/day for three to six weeks

Kerion
Consider addition of systemic glucocorticoid therapy
May improve discomfort, no data on improved healing

Consider treating household contacts with antifungal shampoo


Selenium sulfide 2.5% Ketoconazole 2% Apply for 5 min 3x/wk

Pediculosis Capitis

Google Images

Pediculosis Capitis
Affects a greater number of elementary school students in North America than all other communicable diseases combined Prevalence = 25% Direct contact with the head of an infested person
Primary mode of transmission

Head Louse (Pediculus humanus capitis)

Google Images

Pediculosis Capitis
Head louse
Gray-white, mobile insect 3 to 4 mm in length Life span of the female = 1 month Survive up to 55 hours without a host 7 to 10 eggs/day on base of a host hair (nit) Eggs hatch in eight days Nymphs mature in 8 days

Pediculosis Capitis
Itching of the scalp, neck, and ears
Allergic reaction to lice saliva

Persistent or recurrent pyoderma about the neck and ears Cervical and nuchal lymphadenopathy Febrile episodes
Associated with secondary staph infection

Pediculosis CapitisDx
Combing wet or dry hair with a finetoothed nit comb Woods lamp
Nits will fluoresce pale blue

Pediculosis CapitisRx
Pyrethroids
1st line Rx

Malathion Benzyl alcohol Spinosad (Natroba) Topical ivermectin Permethrin + oral TMP-SMX

Pediculosis CapitisRx
Children with pediculosis capitis do not need to be excluded from school Household members and close contacts should be examined for infestation Individuals who share bedding with the affected person should be treated prophylactically

Case #14

Jefferson Clinical Images Database. http://jeffline.jefferson.edu/ JCI/.

Case #14
A 43 y/o male park ranger/lifeguard noted this slowly expanding rash on his back. In addition to being tired and achy, it was ruining his appearance, and he was really ticked off about it.

Jefferson Clinical Images Database. http://jeffline.jefferson.edu/JCI/.

14. You would recommend treatment with which one of the following?
A. Benzathine penicillin G 2.4 MU IM B. Doxycycline C. Acyclovir D. Fluconazole

Jefferson Clinical Images Database. http://jeffline.jefferson.edu/JCI/.

Erythema MigransLyme Disease

Jefferson Clinical Images Database. http://jeffline .jefferson.edu/JCI/.

Lyme Disease
Due to infection with Borrelia burgdorferi Bite from Ixodes ricinus complex ticks

Lyme Disease
Early localized disease
Erythema migrans (EM)
Usually occurs within 1 month of tick bite

With or without constitutional symptoms

Lyme Disease
Early disseminated disease
Usually weeks to months after bite

Multiple EM lesions Neurologic manifestations


Meningitis, cranial neuropathy, and motor or sensory radiculoneuropathy

Cardiac
Atrioventricular heart block Myopericarditis

Lyme Disease
Late Lyme disease
May develop months to a few years after the initial infection Arthritis
May be preceded by migratory arthralgias Intermittent or persistent Few large joints, especially the knee

Neurologic manifestation
Subtle encephalopathy Polyneuropathy

Erythema MigransLyme Disease

Jefferson Clinical Images Database. http://jeffline.jefferson.edu/JCI/.

Erythema Migrans
Appears at the site of the tick bite Usually within 7 to 14 days after the bite
3 to 30 days

Slow expansion of lesion Occurs in 80% of cases


Only 25% of patients recall tick bite

Erythema Migrans
Occur in axilla, inguinal region, popliteal fossa, at belt line Usually not painful
May burn or itch Warm to touch

Expand slowly over days or weeks Classical central clearing


Only seen in 10% of cases

Can reach diameter > 20 cm

Lyme DiseaseDx
Serologic studies
Not recommended if EM rash in present
Treatment is indicated if rash present

Best done in disseminated or late disease

ELISA
Confirmatory Western blot if ELISA is positive

PCR
CSF
If seropositive and further confirmation needed

Lyme DiseaseRx
Early Rx for 10-21 days
Most patients have complete resolution of symptoms within 20 days

Doxycycline Amoxicillin Cefuroxime

Lyme DiseaseRx
Early disseminated
Asymptomatic or isolated facial nerve palsy
Same Rx as for early disease

Cardiac or meningitis
IV ceftriaxone, cefotaxime, Pen G

Lyme DiseaseRx
Late
Arthritis
Doxycycline or amoxicillin x 28 days Retreat x 28 days if not better IV ceftriaxone x 2-4 weeks if still not improved

Neurologic disease
IV ceftriaxone, cefotaxime, or PCN G x 28 days

Rocky Mountain Spotted Fever

Jefferson Clinical Images Database. http://jeffline.jefferson.edu/JCI/.

Meningococcemia

Google Images

Secondary Syphilis

Jefferson Clinical Images Database. http://jeffline.jefferson.edu/JCI/.

Classical Childhood Exanthems


Exanthem
Widespread macular rash Usually accompanied by systemic symptoms
Fever, malaise, and headache

Classical Childhood Exanthems


Rubeola (measles) = 1st disease Scarlet fever = 2nd disease Rubella (German measles) = 3rd disease Staph. aureus = 4th disease Erythema infectiosum = 5th disease Roseola = 6th disease

Rubeola

Google Images

Rubella

Google Images

Roseola

Jefferson Clinical Images Database. http://jeffline .jefferson.edu/JCI/.

Erythema Infectiosum (5th Disease)

Images Dr. Richard P. Usatine

Varicella

Images Dr. Richard P. Usatine

Case #15

Jefferson Clinical Images Database. http://jeffline.jefferson.edu/JCI/.

Case #15
A 9 y/o girl presents to the office with an uncomfortable rash on her face.

Jefferson Clinical Images Database. http://jeffline.jefferson.edu/JCI/.

15. This is best treated with drug therapy targeted at which one of the following?
A. Candida albicans B. Herpes simplex C. Propionibacterium acnes D. Staph. aureus

Jefferson Clinical Images Database. http://jeffline.jefferson.edu/JCI/.

Impetigo

Google Images

Impetigo
Contagious superficial bacterial infection Primary
Direct bacterial invasion of previously normal skin

Secondary
Infection at sites of minor skin trauma
Abrasions, minor trauma, and insect bites

Infection at sites of underlying conditions


Eczema

Impetigo
Most common in children ages 2-5 y/o
Older children and adults may also be affected

Usually occurs in warm, humid conditions Easily spread among individuals in close contact
Poverty Crowding Poor hygiene

Impetigo
Types
Non-bullous impetigo
Most common form Lesions begin as papules, surrounded by erythema, that progress to pustules Pustules enlarge and form thick crusts with a characteristic golden appearance Principal pathogen is S. aureus
Beta-hemolytic streptococci accounts for a minority of cases

Impetigo
Types
Bullous impetigo
Vesicles enlarge to form flaccid bullae with clear yellow fluid Become darker, turbid Ruptured bullae leave a brown crust Usually due to Staph. aureus

Ecthyma
Punched-out ulcers Covered with yellow crust With raised violaceous margins

Impetigo
Associated with development of
Poststreptococcal glomerulonephritis Rheumatic fever

ImpetigoTreatment
Topical therapy
Limited number of lesions without bullae Mupirocin (Bactroban)

Oral antibiotics
Dicloxacillin, cephalexin, clindamycin If suspect MRSA, or PCN allergic
Clindamycin TMP-SMX Linezolid (Zyvox)

Erysipelas

Google Images

Erysipelas
Similar to cellulitis Involves the upper dermis and superficial lymphatics Lesions are raised above the level of surrounding skin
Clear line of demarcation between involved and uninvolved tissue

Butterfly-like involvement of the face. Involvement of the ear


Milian's ear sign

Erysipelas
Acute onset of symptoms
Systemic manifestations including fever and chills

Infections involving the medial third of the face


Around the eyes and nose Can be complicated by septic cavernous sinus thrombosis

Vast majority of cases are caused by betahemolytic streptococci

ErysipelasTreatment
Systemic manifestations, fever/chills
Parenteral antibiotics
Ceftriaxone Cefazolin

Mild infection or after initial parenteral antibiotics


Penicillin-V Amoxicillin Cephalexin Clindamycin

Total duration of Rx = 5-10 days

Herpes (Simplex) Labialis

Google Images

Herpes Simplex
Gingivostomatitis and pharyngitis
Most frequent clinical manifestations of primary/first-episode herpes simplex virus (HSV-1) infection

Herpes labialis
Most frequent sign of reactivation disease

Herpes Labialis
Incidence = 1.6 per 1000 patients/ yr Prevalence = 2.5 per 1000 patients/yr Approximately 3% of infected patients suffer relapses

Herpes Labialis
Recurrences
Commonly involve oral cavity and lips Most patients (> 85%) develop prodromal symptoms
24 hours before painful lesions at the lip borders

Most lesions are small (< 100 mm3) Progress from vesicle to crust in about 8 days Recurrences are not associated with systemic symptoms

Herpes Labialis
Stimuli for reactivation
Fever Menstruation Sunlight URIs

Herpes SimplexDx
Viral culture HSV PCR assays

Herpes Simplex (Labialis)Treatment


Mild self-limited disease
Does not require treatment

Recurrent symptomatic herpes labialis and an identifiable prodrome


Episodic oral antiviral therapy with
Acyclovir Famciclovir Valacyclovir

Frequent recurrences > 4 episodes/year


Or recurrences associated with serious complications, such as recurrent aseptic meningitis
Chronic suppressive therapy

Herpes Simplex (Labialis)Treatment


Antiviral therapy with topical creams or ointments are, at best, of modest benefit Sunscreen may be helpful

Herpes SimplexTreatment
Acyclovir (Zovirax)
Primary = 400 mg TID x 7 days, 200 mg 5x/day x 7 days Recurrent = 400 mg 5x/day x 5 days, 400 mg TID x 5 days, 800 mg TID x 2 days Suppression = 400 mg BID

Famciclovir (Famvir)
Primary = 250 mg TID x 7 days Recurrent = 1,500 mg once, 750 mg BID x 1 day, 125 mg BID x 5 days, 1 g BID x 1 day Suppression = 250 mg BID

Valacyclovir (Valtrex)
Primary = 1 g BID x 7 days Recurrent = 2 g once, 1 g daily x 5 days, 500 mg BID x 3 days Suppression = 500 mg or 1 g daily

Perleche (Angular Cheilitis)

Google Images

Perleche (Angular Cheilitis)


Acute or chronic inflammation of the angles of the lips Causes
Mechanical trauma Infection
Candida albicans Staph. aureus

Ill-fitting dentures Sicca symptoms (dry mouth) Poor oral hygiene

KOH prep is useful to r/o candida

Perleche (Angular Cheilitis)Rx


General measures
Improving denture fit Oral hygiene Salivary substitutes (sialogogues)

Zinc oxide Vaseline Topical antifungal azoles


Miconazole Clotrimazole BID x 1-3 wks

Mupirocin BID x 1-2 wks

Pemphigus Vulgaris

Google Images

Angioedema

Google Images

Angioedema
Self-limited, localized subcutaneous (or submucosal) swelling Due to extravasation of fluid into interstitial tissues Areas with loose connective tissue
Face, lips, mouth, throat, larynx, uvula, extremities, genitalia

Angioedema
Mast cell-mediated angioedema
Urticaria, flushing, generalized pruritus, bronchospasm, throat tightness, and/or hypotension

Bradykinin-induced angioedema
ACE No allergic manifestations

AngioedemaCauses
Hereditary angioedema due to C1 inhibitor deficiency = 23% Acquired angioedema due to C1 inhibitor deficiency = 2% ACE inhibitors = 11% Food, drug, bite, environmental allergen, physical stimulus = 16% Chronic infection or autoimmune disease = 7% No trigger could be identified = 38%

AngioedemaRx
Airway management Allergic angioedema
H1 and H2 antihistamines Glucocorticoids
Methylprednisolone 60-80 mg Prednisone 20-40 mg/day

C1-inhibitor deficiency (hereditary angioedema)


Purified C1 inhibitor concentrate FFP

AngioedemaRx
ACE inhibitor-induced angioedema
Airway management Discontinue ACE inhibitor Additional therapies - severe or persistent symptoms
Icatibant synthetic bradykinin B2 receptor antagonist Ecallantide - recombinant protein used for hereditary angioedema Fresh frozen plasma Purified C1-inhibitor concentrate

Aphthous Ulcer

Google Images

Aphthous Ulcer
Painful oral lesions
Localized, shallow, round to oval ulcers with a grayish base Heal within 10-14 days without scarring

Recurrent aphthous ulcers


Hereditary Trauma Hormonal factors Food or drug hypersensitivity Immunodeficiency Emotional stress IBD Celiac disease

Aphthous UlcerRx
Topical therapy
Triamcinolone in Orabase Fluocinonide gel with Orabase Topical analgesics

Silver nitrate cautery

Aphthous UlcerRx
Recalcitrant ulcers
Colchicine Dapsone Pentoxifylline Alpha interferon Levamisole Thalidomide

Venous Lake

Google Images

Case #16

Jefferson Clinical Images Database. http://jeffline.jefferson.edu/JCI/.

Case #16
A 39 y/o female c/o a lump on her back that has gotten bigger and smaller over the past several years, and has occasionally drained some cheesy looking material.

Jefferson Clinical Images Database. http://jeffline .jefferson.edu/JCI/.

16. You tell her that the material within the lesion is most likely which one of the following?
A. Keratin B. Sebum C. Pus D. Dirt

Jefferson Clinical Images Database. http://jeffline.jefferson.edu/JCI/.

Epidermal Cyst

Jefferson Clinical Images Database. http://jeffline.jefferson.edu/JCI/.

Epidermal Cyst

Google Images

Epidermal Cyst

Images Dr. Richard P. Usatine

Epidermal Cyst
Also known as
Epidermoid cysts Epidermal inclusion cysts Sebaceous cysts

Most common cause of cutaneous cysts Have characteristic central punctum Cyst wall produces keratin, not sebum Lesions may remain stable or progressively enlarge Spontaneous inflammation and rupture can occur

Epidermal Cyst
Inflamed, uninfected epidermal cysts often resolve spontaneously without therapy
Tend to recur

Injection of triamcinolone
3 mg/mL for the face 10 mg/mL for the trunk Can hasten the resolution of inflammation and may prevent need for I&D

Epidermal CystRx
Excision
Best accomplished when the lesion is not inflamed 4-6 weeks after inflammation has resolved

Infected, fluctuant cysts


Larger, more erythematous, and more painful than sterile inflamed cysts Require I&D Drain/wick for at least 24hrs

Epidermal CystRx
Oral antibiotics
Cellulitis or cysts not responding to I&D Augmentin Tetracycline Erythromycin Bactrim

Lipoma

Jefferson Clinical Images Database. http://jeffline .jefferson.edu/JCI/.

Keloid

Google Images

Keloid
Benign, fibroproliferative lesions Hyperproliferation of fibroblasts
Response to trauma
Piercings Lacerations Secondarily infected skin lesions Surgery

Occasionally occur de novo

KeloidRx
Intralesional corticosteroids
1st line therapy 70% will improve Recurrence = 50% at 5 yrs Complications
Cutaneous atrophy Hypopigmentation

Triamcinolone 5-40 mg/mL


Inject 0.1-0.5 mL via 30 gauge needle to make keloid blanch Repeat q month

KeloidRx
Excision
Combine with steroid injections

Silicone gel sheeting Cryosurgery Pressure earrings Radiation therapy Interferon alfa Intralesional 5-FU Intralesional verapamil Laser therapy Imiquimod

Neurofibroma

Jefferson Clinical Images Database. http://jeffline.jefferson.edu/JCI/.

Xanthelasma

Google Images

Xanthelasma
Cholesterol-filled, soft, yellow plaques Medial aspects of the eyelids bilaterally Middle-aged/older adults Hyperlipidemia 50% of patients with xanthelasma

Xanthelasma
Lipid lowering (statins)
May induce regression of xanthelasma in some patients

Therapy for cosmetic reasons


Surgery CO2 laser Trichloroacetic acid

Answers
9. B 10.D 11. B 12. C 13. C 14. B 15. D 16. A

Managing Common Cutaneous Problems (Part 2)


Gary Levine, MD, FAAFP Associate Professor Brody School of Medicine at East Carolina University Greenville, NC

Selected Issues in Womens Health


Contraception, Infertility, Cancer
David Glenn Weismiller, MD, ScM, FAAFP Department of Family Medicine
The Brody School of Medicine at East Carolina University

Disclosure Statement
Dr Weismiller has nothing to disclose.

The AAFP has selected all faculty appearing in this program. It is the policy of the AAFP that all CME planning committees, faculty, authors, editors, and staff disclose relationships with commercial entities upon nomination or invitation of participation. Disclosure documents are reviewed for potential conflicts of interest and, if identified, they are resolved prior to confirmation of participation. Only those participants who had no conflict of interest or who agreed to an identified resolution process prior to their participation were involved in this CME activity.

Learning Objectives
1. Discuss what is known about risk factors for common malignancies. 2. Recommend methods of screening for cancer in women. 3. Describe principles of patient selection regarding contraceptives. 4. Discuss the contraindications for various contraceptive methods. 5. State the indications regarding methods of emergency contraception. 6. Summarize the basic steps for the initial evaluation of the infertile couple.

Endometrial Ovarian Cervical

CANCER

1. Which of the following statements is true when considering gynecologic malignancies and care?
A. Endometrial cancer is considered to be caused by overexposure to unopposed progesterone stimulation. B. Transvaginal ultrasound is the preferred modality in evaluating postmenopausal women with uterine bleeding. C. Combined hormonal contraception is associated with a decreased incidence of ovarian cancer. D. HPV testing is a routine adjunctive screening test for women < 30 years of age. E. The upper age limit recommended by ACIP for starting the HPV vaccine is 21 years of age.

Endometrial Cancer
Key Facts
Most common gynecologic malignancy in the USA (adenocarcinoma) Believed to be caused by overexposure to unopposed estrogen stimulation (endogenous and/or exogenous) Abnormal uterine bleeding is the presenting sign in 85% of women with endometrial cancer. Strongest association with reduced risk: Combined hormonal contraception use
50% reduction in risk Protection for 10-15 years after discontinuation

Endometrial Cancer Risk and Protective Factors


Risk

Protective

Advancing age Progesterone Obesity* OCPs Nulliparity Cigarette smoking Early menarche Multiparity Late menopause Breastfeeding Chronic anovulation Physical activity Unopposed exogenous estrogen use * Obesity leads to increased estrogen Tamoxifen levels from peripheral conversion of androstenedione. The presence of diabetes and Hypertension HTN as risk factors may simply reflect the high Diabetes
incidence of obesity in patients with these disorders. All reduce exposure to unopposed estrogens.

SERMs and Endometrial Cancer


Although raloxifene has estrogen-like effects on the uterus, it has NOT been shown to increase the risk of endometrial cancer. (SOR A) Tamoxifen is a selective estrogen receptor modulator that has estrogen-like effects. While it has a protective effect on breast tissue, its effect on the uterus INCREASES the risk of endometrial cancer. (SOR A)

Abnormal Bleeding Who Needs Endometrial Biopsy?


Women
> age 45 with suspected anovulatory bleeding
ACOG 2012

Between 19 and 45 years who do not respond to medical therapy OR have prolonged periods of anovulation
ACOG 2012

Using tamoxifen
ACOG 2002; No. 39

What Are We Looking for on the Biopsy?


Cytologic atypia is the SINGLE most important histologic finding. Only ATYPICAL hyperplasia has a significant risk of developing into endometrial cancer.
29% progresses to invasion. Need to rule out cancer if atypia is present.

Endometrial hyperplasia is a BENIGN condition, not a cancer precursor.

Triage Guidelines Reproductive Age Women


No cytologic atypia
Simple EMHP with abnormal bleeding
Progestin withdrawal for 6 months, then rebiopsy

Complex (adenomatous) EMHP


Progestin withdrawal, then rebiopsy

Cytologic atypia
High-dose progestins, megestrol, or medroxyprogesterone (Depo-Provera) for 3 months, then rebiopsy

Triage Guidelines Postmenopausal Women


No cytologic atypia
Progestins for 6 months, then rebiopsy TAH for recurrent EMHP or bleeding

Cytologic atypia (substantial risk of deeply invasive or poorly differentiated cancer)


Hysterectomy

Postmenopausal Bleeding: An Evidence-Based Workup


H&P, Pap CBC, STD testing*

Perform EMB or TVUS

EMB

Abnormal results

TVUS

Normal results

Refer for treatment

ET 4 mm ( 95% sensitivity)

ET 4 mm

Uterine pathology

Bleeding continues?

Atrophic endometrium

Perform SIS, EMB, or hysteroscopy with biopsy

Refer for treatment

No

Yes

Begin observation period

Perform TVUS, SIS, or hysteroscopy

Can the content of the endometrial stripe be reliably assessed by TVUS?

TVUS for Endometrial Cancer: Meta-analysis


Tabor et al. Obstet Gynecol 2002;99 Not sensitive enough to use as the only test in postmenopausal women with abnormal uterine bleeding
50% false positive rate
Hyperplasia associated with the thickest endometrium, but not most serious disease.

4% of cancers missed even when liberal guidelines for referral to D&C are used. Cut-off should be < 4mm if women at high risk. Conclusion: Obtain histologic sample if possible.

Ovarian Cancer
The most lethal of the gynecologic malignancies
Epithelial ovarian is the most common type (8595%). Risk peaks between ages 65 and 75. < 5% of women are genetically predisposed.

Majority have no identifiable risk factors (90%). Highest risk factor: genetic predisposition
90% are inherited mutations in either BRCA1/2 genes.

Assess response to treatment by serial Ca-125 levels.


90% correlation with disease progression

Ovarian Cancer Prevention


OCPs and a decreased incidence 40% reduction among all users 50% reduction with use > 5 years Up to 80% reduction with use > 10 years Protective effect persists up to 15 years after discontinuation. Breastfeeding
Each month of breastfeeding results in a 1%-2% decrease.

Tubal sterilization
Decreases risk by ~ 18%-40%; mechanism for the protective effect is unknown; some experts theorize it stops carcinogens from reaching the ovaries after they enter the body via the vagina.

Avoid talc powders in genital hygiene. Greater than one full-term pregnancy prior to age 35 Prophylactic oophorectomy

Ovarian Cancer
Risk Factors Summary
Increased Risk
Delayed childbearing Early menarche Endometriosis ERT > 5 years Family history suggesting genetic predisposition Genetic syndromes High-fat diet Late menopause Low parity

Decreased Risk
Breastfeeding > 18 months Early menopause Multiparity (risk decreases with each additional pregnancy) Hysterectomy Late menarche Low-fat diet OCP use Tubal ligation

Cervical Cancer
Cervical cancer is an STD caused by HPV!
Worldwide the fourth leading cause of cancer deaths in women
Developing countries second most common cause of cancer deaths in women

Screening programs have dramatically reduced the rate in screened population.


70% reduction in the US over the past five decades

Condoms appear to be an effective barrier against HPV transmission.


HPV infection are primarily transmitted through contact with infected skin or mucosal surfaces.

Screening for Cervical Cancer


Group
Women ages 21-29 Women ages 30-65

USPSTF 2012*
Screen with cytology q 3 years. (A) Screen with cytology q 3 y OR co-testing (cytology/HPV) testing q 5 years. (A) Do not screen. (D)

ACS, ASCCP, ACCP 2012*; ACOG 2013*


Screen with cytology q 3 years. (A) Screen with co-testing (cytology/HPV) q 5 years (preferred) or cytology q 3 y. (A)

Women < age 21 Women > 65 who have had adequate prior screening and are NOT at high risk

Do not screen. (D) The ACS/ASCCP/ASCP/ACOG guidelines define adequate prior screening: 3 consecutive (-) cytology results or 2 consecutive (-)HPV results within 10 y before cessation of screening, with the most recent test occurring within 5 y. Do not screen. (D)

Women after hysterectomy with removal of cervix AND no history of HGSIL or cervical CA Women < age 30

Do not screen with HPV testing (alone or with cytology) (D)

*This recommendation statement applies to all women who have a cervix, regardless of sexual history

Resources

www.asccp.org

Essential Changes from 2006 Guidelines


Cytology reported as negative but lacking endocervical cells managed without early repeat. CIN 1 on endocervical curettage should be managed as CIN 1, NOT as a positive ECC. Cytology reported as unsatisfactory requires repeat even if HPV negative. Genotyping triages HPV-positive women with HPV type 16 or type 18 to earlier colposcopy only after negative cytology; colposcopy is indicated for all women with HPV and ASC-US, regardless of genotyping result.

Essential Changes from 2006 Guidelines


For ASC-US cytology, immediate colposcopy is NOT an option. The serial cytology option for ASC-US incorporates cytology at 12 months, NOT 6 months and 12 months, and then if negative, cytology every 3 years. HPV-negative and ASC-US results should be followed with co-testing at 3 years rather than 5 years. HPV-negative and ASC-US results are insufficient to allow exit from screening at age 65 years. The pathway to long-term follow-up of treated and untreated CIN 2+ is more clearly defined by incorporating co-testing.

Essential Changes from 2006 Guidelines


More strategies incorporate co-testing to reduce followup visits. Pap-only strategies are now limited to women younger than 30 years, but co-testing is expanded even to women younger than 30 years in selected circumstances. Women aged 21-24 years are managed conservatively.

Prevention
Condom use
May reduce the risk for HPV-associated diseases (eg, genital warts and cervical cancer) Use associated with higher rates of regression of CIN and clearance of HPV infection in women; regression of HPVassociated penile lesions in men Limited number of prospective studies have demonstrated a protective effect of condoms on the acquisition of genital HPV.

Tobacco cessation
Quit

Vaccination

Selected Age
ACIP Recommendation (CDC) 2014
The recommendation for HPV vaccination for CHILDREN ages 11-12 is based on the following:
Studies suggesting that HPV vaccines among adolescents will be safe and effective Can be started as young as 9 years of age The high antibody titers (persisting at least 5 years in initial clinical trials) achieved after vaccination at this age ACIP does not express a preference for either of the vaccine types. Vaccine is approved for use up until age 26.

Logistics of HPV Vaccination


3-dose schedule; second dose 1-2 months after the first dose; third dose 6 months after first dose
Minimum interval between first and second doses 4 weeks; between second and third dose 12 weeks; between first and third dose 24 weeks

Whenever possible, the same HPV vaccine product should be used for all doses in the series.

CONTRACEPTION

2. In considering contraception in women, which of the following is true?


A. Triphasic OCPs are recommended as the first choice for women starting combined OCPs. B. Decreased libido is a possible side effect of excess progesterone. C. A progesterone-only method of contraception would be contraindicated in a patient > 35 years of age who smokes. D. With the exception of Rifampin, antibiotics have little effect on the efficacy of combined hormonal contraception. E. Natural family planning tends to hinder a couples communication and forms of sexual expression.

Combined Hormonal Options


Pill
Monophasic pills recommended as the first choice for women starting combined OCPs (Cochrane 2005). 25-year mortality from all causes same for OCP users vs non-users.

Ring Patch
Highly efficacious in women < 90 kg Safety warning (FDA)
~ 60% more estrogen per cycle than 35 mcg pill (11/2005) > 3x risk of VTE compared to combined OCP

Inhibit ovulation at pituitary and hypothalamus Estrogen


Ethinyl estradiol

Progestins
Drospirenone no risk of hyperkalemia

Descending order of androgenic potency

Progestins
First
Original pills

Second
Levonorgestrel
Alesse Levlite Nordette Levlen

Third
Norgestimate
Ortho-Cyclen Ortho-Tri-cyclen

Fourth
Drospirenone
Yasmin Yaz

Desogestrel
Mircette Desogen Orthocept Cyclessa

Anti-androgenic

Norgestrel
LoOvral

Norethindrone acetate
Loestrin

Norethindrone
Ortho-novum Ovcon

1/35

35 1/35

Ethynodiol diacetate
Demulen

Combined Contraceptives Benefits


Less risk of ectopic pregnancy Increases bone mass
Reduces risk of postmenopausal hip fractures

Relieves dysmenorrhea Improves symptoms of PCOS


High estrogen/progestin ratio

Low-dose pills useful for management of perimenopause

Combined Contraceptives Decrease


Iron deficiency anemia Fibrocystic breast disease Functional ovarian cysts (use high estrogen content)/fibroids Pelvic inflammatory disease Cervical mucus/reduced menstrual blood flow Less retrograde menstruation Cancer
Ovarian and endometrial (OR 0.57; NNT 60)* cancers Protective effects persist up to 20 years after discontinuation. Colorectal cancer (OR 0.86; NNT 132)*

Endometriosis (use strong progestin component)


* Gierisch JM et al. Cancer Epidemiol Biomarkers Prev. 2013 Sep 6 [e-pub ahead of print]. (http://dx.doi.org/10.1158/1055-9965.EPI-13-0298)

Combined Contraceptives Side Effects (Excess)


Estrogen
N/V Bloating/edema Hypertension Migraine HA Breast tenderness Decreased libido Weight gain Heavy bleeding Leukorrhea

Progestin
Acne
(Ortho-Tri-Cyclen approved for treatment)

Increased appetite Hypertension Fatigue Depression Hirsutism Vaginal yeast infection

Combined Contraceptives Side Effects (Deficiency)


Estrogen
Spotting/breakthrough Amenorrhea Vaginal dryness

Progesterone
Amenorrhea Late breakthrough or heavy bleeding

Drug Interactions with Combined Hormonal Contraception


Drugs likely to lead to contraceptive failure
Rifampin otherwise little effect from antibiotics [SOR:A] Anticonvulsants (significant effect) except valproic acid Antifungals (griseofulvin) HIV medications

Extended vs 28-day Cycle OCPS Cochrane 2005


Similar pregnancy rates, safety profiles, compliance Some discontinuation rates for bleeding problems
Only 16% of women will be amenorrheic in the first three months.

Fewer headaches, less genital irritation, fatigue, bloating, dysmenorrhea

Progestin Only Options


Pill
Must be taken q day at SAME time (2 h window)
Back-up if > 3 h late

Implant
q 3 years < 125% IBW

IUS
q 5 years

DMPA
q 3 months

No effect on BP; risk of VTE, CVA, MI [SOR: B] Most common side effect: Irregular bleeding

Management of Unscheduled Bleeding in Women Using Contraception


Contraceptive
DMPA

Preferred Treatment
Expectant management 7-14 days oral estrogen (1.25 mg conjugated estrogen or 2 mf micronized estradiol Transdermal patch (0.1 mg estradiol/24 h) 10-20 days of low-dose combined OCP Expectant management Low-dose combined OCP for 10-20 days (not studied) NSAID for 5-7 days Take at same time each day and minimize missed doses. NSAID for 5-7 days (eg, ibuprofen 400 mg, naproxen 250 mg, or mefanamic acid 500 mg TID)

Etonogestrel implant

Progestin pills Levonorgestrel IUD

Edelman A and Kaneshiro B. Management of unscheduled bleeding in women using contraception. www.uptodate.com, 2013.

DMPA
Side effects
Weight gain, amenorrhea, hair loss, bone loss

Loss of bone density


Should be used for > 2 y only if other methods are inadequate [SOR: B] No current recs for BMD measurements dietary calcium + vitamin D may be partially protective.

Progestin-Only Methods More Appropriate Than Combined


ACOG 2006
Smoking or obesity AND over age 35 [SOR B, A; respectively] Hypertension with vascular disease or > age 35 [SOR B] Lupus with vascular disease, nephritis [SOR A] Migraine with focal aura [SOR B] Current or personal history of VTE associated with pregnancy or estrogen unless on anticoagulation [SOR A] Coronary artery/cerebrovascular disease [SOR C]

Intrauterine Contraceptives Mechanisms of Action

Source: Barr Pharmaceuticals, Inc.

Levonorgestrel-Releasing Intrauterine System (LNG-IUS, Mirena)


Inhibits fertilization Thickens cervical mucous Inhibits sperm function Thins and suppresses the endometrium

Copper-Releasing Intrauterine Contraceptive (ParaGard T380A) Inhibits fertilization Releases copper ions (Cu2+) that reduce sperm motility May disrupt the normal division of oocytes and the formation of fertilizable ova

Jonsson B, et al. Contraception. 1991;43:447-458; Videla-Rivero L, et al. Contraception. 1987;36:217-226; Kulier R, et al. Cochrane Database Syst Rev. 2006;3: CD005347.

Considerations for IUDs


IUD insertion, not IUD use, is associated with PID.
Cochrane Systematic Review (Grimes, Mohllajee) ACOG Practice Bulletin 2005

DO NOT cause future infertility Nulliparas can use an IUD.


Uterus sounds to depth of a minimum 6 cm

Guidelines for IUDs


Organization
ACOG 2007 ACOG 2007

Recommendation
Asymptomatic women may use an IUD within 3 months of treated pelvic infection or septic abortion. All adolescents should be screened for GC and chlamydia prior to insertion.

Cochrane 2007
CDC 2010

No benefit from doxycycline or azithromycin prior to insertion.


Evidence is insufficient to recommend the removal of IUDs in women diagnosed with acute PID. However, caution should be exercised if the IUD remains in place, and close clinical follow-up is mandatory. The rate of treatment failure and recurrent PID in women continuing to use an IUD is unknown, and no data have been collected regarding treatment outcomes by type of IUD (eg, copper or levonorgestrel).

Strategies to Reduce Barriers and Increase Use of Implants and IUDS


Encourage implants and IUDS for all appropriate candidates including nulliparous women and adolescents. Adopt same-day insertion protocols.
Screening for chlamydia, gonorrhea, and cervical dysplasia SHOULD NOT be required before implant or IUD insertion but may be obtained on the day of insertion, if indicated.
ACOG Committee Opinion No. 450. Increasing use of Contraceptive Implants and Intrauterine Devices to Reduce Unintended Pregnancy. Obstet Gynecol. 2009;114(6):1434-1438.

Barrier Methods Key Points


Sponge
Does not protect against STIs (according to manufacturer) Non-menstrual toxic shock syndrome (sponge, diaphragm, cap)
2 cases/100,000 users per year

Diaphragm
Increased incidence of UTI

Latex condom
Consistent use results in 80% reduction of HIV. Use only water-based lubricants.

Natural Family Planning


Pluses
No adverse drug effects No medication or device cost; cannot run out of method Immediately reversible Acceptable to all major religions Expands couples communication and forms of sexual expression

Minuses
No non-contraceptive benefits of some other methods Requires periodic abstinence Requires intensive education

Contraceptive Use during Lactation


All family planning choices are available to the postpartum lactating woman. Choice and clinical ramifications merit additional counseling.

WHO Medical Eligibility Criteria


Duration of BF method Progestin- Progestin- Progestin- Combined Low-dose only pills only only patch or combined depots implants/ ring IUD

< 6 weeks PP > 6 w to < 6 m PP (primarily breastfeed)

3
1 1

3
1 1

3
1 1

4
3 2

4
3 2

> 6 m PP

1 No restriction 2 Generally use 3 Not usually recommended 4 Not to be used

So How Does Choice Impact Lactation?


LAM
Abstinence/ Periodic Abstinence/ NFP Methods Barrier Methods IUD
Copper

Progestin-only
Pills Injectables

Sterilization

Implants Levonorgestrel IUD

Combined pill Patch Ring

No known impact on lactation

Little to no known impact on lactation

Some reports of Expected to negative impact have on lactation negative impact on lactation

3. Which of the following is true regarding Emergency Contraception (EC)?


A. Use of EC is recommended no longer than 72 hours after unprotected intercourse. B. There is significant evidence that EC is unsafe for women with contraindications to OCPs or for those with medical conditions. C. Nausea and vomiting following use of combined OCPs for EC is uncommon. D. Levonorgestrel (Plan B) is administered as a single dose.

Postcoital Treatments for Preventing Pregnancy


Recommended within 120 Hours
Option Combination OCP Levonorgestrel (Plan B One-Step) Ulipristal actetate (ELLA) Treatment 2 doses (12 h apart) Single dose (1.5mg) Comments N/V not uncommon; prophylactic emetic Single dose simplifies use; no increase in side effects Progesterone agonist/antagonist; inhibits/delays ovulation Failure rate < 1%

Single dose

Copper IUD (Paragard)

Emergency Contraception (EC)


Prevent fertilization by inhibition of ovulation (hormonal). Use after implantation does not interrupt an established pregnancy.
Grimes et al. Ann Intern Med. 2002:137

EC Indications ACOG 2005


Inadequately protected or unprotected intercourse in women who do not desire pregnancy (SOR A) No evidence that EC is unsafe for women with contraindications to OCPs or for those with medical conditions Should be offered up to 120 hours after unprotected intercourse (SOR B)

INFERTILITY

4. Infertility is defined is an inability to conceive in what time period of attempted conception?


A. 6 months B. 12 months C. 18 months D. 24 months E. 36 months

Infertility
Am Soc Reprod Med Practice Comm 2000
Defined: 1 year of attempted conception without successful pregnancy
85% of fertile couples would have been successful by this time. Earlier evaluation
Oligomenorrhea/amenorrhea Age > 35 years Known or suspected pelvic pathology

Etiology
Female factors
Ovarian dysfunction, 40% Tubal factors, 30% Endometriosis, 15% Other, 10% Uterine/cervical, 3%

Male factors
Unknown, 40%-50% Primary hypogonadism, 30%-40% Altered sperm transport, 10%-20% Secondary hypogonadism, 1%-2%

Essential History
Sexual
Frequency of intercourse Use of lubricants, etc Erectile dysfunction Dyspareunia

Drug or alcohol use


Particularly has a toxic effect on sperm

Caffeine
Interferes with muscle contraction of fallopian tube

Medications including nonprescription Chronic disease

5. Which of the following would be considered in the primary workup of infertility?


A. Hysterosalpingogram B. Assessment of sperm-mucus interaction C. Urinary FSH D. Serum testosterone level E. Serum catecholamine level

Primary Objectives at Workup


Rule Out
Azoospermia
Anovulation

Procedure
Semen analysis
Urinary LH and mid-luteal phase progesterone levels

Tubal obstruction
Uterine cavity anomalies Decreased ovarian reserve

Hysterosalpingogram (HSG) or laparoscopy


HSG or sonohysterogram FSH on day 3 of cycle
FSH (> 15-29 IU/L) are associated with: Poor ovarian response to exogenous gonadotropins Reduced likelihood of successful conception

Ovulation Factors
25% of infertile women are anovulatory.
Majority with PCOS

Clomiphene-resistant women (usually PCOS)

Insulinizing-Sensitizing Drugs for PCOS


Metformin is recommended as first-line agent for anovulatory women with PCOS; also decreases hirsutism and menstrual irregularities. Lord et al. BMJ. 2003;327
Cochrane Update 2005 Full dose 850 mg BID First week of taking metformin, upset stomach/diarrhea common; side effect can be reduced by taking it after food and by starting with a very low dose (250 mg) and increasing slowly by 250 mg per week.

Metformin + clomiphene more effective in achieving ovulation than clomiphene alone No long-term safety data in young women

Urine LH Monitoring
Commonly used to assist diagnostic and therapeutic timing
Typically ~ half-way through cycle Ovulate in 24-36 hours

BBT is retrospective, LH is prospective.

Mid-luteal Progesterone Level


Measure 7 days after estimated ovulation detected by LH surge (or basal body temperature chart). Should be at least 6.5 ng/mL, preferably > 10 ng/mL to confirm ovulation

If Not Ovulating
Check FSH to ensure that patient is not menopausal.
Premature ovarian failure consider donor oocytes

Look for: Systemic disease Anorexia nervosa, low body fat PCOS/chronic hyperandrogenic anovulation Hypothalamic dysfunction Stress

Exclude Other Etiologies


Prolactinoma
MRI to rule out pituitary adenoma

Virilizing tumors of adrenal or ovarian tumors Congenital adrenal hyperplasia Cushings syndrome

Luteal Phase Defect


Poorly understood subtle hormonal alterations linked with infertility and habitual abortion May be related to chronic stress
Catecholamines block progesterone receptors.

Treatment: Clomiphene or progesterone in luteal phase

Other Female Factors


Factor
Tubal

Recommendation

Tubal disease diagnosed by HSG, confirm with laparoscopy; treatment typically IVF Cervical Previous cryotherapy, LEEP, conization; inutero DES exposure Endometrial HSG: fibroids, polyps, anomalies; may need sonohysterogram/hysteroscopy Peritoneal Accounts for majority of infertility in young (endometriosis) women. Surgical ablation preferred over medical treatment IF pregnancy desired

6. In considering the male factor in infertility, which of the following is true?


A. Advancing male age is a primary risk factor for infertility. B. There is good evidence that treatment of a varicocele improves a couples chance of conception. C. Prior paternity is a key element of the male history. D. A serum testosterone level should be done in conjunction with a semen analysis.

Male Factor
History: Paternity, surgery, alcohol use, smoking, marijuana, medications

Physical: Testicular
volume, hernia, prostate, penile discharge

Tests: Sperm
analysis, sperm penetration assay

Treatment:
Intrauterine insemination, IVF, donor

Varicocele Surgery or Embolization


Meshwork of distended blood vessels in the scrotum Result of dilatation of spermatic vein No evidence that treatment improves couples chance of conception when compared to expectant management
From: Wikipedia

IVF
Indications
Tubal disease Persistent infertility after initial unsuccessful treatment

Risks
Multiple births Ovarian hyperstimulation

Pregnancy rates
29% with one cycle versus 4% with no treatment

Answer Key
1. 2. 3. 4. 5. 6. C D D B A C

Female Factors Related to Infertility Summary

Evaluate Todays Sessions

www.aafp.org/reportcme/boardrev/indianapolis

Board Review Express March 13-16, 2014 Indianapolis, Indiana


Day 3 - Saturday, March 15
7:00 - 8:00 am 8:00 - 8:30 am 8:30 - 9:00 am 9:00 - 9:30 am 9:30 - 9:45 am 9:45 - 10:00 am 10:00 - 10:30 am 10:30 - 11:00 am 11:00 - 11:30 am 11:30 - 12:00 pm 12:00 - 12:15 pm 12:15 - 1:15 pm 1:15 - 1:45 pm 1:45 - 2:15 pm 2:15 - 2:45 pm 2:45 - 3:15 pm 3:15 - 3:30 pm 3:30 - 3:45 pm 3:45 - 4:15 pm 4:15 - 5:00 pm 5:00 - 5:30 pm 5:30 - 5:45 pm 5:45 pm Breakfast Provided Abnormal Uterine Bleeding - David Weismiller, MD, ScM, FAAFP Renal I - Gary Levine, MD, FAAFP Common Issues in the Elderly Part 1 - Laurence Robbins, MD Q&A Break Common Issues in the Elderly Part 2 - Laurence Robbins, MD Renal II - Gary Levine, MD, FAAFP Acute & Chronic Cognitive Diseases - Laurence Robbins, MD Peripheral Vascular Disease - Robert Dachs, MD, FAAFP Q&A Lunch Acute CVA & TIA - Robert Dachs, MD, FAAFP Pharmacology in the Elderly - Laurence Robbins, MD The Surgical Abdomen - Robert Dachs, MD, FAAFP Managing Dysrhythmias - Jonathon Firnhaber, MD, FAAFP Q&A Break Trauma & Wound Care - Robert Dachs, MD, FAAFP ACS & Hyperlipidemia - Jonathon Firnhaber, MD, FAAFP Bites, Stings & Other Emergency Things - Robert Dachs, MD, FAAFP Q&A Recess

Abnormal Uterine Bleeding


David G Weismiller, MD, ScM, FAAFP
Department of Family Medicine
The Brody School of Medicine at East Carolina University
weismillerd@ecu.edu

Maternity and Gynecologic Care AAFP Reprint No. 261 http://www.aafp.org/dam/AAFP/documents/medical_education_residency/pro gram_directors/Reprint261_Maternity.pdf

Disclosure Statement
Dr Weismiller has nothing to disclose.

The AAFP has selected all faculty appearing in this program. It is the policy of the AAFP that all CME planning committees, faculty, authors, editors, and staff disclose relationships with commercial entities upon nomination or invitation of participation. Disclosure documents are reviewed for potential conflicts of interest and, if identified, they are resolved prior to confirmation of participation. Only those participants who had no conflict of interest or who agreed to an identified resolution process prior to their participation were involved in this CME activity.

Learning Objectives
1. Formulate evaluation and treatment plans related to various high-risk conditions and reproductive complications, including dysfunctional uterine bleeding and potential cancers. 2. Clarify the appropriate diagnostic strategies for evaluating abnormal bleeding. 3. Assess training needs for procedures used to evaluate and treat abnormal uterine bleeding. 4. Formulate a treatment plan for women with abnormal uterine bleeding including dysfunctional uterine bleeding, menorrhagia, and amenorrhea.

1. When considering abnormal uterine bleeding, which of the following statements is true?
A. B. C. D. E. Up to 20% of women (at any age) presenting with heavy bleeding will have an underlying bleeding disorder (disorder of hemostasis). Between the ages of 40 and menopause, pregnancy is one of the common etiologies. Measurement of endometrial thickness in premenopausal women is exceedingly helpful in the evaluation. Hirsutism associated with PCOS is best treated with metformin. Bleeding associated with the endometrium under minimal estrogen stimulation (too thin) is best treated with progestins.

Definition
Menstrual flow outside of normal volume, duration, regularity, or frequency

Nomenclature
2011 International Federation of Gynecology and Obstetrics

New classification scheme Standardize terminology Abnormal Uterine Bleeding (AUB)


Paired with descriptive term
Heavy menstrual bleeding (instead of menorrhagia) Intermenstrual bleeding (instead of metrorrhagia)

Paired with etiology or etiologies

Basic PALM-COEIN Classification System for the Causes of Uterine Bleeding in Non-pregnant Women of Reproductive Age
Munro et al. Int J Gynaecol Obstet. 2011;113:3-13. Descriptor Etiology

Abnormal Uterine Bleeding (AUB) Heavy menstrual bleeding Intermenstrual bleeding

PALM: Structural Causes Polyp (AUB-P) Adenomyosis (AUB-A) Leiomyoma (AUB-L) Malignancy and hyperplasia (AUB-M)

COEIN: Nonstructural Causes Coagulopathy (AUB-C) Ovulatory (AUB-O) Endometrial (AUB-E) Iatrogenic (AUB-I) Not yet classified (AUB-N)

Nomenclature
2011 International Federation of Gynecology and Obstetrics

New classification scheme Standardize terminology Abnormal Uterine Bleeding (AUB)


Paired with descriptive term
Heavy menstrual bleeding Intermenstrual bleeding

Paired with etiology or etiologies

Recommend discontinuation of the term DUB

Pathophysiology
Why do we have abnormal uterine bleeding?
Structural (PALM) Systemic (both inherited and acquired) Ovulatory AUB
Hypothalamic-pituitary-ovarian axis intact Steroid hormone profiles are normal. Once regular menses has been established during adolescence, ovulatory AUB account for most cases
Abnormal prostaglandin synthesis and receptor upregulation Increased local fibrinolytic activity Increased tissue plasminogen activator activity

Ovulatory dysfunction AUB


Spectrum of disorders ranging from amenorrhea to irregular heavy menstrual periods Typically result of endocrinopathy eg, PCOS Mechanisms of abnormal bleeding related to unopposed estrogen

Diagnosis
Medical history
Family history of AUB or other bleeding problems
Up to 20% of women (at any age) presenting with heavy bleeding will have an underlying bleeding disorder (disorder of hemostasis).

Clinical Screening for an Underlying Disorder of Hemostasis


Initial screening structured by medical history; (+) screen comprises any of the following:
Heavy menstrual bleeding since menarche One of the following:
Postpartum hemorrhage Surgery-related bleeding Bleeding associated with dental work

(+) Screen
Further evaluation to include:
Testing of:
von Willebrand factor Ristocetin cofactor

Consultation with hematologist

Two or more of the following:


Bruising 1-2x/month Epistaxis 1-2x/month Frequent gum bleeding Family history of bleeding symptoms

Diagnosis
Medical history
Family history of AUB or other bleeding problems
Up to 20% of women (at any age) presenting with heavy bleeding will have an underlying bleeding disorder (disorder of hemostasis)

Medication or herb remedies


Warfarin Heparin NSAID Motherwort Hormonal contraceptives Gingko Ginseng

Diagnosis
Physical findings
Weight Signs of PCOS
Hirsutism Acne

Signs of thyroid disease Signs of insulin resistance


Acanthosis nigricans on the neck

Acanthosis Nigricans

Diagnosis
Physical findings
Weight Signs of PCOS
Hirsutism Acne

Signs of thyroid disease Signs of insulin resistance


Acanthosis nigricans on the neck

Look at the vagina and cervix lesions

Laboratory Tests
Pregnancy test (blood or urine) CBC
Anemia Thrombocytopenic bleeding

Screening for bleeding disorders (Level A)


CBC with Plts, PT/PTT; fibrinogen and thrombin time are optional; bleeding time neither sensitive or specific do not need.
All adolescents with heavy menstrual bleeding Adult patients with (+) screen

TSH (Level B) Chlamydia trachomatis (Level B) Pap if needed

Imaging
Transvaginal US TVUS images are not adequate or further evaluation of cavity is necessary
Saline infusion sonohysterography (Level A) Hysteroscopy

MRI is NOT a primary imaging modality for AUB.

Who Needs Imaging with AUB?


Literature is unclear. Some suggestions:
Abnormal physical exam, eg, enlarged or globular uterus on bimanual Symptoms persist despite treatment in the setting of normal pelvic exam (Level C).
TVUS, or biopsy, or both

Clinical suspicion for polyps or submucosal leiomyomas


Sonohysterography Hysteroscopy
Numerous recent studies have demonstrated that DH had a significantly better diagnostic performance than SH and TVUS and was significantly more precise in the diagnosis of intracavitary masses. Hysteroscopy not only has increased accuracy for identifying the etiology of AUB, compared with D&C, but also offers the possibility of in-office use.

NO measurement of endometrial thickness in premenopausal women not helpful

Endometrial Tissue Sampling


Primary role: Carcinoma or premalignant lesions present AUB and:
> 45 years (Level C)
First-line test

< 45
History of unopposed estrogen
Obesity PCOS

Failed medical management Persistent AUB

2. In considering the results of an endometrial biopsy, which of the following statements is true?
A. Secretory endometrium is normal in the follicular phase. B. Proliferative endometrium is normal in menopause. C. Atrophic endometrium is seen in women on Depo-Provera. D. Hyperplasia is the advanced effect of unopposed progestin.

EMB
Results
Proliferative normal in the follicular phase
When associated with abnormal bleeding, confirms anovulation and the effect of unopposed estrogen

Secretory/menstrual confirms ovulation has occurred Hyperplasia advanced effect of unopposed estrogen atypia = premalignant Atrophic seen in menopause or effect of OCPs, Depo-Provera, and continuous ERT

Aged-Based Common Differential Diagnosis


13-18 Persistent anovulation
Immaturity of HPO axis and represents normal physiology Hormonal contraceptive use Pregnancy Pelvic infection Coagulopathies Tumors

19-39 Pregnancy Structural lesion


Leiomyoma Polyp

Anovulatory cycles
PCOS

Hormonal contraceptive use Endometrial hyperplasia

Age-Based Common Differential Diagnosis


40 Years to Menopause
Anovulatory
Represents normal physiology in response to declining ovarian function

Endometrial hyperplasia Endometrial carcinoma Endometrial atrophy Leiomyomas

Endometrial Axis

From: Google Images

At What Point in the Evaluation Is Therapy Appropriate?


Lower risk empiric trial of therapy
< 45 Adolescent No enhanced risk of endometrial hyperplasia, neoplasia, structural abnormality

Higher risk initiation of therapy only after complete diagnostic evaluation


> 45 Prolonged anovulatory cycles Persistent bleeding despite therapy

Treatment of Thick Endometrium


What is the therapy?
Goal is to reduce the thickness of the endometrium. Progestins or OCPs

Who Has THICK Endometrium?


Women who:
Are obese Have chronic anovulation Have PCOS Are taking unopposed estrogen (uncommon)

AUB
Emergency Management

Isnt TOO much Estrogen what one is trying to correct? Why give MORE?

IV conjugated estrogen 25 mg q 4 hours until bleeding slows for 12 hours Believed to be a stimulus for clotting at the capillary level Promotes rapid growth of endometrium to cover denuded endometrial surface and stop bleeding 75% will be controlled in 6 hours. Oral conjugated estrogen 10 mg/day in four divided doses can be substituted for IV estrogen. Start OCPs or 10 days of monthly progestin (cyclic) after bleeding stops to prevent recurrence.

AUB
Non-emergency Management
One combined hormonal OCP for 7 days
If the flow stops within12-24 hours, the diagnosis of DUB can be confidently made. Stop combined OCPs for 7 days after one week of therapy. Begin regular OCPs for the next 3-6 months.

Alternative
Cyclic administration of progesterone for 3 months

When the Endometrium Is Too Thin because of Heavy Continuous Bleeding


The endometrium is shed and minimal. It may have previously been too thick. Treatment
Estrogen followed by progestin

Endometrium Is Too Thin Typically Associated with


Endometrium has minimal estrogen stimulation. High progestational OCP
e.g. LoOvral, Nordette

Progestin-only contraception
Depo-Provera, Minipill, IUD

Endometrium becomes thinner and potentially atrophic.

Treatment
Add estrogen, change from minipill to combination OCP or change OCP. General rule: Using progestins when the endometrium is too thin makes a bad situation worse.

Who Has THIN Endometrium?


Women who:
Have heavy continuous uterine bleeding Are using a progestin-dominant OCP, IUD, Depo-Provera, or minipill (progestin only) Are excessively thin or have low body fat may be hypoestrogenic Eating disorders Elite athletes
Marathon runners, gymnasts, skaters

Treatment of AUB (Cochrane Review)


Medical
NSAIDs prostacyclin (platelet
anti-aggregating vasodilator)
Decrease blood flow 20%-50%. Mefenamic acid 500 mg po TID

Surgical

Myomectomy Hysterectomy Ablation


1st generation
Resection Laser Rollerball

Tranexamic acid (Lysteda) Danazol 200 mg po q day


Anti-estrogen Decrease blood flow 50%. Side effects limit use.

OCPs Continuous OCPs Oral continuous progestins (d 5-26) Levonorgestrel IUS


80% decrease in blood loss

2nd generation
Cryoablation* Laser Intrauterine thermotherapy Radiofrequency ablation* Thermal balloon ablation

Microwave ablation*

Endometrial Ablation Methods


Rule out preinvasive and invasive endometrial lesions before procedures. Must have completed childbearing and tolerate some menstrual bleeding.

Treatment of AUB
Cochrane 2005
Endometrial ablation significantly more effective with fewer side effects than medical therapies No significant differences between levonorgestrel IUD and any surgery in satisfaction rates or quality of life
> 64% of women scheduled for hysterectomy canceled the surgery after Mirena.

Hysterectomy has high satisfaction rates but risks of major surgery.

Society of Gynecologic Surgeons


Wheeler TL II, Murphy M, Rogers RG, et al; Society of Gynecologic Surgeons Systematic Review Group. Clinical practice guidelines for abnormal uterine bleeding: hysterectomy versus alternative therapy. J Minim Invasive Gynecol. 2012;19(1):8187.

Systematic Review Group (SRG) of the Society of Gynecologic Surgeons


Tradeoffs between treatment effectiveness and the risk of serious adverse events between hysterectomy, ablation, and the LNG-IUS SRG was able to conclude ONLY that there was moderate strength of evidence supporting the statement that bleeding is better controlled following hysterectomy than following ablation. Three studies found statistically significant differences in validated dimensions of the SF-36 questionnaire favoring hysterectomy for pain, general health, vitality, and social function over ablation.

3. Which of the following statements is true when considering a patient with amenorrhea?
A. Amenorrhea is rarely associated with PCOS. B. It is unusual to find a positive pregnancy test in the patient presenting with amenorrhea. C. The relative estrogen status can be evaluated with a progestin challenge test. D. A TSH is not necessary in the evaluation.

Amenorrhea Systematic Approach


Focus on the signs and symptoms that suggest an underlying cause.
Thyroid disease: Hypo- or hyperthyroidism Estrogen deficiency: Menopause Androgen excess: Tumor, PCOS Pituitary tumors: Prolactinoma

Steps in Evaluation
Step 1 Rule out pregnancy. Step 2 TSH to evaluate for hypo- or hyperthyroidism Prolactin to evaluate for pituitary tumor (fasting, no breast stimulation) Step 3 Determine the relative estrogen status.

Relative Estrogen Status Progestin Challenge Test


5-10 mg medroxyprogesterone acetate po q day x 10 days Any bleeding within 2-7 days is positive.

Amenorrhea
TSH Prolactin Progestin challenge

Elevated TSH or prolactin

(+) Withdrawal bleed

Normal TSH and prolactin


Hypothyroidism Pituitary disease Anovulation

Anovulatory Amenorrhea
Amenorrheic women with adequate estrogen ([+] Progestin challenge test) Anovulatory, frequently obese, +/ PCOS
Progesterone is NOT being adequately produced in luteal phase. Unopposed estrogen stimulation Risk of endometrial cancer is increased. Treatment
Progestin 10 mg q day 7-10 days every month or OCPs

Amenorrhea
TSH (nl), prolactin (nl), and progestin challenge () withdrawal bleed

Outflow tract obstruction


(Ashermans, Mullerian agenesis)

Estrogen and progestin challenge test

No withdrawal bleed

Amenorrhea
(+) Withdrawal bleed following estrogen and progestin challenge Measure FSH and LH

Low
Normal MRI
Hypothalamic amenorrhea

Normal

High

Ovarian failure

Hypothalamic Amenorrhea
Hypogonadotropic Hypogonadism
Low or normal FSH/LH, normal prolactin, low levels of endogenous estrogen, normal MRI of sella (+) Withdrawal bleed following estrogenprogestin challenge test Usually diagnosed by exclusion of pituitary lesions Anorexia/bulimia, stress, high-intensity exercise, chronic illness

Hypothalamic Amenorrhea
Amenorrheic Women with Inadequate Estrogen
Risk of decreased bone density (10%-20%)
Cannot be completely overcome with supplemental calcium or weight-bearing exercise ? Rate of fractures

Although OCPs improve lumbar and total bone mineral, effect on fractures unknown Increase BMI > 20 to restore menses Decrease intensive exercising.

Ovarian Failure High FSH/LH


Premature ovarian failure
< 40 not always reversible Autoimmune, genetic, chemotherapy, mumps

Postmenopausal ovarian failure Absence of secondary sex characteristics


Gonadal dysgenesis Turner syndrome (most common form)

4. Which of the following drugs is not uncommonly used in the management of PCOS?
A. Rifampin B. Clomiphene C. Insulin D. Naprosyn E. Megace

What Is Polycystic Ovarian Syndrome?


Consensus Infrequent or no ovulation, infertility
Oligomenorrhea or amenorrhea

Hyperandrogenism
Presence of hirsutism or biochemical hyperandrogenemia

Polycystic ovaries on ultrasound


Presence of one polycystic ovary is sufficient to provide diagnosis.

Appears between puberty and mid-20s

Suggested Evaluation Physical


BP BMI
> 30 = obese

Waist circumference to determine body fat distribution


> 35 inches = abnormal

Stigmata of hyperandrogenism and insulin resistance


Acne, hirsutism, androgenic alopecia, acanthosis nigricans

PCOS
Diagnosis (AES 2009)
Androgen excess (clinical and/or biochemical) Ovarian dysfunction (oligo-anovulation and/or polycystic ovarian morphology) Exclusion of other causes of androgen excess and ovulatory disorders

Suggested Evaluation Laboratory/Imaging


Documentation of biochemical hyperandrogenemia
Total testosterone and sex hormone-binding globulin or Bioavailable and free testosterone

Exclusion of other causes of hyperandrogenism


TSH Prolactin 17-hydroxyprogesterone (non-classical congenital adrenal hyperplasia due to 21 hydroxylase deficiency)
Random normal level < 4 ng/mL or Morning fasting level < 2 ng/mL

Consider screening for Cushing syndrome and other rare disorders such as acromegaly.

Transvaginal pelvic ultrasound

Diagnostic Evaluation of Abnormal Uterine Bleeding


Imaging Evaluation
Saline infusion sonohysterography (SH)
Superior to TVUS in the detection of intercavitary lesions (eg, polyps, submucosal leiomyomas Can distinguish between focal versus uniform thickening of the endometrium and structural abnormalities

Transvaginal ultrasonography (TVUS)


Helpful for evaluating the myometrium itself Sensitivity and specificity for evaluating intracavitary pathology are low.

Diagnostic hysteroscopy (DH)


Numerous recent studies have demonstrated that DH had a significantly better diagnostic performance than SH and TVUS and was significantly more precise in the diagnosis of intracavitary masses. Hysteroscopy not only has increased accuracy for identifying the etiology of AUB, compared with D&C, but also offers the possibility of in-office use.

MRI
May be useful to guide the treatment of myomas

Suggested Evaluation Laboratory


Evaluation for metabolic abnormalities
2-hour glucose tolerance test

Fasting lipid and lipoprotein level

Hyperinsulinemia
Insulin sensitivity DECREASES. Insulin release and circulating insulin INCREASE; normal glucose tolerance unless there is metabolic syndrome present. 45% of PCOS patients will have impaired glucose tolerance of type 2 diabetes. Practical world: fasting glucose, Hgb A1c

Insulin Resistance
Insulin resistance stimulates ovarian androgen production leading to anovulation.
Prolonged anovulation can lead to development of enlarged ovaries with multiple cysts that were first seen on US, thus the name of the syndrome.

Hyperinsulinemia and hyperandrogenemia interfere with the secretion of gonadotropins from the pituitary gland, resulting in changes to the mid-cycle LH surge and its diurnal variation.

Management of PCOS
Oligomenorrhea and amenorrhea
OCPs# (combination pill, low dose) Monthly progesterone

Hirsutism
OCPs Spironolactone* Finasteride

Insulin resistance Metformin


# Cochrane 2007. * Farquhar et al. Cochrane 2004. ^ Farquhar et al. Cochrane 2005.

Infertility
Clomiphene^ Metformin ? Pioglitazone

Key Points

ACOG Practice Bulletin 108: Polycystic Ovary Syndrome. Obstet Gynecol. 2009;114(4):936-949. Radosh L. Drug treatments for PCOS. Am Fam Physician. 2009;79(8):671-676.

Improving insulin sensitivity with insulinsensitizing drugs is associated with decrease in circulating androgen levels, improved ovulation rate, and improved glucose tolerance. Metformin or clomiphene alone or in combination are first-line treatment for ovulation induction.
No success add exogenous gonadotropins or laparoscopic ovarian surgery.

Perimenopausal Bleeding (Endometrial Biopsy First)


Progestins Provera 5-10 mg for 12 days/month
Prevents endometrial hyperplasia

OCPs (agent of choice if nonsmoker expert opinion)


Regulate cycles and control bleeding, contraception

Levonorgestrel IUD (Mirena)


Induces amenorrhea, may cause atrophy

HRT sequential more effective than continuous


Prevents hyperplasia but NO contraception

Triage Guidelines
Reproductive Age Women No cytologic atypia
Simple EMHP with abnormal bleeding
Progestin withdrawal for 6 months, then rebiopsy

Complex (adenomatous) EMHP


Progestin withdrawal, then re-biopsy

Cytologic atypia
High-dose progestins, Megace, or DepoProvera for 3 months, then re-biopsy

What Are We Looking For on the Biopsy?


Cytologic atypia is the SINGLE most important histologic finding. Only ATYPICAL hyperplasia has a significant risk of developing into endometrial cancer.
29% progresses to invasion. Need to rule out cancer if atypia is present.

Endometrial hyperplasia is a BENIGN condition, not a cancer precursor.

Transvaginal Ultrasound?
Saline infusion sonohysterography is more sensitive and specific for the detection of endometrial abnormalities than transvaginal ultrasonography (SOR C).

5. A 55-yo postmenopausal woman presents with two days of vaginal bleeding (spotting). She initiated hormone replacement therapy 10 months ago because of significant nocturnal hot flashes. Which of the following statements is most accurate? A. Irregular bleeding is uncommon after HRT is initiated. B. Postmenopausal women on hormone therapy for > 4 months who experience bleeding require prompt evaluation.

C. Postmenopausal women on HRT for <12 months who experience bleeding may be observed for one year before diagnosing abnormal uterine bleeding D. The sensitivity of endometrial biopsy for the detection of endometrial abnormalities is 50%

Postmenopausal Bleeding
Irregular bleeding is common after HRT is initiated and improves within 6-12 months for most women.
Evaluate Cyclic HRT, experience unusually prolonged or heavy bleeding that occurs near the end of the progestogen phase of the cycle, or breakthrough bleeding that occurs at any other time. Continuous HRT, experience bleeding that persists > 6-12 months or that occurs after amenorrhea has been established. HRT < 12 months may be observed for 1 year before diagnosing abnormal uterine bleeding. Postmenopausal on no HRT or HRT > 12 months with bleeding

Practice Recommendations
Screen all women with postmenopausal vaginal bleeding for endometrial cancer [SOR:A]. Use transvaginal ultrasound for the initial study for patients at low risk for endometrial cancer, and endometrial biopsy for those at higher risk [SOR:B]. Use saline infusion sonography as a second step in the evaluation of postmenopausal bleeding if the diagnosis remains unclear after a biopsy or the bleeding persists despite a normal initial workup [SOR:B].

SUMMARY

Uterine evaluation

Enhanced risk of hyperplasia or neoplasia or both

Enhanced risk of a structural abnormality

Yes

No

Yes

Office endometrial biopsy Yes Adequate specimen? No

TVUS

Normal cavity?

No Yes
Hysteroscopy +/ biopsy

or

SIS

Atypical hyperplasia / CA

No

AUB-E or O (presumptive)

Target lesion? No Yes


Cant assess

Yes Consider MRI Management of AUB-M AUB-L, AUB-P, AUB-A

Pharmacologic Treatment of Abnormal Uterine Bleeding

Summary
Type/Medication
Anovulatory bleeding Combination contraceptives MPA (Provera) Endometria hyperplasia without atypia MPA (Provera) Megestrol (Megace) Levonorgestrel IUS Ovulatory Bleeding Levonorgestrel IUS MPA (Provera) NSAIDS Tranexamic acid (Lysteda)

SORT Key Recommendations for Practice Summary


Adolescents with excessive uterine bleeding should be evaluated for bleeding disorders, such as von Willebrand disease (SOR C). Saline infusion sonohysterography is more sensitive and specific for the detection of endometrial abnormalities than transvaginal ultrasonography (SOR C). The levonorgestrel-releasing intrauterine system (Mirena) is an effective treatment for menorrhagia, with patient satisfaction scores similar to endometrial ablation and hysterectomy (SOR A). NSAIDs are effective in reducing heavy menstrual blood flow. There is no evidence that one NSAID is more effective than another (SOR B).

6. Which of the following is a true statement regarding natural menopause?


A. An FSH > 20 IU/mL is the most accurate biologic marker for menopause. B. Postmenopause is defined as the period after menopause and begins following 6 months of spontaneous amenorrhea. C. There appear to be no cultural differences in the reporting or experiencing of hot flashes. D. Healthy women > 40 years of age who are nonsmokers can safely use combination estrogen-progestin contraceptives.

Vasomotor Instability Hot Flashes


Probably hypothalamic origin
Menopause Thyroid disease Panic or anxiety disorder Insulinoma Autoimmune disorders Pheochromocytoma Carcinoid syndrome Tamoxifen and raloxifene

Influences on Hot Flashes


Cultural
More prevalent in African American and Latin American women than in white women Less common in Chinese and Japanese women

Other variables associated with increased reporting of hot flashes


Cigarette smoking Potential risk factors with inconsistent association Maternal history Early age of menarche and menopause onset History of irregular menses Higher BMI Alcohol use Hot/humid weather

Treatment of Hot Flashes


All oral, transdermal, topical gel, and emulsions estrogen/testosterone preparations are FDA-approved.
Organization
North American Menopause Society (NAMS) 2007 ACOG Cochrane 2006 AHRQ March 2005

Recommendation
Treatment of moderate to severe hot flashes is the primary indication for oral estrogen therapy. Primary indication for hormone replacement therapy (HRT) HRT significantly reduces the frequency and severity (75%) of hot flashes. Studies on reduction of vasomotor symptoms are conclusive ONLY for estrogen.

Hot Flashes: Other Hormonal Therapies


OCPs Androgen-estrogen therapy (Estratest) Custom hormone preparations
(Compounding pharmacist)

Highly effective Known contraindications Approved for treatment of moderate to severe flashes not improved by estrogen alone Data are lacking. Not adequately studied for any indication No data they are safer than conventional therapy

Atrophy of Vagina (Dryness)


Suckling et al. Cochrane 2006 Local estrogens equally effective Vaginal estradiol ring (worn 90 days) preferred Vaginal estradiol tablets Vaginal cream (Premarin)
More uterine bleeding, breast pain than tablets Significantly more endometrial stimulation than estradiol ring

7. Which of the following is true regarding combination estrogen-progesterone hormone replacement therapy (HRT)?
A. B. HRT reduces the risk of osteoporosis. HRT increases the risk of colon cancer. C. HRT decreases the risk of breast cancer. D. HRT has no effect on the risk of stroke.

HRT Womens Health Initiative Study*


Proven benefits
Reduced risk of osteoporosis and related fractures (34%) Decreased colon cancer risk (37%) Improvement of vasomotor symptoms
* Writing Group for the Womens Health Initiative. JAMA. 2002;288:321-333.

Increases the risk of the following:


Breast cancer (26%) CVA (41%) MI (29%) Venous thromboembolic events*

Previous thromboembolic disease is the only ABSOLUTE contraindication to HRT. Heart disease, breast cancer, and endometrial cancer are RELATIVE contraindications.

USPSTF 2005: Chemoprevention for Combination HRT in Postmenopausal Women


Recommends AGAINST routine use of combined HRT for prevention of chronic conditions in postmenopausal women

Androgen Therapy
The FDA has not approved any use of androgens alone for symptoms that may be attributable to androgen deficiency (which may also be attributable to estrogen deficiency).
Low libido Decreased sexual response Decreased sense of well-being Poor concentration Fatigue

Use of androgens is considered off-label. Adverse effects with use at supraphysiological levels
Acne Hirsutism HDL

Testosterone Therapy Cochrane 2006


Addition of testosterone to HRT regimens improved sexual function scores in postmenopausal women.
Significant adverse effect: HDL reduction

Conclusion: Limited number of trials cannot conclude efficacy and safety

8. A 52-yo patient currently takes no prescribed or over-the-counter medications and declines estrogen replacement therapy. Which one of the following would be most effective for relieving this patients menopausal symptoms?

A. B. C. D.

Venlafaxine (Effexor) Black cohosh Soy protein Vitamin E

Hot Flashes Other Therapies


Significant reduction Gabapentin 100 mg/d Clonidine 0.1 mg/d Venlafaxine (SSRI) 37.5-75 mg/d Some reduction Methyldopa 500 mg/d (frequent side effects) No better or only slightly better than placebo Vitamin E Soy protein Red clover Black cohosh

Renal Disease 1

Gary Levine, MD, FAAFP Associate Professor Brody School of Medicine at East Carolina University Greenville, NC

Faculty Disclaimer
Gary Levine, MD, returned disclosures indicating that indicating that he has no affiliation or financial interest in any organization(s).

The AAFP has selected all faculty appearing in this program. It is the policy of the AAFP that all CME planning committees, faculty, authors, editors, and staff disclose relationships with commercial entities upon nomination or invitation of participation. Disclosure documents are reviewed for potential conflicts of interest and, if identified, they are resolved prior to confirmation of participation. Only those participants who had no conflict of interest or who agreed to an identified resolution process prior to their participation were involved in this CME activity.

Special Thanks
Americo D. Fraboni, MD, FAAFP
Assistant Clinical Professor Department of Family Practice & Community Health University of Minnesota Medical School Minneapolis, Minnesota

Learning Objectives
1. Cite the causes and explain the management of acute kidney injury. 2. Discuss the common metabolic issues seen in the critically ill patient as a result of renal compromise. 3. Discuss the principles of neurohormonal antagonism and the role of the kidney in the management of heart failure.

Functions of the Kidney


Maintenance of the extracellular fluid environment
Excretion of metabolic waste products
Urea Creatinine Uric acid

Water balance Electrolyte balance Acid-base balance

Functions of the Kidney


Hormone secretion
Regulation of systemic and renal hemodynamics
Renin Angiotensin II Prostaglandins Bradykinin

Red blood cell production


Erythropoietin

Calcium/phosphorus regulation
1,25-dihydroxycholecalciferol; calcitriol

1. The most cost effective test in evaluating renal disease is?


A. B. C. D. Urine analysis Ultrasound 24-hour urine for CrCL Complete metabolic panel

Urinalysis
The most cost-effective test in evaluating renal disease Always perform your own microscopic exam

Urinalysis
If there is a positive dipstick test for blood
But few RBCs
Hemolysis Rhabdomyolysis

Urinary dipstick for protein only measures albumin


Bence Jones protein will be missed Urinary protein also varies with the hydration status of the patient

Urinalysis
Positive leukocyte esterase dipstick test
Usually indicates UTI If WBCs and no bacteria, think urethritis

Large numbers of WBC may not always indicate a UTI


Glomerular disease Interstitial nephritis
NSAIDs

Chronic cystitis

Urinalysis
Hyaline and granular casts can be normal RBC and WBC casts are always abnormal
Glomerulonephritis Pyelonephritis

2. The Cockroft-Gault and MDRD formulas both use age, creatinine, and which of the following variables in determining GFR?
A. B. C. D. Sex Ethnicity Microalbumin Actual body weight

Glomerular Filtration Rate (GFR) Assessment


Two common calculations
Cockroft-Gault estimated CrCl Modified MDRD (Modification of Diet in Renal Disease) formula

24-hour urine creatinine clearance

Formulas to Assess Renal Function


Cockcroft-Gault (really CrCl, not GFR)
eCrCl (cc/min) = (140 - age yrs) x IBW kg/SeCr mg/dl x 72) x (0.85 if female)

MDRD
eGFR = 186 x (SeCr mg/dl) - 1.154 x (age yrs) 0.203 x (0.742 if female) x (1.212 if African Am)

Acute Kidney Injury


Creatinine is a metabolic waste product excreted by the kidneys
Filtered through the glomerulus into the tubules then excreted It is also secreted by tubular cells
Certain medications can inhibit tubular secretion and falsely elevate the serum creatinine level: trimethoprim, sulfamethoxazole, cimetidine

Protein Excretion
24-hour urine collection
Random spot urine protein excretion
Normal
< 150 mg/24 hour in the non-pregnant patient < 300 mg/24 hour in the pregnant patient 3 grams/24 hr = nephrotic syndrome

Urine microalbumin/creat
< 30 mg/g normal 30-300 mg/g - microalbuminuria > 300 mg/g - macroalbuminuria

Acute Kidney Injury (AKI)

3. An 70 y/o, 70 kg man is admitted to the hospital with pyelonephritis. His admission creatinine is 1.5 mg/dL. The following morning, his creatinine is 2.8 mg/dL. He has voided 300 ml of urine over the past 10 hrs. According to KDIGO criteria, he has which one of the following: A. B. C. D. E. Stage 1 AKI Stage 2 AKI Stage 3 AKI Stage 4 AKI Unstageable AKI

Acute Kidney Injury


Acute kidney injury
Historically referred to as acute renal failure Early recognition facilitates interventions
Prevent worsening Prevent multi-organ system failure (SOR C)

Acute Kidney Injury


Prevalence in US
1% (community-acquired) Up to 7.1% (hospital-acquired) of all hospital admissions Non-ICU mortality rate is ~10%

Affects 15-20% of pts in ICUs


Reported mortality rates > 50%; up to 80% if renal replacement therapy (RRT) or dialysis required

Most common causes of death are


Infectious complications Cardiorespiratory complications

Acute Kidney Injury


Dx based on
Changes in GFR Changes in serum creatinine Urine output Need for renal replacement therapy (dialysis)

Acute Kidney Injury


KDIGO criteria for AKI
Kidney Disease: Improving Global Outcomes (KDIGO)

Increase in serum creat of 0.3 mg/dL) within 48 hours OR Increase in serum creat of 1.5 times baseline within the prior 7 days OR Urine volume < 0.5 mL/kg per hour for more than 6 hours

Acute Kidney Injury


KDIGO severity of AKI
Stage 1 AKI
(1.5 - 1.9) x baseline increase in serum creat OR 0.3 mg/dL increase in the serum creat OR Urine output < 0.5 mL/kg per hour for 6 to 12 hrs

Stage 2 AKI
(2.0 - 2.9) x baseline increase in the serum creat OR Urine output < 0.5 mL/kg per hour for 12 hours

Acute Kidney Injury


KDIGO severity of AKI
Stage 3 AKI
3 x baseline increase in the serum creat OR Increase in serum creat to 4.0 mg/dL OR Urine output of < 0.3 mL/kg per hour for 24 hours OR Anuria for 12 hours OR Initiation of renal replacement therapy OR Decrease in estimated GFR to < 35 mL/min in children < 18 y/o

4. The most common cause of acute kidney injury is which of the following?
A. B. C. D. Ureteral calculus with hydronephrosis Dehydration Acute tubular necrosis Acute interstitial nephritis

Causes of AKI
Prerenal Intrarenal
Tubular Glomerular Interstitial Vascular
Account for 75% of all AKI

Postrenal

Causes of AKI
Acute tubular necrosis (ATN) - 45% Prerenal disease - 21% Acute on chronic kidney disease - 13%
ATN with prerenal disease

Urinary tract obstruction - 10%


BPH

Glomerulonephritis or vasculitis - 4% Acute interstitial nephritis - 2% Thromboembolic disease - 1%

Causes of AKI Prerenal Azotemia


Actual intravascular volume depletion Diseases that lead to decreases in the effective arterial blood volume Medications that reduce renal perfusion

Prerenal Causes of AKI


Volume depletion
Bleeding Dehydration
Gastrointestinal volume loss Urinary volume loss Cutaneous losses

Prerenal Causes of AKI


Decreased effective renal perfusion
Shock CHF
Effective circulating volume depletion
Cardiorenal syndrome

Cirrhosis
Hepatorenal syndrome

Thromboembolic disease

Prerenal Causes of AKI Medications


NSAIDs
Block cyclo-oxygenase increase thromboxane A2 afferent vasoconstriction decreased glomerular perfusion

ACE inhibitors
Block production of angiotensin II vasodilation of postglomerular efferent vessels decreased glomerular pressure may cause azotemia

5. Which of the following drugs is commonly associated with allergic interstitial nephritis?
A. B. C. D. Tegretol Allopurinol Omeprazole Fluoxetine

Causes of AKI Intrarenal


Tubular/interstitial Glomerular Vascular

Intrarenal Causes Tubular


Injury most often caused by
Ischemia and/or Nephrotoxins

Renal Causes of AKI


Tubulointerstitial diseases (acute)
Acute tubular necrosis (ATN) Acute interstitial nephritis (AIN)
Usually drug-induced

Multiple myeloma Hypercalcemia Tumor lysis syndrome Acute phosphate nephropathy


Following a phosphate-containing bowel preparation prior to colonoscopy or surgery

Intrarenal Causes
Acute tubular necrosis ATN
Initiation phase (initial insult) Maintenance phase (1-2 wks) Recovery phase (marked diuresis and slow return of kidney function) No therapy has been shown to hasten recovery

Intrarenal Causes Acute Interstitial Nephritis (AIN)


Causes
Allergic reaction to a drug Autoimmune diseases Infection Infiltrative diseases

Symptoms
Fever Rash Elevated serum and urine eosinophils

Immediate withdrawal of drug and supportive care are essential


Corticosteroids may be beneficial

Drugs Commonly Associated with AIN


Cephalosporins Ciprofloxacin Penicillin Rifampin

Sulfonamides

Allopurinol

Thiazides Furosemide

Cimetidine

NSAIDs

Phenytoin

Renal Causes of AKI


Tubulointerstitial diseases (chronic)
Polycystic kidney disease Nephrocalcinosis
Hypercalcemia and/or hypercalciuria

Sarcoidosis Sjgren's syndrome Reflux nephropathy


Children/adolescents

Medullary cystic kidney disease


Autosomal dominant

Intrarenal Causes Glomerular


An uncommon cause of AKI Systemic manifestations
Fever Rash Arthritis

Urine findings
RBC casts Hematuria Proteinuria

Renal consult and biopsy may be required

Renal Causes of AKI


Glomerular diseases (nephritic)
Postinfectious glomerulonephritis IgA nephropathy Thin basement membrane disease Hereditary nephritis Henoch-Schnlein purpura Mesangial proliferative glomerulonephritis Rapidly progressive glomerulonephritis Fibrillary glomerulonephritis Membranoproliferative glomerulonephritis Vasculitis
Mixed cryoglobulinemia

Renal Causes of AKI


Glomerular diseases (nephrotic)
Minimal change disease Focal glomerulosclerosis Mesangial proliferative glomerulonephritis Membranous nephropathy Diabetic nephropathy Preeclampsia IgA nephropathy Primary amyloidosis Light chain deposition disease Benign nephrosclerosis Postinfectious glomerulonephritis (later stage)

Intrarenal Causes Vascular


Microvascular
Presents as microangiopathic hemolytic anemia and ARF Secondary to small vessel thrombosis or occlusion

Macrovascular
Renal artery stenosis or thrombosis Atheroembolism secondary to:
Atrial fibrillation Aortic disease Acute dissection

Renal Causes of AKI


Vascular disease (acute)
Vasculitis
Wegener's granulomatosis

Thromboembolic disease Hemolytic-uremic syndrome/thrombotic thrombocytopenic purpura Malignant hypertension Scleroderma

Renal Causes of AKI


Vascular disease (chronic)
Hypertensive nephrosclerosis Renal artery stenosis Atheroembolic disease

Causes of AKI Postrenal


Obstruction of the outflow tracts of the kidneys Most are readily reversible Recovery of renal function is directly proportional to the duration of the obstruction Renal US recommended to assess for hydronephrosis

Postrenal Causes of AKI


Obstructive uropathy
Anatomic abnormalities
Urethral valves Ureterovesical or ureteropelvic junction stenosis

Stricture Renal/ureteral calculi BPH Prostate cancer Retroperitoneal or pelvic neoplasms

6. You are reviewing the lab findings of a 64 y/o male hospitalized with AKI, who has no h/o of any long-term medication use. Renal function has been normal, but now the Cr = 2.8 mg/dL, BUN = 60 mg/dL and FENa = 0.75%, urine sp gr = 1.025, and urine sediment shows only hyaline casts. Based on these findings, which one of the following conditions is most likely?
A. B. C. D. E. Hypovolemia due to vomiting Acute pyelonephritis Interstitial nephritis Hypovolemia due to diuretics Obstruction due to BPH

FENa
Fractional excretion of sodium
FENa = 100 x (U Na x Plasma Creat) / (Plasma Na x U Creat)

FENa interpretation
< 1% = prerenal 1% - 2% = renal > 2% = ATN

FENa
Limitations
May be < 1% if ATN superimposed on chronic prerenal disease (CHF, cirrhosis)
Can occur with certain types of kidney injury including contrast induced nephropathy

May not be < 1% in prerenal states in patients with preexisting CKD Can be > 1-2% if diuretics used even in prerenal Can be < 1% in early obstruction, > 1 with chronic obstruction

Renal Biopsy in AKI


Prerenal and postrenal causes of acute kidney injury have been excluded Cause of intrinsic renal injury is unclear Clinical assessment and laboratory investigations suggest a diagnosis that requires confirmation before disease-specific therapy is instituted
Immunosuppressants

May may need to be performed urgently


Oliguria who have rapidly worsening acute kidney injury, hematuria, with RBC casts.

Drugs Associated with Nephrotoxicity


Analgesics Acetaminophen ASA NSAIDS Antidepressants Amitriptyline Doxepin Fluoxetine Lithium Antihistamines Diphenhydramine, doxylamine Antimicrobials Acyclovir Aminoglycosides Amphotericin B Beta-lactams Antimicrobials (cont) Foscarnet Ganciclovir Pentamidine Quinolones Rifampin Sulfonamides Vancomycin Antiretrovirals Adefovir Cidofovir Tenofovir Indinavir Benzodiazepines Calcineurin inhibitors Cyclosporine Tacrolimus

Cardiovascular agents ACE I ARB Clopidogrel Ticlopidine Statins Chemotherapeutics Carmustine Cisplatin Interferon-alfa Methotrexate Mitomycin-C Contrast dye
Diuretics Loops Thiazides Triamterene

Drugs of abuse (cont) Methadone Methamphetamine Herbals Chinese herbals with aristolochic acid Proton pump inhibitors Lansoprazole Omeprazole Pantoprazole Others Allopurinol Gold therapy Haloperidol Pamidronate Phenytoin Quinine Ranitidine Zoledronate

Drugs of abuse Cocaine Heroin Ketamine

Nephrotoxic Meds
NSAIDs Lithium Antibiotics
Aminoglycosides Vancomycin Amphotericin B Acyclovir

Nephrotoxic Meds
ACE/ARB Diuretics Methotrexate Cisplatin

Other Important Medications in Renal Disease


Metformin
Should not be used if creatinine
1.5 (men) > 1.4 (women)
CC < 60 mL/min

Increased risk of lactic acidosis

Other Important Medications in Renal Disease


Enoxaparin
Dont use if creat > 2 mg/dL

Nitrofurantoin
Needs CC > 60 mL/min for clinical effectiveness & avoid toxicity

Allopurinol
Adjust dose for CC

7. An 81 y/o male is scheduled to have a CT of his abdomen with contrast to assess for a tumor. He has COPD, type II DM, with a serum Cr of 1.5mg/dL (nl = 0.6-1.5)
Which one of the following would decrease the likelihood of contrast-related nephropathy?
A. Oral acetylcysteine BID 24 hr prior to the procedure and the day of it B. Oral prednisone on the morning of the procedure C. Oral enalapril (Vasotec) 24 hrs prior to the procedure D. Use of a hyperosmolar contrast medium

Contrast-Induced Nephropathy
Due to iodinated contrast agents 3rd leading cause of AKI in hospitalized patient
Increases mortality, morbidity, and length of hospitalization

Rapid and often irreversible decline in renal function


Inc in serum creat 0.5 mg/dL Inc > 25% above baseline

Follows a predictable time of onset Potentially preventable

Contrast-Induced Nephropathy
Radiocontrast media
Best avoided in patients with, or at risk for, AKI
CKD DM
Metformin

SS disease Dehydration Elderly

Management of At-Risk Patients BEFORE a Dye Study


Stop all diuretics, ACE-I/ARB, & metformin Isotonic solution IV hydration
Favored over hypotonic solutions & oral hydration

Alkalinize the urine


D5W with 3 amps NaHCO3 1cc/kg/hr at least 4-6 hrs prior to exam 1/4NS with 2 amps NaHCO3 (patients with diabetes) Lower risk of CIN, compared with 600-mg doses (3.5% versus 11%).

Acetylcysteine 1200 mg bid the day before and the day of the exam

8. Systemic manifestations of acute kidney injury include which of the following biochemical disturbances?
A. Hypokalemia B. Metabolic alkalosis C. Peripheral insulin resistance and glucose intolerance D. Decreased BUN

Systemic Manifestations of AKI


Fluid, electrolyte, & serum biochemical disturbances Anuria, oliguria, polyuria/polydipsia Dehydration Gastrointestinal disturbances Anorexia Vomiting and diarrhea Hematological disturbances Platelet function defect/bleeding tendencies Blood loss anemia

Azotemia: increased urea and creatinine


Metabolic acidosis Hyperphosphatemia Hyperkalemia Hypercalcemia/hypocalcemia

Halitosis
Oral ulceration/stomatitis

Lymphopenia
Neutrophilia

Peripheral insulin resistance and glucose intolerance

Gastropathy, gastritis, gastric and duodenal ulceration and bleeding

Systemic Manifestations of AKI


Cardiovascular and pulmonary disturbances Systemic arterial hypertension Uremic pneumonitis Neuromuscular disturbances

Weakness Lethargy Depression Uremic encephalopathy Coma/death

Management of AKI
Patients with AKI generally should be hospitalized
Unless mild and clearly resulting from an easily reversible cause

Close collaboration among primary care physicians, nephrologists, hospitalists, and other subspecialists is essential Management is primarily supportive

Management of AKI
Treat underlying cause if identified
Sepsis, CHF, DKA, other catastrophic illness

Treat reversible causes


Volume depletion

Avoid exposure to aggravating factors


Contrast studies Toxic medications/NSAIDs ACE-inhibitors/ARB (rare)

Treatment of AKI
Assure adequate renal perfusion
Achieve and maintain hemodynamic stability
Goal is mean arterial pressure > 65 mm Hg

Avoid hypovolemia Isotonic solutions are preferred over hyperoncotic solutions


NS > dextrans, hydroxyethyl starch, albumin

Treatment of AKI
Volume overload
Furosemide IV q 6 hrs is the initial Rx
20-100 mg initially If inadequate response after 1 hr, double the dose Repeat process until adequate urine output

Ultra-filtration via dialysis (last resort)

Treatment of AKI
Correct
Electrolyte abnormalities Symptomatic uremia

Prevent complications
Including nutritional deficiencies

Hyperkalemia Rx (Initial Rx)


Calcium
Calcium gluconate 10% solution 10 mL IV
Cardio protective/membrane stabilizer Temporarily reverses the neuromuscular effects of hyperkalemia

Insulin*
10 units IV and glucose 25 gm

Inhaled beta-agonists* Sodium bicarbonate*


3 ampules in 1 L of 5% dextrose
*Temporarily shift K+ intracellularly

Hyperkalemia Rx(K+ Elimination)


Sodium polystyrene sulfonate (Kayexalate)
Orally
25-50 g mixed with 100 mL of 20% sorbitol

Rectally
50 g in 50 mL of 70% sorbitol and 150 mL of tap water

Acidosis
Sodium bicarbonate
(if serum level < 15 mEq/L or pH < 7.2)
Given IV or PO Amount based on Bicarb deficit equation
Bicarb deficit (mEq/L) = 0.4*wt(kg) * (24 pts serum bicarb level)

Arm and Hammer baking soda provides approx 50 mEq of sodium bicarb per rounded tsp

Acute Kidney Injury (AKI)


Nonoliguric renal failure patients fare better than those with oliguria Use of diuretics to stimulate urine output actually increases mortality and does not promote recovery of renal function (SOR B)

Acute Kidney Injury (AKI)Rx


Dopamine (low-dose)
No benefit (SOR A).
Preventing AKI Need for dialysis Reduce hospital or ICU lengths of stay Mortality
Bellomo R, Chapman M, Finfer S, et al. Lancet, 2000.

9. In patients with AKI, urgent dialysis is not indicated in which of the following situations?
A. Hyperkalemia refractory to medical therapy B. Volume overload unresponsive to diuretics C. Metabolic acidosis with pH = 7.25 D. Lithium overdose E. Uremic pericarditis

Acute Kidney Injury


Indications for dialysis in patients with AKI
Metabolic acidosis
pH < 7.1 Administration of bicarbonate is not indicated or effective
Volume overload Ketoacidosis

Uremia
Pericarditis/pleuritis Neuropathy Encephalopathy/altered MS

Acute Kidney Injury


Indications for dialysis in patients with AKI
Fluid overload refractory to diuretics Hyperkalemia
K+ > 6.5 or rapidly rising Refractory to medical therapy

Poisonings and intoxications


Ethylene glycol, lithium

AKIPrevention
Vasopressors are recommended for persistent hypotension despite fluid resuscitation Hepatic failure/cirrhosis
Avoid hypotension/GI bleeding Albumin infusion during large volume paracentesis

AKIPrevention
Cancer chemotherapy
Hydration and allopurinol (Zyloprim) administration a few days before chemotherapy initiation

Avoid nephrotoxic medications Exposure to radiographic contrast agents


Optimize volume status Isotonic normal saline or sodium bicarbonate in high-risk patients who are not at risk of volume overload N-acetylcysteine

AKIPrevention
Rhabdomyolysis
Maintain adequate hydration Alkalinization of the urine with intravenous sodium bicarbonate

Surgery Preop
Adequate volume resuscitation/prevention of hypotension Consider holding ACE/ARB/aldosterone blockers Treat sepsis

AKIPrognosis
Patients with acute kidney injury
More likely to develop chronic kidney disease Higher risk of end-stage renal disease Higher risk of premature death

Neurohormonal Antagonism & The Role of the Kidney in Heart Failure

10. Which of the following statements is true?


A. ACE inhibitors enhance angiotensin 2 vasoconstriction of efferent arterioles B. Angiotensin 2 inhibits secretion of aldosterone and vasopressin C. ACE-I therapy should be stopped if a 20% increase in serum creatinine is noted D. Type 1 cardiorenal syndrome is defined as CHF resulting from AKI

Neurohormonal Renal Effects


Renin
Released by renal juxtaglomerular apparatus In response to
Fall in BP Decreased blood volume Decreased sodium concentration in the distal tubule Sympathetic stimulation

Angiotensin 1
Formed from renin-induced conversion of angiotensinogen

Neurohormonal Renal Effects


Angiotensin 2
Formed from angiotensin 1 via ACE Direct arterial vasoconstriction
Constriction of efferent arterioles

Results in aldosterone secretion from adrenal gland


Sodium retention

Results in vasopressin release from posterior pituitary


H2O retention at distal tubule

http://www.kidney.org/professionals/KDOQI/guidelines_bp/images/figure57.jpg

Renal Autoregulation
Enables the kidney to maintain fairly constant renal blood flow and GFR as mean arterial pressure varies between 80 and 160 mm Hg. Myogenic reflex causes afferent arteriole to constrict or dilate in response to changes in intraluminal pressure. Angiotensin II mediated efferent arteriole constriction provides support for GFR when renal perfusion pressure decreases.

Renal Autoregulation in Chronic Hypertension


As systolic pressure rises above 160 mm Hg, constriction of preglomerular vessels is overcome by high pressure. With progressive renal injury, autoregulation is impaired and intraglomerular (IG) pressure begins to vary directly with systemic pressure. High intraglomerular pressure can lead to injury and rapid loss of renal function.

Wikapedia

Treatment-Induced Decline in Renal Function


ACE-I/ARB dilate efferent arteriole, exaggerating decline in IG pressure. A 20-30% increase in creatinine, which then stabilizes, represents a hemodynamic change, and not a structural change.

1997 study following GFR after initiation of ACE-I


Those with largest initial increase in creatinine had most stable renal function over time Creatinine reversed with discontinuation of drug
Kidney Int 1997;51:793-7.

Treatment-Induced Decline in Renal Function


If creatinine increases by more than 30%, agent (ACE-I or ARB) should be discontinued and other causes of renal dysfunction should be evaluated.

Causes of > 30% ACE-I/ARB-Induced Decline in Renal Function


Bilateral renal-artery obstruction (usually > 70%) Absolute or effective reduction in intravascular volume:
Aggressive diuresis Poor oral intake Moderate-to-severe congestive heart failure

Renal vasoconstriction:
NSAIDs Early sepsis

Heart Disease and Kidney Disease


Acute or chronic dysfunction of the heart or kidneys can induce acute or chronic dysfunction in the other organ. Mortality is increased in patients with heart failure (HF) who have a reduced glomerular filtration rate (GFR)
Mortality increased by approximately 15% for every 10 mL/min reduction in estimated GFR

Patients with chronic kidney disease have an increased risk of both atherosclerotic cardiovascular disease and heart failure Cardiovascular disease is responsible for up to 50 percent of deaths in patients with renal failure

Cardiorenal Syndrome (CRS)


National Heart, Lung, and Blood Institute definition Type 1 (acute)
Acute HF results in acute kidney injury (AKI).

Type 2
Chronic cardiac dysfunction (chronic CHF) causes progressive chronic kidney disease (CKD)

Cardiorenal Syndrome (CRS)


Type 3
Abrupt and primary worsening of kidney function causes acute cardiac dysfunction Renal ischemia or glomerulonephritis May be manifested by CHF

Type 4
Primary CKD contributes to cardiac dysfunction Manifested by coronary disease, HF, or arrhythmia.

Type 5 (secondary)
Acute or chronic systemic disorders cause both cardiac and renal dysfunction Sepsis, diabetes

Cardiorenal syndrome (CRS)


Possible mechanisms by which acute HF leads to worsening kidney function (type 1 & 2 CRS)
Neurohumoral adaptations - result in vasoconstriction
Activation of the sympathetic nervous system Renin-angiotensin-aldosterone system Release of vasopressin

Reduced renal perfusion Increased renal venous pressure


Reduces GFR

Right ventricular dysfunction

2014 UpToDate

ACE-Inhibitors
Block the conversion of angiotensin I to angiotensin II Lower arteriolar resistance Increase venous capacity Decrease cardiac output/stroke work Increase sodium excretion Increase levels of renin, angiotensin 1, and bradykinin

http://www.kidney.org/professionals/KDOQI/guidelines_bp/images/figure57.jpg

ACE-Inhibitors
Cause a central enhancement of parasympathetic activity
Break cycle of sympathetic system activation Reduce plasma norepinephrine levels in CHF
May reduce the prevalence of malignant cardiac arrhythmias Reduction in sudden death Interrupt the downward spiral in cardiac function in congestive heart failure

Cardiorenal Syndrome (CRS)


Possible mechanisms by which AKI & CKD lead to worsening cardiac function (type 3 & 4 CRS)
Volume overload Hypertension Acidosis Electrolyte abnormalities Anemia

Answers
1. 2. 3. 4. 5. 6. 7. 8. 9. 10. A A A C B A A B C D

Renal Disease 1
Gary Levine, MD, FAAFP Associate Professor Brody School of Medicine at East Carolina University Greenville, NC

Common Issues in the Elderly: Part 1


Laurence Robbins, MD Associate Professor of Medicine University of Colorado School of Medicine Denver, Colorado

Disclosure
Dr Robbins has nothing to disclose.

It is the policy of the AAFP that all individuals in a position to control content disclose any relationships with commercial interests upon nomination/invitation of participation. Disclosure documents are reviewed for potential conflicts of interest. If conflicts are identified, they are resolved prior to confirmation of participation. Only participants who have no conflict of interest or who agree to an identified resolution process prior to their participation were involved in this CME activity.

Learning Objectives
Identify key concepts in systems of care for the elderly. Identify common gait disorders in the elderly including Parkinsons Disease. Manage common geriatric syndromes.

Case Discussion
1. A 72-yo woman seeks your advice. Her daughter has encouraged her to transfer the title for her home to her children to protect it for her family if she were to need nursing home care. She wonders if this is really necessary because she has Medigap insurance. She also says that if she ever needed to live permanently in a nursing home, she would not want to be kept alive and she understands that Medicare would cover her hospice care.

Question 1
A. Shes correct, Medicare hospice would pay for her nursing home care. B. Her Medicare C (Medigap) will cover most of the cost of nursing home care. C. Putting the home in a trust will protect it for her family in two years D. If her husband is still alive, he could stay in the home if she needed nursing home care under Medicaid.

Medicare will only pay for skilled, rehabilitative care in nursing homes (20 days after at least a three-day hospital stay; patient pays a co-payment of $152 per day for day 21-100 of skilled care) Medicare hospice benefit is for home hospice and does not include cost of inpatient care. Custodial nursing home care is paid for primarily out of patients pocket. Medicaid only pays after spending down.

Surviving spouse is often allowed to keep the couples home, one car, and a very limited amount of other assets (as little as $3,000). Any asset transfer to family members other than a spouse must occur at least three years prior to the need for nursing home care (five years if in a trust).

2. The largest portion of the Medicare budget is spent on:


A. Rare and expensive surgical procedures B. Hospital costs in the last month of life C. Overhead for intermediaries D. Hemodialysis E. The 20% of Medicare recipients with five or more chronic diseases

Impact of Multiple Chronic Diseases


Nearly half of Medicare enrollees have at least 3 chronic conditions; > 20% have 5 or more. Enrollees with at least 3 chronic conditions account for nearly 90% of Medicares annual budget. Two-thirds of Medicare budget spent on 20% with 5 or more chronic diseases.

Case Discussion
Three months ago, 71-yo complained of increased falls and weak right knee. Chronic severe low back pain. Minimal neck discomfort. Referred to orthopedics. Mental status normal. Cranial nerves normal. Stiff-legged gait. Lower extremity increased tone and hyperreflexia; bilateral lower ext weakness. Position sense impaired.

3. Your next step is:


A. B. C. D. E. Order CT of head. Order TSH, B12, blood sugar. Order MRI of cervical spine. Order MRI of lumbosacral spine. Order EMG of lower extremities.

Neurological Gait Disorders


Peripheral neuropathy: Distal sensory and motor signs only Lumbosacral: Lesion below end of spinal cord (T12) = no upper motor neuron signs Cervical: Upper motor signs; no cranial nerve or gray matter signs (eg, dementia) Brain: cranial nerve and gray matter signs, extrapyramidal signs

End of spinal cord

Weakness (not complete paralysis) of a group of muscles (not a single muscle); minimal muscle atrophy Increased muscle tone (spasticity, rigidity) Hyperreflexia (+/- clonus) Babinski response

Upper Motor Neuron Signs

Cervical Myelopathy
Cervical myelopathy usually due to degenerative spine changes; may have little neck pain and no radicular symptoms. Upper motor neuron signs often present. Paresthesias and loss of position sensation may be caused by cervical myelopathy but may also have peripheral neuropathy.

Surgery for Cervical Myelopathy


Better response to surgery if shorter duration, milder symptoms (better if not walker dependent pre-op)

Management
Image neck (MRI) if candidate for surgery. Check B12, TSH, glucose (since he has signs of posterior column sensory loss).

Case Discussion
74-yo man complains of exertional pain in back of thighs that limits walking; pain is worse walking downhill than uphill; some relief with rest and leaning forward. On exam, normal cranial nerves, DTRs, and upper extremity strength; mild weakness of quadriceps bilaterally; labs including alk phos and PSA are normal.

4. The next step is:


A. B. C. D. Order CT of head. Order CT of spine. Order CT of lumbosacral spine. Prescribe analgesic, order physical therapy. E. Order EMG of lower extremities.

Surgical vs Nonsurgical Therapy for Lumbar Spinal Stenosis


(Weinstein JN. N Engl J Med. 2008;358:794-810.)

One of three randomized to surgery didnt get surgery; two of five nonsurgical group had surgery. Benefit of surgery waned over time (2 years). Often patients fear they will get worse without surgery, but the majority of patients in the nonsurgical group showed small improvements in all outcomes.

Case Discussion
An 80-yo man is referred for evaluation of possible depression; is he a Ritalin candidate? Hes accompanied by his wife who describes how much more difficulty ambulating hes had since esophagectomy for cancer 18 months ago. His medications include hydrochlorothiazide, lisinopril, metoclopramide, valproic acid, and prochlorperazine prn.

On exam, the patient has a flat affect and blinks little. He has severe seborrhea. He slowly rocks bath and forth in his chair when asked to stand but is unable to propel himself to a standing position. When helped up to a standing position, he has trouble initiating his gait, then takes a few small steps and freezes.

5. Your next step is:

A. Stop the metoclopramide (eg, Reglan). B. Stop the prochlorperazine (Compazine). C. Stop the valproate (Depakote). D. Stop only 1 and 3. E. Stop all 3.

Pearls
Drug-induced parkinsonism can occur with medications not usually considered culprits (metoclopramide, valproic acid, prochlorperazine, etc). Resting tremor, asymmetric rigidity/tremor, and response to levodopa best predict correct diagnosis of PD.

Case Discussion
The same 80-yo patient returns one week later after withdrawal of medications. He now can stand without assistance but still has difficulty initiating his gait and walks with small steps.

6. The best treatment choice now is:


A. B. C. D. E. Entacapone (Comtan) 200 mg qid Pramipexole 0.5 mg tid Selegiline 5 mg bid Amantadine 100 mg bid Carbidopa/levodopa 25/100 tid

When to Start Drug Rx in the Elderly?


Functional decline: dominant side more affected, interference with ADLs and gait. Why delay drug treatment? Medications often associated with side effects in elderly and dont prevent progression. Cost of medication is high.

Medications for PD
Anticholinergics (eg, trihexyphenidyl) Amantadine MAO Inhibitor (selegiline, rasagiline) Carbidopa/L-dopa (Sinemet) Dopamine agonists (eg, pramipexole, ropinirole) COMT inhibitors (eg, entacapone)

Carbidopa/Levodopa (Sinemet)
Most effective med for gait (bradykinesia, rigidity); tremor response variable. Carbidopa prevents peripheral breakdown of levodopa; > 75 mg daily for max effect. Begin 25/100 bid or tid; may use 25/250 as dose increased; avg patient needs about 500 to 1,000 mg L-dopa/day. 50/200 SA reduces dose frequency 1/3. Avoid taking regular with food but eat with SA.

Treating PD in the Elderly


Carbidopa/levodopa is the most effective medication for PD; optimize dose before adding other drugs in the elderly. Anticholinergics and amantadine have little role in treating elderly PD patients. COMT inhibitors are very expensive for modest gain. Dopamine agonist more likely to cause delusions/hallucinations.

Case Discussion
An 87-yo nursing home resident returns from the hospital after treatment for wrist fracture. She completed a course of antibiotics and takes prn oxycodone for pain. She has frequent incontinence of small volumes of liquid stool.

7. Your first step is:


A. Order test for C. difficile. B. Order stool culture. C. Perform hemoccult test. D. Arrange colonoscopy. E. Check for fecal impaction.

Case Discussion
Two days later, the patient develops severe diarrhea and has a temperature of 38.5C (101F). White count is 16,000. Clostridium difficile toxin titre is positive.

8. Your next step is:

A. Loperamide 2 mg after each loose stool. B. Begin vancomycin 125 mg qid. C. Begin metronidazole 500 mg tid. D. Begin metronidazole 250 mg tid. E. Begin ciprofloxacin 250 mg bid.

C. Difficile in the Elderly


Metronidazole for mild to moderate disease; vancomycin for severe Severe disease: 2 or more factors (age > 60, temp > 38.3C, albumin < 2.5, WBC > 15,000, creatinine increase 50%) OR WBC > 15,000 and creatinine 1.5x baseline (no agreement on standard definition of severe disease)

9. The most common cause of uncorrectable visual acuity > 70 years is:
A. B. C. D. E. Macular degeneration Cataracts Glaucoma Diabetic retinopathy Presbyopia

Age-Related Macular Degeneration (ARMD)


Leading cause of new blindness > 55 yo. Macula has highest concentration of photoreceptors in retina. Provides visual acuity and color vision; degeneration leads to loss of central vision (impairs reading, face recognition, driving).

Case Discussion
What causes this type of visual loss?

http://media.photobucket.com/image/glaucoma/ellytza/everything/glaf2.jpg?o=35

10. What causes this type of visual loss?

A. Age-related macular degeneration B. Cataracts C. Glaucoma D. Diabetic retinopathy E. Presbyopia

Glaucoma: Definition
Characteristic optic neuropathy and visual field changes often, but not always, associated with increased intraocular pressure.

11. Presbycusis is typically associated with the reduction in:


A. Hearing threshold at all frequencies B. Hearing threshold at higher frequencies C. Hearing threshold at lower frequencies D. Hearing threshold at midrange frequencies E. Speech discrimination only

Typical Presbycusis: Blue Line

Case Discussion
An 82-yo man develops a pressure ulcer on his left lateral malleolus that is covered by a thick eschar.

12. The most appropriate management is:


A. Apply collagenase with occlusive dressing. B. Surgical debridement. C. Apply wet to dry dressings. D. Apply biofilm. E. Apply platelet-derived growth factor cream.

Case Discussion
A 78-yo woman with history of osteoarthritis comes to your office for four-week history of early morning shoulder and hip discomfort. Labs are normal except for erythrocyte sedimentation rate of 52 mm/h.

13. You recommend the following treatment:


A. Indomethacin 25 mg bid B. Naproxen 250 mg bid C. Prednisone 60 mg daily D. Prednisone 15 mg daily E. Oxycodone 10 mg q 4h prn

Polymyalgia Rheumatica
Rare < 50 yo, avg age 70 Bilateral aching/morning stiffness (> 30 minutes) for at least one month, and involving at least two of the following three areas: neck or torso, shoulders or proximal regions of the arms, and hips or proximal aspects of the thighs Sed rate > 40 or elevated C-reactive protein

Case Discussion
An 84-yo woman has repeated falls due to near syncope in the nursing home, most often when she is returning to her room after lunch or dinner. Her medications include HCTZ 25 mg daily and lisinopril 10 mg daily for hypertension.

14. Your next step is:


A. Stop the hydrochlorothiazide. B. Stop the lisinopril. C. Start fludrocortisone. D. Encourage the staff not to allow her to stand up for 45-60 minutes after a meal. E. Begin offering smaller more frequent meals.

Postprandial Orthostatic Hypotension


Syndrome of orthostatic hypotension occurring 30-45 minutes after a meal. May be ameliorated by caffeine or smaller meal. Prevent by having patient remain seated for 45 minutes after meals and avoiding hypotensive medications at mealtime.

Postprandial hypotension or orthostasis is common in the elderly and should be considered in the differential diagnosis of near syncope or syncope. Caring for patients in nursing homes may require rethinking of conventional wisdom (eg, small frequent meals not feasible).

Answers
1. 2. 3. 4. 5. 6. 7. D E C D E E E 8. B 9. A 10. C 11. B 12. B 13. D 14. D

Common Issues in the Elderly: Part 2


Laurence Robbins, MD Associate Professor of Medicine University of Colorado School of Medicine Denver, Colorado

Disclosure
Dr Robbins has nothing to disclose.

It is the policy of the AAFP that all individuals in a position to control content disclose any relationships with commercial interests upon nomination/invitation of participation. Disclosure documents are reviewed for potential conflicts of interest. If conflicts are identified, they are resolved prior to confirmation of participation. Only participants who have no conflict of interest or who agree to an identified resolution process prior to their participation were involved in this CME activity.

Learning Objectives
Discuss diagnosis and management of osteoporosis. Review essentials of decision-making capacity. Identify key issues in geriatric prevention. Manage urinary incontinence and male sexual dysfunction in the elderly.

Question #1 Which patient is least likely to have idiopathic osteoporosis?


A. Slender, 61-inch-tall 71-yo Caucasian woman with a T12 vertebral fracture B. 86-yo Caucasian man with a hip fracture C. 65-yo thin, alcoholic Caucasian woman who smokes and has T10-T12 vertebral fracture D. 67-yo obese African-American woman with a T4 vertebral fracture

Short stature, slender, Caucasian women are at greater risk for osteoporosis. Risk of osteoporosis in men becomes similar to women in advanced age (>80). Alcohol and cigarette smoking accelerate this risk. Osteoporotic fractures usually begin in vertebrae under greatest anatomical load (T10-T12) and spread caudal and cephalad.

Why Care about Osteoporosis?


In the US, the majority of patients with fragility fractures do not receive osteoporosis therapy. In one study of women >65 yo with recent hip fracture, 13% were receiving adequate treatment for osteoporosis.

Following a Hip Fracture, Remember the Rule of 4ths


Dead
Institutionalized

Alive, no change in function

Home with new functional deficits

Osteoporosis: WHO Definition


T Score
>-1 SD -1 to -2.5 SD <-2.5 SD <-2.5 SD plus one or more fractures Normal Low bone mass (osteopenia) Osteoporosis Severe or established osteoporosis

Bone Mineral Density Test DXA (Dual Energy X-ray Absorptiometry)


The WHO criteria for osteoporosis are based on DXA. Hip DXA is the best predictor of hip fracture, which is the most clinically relevant site of fracture.

Who Needs BMD Testing ?


Patient category
USPSTF NOF/ISCD

Women age 65 yo

Yes

Yes Yes Yes Yes

Women age 50-64 yo based on risk factors


Men age 70 yo Men age 50-69 yo based on risk factors

Yes
No No

Who Needs BMD Testing?


Patient category
USPSTF NOF/ISCD

All men and women No with a fragility fracture Anyone considering No Rx for osteoporosis Anyone receiving Rx for osteoporosis No

Yes

Yes
Yes

Question #2 Which one of the following is the best test for vitamin D deficiency? A. B. C. D. Serum ionized calcium Serum 1,25 dihydroxyvitamin D level Serum 25-hydroxyvitamin D level 24-hour urine for calcium, alpha hydroxylase

Vitamin D Metabolism
D3 (cholecalciferol) and D2 (ergocalciferol) are both biologically active. Both are converted in liver by hydroxylation into 25 OH vit D. 25 OH vit D is converted in kidney to 1,25 di-OH vit D. 25 OH vit D has low biological activity but is the major form in circulation: best marker for nutritional status re: vit D.

Basic Screening for Secondary Causes of Osteoporosis


CBC, albumin/total protein, alkaline phosphatase (myeloma, cancer) Serum creatinine (CKD) Calcium, phosphorus (hyperparathyroid) TSH (hyperthyroidism) Testosterone (men; hypogonadism)

Who Needs Treatment?


Postmenopausal women and men >50 yo Hip or vertebral fracture Prior fractures and low bone mass T score -2.5 after appropriate evaluation for secondary causes Low bone mass (T score -1 to -2.5) if high risk (eg, glucocorticoid use)
2009. National Osteoporosis Foundation.www.nof.org

Question #3 Which of the following is the USPSTF recommended daily dose of vitamin D and calcium for non-institutionalized postmenopausal women to prevent fracture? A. 20 mcg 25-hydroxyvitamin D + 800 mg calcium B. 400 IU/day of Vitamin D3 + 1000 mg calcium C. 800 IU/day of Vitamin D3 + 1200 mg calcium D. None of the above

Vitamin D and Calcium: USPSTF


Recommends against daily supplementation with D3 400 IU or less and calcium 1000 mg or less. Concludes that evidence is insufficient to assess benefits/harms of higher doses. Guideline does not address vit D and calcium supplements for women or men with established osteoporosis.

Vitamin D and Calcium: USPSTF


Vit D supplementation can reduce risk of fracture (institutionalized elderly > community dwelling) dose >800 IU D3. No clear benefit (or harm) in cancer or cardiovascular risk. NOTE: Nearly all studies of osteoporosis Rx (eg, bisphosphonates) include vitamin D/Ca supplementation.

National Osteoporosis Foundation still recommends calcium and vitamin D supplementation (1200 mg calcium, 800-1000 IU vitamin D3) for all postmenopausal women.

Osteoporosis Prevention and Treatment


Everyone should be counseled about: Weight-bearing exercise Calcium and vitamin D(?) Smoking cessation Avoiding excessive alcohol intake Fall prevention Consider hip protectors.

Question #4 Which of the following has been shown to reduce the risk of hip fracture? A. Raloxifene B. Alendronate C. Ibandronate D. Calcitonin E. All of the above

FDA-Approved Treatment
Antiresorptive (Bone Retaining)

Medication
Alendronate Risedronate Zoledronate Ibandronate Raloxifene Calcitonin Teriparatide

Postmenopausal osteoporosis
+ + + + + + +

Men
+ + + +

GIOP*
+ + + +

Anabolic (Bone Forming)

* GIOPGlucocorticoid induced osteoporosis

Bisphosphonates
Given on an empty stomach with full glass of water Contraindications: Esophageal stricture Achalasia Patient unable to stand or sit upright Renal failure with creatinine clearance of <35 ml/minute Hypocalcemia

Bisphosphonates: Side Effects


GI upset Headache Bone pain Flu-like symptoms (with IV administration)

Bisphosphonates: Osteonecrosis of Jaw


Low incidence. Most often with IV bisphosphonates. Mostly in cancer patients. Maintain oral hygiene. Regular dental check-up.

Bisphosphonates: Bone PainFDA alert


Severe, sometimes incapacitating May occur days, months, or years after beginning bisphosphonates Incidence and risk unknown May be relieved after discontinuation

Bisphosphonate and Atrial Fibrillation (FDA Safety Review)


No clear association Data from 19,687 patients 2 events in most studies

Bisphosphonates: Long-Term Use Linked to Atypical Fractures


Low-energy subtrochanteric femoral and pelvic insufficiency fractures. Poor healing. Consider stopping bisphosphonate after 5-10 years.
J Clin Endocrinol Metab April, 2010, 95(4)

FLEX trial: 1100 women, alendronate therapy averaged 5 years; randomized to continue 5 more years or stop Clinical vertebral fracture risk lower for continued treatment, especially in two groups: those without pre-existing fracture & T score <2.5 or pre-existing fracture & T score <2.0 (NNT 21 & 17 respectively) No data for non-vertebral fracture

Bisphosphonate Rx: How Long?

Question #5 Which osteoporosis therapy has an analgesic effect in the setting of acute vertebral fracture? A. Alendronate B. Calcitonin C. Estrogen D. Teriparatide E. Raloxifene

Calcitonin
Calcitonin: Polypeptide hormone Intranasal formulation for vertebral fracture complicated by pain for analgesic benefit. Weak anti-fracture efficacy. Remember to prescribe calcium and vitamin D! Increased risk of cancer with long-term use? Limit to six months duration.

Raloxifene
Selective estrogen receptor modulator (SERM) Estrogen agonist/antagonist Reduces risk of breast cancer Does not stimulate endometrium Hot flashes Has not been shown to reduce hip fracture risk; less potent than bisphosphonates

Teriparatide (PTH) Parathyroid Hormone (1-34)


Daily SQ injections. Reserved for pts with high risk for fractures: Multiple fractures Extremely low BMD eg, < -3 Intolerant/unresponsive to other Rx FDA black box warning: osteosarcoma (animals); thus, safety and efficacy ? > 2 yrs. Consider substituting bisphosphate > 2 yrs.

Denosumab (Prolia)
Monoclonal antibody against RANKL (receptor that stimulates osteoclastic activity). Reduces vertebral, hip, and non-vertebral fractures. No long-term safety data; 1%-2% severe hypocalcemia. q6 month injection (expensive). Role is uncertain.

Question #6
An elderly woman with severe Alzheimers Disease is admitted to your service and deemed mentally incapacitated. In order to make further management decisions: A. Obtain a Power of Attorney B. Create a living will C. A&B D. None of the above

Power of attorney can be assigned only by someone with the capacity to make decisions (thats why they call them advance directives).

Principles of Decision Making for Incompetent Patients


Documented advance directives Substituted judgment: Someone who knows the patient attempts to make decisions in the manner that the patient would. Best interest standard: Decisions based on predicted outcomes that would most likely promote the patients well-being (beneficence).

Question #7 A daughter comes says that she is the patients Power of Attorney (signed by the patient 15 years ago when she was out of the country).
A. A standard POA gives her the authority to make decisions about her mothers care B. The daughter must apply for guardianship C. The POA must be durable for medical care D. A court must declare patient incompetent first

Standard power of attorney is null and void if the individual becomes incapacitated. Durable power of attorney, specifically for medical affairs, allows the named individual to make medical decisions when the grantor is incapacitated.

Question #8 Your demented patient has gangrene involving of her right foot. The surgeons recommend a BKA. You and the surgeon explain the surgery to the patient and believe she understands the pros and cons. The daughter is the DPOA and objects.
A. B. C. D. Proceed with surgery Ask a psychiatrist to evaluate her Call off the surgery Call the patients son to get his permission

Determining capacity: Capacity to make a given medical decision is an isolated measure. If a patient understands the benefits and risks of an intervention and is consistent in her response, then the DPOA has no authority to overrule the patient. Generally a good idea to have two or more physicians independently confirm that the patient understands her decision.

Question #9 Which of the following is true about tube feeding in advanced dementia? A. Reduces risk of aspiration pneumonia. B. Once in place, low risk of complications. C. Most feeding tubes are inserted during acute hospital stay. D. Improves healing of pressure sores. E. Reduces mortality.

No evidence that feeding tubes reduce the risk of aspiration pneumonia, heal pressure wounds, improve nutritional status, or decrease mortality. 2/3 placed during acute hospitalization with little discussion with family. Transfer to ER for tube-related complications common. Hand feeding often acceptable alternative.
http://onlinelibrary.wiley.com/doi/10.1002/14651858.CD007209.pub2/abstract (Cochrane Review 2009)

Case Discussion
A 75-yo woman has a one-month progressive decline in her baseline mental status. She has anorexia, constipation, intermittent nausea, and a 10-lb weight loss. More recently, she became nonverbal and was unable to ambulate at all and was using a wheelchair. She is disoriented and intermittently incontinent of urine.

Her affect is flat. HEENT exam normal. Neck supple, no goiter. Mucous membranes are dry. Coarse breath sounds in all lung fields. T 38.2C, pulse 84/min, respiratory rate 20/min, blood pressure 152/81 mm Hg, and oxygen saturation 98% on room air.

Question #10 What test is most likely to help with the diagnosis? A. Brain MRI B. Urinalysis C. EKG and troponin D. TSH E. Chest x-ray

Apathetic Hyperthyroidism
Up to one third of elderly patients with hyperthyroidism do not have symptoms of sympathetic overactivity (tachycardia, tremor, nervousness, heat intolerance, increased appetite, more frequent stools, etc). Elderly patients with Graves disease less likely to have goiter. Constipation common and 40% have pulse <100.

Question #11 Which of the following is incorrect for patients > 65 years old according to USPSTF? A. Flu shot annually B. Pneumovax every 5 years C. Pneumovax once after age 65 D. Tdap if first booster for tetanus, then Td only every 10 years

Immunization for Elderly


Per USPSTF, give one pneumococcal vaccine after age 65 (whether or not one received prior to age 65).

Question #12 The USPSTFs stand on screening for abuse of the elderly or vulnerable adult is:
A. B. C. D. E. Screen all patients age > 80. Screen all vulnerable patients age > 65. Screen women age > 85. Screen patients with dementia age > 70. Insufficient evidence to recommend screening.

Elder abuse and neglect not well studied; little data to identify best screening tool or effectiveness of screening. Despite lack of evidence for screening, most states have statutes protecting elders from abuse or neglect (including self-neglect). Adult protective services (APS) protects community dwelling; long-term care ombudsman programs (LTCOP) focus on nursing homes, assisted living, personal care homes.

Identifying At-Risk Elders


Cognitively impaired women age > 80 at greatest risk. Sedation (overmedication), skin tears, dehydration (Na > 147), malnutrition, fractures, pressure sores may be clues. AMA recommended screening geriatric patients for abuse in all practice settings.

Screening Tools
Interview elderly patients by themselves. Ask about family composition and living arrangements. Ask directly about abuse, neglect, or exploitation. 1. Do you feel safe where you live? 2. Who prepares your meals? 3. Who handles your checkbook?

Question #13 The most common cause of lack of sexual activity in older heterosexual couples is: A. Female partners lack of interest B. Female partners medical condition C. Male partners lack of interest D. Male partners medical condition E. A and D

Reason for Cessation of Sexual Intercourse (2000 Interview Cohort)


No partner Partner's illness, loss of desire or capability Own illness, loss of desire or capability 70 60 50 40 30 20 10 0

Percent

Women (n=146)

Men (n=67)

N Beckman et al., BMJ, 2008; 337:a279.

Question #14 The most common reason for cessation of sexual intercourse reported by men:

A. Lack of interest B. Erectile dysfunction C. Performance anxiety D. Premature ejaculation E. Anorgasmia

NSHAP: Sexual Problems in Men


Report problem Bothered by problem

100 80

Percent

60 40 20 0
Erectile Difficulty Climax too soon Anorgasmia

ST Lindau et al., NEJM, 2007; 357: 762-74.

Question #15 What are the physical demands of sexual intercourse for men?
A. HR & BP response = Bruce stage 1 (5 METs) B. HR & BP response = Bruce stage 2 (7 METs) C. HR & BP response = Bruce stage 3 (10 METs) D. HR & BP response = Bruce stage 4 (13 METs)

Physical Demands of Sex


(Palmieri et al. Am J Cardiol. 2007;100:1795-1801)

19 men (55 +/- 8 yrs) treadmill exercise (Bruce protocol) vs sexual intercourse HR & BP response with sex = Bruce stage II (7 METs or 75% of max exercise HR & BP: 6 min, 12% grade, 2.5 mph) Exercise duration predicted intercourse duration (each minute on treadmill added one minute to duration of sex)

Impairments in Physical Activity Non-institutionalized Adults 70 Yrs Old


Women 20% 15% Men

Percent unable to perform the activity

10% 5% 0% Walk 1/4 mile Climb 1 flight Stoop, crouch, kneel

Sexual intercourse with a known partner in familiar place = the ability to climb 1-2 flights of stairs

Question #16 No USPSTF guideline re: screening for urinary incontinence. However, reasons for screening include: A. Effectiveness of pelvic floor exercise and biofeedback for women B. Effectiveness of pelvic floor exercise and biofeedback for men after prostate surgery C. Modest benefit of drug treatments D. All of the above

Question #17 The most common cause of urinary incontinence in women is:

A. B. C. D.

Urge Stress Overflow Functional

Urge Incontinence
Most common type of urinary incontinence Signs and symptoms: Abrupt urgency Frequency Nocturia Volume of leakage may be large or small

Bladder Filling Physiology: Urge


Overactive
Normal

mm Hg

1 5

100

200

300

400

500

600

Question #18 The most effective drug for urge incontinence is:

A. B. C. D. E.

Oxybutynin. Tolterodine. Darifenacin (Enablex). Trospium (Sanctura). All are equally effective.

Urge incontinence meds more effective when combined with behavioral therapy.
Usually do not ablate detrusor overactivity. Efficacy similar; differ by side effects, cost.

Lack of response to one agent does not preclude response to another. In men, check PVR before starting antimuscarinic medication to avoid making urinary retention worse.

Question #19 An 80-yo complains of discomfort in his legs at night when trying to sleep, relieved by standing and walking. His only medication is prophylactic aspirin. The test most likely to reveal a treatable cause of his symptoms is: A. Serum ferritin B. Serum calcium C. TSH D. CPK

Restless Legs Syndrome


Marked disagreeable discomfort in the lower extremities that occurs only at rest and is immediately relieved by movement 20% of pts > 80 yo; sleep disturbance freq. Check serum ferritin; if low, give 2-month trial of iron replacement. RX: Pramipexole 0.125 mg, Sinemet CR 50/200, clonazepam 0.5-1.0 mg, or oxycodone 5 to 10 mg hs.

Answers
1. D 2. C 3. D 4. B 5. B 6. B 7. B 8. A 9. C 10. D 11.B 12. E 13. D 14. B 15. B 16. D 17. A 18. E 19. A

Renal Disease 2
Gary Levine, MD, FAAFP Associate Professor Brody School of Medicine at East Carolina University Greenville, NC

Faculty Disclaimer
Gary Levine, MD, returned disclosures indicating that indicating that he has no affiliation or financial interest in any organization(s).

The AAFP has selected all faculty appearing in this program. It is the policy of the AAFP that all CME planning committees, faculty, authors, editors, and staff disclose relationships with commercial entities upon nomination or invitation of participation. Disclosure documents are reviewed for potential conflicts of interest and, if identified, they are resolved prior to confirmation of participation. Only those participants who had no conflict of interest or who agreed to an identified resolution process prior to their participation were involved in this CME activity.

Special Thanks
Americo D. Fraboni, MD, FAAFP
Assistant Clinical Professor Department of Family Practice & Community Health University of Minnesota Medical School Minneapolis, Minnesota

Learning Objectives
1. State the major points of the National Kidney Foundation Kidney Disease Outcomes Quality Initiative (NKF KDOQI) for chronic kidney disease (CKD). 2. Accurately identify, screen, evaluate, and classify patients who are at risk of, or have the diagnosis of, CKD. 3. Reduce the risk for progression of CKD to endstage renal disease (ESRD) by applying appropriate, proven therapeutic interventions early in the disease process. 4. Describe the management strategies for anemia, bone disease, malnutrition, and electrolyte abnormalities in the later stages of CKD.

Chronic Kidney Disease (CKD)

KDOQI Guidelines
KDOQI Clinical Practice Guideline for Diabetes and CKD: 2012 Update

CKD
27 million adults in the United States 13% of adult Americans 44% of persons > 65 y/o > 500,000 Americans were treated for end-stage renal disease in 2007 Incidence and prevalence of CKD among US adults have increased dramatically since 1991 Associated with increased mortality, morbidity, and health care costs Patients with CKD Have significantly increased risks of cardiovascular disease and stroke Are more likely to die of cardiovascular disease than to require dialysis

CKDRisk Factors
Autoimmune disease Diabetes Exposure to certain chemicals and environmental conditions
Lead, cadmium, arsenic, mercury, uranium

Exposure to certain drugs Family history of CKD Hypertension Low birth weight Low income or education Lower urinary tract obstruction

CKDRisk Factors
Minority status (eg, blacks, American Indians, Asians, Pacific Islanders) Neoplasia Nephrolithiasis Older age Recovery from acute kidney injury Reduction in kidney mass Systemic infections Urinary tract infections

CKDEtiology
Diabetic kidney disease
Type 2 diabetes Type 1 diabetes 33% 6%

CKDEtiology
Nondiabetic kidney disease Vascular diseases
Hypertension, ischemic renal disease

25% 18%

Glomerular diseases

Primary: lupus nephritis, vasculitis, membranous nephropathy, minimal change disease, focal segmental glomerulosclerosis, immunoglobulin A nephropathy Secondary: infections (eg, hepatitis B and C, human immunodeficiency virusassociated bacterial endocarditis), amyloidosis, heroin use, malignancy (eg, leukemia, Hodgkin lymphoma, carcinoma)

Cystic diseases
Polycystic kidney disease

7% 4%

Tubulointerstitial disease

Urinary tract infections, nephrolithiasis, obstruction, sarcoidosis, multiple myeloma, drug toxicity (eg, proton pump inhibitors, lithium, nonsteroidal anti-inflammatory drugs)

Causes of CKD that Lead to ESRD and Transplant


Chronic glomerulonephritis Diabetic nephropathy HTN nephropathy
~ 25% of cases

Polycystic kidney disease Chronic pyelonephritis Renal calculi

11. Stage 4 chronic kidney disease is defined as an estimated GFR of?


A. B. C. D. E. > 90 60-90 30-60 15-30 < 15

Definition of CKD
Kidney damage for > 3 mo
Structural or functional abnormalities
With or without decreased GFR
Pathological abnormalities or Markers of kidney damage, including abnormal blood, urine, or imaging tests

GFR < 60mL/min/1.73 for > 3 mo


With or without kidney damage

Chronic Kidney DiseaseStages


Stage 1
Normal GFR > 90 mL/min and persistent albuminuria

Stage 2
GFR between 60 to 89 mL/min and persistent albuminuria

Stage 3
3aGFR between 45 and 59 mL/min 3bGFR between 30 and 44 mL/min

Stage 4
GFR between 15 and 29 mL/min

Stage 5 = end-stage renal disease (ESRD)


GFR < 15 mL/min

Chronic Kidney DiseaseAlbuminuria


A1
< 30 mg/day
Normal to mildly increased

A2
30 to 300 mg/day
Moderately increased

A3
> 300 mg/day
Severely increased

Chronic Kidney Disease

Management by Stage
Stage I GFR > 90
Dx and Tx of comorbid conditions, slowing progression, CV risk reduction

Stage II GFR 60-90

Estimating progression
Evaluating and Tx of complications Prep for kidney replacement, Nephrology referral Replacement/dialysis

Stage III GFR 30-60 Stage IV GFR 15-30 Stage V GFR < 15 (ESRD)

Chronic Kidney Disease


Evaluation should include:
Diagnosis (type of renal disease) Comorbid conditions Severity, assessed by level of kidney function Complications, related to level of kidney function Risk for loss of kidney function Risk for cardiovascular disease

Chronic Kidney Disease


Evaluation of CKD
Laboratory evaluation Radiology evaluation GFR assessment Protein excretion Kidney biopsy

Laboratory Evaluation for CKD


UA with microscopic exam CBC CMP Calcium & phosphorous level Uric acid level 24-hour urine CrCl or estimated GFR 24-hour urine or random protein excretion

Laboratory Evaluation for CKD


Serum protein electrophoresis (SPEP) HepBsAg, Hepatitis C antibody; HIV ANA C3, C4, & CH50 ANCA Anti-GBM antibody

Radiology Evaluation for CKD


Renal ultrasound CT scan of the kidneys and liver MRI of the kidneys Renal angiogram Voiding cystourethrogram

12. Nephrogenic systemic fibrosis most often occurs as the result of which of the following?
A. Chemotherapy with alkylating agent B. RBC transfusion C. Platelet transfusion D. Peritoneal dialysis E. MRI with gadolinium contrast

Nephrogenic Systemic Fibrosis (NSF)


Patients with stage 4-5 CKD (GFR < 30 mL/min) are at risk of developing NSF if gadolinium (Gd3+)-based contrast agents (Gd-CA) are used for MR imaging Risk may be as high as 18% in some some subgroups

Gd3+ attaches to endogenous anions (phosphate, hydroxide, carbonate, citrate), forming insoluble salts that deposit in tissues and initiate fibrosis

Nephrogenic Systemic Fibrosis (NSF)


NSF is characterized by:
Discolored skin plaques
Can be itchy and painful

Contractures of joints
Potential for complete loss of range of motion

Potentially widespread fibrosis


Liver, lungs, muscles, and heart

NSF may be more likely in patients with recent inflammatory event


Surgical intervention, infection, or thrombotic event

Indications for a Renal Biopsy


Hematuria with a low GFR or proteinuria Nephrotic range proteinuria CKD of unknown cause
Normal or large kidneys on US

AKI of unknown cause Patient wants/needs to know

Contraindications to Renal Biopsy


Uncorrectable bleeding tendency Small kidneys < 9 cm Single (functioning) kidney Severe HTN Multiple large cysts Hydronephrosis Active infection Uncooperative patient Unwilling to give consent

Monitoring CKD
eGFR should be obtained at least yearly More often in patients with: GFR < 60 mL/min/1.73 m2 Fast GFR decline in the past Risk factors for faster progression Ongoing treatment to slow progression Exposure to risk factors for acute GFR decline

Chronic Kidney Disease Treatment to Include:


Specific therapy, based on diagnosis Evaluation & management of comorbid conditions Slowing the loss of kidney function Prevention & treatment of
Cardiovascular disease Complications of decreased kidney function

Preparation for ESRD and kidney replacement Rx Replacement of ESRD by dialysis and transplantation, if signs and Sxs of uremia are present Self-management behaviors should be incorporated into the Tx plan at all stages of CKD

Chronic Kidney Disease


Self-management strategies should be key components of a multifaceted treatment plan with attention to multiple behaviors: (C)
Monitoring and treatment of hyperglycemia Blood pressure Nutrition Smoking cessation Exercise Adherence to medicines

Chronic Kidney Disease


Review of medications at all visits for:
Dosage adjustment based on level of CKD Detection of potentially adverse effects on renal function or complications of CKD Detection of drug interactions Therapeutic drug monitoring, if possible

New CKD Guidelines (2013)


Released 12/28/12 (available January 2013)

CKD classification should be based on cause, GFR category, and albuminuria category
GFR stage 3 is now subdivided into 3a and 3b

Estimated GFR should be determined using creatinine


Risk for overall mortality, CV disease, & end-stage CKD
Should be defined via eGFR and albuminuria

Patients with very low GFR (< 15) or very high albuminuria (> 300)
Should be referred to nephrology in a timely manner

Patients with progressive CKD


Should be managed in a multidisciplinary care setting, including MDs, nurses, dietitians, and social workers

Chronic Kidney Disease


Hypertension Anemia Acidosis Hyperkalemia Metabolic bone disease Malnutrition Cardiovascular disease Hyperlipidemia

13. Which of the following statements are true about CKD and hypertension?
A. Most patients with elevated Cr levels can be treated safely with ACE-I or ARB antihypertensives JNC-8 recommends a target BP of 130/90 KDOQI recommends a target BP of 140/90 HTN in patients with CKD is typically not responsive to diuretic therapy

B. C. D.

JNC-8 GuidelinesBP Target


CKD
140/90

General population
< 60 y/o = 140/90 > 60 y/o = 150/90

DM
140/90

JNC-8 GuidelinesInitial Rx
CKD
ACE-I or ARB

African American
Thiazide diuretic or CCB

Caucasian
Thiazide diuretic, CCB, ACE-I, or ARB

KDOQIHypertension Guidelines
Target BP for CVD risk reduction in CKD should be < 130/80 mm Hg (B)

KDOQIHypertension Guidelines
Antihypertensive therapy should be used in CKD to
Lower blood pressure (A) Reduce the risk of CVD (A) Slow progression of kidney disease (A)

Modifications to antihypertensive therapy should be considered based on the level of proteinuria during treatment (C)

KDOQIHypertension Guidelines
BP should be measured at each health encounter (A). Initial evaluation should include barriers to self-management, adherence to diet and other lifestyle modifications, and adherence to pharmacological therapy (B)

KDOQIHypertension Guidelines
Patients should be taught to measure and record their BP whenever possible (C). Ambulatory BP monitoring should be considered for patients with CKD and (C):
Suspected white coat hypertension Resistant hypertension Hypotensive symptoms while taking antihypertensive medications

KDOQIHypertension Guidelines
Patients with resistant hypertension should undergo additional evaluation to ascertain the cause (B). For patients in whom there is a clinical suspicion of RAD
Obtain a noninvasive screening test for RAD (A) Refer to a kidney disease or hypertension specialist for evaluation (C). Patients found to have hemodynamically significant RAD should be referred to a kidney disease or hypertension specialist for management (C).

KDOQIHypertension Guidelines
All antihypertensive agents can be used to lower BP in CKD. Multidrug regimens will be necessary in most patients with CKD to achieve therapeutic goals. Patients with specific causes of kidney disease and CVD will benefit from specific classes of agents. Patients with CKD should be considered in the "highest-risk" group for CVD for implementing recommendations for pharmacological therapy (A).

KDOQIHypertension Guidelines
Antihypertensive agents should be prescribed as follows, when possible:
Preferred agents for CKD should be used first (A). Diuretics should be included in the antihypertensive regimen in most patients (A). Choose additional agents based on cardiovascular disease-specific indications to achieve therapeutic and preventive targets and to avoid side effects and interactions (B).

KDOQIHypertension Guidelines
Long-acting agents should be used when possible (B). Two agents may be considered as initial therapy for SBP > 20 mm Hg above goal (C).

KDOQIHypertension Guidelines
Target blood pressure in diabetic or nondiabetic kidney disease should be <130/80 mm Hg ACE-I or ARB
Patients with diabetic kidney disease
With or without hypertension

Patients with non-diabetic kidney disease


Spot urine total protein to creatinine ratio 200 mg/g
With or without hypertension

KDOQIHypertension Guidelines

http://www.kidney.org/professionals/KDOQI/guidelines_bp/images/table104.jpg

KDOQIHypertension Guidelines

http://www.kidney.org/professionals/KDOQI/guidelines_bp/images/table111.jpg

KDOQIHypertension Guidelines
ACE-I & ARBs can be used safely in most patients with CKD.
Patients should be monitored for dec BP, dec GFR, and hyperkalemia (A) ACE-I or ARB can be continued if:
GFR decline over 4 months is < 30% from baseline value (B); Serum potassium is 5.5 mEq/L (B).

ACE-I & ARBs should be used at mod to high dose (A). ACE-I & ARBs can be used as alternatives to each other (B). ACE-I & ARBs can be used in combination (??) (C)

KDOQIHypertension Guidelines
Most patients with CKD should be treated with a diuretic (A) Use thiazide diuretics if GFR 30 mL/min (CKD 1-3) (A) Use loop diuretics if GFR < 30 mL/min (CKD 4-5) (A) Loop diuretics in combination with thiazides can be used for patients with ECF volume expansion/edema (A) Potassium-sparing diuretics should be used with caution: GFR < 30 mL/min (CKD Stages 4-5) (A) With ACE-I or ARBs (A) Risk factors for hyperkalemia (A) Patients treated with diuretics should be monitored for: Volume depletion & hypokalemia (A)

14. Which of the following is true regarding anemia in CKD?


A. Packed red cell transfusion is indicated for Hb < 10 B. Target Hb for patients with CKD-4 receiving EPO is 10 C. Most anemia in patients with CKD is due to iron deficiency D. Therapy with erythropoiesis-stimulating agents is associated an with increased risk of MI and thromboembolic events

KDOQIAnemia Guidelines
Hb testing should be carried out annually in all patients with CKD Dx of anemia should be made and further evaluation should be undertaken if Hb < 13.5 g/dL in adult males < 12.0 g/dL in adult females

Anemia of Chronic Disease


Chronic kidney disease Acute infections Chronic infections
Tuberculosis Infective endocarditis Chronic urinary tract infection Chronic fungal infection

Anemia of Chronic Disease


Chronic inflammatory disorders
Osteoarthritis Rheumatoid arthritis Collagen vascular disease Polymyalgia rheumatica Acute and chronic hepatitis Decubitus ulcer

Anemia of Chronic Disease


Malignancy
Metastatic carcinoma Leukemia Lymphoma Myeloma

Protein energy malnutrition

KDOQIAnemia Guidelines
Initial assessment of anemia should include the following tests:
CBC Absolute reticulocyte count Serum ferritin Serum transferrin saturation

Evaluation of Anemia
Microcytic
Ferritin Iron, TIBC

Macrocytic
B12
Methylmalonic acid level

Folate Retic count

Normocytic or high RDW


Creatinine, LFTs, SPEP Hemoglobin electrophoresis Bone marrow aspiration/biopsy

Anemia
Anemia of chronic disease
Ferritin level is normal Serum iron is low Total iron-binding capacity (TIBC) is low Fe/TIBC > 15%

Anemia of Chronic DiseaseRx


Treat underlying cause of the ACD Erythropoiesis-stimulating agent
Continuing symptomatic anemia despite Rx of underlying cause

Red cell transfusion


If continue to have symptomatic anemia despite
Treatment of the underlying cause Use of an erythropoiesis-stimulating agent

KDOQIAnemia Guidelines
Dialysis and nondialysis patients with CKD receiving ESA therapy
Hb target should be 11.0 to 12.0 g/dL Not > 13.0 g/dL Monitor Hb monthly

Monitor iron status targets at least q 3 months


Ferritin > 100 (> 200 HD) TSAT > 20%

Erythropoietin (EPO)
Glycoprotein growth factor Primary stimulus to erythropoiesis Promotes differentiation of CFU-E into normoblasts and erythrocytes 90% produced by the kidney 10% produced by the liver Decreased O2 delivery
Primary stimulus to erythropoietin release Due to anemia or hypoxemia

Erythropoiesis-Stimulating Agents (ESAs)


Rx anemia
Chronic renal failure Hemodialysis Cancer Cancer treatment-related

Erythropoiesis-Stimulating Agents (ESAs)


Darbepoetin (Aranesp) Epoetin alfa (Epogen, Procrit) Equally effective Supplemental iron
Maintain transferrin saturation 20% Maintain serum ferritin 100 ng/mL

Erythropoiesis-Stimulating Agents (ESAs)


Black box warnings
Increased mortality and serious CV events in CKD
MI, CVA, CHF

Increased mortality and tumor progression in cancer patients Increased thromboembolic events in surgery patients

Contraindicated in uncontrolled hypertension

KDOQIAnemia Guidelines
Red blood cell transfusions should be used judiciously in patients with CKD
Potential development of sensitivity affecting future kidney transplantation

No specific Hb trigger requires transfusion Target Hb for EPO is not a transfusion trigger

15. The most common acid base abnormality seen in patients with CKD is:
A. Metabolic acidosis B. Metabolic alkalosis C. Respiratory acidosis D. Respiratory alkalosis E. Mixed metabolic and respiratory acidosis

Metabolic Acidosis
pH < 7.37
Primary: HCO3 < 24 Compensatory: PCO2 < 40

Metabolic Acidosis
Increased anion gap
Normal anion gap

EquationsAnion gap
AG = NaClCO2 Normal AG = 12 +/- 2 High AG
Overproduction or underexcretion of organic acids

Low AG
Low albumin, hyperlipidemia, paraproteinemia, lithium toxicity

Metabolic AcidosisIncreased AG
Renal failure
GFR < 20 AG usually < 20 HCO3 usually > 15

Lactic acidosis
Tissue hypoxia
Bowel ischemia

Occult disorders
Hypoglycemia, CA, DM, seizures, metformin

Metabolic AcidosisIncreased AG
Diabetic ketoacidosis
Hyperglycemia & ketosis Beta-hydroxybutyrate, acetoacetate

Alcoholic ketoacidosis
Normal glucose & ketosis Beta-hydroxybutyrate

Metabolic AcidosisIncreased AG
Alcohol toxicity
Increased osmolar gap

Methanol Ethylene glycol


Oxalate crystals in urine

Paraldehyde

Salicylate intoxication
May see initial respiratory alkalosis

Metabolic AcidosisNormal AG
GI loss of HCO3
Diarrhea Fistulas Urinary diversions (ileal conduit)

Metabolic AcidosisNormal AG
Renal loss of HCO3
Distal RTA
Type 1 classical (hypokalemia)
Usually in Sjgren's or myeloma Urine pH > 5.5

Type 4 hyperkalemic
Seen in DM & renal interstitial disease

Proximal RTA (type 2)


Urine pH < 5.5 Fractional excretion of HCO3 > 15% Inherited and autoimmune disorders

Metabolic Acidosis and CKD


Due to decreased hydrogen ion excretion HCO3 < 22 mEq/L
< 5% in CKD 1 & 2 Increases linearly with worsening GFR 25% in nondialysis CKD-5

As patient approaches CKD-5


HCO3 stabilizes between 12-20 mEq/L

HCO3 < 22 associated with


Increased progression of CKD Increased overall mortality

Effects of Acidosis in CKD


Bone resorption and osteopenia Increased muscle protein catabolism Aggravation of secondary hyperparathyroidism Reduced respiratory reserve Exhaustion of body buffer systems Reduced Na+-K+-ATPase activity in RBCs Resistance to growth hormone and insulin Hypertriglyceridemia Systemic inflammation Hypotension and malaise

KDOQIMetabolic Acidosis Guidelines


In CKD Stages 3, 4, and 5
Total serum CO2 should be measured

Serum levels of total CO2 should be maintained at 22 mEq/L (Evidence) If necessary, supplemental alkali salts should be given to achieve this goal (OPINION)

Treatment of Acidosis in CKD


Na bicarbonate
0.5- 1 mg/kg/day Target HCO3 = 23-29

Na citrate (eg, Shohls solution)


Metabolized to bicarbonate Less bloating Do not use with aluminum antacids
Enhances intestinal aluminum absorption

Ca citrate Ca acetate (PhosLo, Eliphos, Phoslyra) Ca carbonate (Tums)

16. Which of the following patterns is commonly found in patients with CKD?
A. High Ca++, low PO4, high PTH B. Low Ca++, high PO4, high PTH C. High Ca++, high PO4, low PTH D. Low Ca++, low PO4, low PTH E. Low C++, high PO4, low PTH

CKD-MBD
Chronic kidney disease-mineral and bone disorder (CKD-MBD)
KDIGO terminology

Renal osteodystrophy
Older term Used to define bone pathology observed on biopsy

Characterized by
Abnormalities of calcium, phosphorus, parathyroid hormone (PTH), or vitamin D metabolism Abnormalities in bone turnover, mineralization, volume linear growth, or strength, and/or Extraskeletal calcification

CKD-MBD
Secondary hyperparathyroidism
Encompasses most of the biochemical abnormalities that characterize CKD-MBD

Causes of secondary hyperparathyroidism


Phosphate retention Decreased free Ca++ Decreased 1,25-dihydroxyvitamin D (calcitriol)

KDOQIMBD Guidelines
Measure serum calcium, PO4, intact PTH in all patients with GFR < 60 mL/min (CKD-3)

KDOQIMBD Guidelines
In CKD Stages 3 & 4
PO4 should be maintained between 2.7 and 4.6 mg/dL. (OPINION)

In CKD-5
PO4 should be maintained between 3.5 and 5.5 (EVIDENCE)

KDOQIMBD Guidelines
Dietary phosphorus should be restricted to 800 to 1,000 mg/day when serum PO4 levels are elevated Serum PO4 should be monitored every month following the initiation of dietary phosphorus restriction. (OPINION)

KDOQIMBD Guidelines
In CKD 3 & 4
Use PO4 binders if PO4 or intact PTH levels cannot be controlled despite dietary phosphorus restriction. (OPINION) Calcium-based phosphate binders are effective

In CKD-5
Both calcium-based phosphate binders and other non-calcium-, non-aluminum-, non-magnesiumcontaining phosphate-binding agents are effective. (OPINION)

Treatment of Hyperphosphatemia
Dietary phosphate restriction = 900 mg/day Oral phosphate binders
Calcium-containing
Calcium carbonate Calcium acetate (PhosLo, Eliphos, Phoslyra)
1300-2600 mg with each meal

Non-calcium
Sevelamer (Renegel, Renleva) 800-1600 mg TID Lanthanum (Fosrenol) 1500-3000 mg/day

Treatment of Hyperphosphatemia
Calcium-containing binders
Usually cheaper Better if pt has low Ca+

Non-calcium-containing binders
Better with normal or high Ca++ On Vit D supplementation

Avoid aluminum hydroxide


Except for short term Rx May cause aluminum toxicity

KDOQIMBD Guidelines
CKD 3 & 4 :
Serum Ca++ should be maintained within the "normal" range (EVIDENCE)

CKD 5
Serum Ca++ should be maintained within the lower end of the normal range (8.4 to 9.5 mg/dL)

KDOQIMBD Guidelines
CKD 3 & 4
If intact PTH is > target range
Serum 25-hydroxyvitamin D should be measured If serum 25-hydroxyvitamin D is < 30 ng/mL
Rx Vit D2 (Evidence)

If serum 25-hydroxyvitamin D is > 30 ng/mL


If Ca++ < 9.5 mg/dL & PO4 < 4.6 mg/dL Rx oral vitamin D sterol (calcitriol, alfacalcidol, or doxercalciferol) (OPINION)

KDOQIMBD Guidelines
CKD-5
If intact PTH are > 300 g/mL
Rx vitamin D sterol (calcitriol, alfacalcidol, paricalcitol, or doxercalciferol (OPINION) If Ca++ > 10.2 mg/dL
Use a noncalcium-, nonaluminum-, nonmagnesiumcontaining phosphate binder. (OPINION) Vitamin D Rx should be reduced or discontinued

KDOQIMBD Guidelines
Parathyroidectomy is recommended
Severe hyperparathyroidism (PTH > 800 pg/mL) Hypercalcemia and/or hyperphosphatemia that are refractory to medical therapy. (OPINION)

KDOQIMBD Guidelines
Bone x-rays are not indicated for the assessment of bone disease of CKD Bone mineral density should be measured by DEXA in patients with fractures and in those with known risk factors for osteoporosis (OPINION) Iliac crest bone biopsy should be considered in patients with CKD-5 who have:
Fractures with minimal or no trauma (pathological fractures); (OPINION) Intact plasma PTH levels between 100 and 500 pg/mL
Unexplained hypercalcemia, severe bone pain, or unexplained increases in bone alkaline phosphatase activity; (OPINION)

KDOQI Nutrition Guidelines


Dietary and other therapeutic lifestyle modifications are recommended as part of a comprehensive strategy to lower blood pressure and reduce CVD risk in CKD. Dietary sodium intake of less than 2.4 g/d should be recommended in most adults with CKD and hypertension (A). Referral to a registered dietitian should be considered to help patients achieve dietary recommendations (C).

Diabetic Nephropathy
Reduction of dietary protein intake to 0.81.0 g/kg/d in earlier stages and to 0.8 g/kg/d in later stages of CKD may improve renal function and is recommended (B) A recent small 4-year RCT showed that moderately low-protein diet (0.9 g/kg/d) reduced the risk of ESRD or death by 76% in type 1 diabetics (ARR 17%, NNT=6)

Malnutrition
Concern for malnutrition
Hx of poor nutritional intake accompanied by unintentional weight loss or low body weight
BMI less than 18.5 Unintentional loss > 2.3 kg or 5% of body weight over one month Unintentional loss > 4.5 kg or 10% of body weight over six months

Malnutrition
Concern for malnutrition
Physical exam findings
Temporal muscle wasting Sunken supraclavicular fossae Decreased adipose stores Signs of vitamin deficiencies

Malnutrition
Factors associated with risk of malnutrition in hospitalized elderly
Impaired cognition or delirium Poor appetite, nausea, or constipation
Due to underlying illness Side effects of medications

Restriction of movement
Restraints

No access to dentures Difficulty in self-feeding Severely restricted diet orders


NPO

Harris-Benedict Equation
Men
BMR = 88.362 + (13.397 x weight in kg) + (4.799 x height in cm)(5.677 x age in years)

Women
BMR = 447.593 + (9.247 x weight in kg) + (3.098 x height in cm)(4.330 x age in years)

Harris-Benedict Equation
Little to no exercise
Kcal/day = BMR x 1.2

Light exercise (13 days/week)


Kcal/day = BMR x 1.375

Moderate exercise (35 days/week)


Kcal/day = BMR x 1.55

Heavy exercise (67 days/week)


Kcal/day = BMR x 1.725

Very heavy exercise (2x/day, extra heavy workouts)


Kcal/day = BMR x 1.9

17. Which of the following statements regarding lipid management in CKD-5 are true according to the KDOQI guidelines?
A. Fasting lipid panel should be done every other year B. The target LDL is < 70 C. The target triglyceride level is < 250 D. Hypothyroidism should be ruled out as a treatable cause of secondary dyslipidemia

KDOQI Lipid Guidelines


All adults and adolescents with CKD should be evaluated for dyslipidemias (B)
Fasting lipid profile
Total cholesterol, LDL, HDL, and triglycerides. (B)

Annually in CKD-5

CKD-5 lipid targets


Triglycerides < 500 (B) LDL to < 100 mg/dL (B) Non-HDL cholesterol to < 130 mg/dL. (C)

KDOQI Lipid Guidelines


CKD-5 patients with dyslipidemias
Should be evaluated for remediable, secondary causes. (B)

KDOQI Lipid Guidelines

http://www.kidney.org/professionals/KDOQI/guidelines_lipids/jpegs/Table24.jpg

Diabetic Nephropathy
Develops in 20-40% of diabetics and is the leading cause of ESRD Persistent microalbuminuria (30-299 mg/24h) is a marker for the development of diabetic nephropathy in type 2 diabetics Progression to macroalbuminuria (> 300 mg/24h) predicts progression to ESRD

Diabetic Nephropathy
Screening should be initiated at time of diagnosis of type 2 diabetes as ~7% already have microalbuminuria Screen type 1 diabetics 1-5 years from initial diagnosis If absent, continue to screen annually Abnormal results should be confirmed with 2 out of 3 samples over 3-6 months due to variability in urine albumin excretion (UAE)

Diabetic Nephropathy
Random spot urine for albumin-tocreatinine ratio is preferred
Normal: < 30 mcg/mg creatinine Microalbuminuria: 30-299 mcg/mg Macroalbuminuria: >300 mcg/mg

4- or 24-hour collections may also be used, but are more cumbersome and are prone to collection error

Diabetic Nephropathy
Progression from micro- to macroalbuminuria can be delayed with intensive blood glucose and blood pressure control ACE inhibitors and ARBs are first-line therapy, even if blood pressure is normal Antiproteinemic effect is independent of blood pressure lowering

KDOQIDiabetes & CKD


Use ACE-I or an ARB in normotensive patients with diabetes and albuminuria levels > 30 mg/g who are at high risk of CKD or its progression. (2C) Do not use ACE-I or ARB for the primary prevention of CKD in normotensive normoalbuminuric patients with diabetes. (1A)

18. Indications for nephrology referral in patients with CKD include which of the following?
A. GFR < 90 mL/min B. BP > 130/90 despite treatment with an ACE-I and diuretic C. Presence of a new recently trained, highly indebted nephrologist in town D. Hb < 10 g/dL E. Diabetes and a urine albumin/creatinine ratio 100-300 mg/g

CKD- Indications for Nephrology Referral


Acute, complex, or severe cardiovascular disease Anemia of CKD Bone and mineral disorder of CKD Difficult to manage adverse effects of medications Hyperkalemia
K+ > 5.5 mEq/L (despite treatment)
Urinary protein/creatinine ratio > 500 to 1,000 mg/g Urinary albumin/creatinine ratio > 300 mg/g

Refractory proteinuria Resistant hypertension


Target blood pressure not achieved with use of at least three BP drugs

Stage 4 CKD
GFR < 30 mL/minute

Unexplained decrease in GFR > 30 percent over 4 months

CKDEvidence-Based Guidelines
ACE inhibitor or an angiotensin II receptor blocker
Nondiabetic kidney disease and a random urine total protein-tocreatinine ratio > 200 mg/g Diabetic kidney disease (A)

Concurrent use of ACE inhibitors and ARBs


Should be avoided because of symptomatic hypotension and worsening kidney function (A)

Erythropoiesis-stimulating agents
Hb goal should not exceed 11 g/dL
Due to the risk of major cardiovascular events (A)

Avoid gadolinium if
GFR < 30 mL/min per minute AKI due to hepatorenal syndrome
Or in the perioperative liver transplantation period (B)

Answers
11. D 12. E 13. A 14. D 15. A 16. B 17. D 18. D

Renal Disease 2
Gary Levine, MD, FAAFP Associate Professor Brody School of Medicine at East Carolina University Greenville, NC

Acute and Chronic Cognitive Impairment


Laurence Robbins, MD Associate Professor of Medicine University of Colorado School of Medicine Denver, Colorado

Disclosure
Dr Robbins has nothing to disclose.

It is the policy of the AAFP that all individuals in a position to control content disclose any relationships with commercial interests upon nomination/invitation of participation. Disclosure documents are reviewed for potential conflicts of interest. If conflicts are identified, they are resolved prior to confirmation of participation. Only participants who have no conflict of interest or who agree to an identified resolution process prior to their participation were involved in this CME activity.

Learning Objectives
1. Review delirium. 2. Describe differential diagnosis of dementia. 3. Discuss evaluation of cognitive impairment. 4. Review prevention and treatment of dementia.

Case Discussion
A 79-yo woman with mild dementia is 2 days post-op for an elective right total hip arthroplasty. The nurses note that she was trying to get out of bed and screamed at them when they put her back to bed. When you see her, she is somnolent but arousable. You ask her where she is, but she just picks at the sheets and speaks nonsensically.

1. The appropriate next step is:


A. Order a chest x-ray. B. Order a CT head with contrast. C. Order EEG. D. Do no further testing; this is predictable progression of dementia. E. Review her outpatient medications.

Diagnostic Criteria for Delirium


Acute disturbance of cognition (inattention: cant focus, shift, or sustain attention) KEY: Rapid onset (hours to days), fluctuation Tactile or visual delusions common (auditory hallucinations rare)

Causes of Delirium
D E L I R I U M Drugs (toxicity and withdrawal) Electrolyte disturbance Lack of drugs, liver disease Infection Reduced sensory input Intracranial Urinary retention/fecal impaction Myocardial/metabolic/pulmonary

Risk Factors for Delirium


Use of restraints Four or more medicines in 24 hours Use of indwelling urinary catheter History of dementia, stroke, or Parkinsons disease

Is Neuroimaging Always Needed?


Neuroimaging unnecessary if: Clinical evaluation discloses an obvious treatable medical illness or problem. No evidence of trauma or new focal neurologic signs. Patient is arousable and able to follow simple commands

The Yale Delirium Prevention Program


Risk factors
Cognitive impairment Sleep deprivation Immobilization Vision impairment

Intervention
Reality orientation Therapeutic activities Non-pharmacologic sleep protocol Early mobilization Vision aids

Hearing impairment Dehydration

Amplifying devices Early recognition and volume repletion

Impact of Delirium Prevention


Reduced number of delirium episodes Reduced total days of delirium Did not reduce severity of delirium or risk of recurrence

2. Normal aging may be associated with which of the following?


A. Short-term memory loss B. Difficulty with calculations C. Word-finding difficulties D. Difficulty remembering names E. Reduction in vocabulary

Normal Aging Changes in Cognition


Slowing in rate at which information can be received and processed Reduction in explicit memory (eg, the ability to recall a specific name, number, or location on demand)

Case Discussion
76-yo woman is brought to see you by her daughter, who is concerned about her failing memory. Six months ago, the daughter took over management of her mothers checkbook after she failed to pay bills. Her mother seems unable to knit, something she enjoyed for years. She has difficulty finding the right words to complete a thought.

3. What is your diagnosis?


A. This patient has dementia. B. This patient is depressed. C. This patient is delirious. D. This patient has mild cognitive impairment

Criteria for Dementia


Acquired impairment of short- and longterm memory and at least 1 of the following: abstract thinking, judgment, language, praxis, visual recognition, constructional abilities, or personality Severe enough to interfere with daily function Gradual decline and progression (ie, absence of delirium)

Dementia; Prevalence & Cost


(Hurd et al. N Engl J Med. 2013,368:1326.)

Subsample (856 persons) of Health and Retirement Study 3-4 hr in-home cognitive assessment for dementia Dementia prevalence @ 15% for age > 70 Annual cost @ $50,000 per demented person Projected cost 2040 @ $379 to $511 billion/yr

Cases of Alzheimer s Disease by Age per 100,000

4. The hallmarks of Alzheimers disease are:


A. Memory loss, personality change, delusions B. Memory loss, ataxia, mood changes C. Memory loss, aphasia, apraxia, agnosia, executive dysfunction D. Memory loss, depression, abulia E. Memory loss, acalculia, spasticity

Criteria for Alzheimers Disease


Memory loss plus 1 or more: aphasia, apraxia, agnosia, executive dysfunction Usually few motor signs apparent early Subtle behavioral and personality changes early

Screening Questions for Alzheimers Dementia


Aphasia: cant come up with words, substitutes words, new words Apraxia: has difficulty using utensils, tools Agnosia: doesnt recognize familiar people; gets lost in familiar surroundings Executive dysfunction: cant manage checkbook, use computer

Features Inconsistent with Alzheimers Disease


Sudden onset Focal neurological findings Seizures, early marked change in personality/behavior Gait disorder early in disease course

Distribution of Neurofibrillary Tangles and Amyloid Plaques

http://media.photobucket.com/image/brain%20anatomy/L Will28684/anatomy.jpg?o=1

RED: heaviest BLUE: lightest

Case Discussion
76-yo ex-college professor complains that his memory just isnt as good as it was. Daughter confirms that he has more difficulty remembering discussions that took place earlier in the day. Hes still paying bills and doing the crossword puzzles. His mental status screening test shows minimal impairment.

5. What is your diagnosis?


A. This patient has dementia. B. This patient is depressed. C. This patient is delirious. D. This patient has minor neurocognitive disorder E. This patient is normal for his age.

Mild Cognitive Impairment or Minor Neurocognitive Disorder


Complaint of memory impairment Objective memory loss (adjusted for age and education) Preserved general cognitive function Intact activities of daily living High risk of developing dementia (5%10% annually) but 40%-70% do not progress

Difficulty with concentration: Depression vs Dementia


Patient comes in alone complaining about memory = depression. Patient brought in by loved one who complains about patients memory = dementia.

Pseudodementia (Dementia Syndrome of Depression)


Some depressed elderly patients will have objective evidence of impaired cognition that improves with Rx. Clues include inconsistent performance on mental status testing, I dont know instead of near miss.

When Depression Mimics Dementia


(Alexopoulos GS, Meyers BS, Young RC, et al. Am J Psychiatry. 1993;150:1693-1699.)

23/57 hospitalized depressed elderly appeared demented (pseudodementia); 34/57 did not have signs of dementia. Among pseudodemented, signs of dementia resolved with Rx of depression. At 3 yrs: 10/23 pseudodemented had signs of dementia in absence of depression (vs 4/34 who were not pseudodemented).

When Dementia Mimics Depression: Abulia


Diffuse frontal lobe disease (eg, vascular dementia) associated with apathy, lack of motivation, flat affect. Dementia and primitive reflexes usually present (eg, grasp reflex, palmomental response). Abulic patient may seem to enjoy activities if others initiate them.

Case Discussion
An 80-yo man has slowly progressive memory loss and word finding difficulties. Family took over his finances 2 months ago. His physical exam is unremarkable. No focal findings on neurological exam.

6. The next step mostly likely to result in improvement in his function?


A. MRI of brain B. CBC, metabolic panel, TSH, B12 C. EEG D. PET scan E. Medication review

Potentially Reversible Dementias


Drugs Hypothyroid Hyperparathyroid B12 Deficiency Subdural Hematoma Other Total 16 7 3 2 2 3 31 (10%)

(Larson et al. 107 cognitively-impaired outpatients. Ann Intern Med. 1984;100:417-23.)

Screening for Dementia


> 50% of persons with dementia have not received a diagnosis of dementia. Practical screening tools improve detection (eg, MMSE).

Mental Status Screening Tests


Mini-Mental Status Exam (MMSE): 12-item, 30-point tool administered in 10-15 minutes St Louis Univ Mental Status (SLUMS) 30-point; includes cutoffs for education/MCI http://medschool.slu.edu/agingsuccessfully/pdf surveys/slumsexam_05.pdf ($1.99 iPhone app) Mini-cog: Draw clock face and 3-word recall http://geriatrics.uthscsa.edu/tools/MINICog.pdf MOCA: www.mocatest.org

Should We Screen for Dementia?


Under current Medicare Pay for Performance: Whether or not patient (> 65 yo) was screened for cognitive impairment using a standardized tool Affordable Care Act will require clinicians to assess for cognitive impairment as part of annual wellness visit. www.healthcare.gov/law/full/index.html

Potential Benefits of Screening


Clarify advance directives while patient still competent. Begin discussion about alternatives to driving, housing alternatives. Prevent financial victimization or self-neglect; remove firearms. Participate in research.

Average Dementia Evaluation


History, PE, mental status testing, comprehensive neuropsychological testing CBC, SMA 6, TSH, VDRL, B12, folate, calcium, U/A Genetic testing Brain imaging (CT or MRI)

Reversible Dementia
1970s: Reversible dementia said to be 5%-10% of all dementia. Early studies flawed; often done in hospital setting (confounded by delirium) and no follow-up to document reversibility. Outpatient studies with follow-up suggest 1% or less are reversible.

Report of the Quality Standards Subcommittee of the American Academy of Neurology


DS Knopman, MD, et al
(http://www.neurology.org/content/56/9/1143.long)

Structural neuroimaging with either a noncontrast CT or MR scan in the initial evaluation of patients with dementia is appropriate. (Guideline) Screening for depression, B12 deficiency, and hypothyroidism should be performed. (Guideline)

Incidence and Causes of Dementia


Record review of 560 consecutive patients newly diagnosed with dementia No cases of reversible dementia due to NPH, subdural hematoma, B12 deficiency, hypothyroidism, or neurosyphilis Conclusion: None of the patients with dementia reverted to normal with treatment of the putative reversible cause.
(Knopman DS, Petersen RC, Cha RH, et al. Arch Neurol. 2006;63:218221.)

Case Discussion
An 80-yo woman has short-term memory loss consistent with Alzheimers dementia. She scores 20/30 on the MMSE. Her family asks about starting donepezil (eg, Aricept).

7. What should you should tell them?


A. It will reverse her dementia.
B. It will delay nursing home placement.

C. It will increase her life expectancy.


D. It may have modest effects on scales measuring cognition and function. E. It will have major side effects and should be avoided.

Summary of Cholinesterase Inhibitor Trials in AD


Nearly 9000 patients in 22 RCT, range 3 to 12 months of donepezil (eg, Aricept), rivastigmine (eg, Exelon), or galantamine (eg, Razadyne) Modest positive benefit in cognitive, behavioral, ADL, and global scales; few side effects; modest side effects (GI) Rare evidence of dose response Clinical outcomes (caregiver burden, nursing home placement, etc) not measured

Memantine
NICE (National Institute of Health and Clinical Excellence) update 2011 based on 6 studies since 2004 (6-mo studies). Behavioral, ADL, and global scales modestly better in moderate/severe dementia. Clinical outcomes (eg, caregiver burden) not measured. ADR: Falls and agitation. Conclusion: Use in moderate dementia if intolerant of CIs or severe dementia.

Medical Foods
Caprylidene (Axona) is a medium-chain triglyceride derived from coconut or palm oil. Theory: Alternative fuel for brain of AD patients (poor uptake of glucose). Small, short-term, manufacturer-sponsored trials show benefit. Approximately $90 per month.

Lifestyle and Dementia


Bronx Aging Study: Education and cognitive leisure activities protective against development of AD (N Engl J Med. 2003;348:2508-2516) 15 minutes aerobic exercise 3x/week reduces likelihood of dementia
(Ann Intern Med. 2006;144(2):73-80)

Australian study of 170 participants with MCI: In randomized, controlled, 24-week trial of home-based physical activity intervention (70,000 steps/week) vs usual care showed modest improvement in cognition
(JAMA. 2008;300:1027-1037)

Does the Diagnosis of a Specific Type of Dementia Matter?

Diagnosis and Treatment of Dementia: The Dismal Failure of Medical Science


No direct evidence linking screening and improved decision making. (USPSTF. Ann Intern Med.
2013;159:601-612)

No current intervention will prevent or delay the onset of dementia.


(http://consensus.nih.gov/2010/docs/alz/AIM_alz.pdf)

Treatment for dementia (cholinesterase inhibitors and NMDA receptor antagonist) is minimally effective. (Ann Intern Med. 2008;148:370-378)

The Dementia Epidemic: Any Good News?


Cognitive function and ageing study suggests that dementia prevalence declined between early 1990s compared to 2008-2011 (8.3% vs 6.5%, age > 65). Linked to better education and reduction of ASCVD/CVA. Obesity, diabetes, and inactivity threaten to reverse this trend.

Case Discussion
80 yo with 12 mos of becoming more sedentary, slowed movement, unsteady gait, 2 falls, no injury. Stepwise progression of deficits. Uses walker. Speech diminished in volume, less distinct. Cant manage finances. No change in mood or personality. Diabetic, smoker, hypertensive. Flat affect. Muscle tone increased, right grip weak, asymmetric reflexes, no tremor. 23/30 on MMSE (deficits in memory and calculations).

8. The most likely diagnosis is?


A. Alzheimers disease B. Picks disease C. Huntingtons disease D. Parkinsons disease E. Vascular dementia

Vascular Dementia
Subcortical or mixed dementia Stepwise progression, prior strokes, focal neuro symptoms/signs Preserved personality but emotional incontinence or apathy common Definitive diagnosis difficult

Case Discussion
A 69-yo man has developed rigidity, a short-stepped gait, and masked facies. He also has become more forgetful (MMSE = 19). His family thinks he sees things that arent real.

9. The most likely diagnosis is?


A. Picks disease B. Alzheimers disease C. Diffuse Lewy body dementia D. Progressive supranuclear palsy E. Parkinsons disease and depression

Diffuse Lewy Body Dementia


Dementia, parkinsonism, and visual hallucinations (may develop severe EPS if prescribed neuroleptics)

Case Discussion
A 64-yo man is brought in by his family after exposing himself in public. He has been urinating in the kitchen sink and refuses to bathe. His MMSE is 26/30. He has some wording finding difficulties.

10. The most likely diagnosis is:


A. Alzheimers disease B. Frontotemporal dementia C. Diffuse Lewy body dementia D. Vascular dementia E. Creutzfeldt-Jakob disease

Frontotemporal Dementia
Picks disease and non-specific degeneration of frontal lobes; corticobasal dementia, progressive supranuclear palsy (Parkinson plus syndromes) Behavioral problems early (disinhibition and/or profound apathy) plus aphasia Memory and visuospatial problems later

Case Discussion
A 76-yo man has increasing difficulty walking. He complains that his feet seem stuck together. He has mild memory loss. He has urge urinary incontinence.

11. You order the following test:


A. B. C. D. E. TSH CT scan Carotid ultrasound EEG Cystoscopy

Normal Pressure Hydrocephalus


Clinical triad of dementia, ataxia, urinary incontinence (wacky, wobbly, and wet) Frequency of NPH and response to shunt surgery controversial Ataxia most responsive; dementia probably least responsive

Case Discussion
An 84-yo woman has developed rapidly progressive dementia over 4 months. She has a low-grade fever, is very rigid, and has myoclonic jerks when startled. EEG shows triphasic sharp wave complexes.

12. This pattern is consistent with which diagnosis?


A. Creutzfeldt-Jakob Disease B. Subdural hematoma C. Cerebral vasculitis D. HIV dementia E. Herpes encephalitis

Creutzfeldt-Jakob Disease
Rapidly progressive dementia over several months with myoclonus Frequently have periodic synchronous bi- or triphasic sharp wave complexes on EEG Tend to be younger patients Transmissible (viral-like prions) Rare (1 per million in US) Variant CJD = mad cow disease

Case Discussion
An 81-yo patient with advanced dementia is hoarding food at her assisted living facility and repeatedly leaving her room wearing only her underwear. She makes sexually inappropriate comments to visitors. The administrator asks you to do something to control these behaviors.

13. You should:


A. Begin donepezil 10 mg at bedtime. B. Begin haloperidol 0.5 mg bid. C. Begin olanzapine (eg, Zyprexa) 2.5 mg hs. D. Begin valproate 250 mg tid. E. Offer to help the staff find ways to manage the behaviors nonpharmacologically.

Which Behaviors Responsive to Medication?


RESPONSIVE: Agitation, depression, delusions, hallucinations, aggression REFRACTORY: Wandering, hoarding/hiding objects, repetitive questioning, apathy, social inappropriateness Behavioral approach to refractory Sx

Do Cholinesterase Inhibitors Treat Neuropsychiatric Symptoms?


(Howard RJ, Juszczak E, Ballard CG, et al. N Engl J Med. 2007;357:1382-1392.)

272 pts in 12-week trial of donepezil (Aricept) for treatment of agitation in Alzheimer s disease (avg MMSE 8/30) Donepezil no more effective than placebo

Popular Drugs for Dementia Tied to Deaths NY TIMES 4/12/05


FDA reviewed all atypical antipsychotics in dementia. 17 placebo-controlled studies, 5106 elderly subjects with dementia, average duration of Rx 10 weeks. Deaths: 4.5% (Rx) vs 2.6% (placebo).

Risk of Death in Elderly Users of Conventional vs Atypical Antipsychotics


(N Engl J Med. Dec 2005;353:2335.)

Retrospective evaluation of Medicare data suggests that death rate in nursing home residents on typical antipsychotics is the same, if not greater than, the atypical antipsychotics.

CATIE-D Results
Randomized trial comparing olanzapine, quetiapine, risperidone, and placebo for agitation/aggression in demented patients Adverse effects offset advantages in the efficacy of atypical antipsychotic drugs for the treatment of psychosis, aggression, or agitation in patients with Alzheimer disease.
Schneider L et al. N Engl J Med. Oct 2006;355:1525.

New Treatments for Dementia: Are They Worth Remembering?


5 FDA-approved medications for Rx of dementia; no evidence yet of clinically significant alteration in disease course. Stop the amitriptyline (et al). Read while you exercise. Searching for a breakthrough, settling for less.

Answers
1. E 2. C 3. A 4. C 5. D 6. E 7. D 8. E 9. C 10. B 11. B 12. A 13. E

Caregiver/Practitioner Resources
Alzheimers Association 1-800-2723900 or www.alz.org Government funded clinical trials in AD can be found at http://clinicaltrials.gov

Additional Reading
Le Couteur DG et al. Political drive to screen for pre-dementia; not evidence based and ignores the harms of diagnosis. BMJ. 2013;347:f5125. Qaseem A et al. Current pharmacologic treatment of dementia. Ann Intern Med. 2008;148:370. (ACP/AAFP guideline) Inouye SK. Delirium in older persons. New Engl J Med 2006;354:11.

Larson E. New Insights into the dementia epidemic. N Engl J Med. 2013;369:2275-2277. Kawas, C. Early Alzheimers disease. N Engl J Med. 2003;349:1056-1063. AD2000 Collaborative Group. Long-term donepezil treatment in 565 patients with Alzheimers disease (AD2000): randomised double-blind trial. Lancet. 2004;363:2105-2115. http://www.nice.org.uk/nicemedia/live/13419/5361 9/53619.pdf (2011 review of CIs and memantine)

Peripheral Vascular Disease


Robert Dachs, MD, FAAFP
Clinical Associate Professor and Director of Research Ellis Hospital Family Medicine Residency Program Albany Medical College Albany, New York

Disclosure Statement
Dr. Dachs has nothing to disclose.

It is the policy of the AAFP that all individuals in a position to control content disclose any relationships with commercial interests upon nomination/invitation of participation. Disclosure documents are reviewed for potential conflicts of interest. If conflicts are identified, they are resolved prior to confirmation of participation. Only participants who have no conflict of interest or who agree to an identified resolution process prior to their participation were involved in this CME activity

Learning Objectives
1. 2. 3.

4.
5.

6.
7.

Recognize the signs and symptoms of abdominal aortic aneurysms and aortic dissection. Define claudication. Describe the physical findings in chronic arterial insufficiency. List some of the means for objective documentation of occlusive disease. Recognize some of the supportive measures for patients with claudication. Describe the benefits of walking programs for patients with claudication. List the medications available for peripheral vascular disease.

Aortic Diseases
Aortic dissection (thoracic) Abdominal aortic aneurysm (AAA)

Abdominal Aortic Aneurysm


1. The most common predisposing factors for the development of an abdominal aortic aneurysm are: 1. History of syphilis and male sex 2. Marfans syndrome and male sex 3. Atherosclerosis and male sex 4. Atherosclerosis and female sex

Abdominal Aortic Aneurysm (AAA)


Aneurysm is > 50% in vessel diameter Normal aorta diameter is 1.8 - 2.0 cm 2-5% prevalence in elderly populations Average age of Dx: 65 13th most common cause of death in US 10th most common cause of death in men

Abdominal Aortic Aneurysm (AAA)


Location: 95% are infra-renal Pathogenesis: Atherosclerosis
(Thoracic and supra-renal aneurysms do occur think Marfans, Ehlers-Danlos, syphilis)

Male to female ratio: 3-8:1

Abdominal Aortic Aneurysm (AAA): Mortality


Rupture: 60% of patients die before arrival Only 50% that do arrive alive, survive Overall mortality rate (with rupture) = 80%

2. Because of the prevalence and its lethality, the USPSTF recommends that ultrasound screening be performed in which patients:
1. One-time screening for men ages 65-75, who have ever smoked 2. One-time screening for men ages 65-75, regardless of smoking history 3. One time screening for both men and women ages 65-75, who have ever smoked 4. One time screening for both men and women ages 65-75, regardless of smoking history

Should You Ultrasound Screen Patients for AAA?

USPSTF Recommendations: 2005


Release Date: Feb. 2005, Ann Intern Med 142: 198-202
(+) One-time screening for AAA by US in men ages 65-75, who have ever smoked. Rating: B Recommendation No recommendation (+ or -) screening for AAA in men 65-75, who have never smoked Rating: C Recommendation

(-) USPSTF recommends against routine screening for AAA in women Rating: D Recommendation

Should You US Screen Patients for AAA?

Why not women???


Chichester trial. (Scott, RA; et al. Br J Surg; 1995)
Methods: 9342 women, ages 65-80 randomly assigned to:
Screening vs. control (no screening) Results: - At 5 years, no difference in AAA mortality - At 10 years, no difference in AAA rupture rate

3. You identified an abdominal aortic aneurysm (AAA) in your patient. At what size (in centimeters) should you refer your patient for surgical intervention?
1. 2. 3. 4.

3 - 3.5 cm 4 - 4.5 cm 5 - 5.5 cm 6 - 6.5 cm

AAA: Risk of Rupture

Used with permission NEJM

Powell JT and Greenhalgh RM. NEJM 348: 1895, May 8, 2003

AAA: Risk of Rupture


Rate of increase = Is exponential!!!! Smaller aneurysms expand slower
(Conversely, larger aneurysms expand faster)

Aneurysm size 3.0 - 3.9 cm 4.0 - 4.0 cm 5.0 - 5.9 cm

Mean yearly increase 0.20 cm 0.34 cm 0.64 cm

Vardulaki, KA et al. Br J Surg 1998; 85: 1674.

4. Your patients AAA diameter is 4.3 cm. He leaves your practice and returns 4 yrs later where you meet him in the ED complaining of severe right flank and abdominal pain. Vitals: 120/60, P = 90, afebrile Labs: H/H = 13/39, Urine = 10-20 RBCs/hpf. Which of the following should be performed?
1. 2. 3. 4.

STAT abdominal ultrasound STAT non-contrasted abdominal CT scan STAT aortogram STAT intravenous pyelogram (IVP)

AAA Rupture: Symptoms


Pain is most common: Abdomen, flank, legs, buttocks, testicular/groin Syncope Vomiting Hypotension (+/-) Pulsatile mass (+/-) Femoral pulses are NORMAL!!! Laboratory: - Hematuria is (+) up to 10-30% of cases!!!!

AAA Rupture: Symptoms


Pain is most common: Abdomen, flank, legs, buttocks, testicular/groin Syncope Vomiting Hypotension (+/-) Pulsatile mass (+/-) Femoral pulses are NORMAL!!! Laboratory: Hematuria is (+) up to 10-30% of cases!!!!

AAA: Graft Complications


Aortoenteric fistula: Distal Duodenum 57%, Esophagus 32% Presents with GI bleed (sentinel hemorrhage) Graft Infection: Fever, source unknown, distal septic emboli (+) blood cultures, CT with gas surrounding graft

5. A 55-year-old male with Hx of long-standing HTN presents to the ED with sudden onset of chest pain. The CXR suggests a widened mediastinum. The most likely diagnosis is:
1. 2. 3. 4.

Ruptured aneurysm Aortic dissection Mycotic aneurysm Coarctation of the aorta

A Pet Peeve
AAA do not dissect Aortic dissections are NOT aneurysms
The pathophysiology is different!!!!

AAA
Atherosclerosis

Aortic dissection
Hypertension

Aortic Dissection: Whos at Risk?


Ages: 40-80, but younger pts also Males: 2-3:1 Hypertension: 70-90%
Marfans, Ehlers-Danlos

Aortic Dissection: Presentation


Patients with aortic dissections will commonly present with ripping, tearing, interscapular back pain and pulse deficits

A. True B. False

Aortic Dissection: Presentation


Patients with aortic dissections will commonly present with ripping, tearing, interscapular back pain and pulse deficits

A. True B. False
Answer: 50% and 15%
IRAD study. JAMA 283: 897, Feb 16, 2000

Aortic Dissection: Location


Stanford Classification Ascending Aorta (60-65%) Type A Descending Aorta (30-35%) Type B (After origin of subclavian artery)

IRAD Study: JAMA, 2000


Symptoms/Findings Any Pain Anterior CP Back pain Abdominal pain Tearing/ripping Migrating Syncope N=464 95% 61% 53% 30% 50% 17% 9% Type A 94% 71% 46% 22% 49% 15% 13% Type B 98% 44% 64% 42% 52% 19% 4%

Hypertensive
Hypotensive/shock

49%
16%

36%
25%

70%
4%

Pulse deficits

15%

19%

9%

Aortic Dissection: Diagnosis


Echocardiography 1) Transthoracic (TTE) 2) Transesophageal (TEE)

CT scanning (with contrast) MRI Aortography


All are acceptable, depends on what you have available!

Aortic Dissection: Management


2 Goals:
Lower blood pressure to BP sys 90-110 - IV nitroprusside Lower velocity of LV ejection - IV esmolol

Pearl: Start with your B-Blocker

Arterial Occlusive Disease


Chronic due to: Atherosclerosis => Claudication Acute due to: Thromboembolic => 5 Ps

6. A 68-year-old male presents with complaints of an aching pain in both thighs when he walks about one block. The pain subsides within about 1-2 minutes after he stops ambulating. The most likely diagnosis is: 1. Claudication
2. Pseudoclaudication due to spinal stenosis 3. Lumbar radiculopathy

4. Bilateral hip degenerative joint disease

Claudication: Definition
Reproducible ischemic muscle pain that occurs with exercise, relieved with rest
Its stable anginaof the legs!!!

Claudication: Presentation
Symptoms are distal to the location of occlusion - Calf symptoms: femoral - popliteal disease - Calf and thigh: profunda femoral artery - Thigh, hip, buttock pain, with impotence: aortoiliac disease (Leriche syndrome)

Chronic Arterial Occlusive Disease


Differential Diagnosis Spinal stenosis Pseudoclaudication Spinal cord tumors Lumbar radiculopathy DJD DVT

7. The best screening test for suspected PVD is:


1. Perform an ankle-brachial index (ABI) 2. Perform bilateral leg ultrasound

3. Perform pulse volume recordings (PVR) 4. Perform lower extremity magnetic resonance arteriogram (MRA)

Used with permission NEJM

White C. N Engl J Med 2007;356:1241-1250

Chronic Arterial Occlusive Disease


Screening: Ankle - Brachial Pressure Index ABI
0.9 - 1.30 0.7 - 0.89 0.4 - 0.69 < 0.4

Interpretation
Normal Mild Moderate Severe

Use higher of 2 brachial pressures if different Use higher of 2 ankle pressures (DP or PT) if different CPT # 93922

Screening: Ankle - Brachial Pressure Index What if the value is markedly elevated?? Example: ankle BPsys = 210 = brachial BPsys = 130

1.6

An ABI > 1.4 indicates noncompressible arteries (calcified vessels) 1.4% of adults > 40 yrs old have ABI > 1.4
- Accounts for approx 20% of all adults with PVD.

Bottom Line: Normal ABI = 0.9 - 1.4

8. Your patient has an ABI of .65 in the R leg and .70 in the L leg. You recommend exercise and smoking cessation.

Which drug therapy does NOT increase walking distance? 1. Atorvastatin (Lipitor) 2. Ramipril (Altace) 3. Clopidogrel (Plavix) 4. Cilostazol (Pletal)

PAD: Management
Risk factor modification Smoking cessation Hypertension Diabetes Antiplatelet therapy - Aspirin - Ticlopidine - Clopidogrel (Plavix)

Intervention
Exercise Cilostazol (Pletal) Lipid lowering agents Ramipril

PAD: Interventions: 1. Supervised exercise training


22 trials: => improve pain-free walking*
Improved pain-free walking by 90 yards

2. Lipid-lowering agents
7 small trials: => improve pain-free walking*

Improved pain-free walking by 96 yards


*Watson L, et al. Cochrane Library, Issue 1, 2009 **Aung PP, et al. Cochrane Library, Issue 4, 2007

PAD: Interventions:
3. Drug therapy for claudication Cilostazol (Pletal)
- Inhibits phosphodiesterase type 3 - Mechanism of action is unclear - 7 randomized, placebo-controlled trials => - Improved pain-free walking* - Approx by 41 yards**
*Robless P, et al. Cochrane Library, Issue 1, 2008 ** Beebe HG, et al. Arch Intern Med 1999; 159: 2041-2050

PAD: Interventions:
3. Drug therapy for claudication
Cilostazol (Pletal)
Side effect: Headache = 34% (vs. placebo 14%)

*****Black Box Warning***** Do not give to patients with CHF

PAD: Interventions:
4. Drug therapy for claudication A new player
Ramipril 10 mg qd
Methods: double-blind randomized trial Results: At 6 months, 75-second increase in painfree walking time, and 255-second increase in maximum walking time.
Ahimastos, AA, et al. JAMA, Feb 6, 2013

PAD:Management

/ Risk factor

modification

Mild/moderate symptoms Exercise Drug therapy

Critical leg ischemia

/ Symptoms
improve

\ Symptoms
worsen

Localize the lesion: Pulse volumeLocalize recording lesion Magnetic resonance angiography (MRA) Conventional angiography

| | | | | | |

Chronic Arterial Occlusive Disease: As Disease Progresses


Pain at rest in foot or toes (sometimes noted as paresthesias/numbness) Worse with legs elevated, relieved with legs dependent Develops leg edema

Chronic Arterial Occlusive Disease: As Disease Progresses


Hair loss, smooth shiny skin Thickened nails Pallor with leg elevation and Rubor with legs dependent Bruits, decreased pulses Cyanosis, ulceration, gangrene

So should you screen all at risk patients with an ABI?


2013: USPSTF notes Insufficient Evidence for Routine Screening for PAD Rating: I level Recommendation

9. A 72-year-old female presents to the ED with sudden severe R leg pain, located from the knee to toes. Her past medical history is significant for HTN and DM. Exam: Vitals: 160/90, pulse = 120 and irregular, afebrile. Lungs are clear. Heart: rapid irregularly, irregular pulse. The right leg is cool to touch, pale in color, and you are unable to obtain posterior tibial or dorsalis pedis pulses.

At this point you should:


1. Start Heparin and immediately consult a vascular surgeon 2. Immediately obtain an ultrasound of the lower extremity 3. Immediately obtain an echocardiogram

4. Immediately obtain an abdominal aortic ultrasound

Acute Arterial Occlusion


Thromboembolic Heart is most common source: 80-90% Presentation: The 5 Ps

-Pain -Pallor -Paresthesia -Pulselessness -Paralysis

Acute Arterial Occlusion


Cardioarterial emboli: Where do they lodge? Lower extremities: 65-70% (usually at bifurcations) Cerebral arteries: 20-25% Upper extremities: 5-10% Visceral arteries: 5-10%
Dont miss: Acute SMA occlusion: Pain out of proportion to physical findings

10. A 76-year-old male with a Hx of HTN, hyperlipidemia, and smoking presents with a painful toe. He denies trauma. No Hx of atrial fibrillation. He has 2+ posterior tibial and 1+ dorsalis pedis pulses. The most likely diagnosis is:

1. 2. 3. 4.

Acute gout Raynauds syndrome Cellulitis Blue toe syndrome

Acute Arterial Occlusion


2. Arterioarterial emboli: The Blue Toe Syndrome Cholesterol or atherothrombotic emboli Occludes small vessels Dont be fooled - Pulses remain present - Often confused with bruising Can involve multiple organs - especially kidneys Can confirm diagnosis with - Skin or muscle biopsy - Cholesterol crystals on fundoscopic exam

Answers
1. C 2. A 3. C 4. B 5. B 6. A 7. A 8. C 9. A 10.D

Unique Geriatric Pharmacologic Issues


Laurence J Robbins, MD Associate Professor of Medicine University of Colorado School of Medicine Denver, Colorado

Disclosure Statement
Laurence J Robbins, MD, has no relevant financial relationships to disclose.

It is the policy of the AAFP that all individuals in a position to control content disclose any relationships with commercial interests upon nomination/invitation of participation. Disclosure documents are reviewed for potential conflicts of interest. If conflicts are identified, they are resolved prior to confirmation of participation. Only participants who have no conflict of interest or who agree to an identified resolution process prior to their participation were involved in this CME activity.

Learning Objectives
1. Identify clinical importance of effects of age on pharmacokinetics and pharmacodynamics. 2. Recognize the risk factors for adverse drug events in older patients and ways to mitigate them. 3. Describe the principles of prescribing for older patients.

Geriatric Pharmacoeconomics
People 65+ People <65 65+ share of prescriptions <65 share of presciptions

100 90 80 70 60 50 40 30 20 10 0 Present 2040

2011: Age 65+ is 13% of US population and buys 33% of prescription drugs. In 2040, will be 25% of population and buy 50% of prescription drugs.

What EBM? The example of cardiovascular drugs: Percentages of all patients in a given age group treated with cardiovascular drugs (Italy) vs percentages in each age group included in cardiovascular drug trials (globally)
Cerreta F et al. N Engl J Med. 2012;367:1972-1974.

1. Which of the following pharmacokinetic factors changes least with age?


A. Absorption B. Distribution C. Metabolism D. Elimination

2. An 80-yo hypertensive woman is switched from atenolol to propranolol to treat tremor. A week later she seems more bradycardic, confused, and despondent. The most likely cause is: A. Taking extra propranolol B. Decreased metabolism of propranolol C. Lipophilia of propranolol D. Reduced elimination of propranolol

3. An 84-yo man has been less alert and his balance is deteriorating. He was taking 5 mg of diazepam twice a day and agrees to stop. One week later, hes no better. Three weeks later hes improved. The most likely explanation is: A. Withdrawing drug too quickly B. Active drug metabolites C. Increased receptor sensitivity D. Patient must have continued taking the diazepam

Effects of Aging on Volume of Distribution (VD)


body water lower VD for hydrophilic drugs lean body mass lower VD for drugs that bind to muscle fat stores higher VD for lipophilic drugs and lipid soluble more likely to get into brain plasma protein (albumin) higher percentage of drug that is unbound (active)

4. Change in which of the following pharmacokinetic factors accounts for the most differences in drug effects with advancing age?
A. Absorption B. Distribution C. Metabolism D. Elimination

5. Which patient is likely to have greater difficulty renally clearing drugs?


A. 50-yo woman, weight 150 lb, serum creatinine level 2.2 mg/dL B. 80-yo woman, weight 110 lb, serum creatinine level 1.4 mg/dL C. They are the same. D. Urine creatinine measurement is required to estimate clearance

Serum Creatinine Does Not Reflect Creatinine Clearance


lean body mass lower creatinine production and less creatinine to clear. Result: In older persons, serum creatinine may stay in normal range, masking decline in creatinine clearance (CrCl).

Cockroft-Gault Equation
(Ideal weight in kg) (140 age) ________________________ x (0.85 if female) (72) (serum creatinine in mg/dL)

50-yo = 33 cc/min
80-yo = 23.6 cc/min

6. An 80-yo on warfarin has an INR of 2.5. The best explanation for increased bleeding risk compared with a younger patient is:
A. Difference in elimination B. Difference in metabolism C. Difference in volume of distribution D. Pharmacodynamic differences

Pharmacodynamics
Pharmacokinetics: What the body does to the drug. Pharmacodynamics: What the drug does to the body.* Elderly are at greater risk for bleeding at any given INR because they are more likely to have additional problems that increase that risk (eg, friable stomach, more likely to fall and suffer head trauma, etc).
* Leslie Z. Benet, in: Drug Metabolism and Drug Toxicity, JR Mitchell and MG Horning (eds.), Raven Press, New York (1984). page 199.

Learning Objectives
1. Identify clinical importance of effects of age on pharmacokinetics and pharmacodynamics. 2. Recognize the risk factors for adverse drug events for older patients and ways to mitigate them.
3. Describe the principles of prescribing for older patients.

7. The most common cause of adverse drug reactions in the elderly is:
A. Decreased renal clearance B. Unrecognized dementia C. Reduced drug metabolism D. Number of medications prescribed E. Inconsistent medication compliance

Risk Factors for ADR


6 or more concurrent chronic conditions 12 or more doses of drugs/day 9 or more medications (potential drug interactions: 6% on 2 meds, 50% on 5 meds, approx. 100% on 8 meds) Prior adverse drug event Low body weight or low BMI Age 85 or older Estimated CrCl < 50 mL/min

Beers Criteria
Comprehensive review and grading of drug-related problems and adverse drug events in older adults Examples: all muscle relaxers, tricyclic antidepressants 2012 version of Beers List:
http://www.americangeriatrics.org/files/documents/ beers/2012BeersCriteria_JAGS.pdf

Beers Other Top Drugs to Avoid


Diphenhydramine, hydroxyzine, and firstgeneration antihistamines Clonidine Amiodarone, dronedarone, class 1 antiarrhythmic drugs Digoxin > 0.125 mg daily All benzodiazepines Glyburide, chlorprompramide Indomethacin, meperidine

Prescription Errors
> 20% of ambulatory older adults receive at least one potentially inappropriate drug (eg, 1/5 receiving cholinesterase inhibitor for Alzheimers also take anticholinergic drug). Nearly 4% of office visits and 10% of hospital admissions result in prescription of medications classified as never or rarely appropriate.

Monitoring Errors
Example: 12%-63% of patients taking ACE inhibitor had inadequate monitoring of K or Cr. Preventable ADEs: Most ADEs do not result from improper choices of drugs or drug doses but instead represent known side effects of drugs that have a rightful place in the therapeutic armamentarium.

Case Discussion
On Friday evening at 8 pm, the answering service puts through a call from a 72-yo man who is followed by a colleague. The patient is coughing and sneezing. In a nasal voice he says Doc, I couldnt pee all afternoon and I just wet myself!

8. Your next step is:


A. Order an antibiotic for him to pick up at his local pharmacy. B. Tell him to take 2 aspirin and call you in the morning. C. Tell him to sit down when he urinates. D. Start alpha blocker (eg, tamsulosin). E. Ask him about over-the-counter drugs.

Over-the-Counter medication
Elderly account for 13%-15% of the population but account for 40% of all OTC purchases. May not consider these medications and may not report to physician because no prescription required. Increasing array of OTCs leads to increased risk of unsuspected ADR, drug interactions (eg, antihistamine/adrenergics can precipitate urinary retention).

Learning Objectives
1. Identify clinical importance of effects of age on pharmacokinetics and pharmacodynamics. 2. Recognize the risk factors for adverse drug events for older patients and ways to mitigate them.
3. Describe the principles of prescribing for older patients.

Case Discussion
Ms Mannie Pils is an 80-yo woman brought to your office by her daughter because she moved her mother from Ohio 1 month ago and wants you to provide her primary care. Her mother had not been taking care of herself and the daughter found her thin and disheveled during a recent visit.

The patient has eaten poorly, lost weight, and spends more time in bed. The daughter thinks her mother was treated for arthritis, asthma, anxiety, gout, constipation, and high blood pressure. The daughter now wants drug refills. Patient seems tired and affect is flat. Her memory is impaired. Her BP is 115/75, pulse is 58 and regular. Mucous membranes are dry; shes photophobic. She cannot arise from a chair without assistance and walks with a shuffle. Her muscle tone seems increased.

9. The most likely cause of this patients decline is:


A. Unrecognized malignancy B. Progressive dementia C. Medication toxicity D. Unrecognized depression

Digoxin 0.25 mg daily Clonidine 0.2 mg po tid Theophylline (delayed release) 300 mg po bid Propranolol 20 mg po bid Verapamil 240 mg (sustained release) daily Diazepam 2 mg po bid Furosemide 20 mg po daily

Indomethacin 25 mg po bid Metoclopramide 10 mg po tid with meals Amitriptyline 50 mg po hs Aspirin 325 mg po daily 3 different laxatives and antacids for prn use (her mother also insists on taking acetaminophen [Tylenol PM] for sleep)

10. Your next step is:


A. Stop all medications immediately. B. Stop all medications except digoxin and theophylline. C. Stop the medications but taper down the clonidine and amitriptyline. D. Stop digoxin and verapamil; increase furosemide for edema. E. Continue all meds but cut doses in half.

When to Be Cautious about Medication Withdrawal


Sudden cessation of amitriptyline may cause a cholinergic rebound syndrome (ie, agitation, borborygmi, diarrhea). Sudden withdrawal of clonidine may cause rebound hypertension but less likely with dose less than 1 mg daily.

ADE Mimic Disease


Bradycardia: Digoxin, verapamil, and propranolol; slow cardiac conduction (note: Verapamil increases digoxin levels 50%-75%) Weight loss: Theophylline and digoxin may cause nausea and dysgeusia (food tastes bad). Clonidine and amitriptyline cause dry mouth (harder to eat) and photophobia.

Side Effects Beget More Polypharmacy


Theophylline causes tremor (propranolol). Verapamil, clonidine, and amitriptyline may cause constipation in the elderly (laxatives). Verapamil and propranolol cause fluid retention and edema (furosemide). Metoclopramide can induce parkinsonism (carbidopa/levodopa). Aspirin and theophylline exacerbate gastroesophageal reflux (antacids).

Digoxin: No Longer a Geriatric Staple


Absence of LV systolic dysfunction or atrial fibrillation with rapid ventricular response (RVR) means digoxin can be discontinued. In atrial fibrillation with RVR, slows rate at rest but not with exertion. In advanced left heart failure, reduces hospital days but does not reduce mortality (unlike ACEI or beta blocker).

11. Which medication may be contributing to her confusion and apathy?


A. B. C. D. E. F. G. H. I. J. Diazepam Digoxin Propranolol Verapamil Indomethacin Metoclopramide Clonidine Diphenhydramine Amitriptyline All of the above

Drugs Impairing Cognition


Everything we prescribe except acetaminophen and docusate. Most often psychoactive meds or those with anticholinergic side effects. Discontinue amitriptyline is always the correct answer on boards.

Drugs Impairing Cognition


Anticholinergics (eg, diphenhydramine, trihexyphenidyl, oxybutynin) Anticonvulsants (phenytoin, gabapentin, valproate) Muscle relaxers (carisoprodol [eg, Soma], cyclobenzaprine [eg, Flexeril]) Antiemetics (prochlorperazine, metoclopramide) Digoxin, clonidine, amantadine, amiodarone Benzodiazepines, antipsychotics

Drug-Induced Cognitive Impairment


> 50% who stop medication will improve. Often a single medication implicated. Patients with drug-induced cognitive impairment were also 3 times more likely to fall. Most offending drugs taken for several years prior to diagnosis.
(Larson EB, Kukull WA, Buchner D, et al. Ann Intern Med. 1987;107:169-173.)

To Reduce Polypharmacy
EBM for patients of this age and disability? Do benefits outweigh known possible adverse effects? If benefit > risk, does the patient have adverse symptoms that may be drug-induced? (If so, switch.) Is there a better drug? If there is no better choice, can the dose be reduced? (Arch Intern Med. 2010;170:1648-1654.)

Results
Mean age @ 83 yo. @ 60% had three or more chronic diseases. Mean number of medications 7.7. @ 60% had medications discontinued; mean of 5 meds eliminated. 2% of meds restarted, 17% not discontinued; 81% meds NOT restarted 19 months follow-up. 88% reported global improvement.

Three More Principles


Danger of silo thinking One size fits all Time to benefit

The Danger of Silos...


81-yo man with CHF (EF 30%), atrial fib, and diabetes transfers primary care to you. Notes dyspnea on exertion with routine ADLs. Medications include furosemide, lisinopril, digoxin, glipizide, warfarin, Toprol XL. Hct 38, K 4.7, creat 2.1, Na 135. A cardiologist recommended spironolactone but patient wary of more medication. Patient asks for your advice.

Spironolactone in Patients with Severe CHF: The RALES Trial


Spironolactone 12.5-25 mg combined with standard CHF reduces morbidity and mortality. (N Engl J Med. 1999;341:709.)

Avg age 65 +/- 12. Trial stopped early @ 2 year avg, because interim analysis determined that spironolactone was efficacious. Death rate: 46% placebo vs 35% spironolactone (relative risk 0.7). Lower risk of death from progressive heart failure and sudden cardiac death. Significant improvement in the symptoms of heart failure.

Conclusion
Spironolactone, in addition to standard therapy with ACE inhibitor and diuretic (and digoxin), substantially reduces the risk of both morbidity and death among patients with severe heart failure.

Rates of Hyperkalemia after Publication of RALES


(N Engl J Med. 2004;351:543.)

Review spironolactone prescription rates among hospitalized patients > 65 who were taking ACE inhibitors (Ontario 19942001) Marked increased use of spironolactone in CHF patients after publication of RALES, average age 78

Findings
Increased rate of hospitalization for hyperkalemia with modest but significant hospital death from hyperkalemia No significant change in gradual decline of hospital readmission for CHF (trend preceded publication of RALES trial)

Conclusions Re: Spironolactone


Use increased dramatically after publication of RALES trial without anticipated benefit. Prescribing for pts (eg, chronic renal disease) who would have been excluded from RALES likely a major factor. Silo thinking: Failure to account for impact of multiple chronic diseases on treatment efficacy and safety.

One Size Fits All


(Risk of Generalizing from Healthy Elderly Trials)

HYVET trial confirms that treating hypertension (> 160/90) in patients > 80 reduces all-cause mortality. HYVET puts the question of the usefulness of treating hypertension to rest and provides important guidance to physicians and writers of such guidelines. (Kostis JB. New Engl J Med. 2008;358:1959.)

HYVET 2-Year Risk of CVA, Untreated vs Treated Hypertension: NNT


X X X X X X X

Frailty and Hypertension


(Odden et al. Arch Intern Med. 2012;172:1162.)

NHANES included 2340 participants age > 65; baseline 20 ft (6 m) walking test; BP checked 34 times; > 140/90 considered elevated. Walk at your usual pace. Walking speed < 0.8m/sec considered frail. 35% increased risk of mortality with systolic (not diastolic) hypertension in fast walkers. No increased mortality in slow walkers; inverse risk of death in those who did not complete walk.

Time to Benefit
(Lee SJ, JAMA on line E1, 12/9/2013)

75 yo with diabetes, advanced neovascular macular degeneration, mild CHF, hypertension, parkinsonism, and renal insufficiency returns for follow-up. Automated quality monitors identify 2 issues: No hemoglobin A1C in 2 years (last was 9.2) and BP > 140/90

Time to Benefit of Tight Control


With poor control of type 1 diabetes, prevalence of retinopathy can reach 90% at ten years after diagnosis. Time to go from 25% to 50% prevalence of retinopathy in type 2 @ 18-20 years. Time to prevention of microalbuminuria approximately five years.

Risks of Tight Control


Patients avg age 62 with Hgb A1C 6.4% had increased mortality and no reduction in CV events compared to A1C 7.5% (3.5 yrs). (ACCORD trial. N Engl J Med.
2008;358:2545.)

Hypoglycemia resulting in need for emergency room visit associated with increased risk of developing dementia; the more such episodes, the greater the risk.
(Alex J et al. JAMA. 2009;302(8):843.)

Role of ACE Inhibitors in Diabetes


Time to see benefit in slowing progression of renal disease (decline in creatinine clearance) may be as little as one year.

Management of Diabetes in Elderly with Comorbidity


Elderly patient living alone, with limited mobility, poor vision = risk for severe consequences of hypoglycemia. Prescribing ACEI may be more important than tighter control of blood sugars.

Practical Management of Elderly with Multiple Chronic Diseases


Choose treatments that have clinical impact within life expectancy of patient and fewest adverse effects. Choose treatments that may have benefit for more than one chronic disease.

Principles of Prescribing for the Elderly


Avoid polypharmacy. Beware of changes in compliance. Parsimony applies to treatment, not just differential diagnosis. Dont assume whats good for a younger patient (or otherwise healthy elderly) with one disease is good for an older patient with multiple diseases (silo thinking and time to benefit).

Answers
1. A 2. C 3. B 4. D 5. B 6. D 7. D 8. E 9. C 10. C 11. J

General Surgery II: The Surgical Abdomen


Robert Dachs, MD, FAAFP
Clinical Associate Professor and Director of Research Ellis Hospital Family Medicine Residency Program Albany Medical College Albany, New York

Disclosure Statement
Dr. Dachs has nothing to disclose.

It is the policy of the AAFP that all individuals in a position to control content disclose any relationships with commercial interests upon nomination/invitation of participation. Disclosure documents are reviewed for potential conflicts of interest. If conflicts are identified, they are resolved prior to confirmation of participation. Only participants who have no conflict of interest or who agree to an identified resolution process prior to their participation were involved in this CME activity

Learning Objectives
1.
2. 3.

Recognize the signs and symptoms of intra-abdominal conditions requiring surgical intervention.
Perform specialized maneuvers to narrow the differential diagnosis of acute abdominal pain. Appropriately utilize labs and radiology tests in the evaluation of acute abdominal pain.

4.

Appreciate that acute abdominal pain in children could be due to different etiologies than adult patients.

Abdominal Pain
Non-surgical Surgical acute abdomen

I. Peritonitis II. Obstruction


-Blood -Pus -Foreign material

Location, Location, Location


RUQ----------------> Hepatitis/GB LUQ-----------------> Gastric/spleen Periumbilical------> Pancreatitis/early appy RLQ-----------------> Appy/GYN (torsion, cysts,
PID, ectopic)

LLQ------------------> Diverticulitis/GYN General/ill-defined-> Ischemia, obstruction

Location, Location, Location


with visceral pain - ill-defined location RUQ----------------> Hepatitis/GB LUQ-----------------> Gastric/spleen Periumbilical------> Pancreatitis/early appy RLQ-----------------> Appy/GYN (torsion, cysts,
PID, ectopic)

LLQ------------------> Diverticulitis/GYN General/ill-defined-> Ischemia, obstruction

Location, Location, Location


when parietal peritoneum involved - localizes RUQ----------------> Hepatitis/GB LUQ-----------------> Gastric/spleen Periumbilical------> Pancreatitis/early appy RLQ-----------------> Appy/GYN (torsion, cysts,
PID, ectopic)

LLQ------------------> Diverticulitis/GYN General/ill-defined-> Ischemia, obstruction

Abdominal Pain and Analgesia: Give It!!


Pain relief does not increase the risk of diagnostic and management errors Dose: morphine 0.1 - 0.2 mg/kg IV
Manterola C, et al. Cochrane Database, 2011 Gallagher EJ, et al. Ann Emerg Med, 2006

Peritonitis and Antibiotics:


Antibiotics are effective in preventing postoperative complications following infection of the peritoneum, but there is not evidence to support that one regimen is superior to another Cochrane Library 2006, Issue 3
Regimens: Must cover Gr (-) aerobes and anaerobes Resistance of Bacteroides
PIP/TZ, AM-SB, TC-CL. Near zero Ertapenem, tigecycline FQ + metronidazole Cefoxitin, cefotetan 4 -25% (?add Metro?)

Nasogastric Tubes and Abdominal Surgery


37 randomized trials, post-op patients 2866 pts with NG tube, 2845 pts (-) NG tube
Results: No NG tube ===>
Earlier return of bowel function Fewer pulmonary infections Trend toward fewer wound infections and ventral hernias Trend toward improved patient comfort, less nausea and vomiting, and decreased length of stay No difference in anastomotic leaks Shorter length of stay Cochrane Library 2010

Acute Abdominal Pain in the Elderly: Be Careful!!!


More likely to require hospitalization More likely to require surgery More likely to be misdiagnosed More likely to die

compared to younger patients


Lyon C, et al. AFP, 2006 Laurell H. et al. Gerontology 2006

Abdominal Wall and Hernias

Suleiman S, et al. AFP Aug 2001, 431-38

Abdominal Wall and Hernias


1. A 72-year-old female presents with a 6 hour Hx of a painful lump in her groin. Some nausea, no vomiting. No fever/chills. Exam: Abdomen: soft, non-tender, R groin with palpable mass below (inferior to) inguinal ligament. Despite gentle, firm pressure, the mass does not disappear.

The most likely diagnosis is: A. Strangulated indirect inguinal hernia B. Reducible direct inguinal hernia C. Incarcerated femoral hernia D. Incarcerated Spigelian hernia

Abdominal Wall and Hernias


Epidemiology: - 1 million hernia repairs are performed each year in the US - inguinal hernia repairs constituting > 3/4 of these cases; - approximately 90% of all inguinal hernia repairs are performed on men Terminology: - Reducible: - Incarcerated: ie. Stuck! (despite gentle firm pressure) - Strangulated: vascular compromise (surgical emergency)

Abdominal Wall Hernias: Types


Indirect inguinal hernia: 75% of all hernias!!! -Most common hernia in both sexes!!! -Men are 5x more common -More likely to incarcerate/strangulate than direct Direct inguinal hernia: 2nd most common Note: You can NOT differentiate between Direct and indirect hernias on exam!!! Femoral hernias: more common in women -Located below inguinal ligament, medial to femoral pulse

Abdominal Hernias: Do You Remember These?


Spigelian hernia: rare, located along lateral border of rectus muscle, below umbilicus, at junction of arcuate line

Richter hernia: When less than full circumference of bowel is trapped

2. A 67 y/o female with a history rheumatoid arthritis presents with 6 hours of sudden, severe diffuse abdominal pain. Medications: Prednisone 15 mg qd x 5 yrs, celecoxib (Celebrex). Vital signs: BP = 140/90, P = 105, RR = 30, afebrile. Her abdomen is diffusely tender, with guarding and rebound.

The best screening study in this situation is:

A. B. C. D.

An upright CXR An abdominal series (flat and upright abdomen) A KUB A CT of the abdomen and pelvis with contrast

An Upright CXR is 80% Sensitive for Free Air Can detect as little as 1-2 mL of gas under the diaphragm or lateral margin of the liver

McCowan C. eMedicine Specialties. Large Bowel Obstruction. 2007 Aug 2.

Pneumoperitoneum: Etiology
Perforated duodenal ulcer - The most common cause
Especially of the anterior aspect of the first part of the duodenum

Perforated peptic ulcer Bowel obstruction Ruptured diverticulum Penetrating trauma Ruptured inflammatory bowel disease (eg, megacolon) Necrotizing enterocolitis Cancer Ischemic bowel After laparotomy; after laparoscopy

Pneumoperitoneum: Mimics
Subphrenic abscess Chilaiditi syndrome:
Bowel interposed between liver and diaphragm------------>

Ali Nawaz Khan. "eMedicine.com: Pneumoperitoneum"

Pneumoperitoneum: Benign etiology


S/P Laparoscopy/laparotomy: air remains up to 6 days

3. The 60 y/o male presents to the ED/office with 3 days of worsening RUQ pain. The pain radiates to the right scapula. (+) nausea and vomiting. He is febrile (temp = 39 C, 102.2 F), P = 122, BP = 90/45 He is jaundiced (total bilirubin = 7.8), RUQ is tender. The most likely diagnosis is: A. Acute cholecystitis B. Ascending cholangitis C. Gallstone pancreatitis D. Gallstone ileus

Gallstone Disease: Who Is at Risk?


First-degree relative with Hx of gallstones Obesity, sedentary life style, cyclic weight change Think recent bariatric surgery!!! Medical conditions: DM, hyperlipidemia, Surgical conditions: short bowel syndrome, terminal ileal resection (think Crohns disease) Drugs: ceftriaxone, post-menopausal estrogen, TPN Hemolytic diseases: Sickle cell disease ==> black pigmented stones

Gallstone Disease
Cholesterol stones most common (80%) Due to bile supersaturation with cholesterol ==> Crystal formation Asymptomatic stones (cholelithiasis) Do NOT recommend prophylatic surgery - 10% will go onto symptoms in 5 yrs Symptomatic stones (biliary colic)
Now consider surgery. Why? Randomized trial noted - Observation = 20% hospitalization rate with recurrent pain
- After 67 months - complication rates Observation = 4% vs. Surgery= 1% Vetrhus M, et al. Scand J Gastroenterol 2002

Complications of Gallstone Disease


Liver

. . . .
GB
Duodenum--

-----Common hepatic duct

-----------Cystic duct ----Common bile duct

Pancreas

Complications of Gallstone Disease


A. Cholecystitis: in 10% of patients with biliary colic - Occluded cystic duct ==> *RUQ pain and tender, +/-fever, WBC **(+) Murphy sign: cessation of inspiration B. Ascending (acute) cholangitis: occluded CBD (choledocholithiasis) 50-70%-Charcots triad----fever, abd pain, and jaundice -Reynolds' pentad---Charcots + confusion + shock
C. Gallstone pancreatitis: obstructed ampulla of Vater ===> reflux of bile D. Gallstone ileus: biliary-enteric fistula, 6/1000 SBO

Gallstone Disease: Evaluation


Clinical Suspicion: Classic history Ultrasound - 99% specific - If high degree of suspicion, but negative US. Think dyskinesia!!! Dyskinesia: gallbladder ejection fraction < 50%
(using a cholecystokinin cholecystoscintigraphy scan in conjunction with typical clinical symptoms) 94% of patients with dyskinesia, symptoms improve or the disease is cured after cholecystectomy1
1Canfield

AJ et al. J Gastrointest Surg 1998

4. Which of the following is NOT associated with acute appendicitis?

A. B. C. D.

Positive (+) Rovsing sign Positive (+) Psoas sign Positive (+) Obturator sign Positive (+) Carnett sign

Appendicitis
Develops in 6-7% of population It is the most common cause of surgical abdomen Due to intra-luminal (eg, appendicolith) OR extra-luminal obstruction (eg, lymphadenopathy)

Rovsings sign: LLQ palpation ==> RLQ pain


Psoas sign: RLQ pain on thigh extension while lying in left lateral decubitus position Obturator sign: RLQ pain with internal rotation of the flexed right thigh

Carnett sign: perform abdominal crunch


(+) if pain persists/worsens --> abdominal wall pathology

Absence of These Signs Does Not Rule Out Appendicitis


Psoas sign: suggests
inflamed appendix is retrocecal irritating iliopsoas muscle

Obturator sign: suggests


inflamed appendix is in pelvis irritating obturator internus muscle

HARDIN DM. Am Fam Physician.1999 Nov1;60(7):2027-2034.

Two More Signs.


Dunphy sign: cough illicits pain (also known as cough sign) Markle sign: patient drops from standing on toes
to the heels with a jarring landing ==> pain

Both suggest peritoneal inflammation

And the rectal exam: is useless!!!


Study: 1024 pts with possible appy + rectal exam Physicians asked opinion before and after exam Result: made no difference1
1 Dixon JM et al. BMJ 1991;302: 386-388.

Appendicitis: Labs
WBC
Elevated in 80-85% of pts But only 60-65% in children and elderly!!!

CRP elevates within 6-12 hrs


Various studies have attempted to combine: WBC + CRP (+/-) neutrophilia Sensitivity: 85% - 99% Specificity: very poor!!!!

Appendicitis: Signs and Symptoms


Helpful for ruling in AP RLQ pain Migration (periumbilical to RLQ) Psoas sign Fever Pain before vomiting Rebound tenderness Helpful for ruling out AP Likelihood Ratio Adults Children 8.4 -3.6 1.9 - 3.1 3.2 2.5 3.2 3.4 2.7 -2.0 3.0
--0.33

Absence of pain before vomiting 0.02 Absence of RLQ pain 0.18 Absence of vomiting --

Adapted from Ebell, M. AFP 2008

5. You are evaluating a 6 year old for abdominal pain. You have a moderate degree of suspicion that the patient may have appendicitis. The American College of Radiology Appropriateness Criteria recommends which of the following as the first choice to evaluate this patient?

A. Abdominal x-ray B. Abdominal ultrasound C. CT scan of abdomen and pelvis D. MRI of abdomen and pelvis

Appendicitis: Diagnostic Imaging


X-ray: rarely useful
Unless appendicolith seen

US: first choice: age < 14yrs and pregnancy CT: study of choice in adults

26 studies (as of 2006) of US in pediatrics Sensitivity: 88%; specificity of 94%1 No advantage to using contrast

MRI: good sensitivity and specificity

greater cost, longer acquisition time, and less clinical availability1Doria AS, et al. Radiology. 2006;241(1):83-94

Intestinal Obstruction
6. The most common cause of intestinal obstruction is:
A. B. C. D.

Adhesions (from previous surgery) Neoplasm Herniation Volvulus

Intestinal Obstruction
Small bowel (85%)
Adhesions from previous surgery (60%) Malignant tumor (20%) Herniation (10%) Inflammatory bowel disease (5%) Volvulus (3%) Misc (2%)

Large bowel (15%)


Neoplasm (60%) Diverticulitis (20%) Volvulus (10-15%) Intussusception (< 5%) Misc

Small Bowel Obstruction


Colicky abdominal pain Nausea and vomiting Diarrhea (early finding) Distention Tympany, high-pitch BS Lack of flatus and constipation Fever/tachycardia (with strangulation) Abdominal tenderness and peritoneal signs (with strangulation)

Symptoms:

Signs:

Late Findings:

Small Bowel Obstruction: Radiography


Dilated loops of bowel --> Paucity of air in colon Air fluid levels

QuickTime and a decompressor are needed to see this picture.

Jackson PG, et al. AFP 2011

Small Bowel Obstruction: Radiography


Dilated loops of bowel Paucity of air in colon Air fluid levels -------->
May be normal: ie, false neg (-) early or in high jejunal or duodenal obstruction

High clinical suspicion and neg(-) X-ray should ==> Non-contrast CT evaluation

From: Wikipedia

Small Bowel Obstruction: Management


Clinically stable: no suggestion of strangulation
NGT, IV hydration, analgesia Nonoperative trial up to 3 days is warranted. Successful 40-70%

If unstable/peritonitis: (+) suggestion of


strangulation.

Go to surgery!!!!

Large Bowel Obstruction


Symptoms
More common in elderly Crampy abdominal pain Sudden onset: think acute obstruction (eg, volvulus) Nausea, vomiting, distension

Signs
Hyper-tympanic to percussion Bowel sounds normal early, then become quiet Fever/tenderness/rigidity - is a bad sign

Cecum is region most likely to perforate

Large Bowel Obstruction


A closed loop obstruction
(with a competent ileo-cecal valve)

Cecum preferentially dilates Dilation > 12-14 cm increases risk of perforation

QuickTime and a decompressor are needed to see this picture.

www.practicalhospital.com

Sigmoid Volvulus
A long mesentery with a narrow base of fixation to the retroperitoneum and elongated, redundant bowel predisposes to the formation of volvulus

Sigmoid colon most common Result of chronic constipation At risk patients: Neuropsychiatric disorders Institutionalized/NH patients Parkinson disease Multiple sclerosis Spinal cord injury Excessive laxatives,enemas
QuickTime and a decompressor are needed to see this picture.

Thorton, S. eMedicine Sigmoid and Cecal Volvulus. 2013 June 7.

Sigmoid Volvulus
Average age: 8th decade
Treatment: endoscopic decompression - Then semi-elective surgery - If no surgery: up to 50% will have recurrence - 12-15% mortality rate

QuickTime and a decompressor are needed to see this picture.

Thorton, S. eMedicine Sigmoid and Cecal Volvulus. 2013 June 7.

Cecal Volvulus
Hereditary condition: congenital incomplete
dorsal mesenteric fixation of the cecum or ascending colon associated with an abnormally elongated mesentery distal to this area of absent mesentery

Women > men Most common: 6th decade of life


QuickTime and a decompressor are needed to see this picture.

Treatment: Surgery Right hemicolectomy

Thorton, S. eMedicine Sigmoid and Cecal Volvulus. 2013 June 7.

Acute Colonic Pseudo-Obstruction (ACPV) Olgilvie Syndrome


Population at risk: older, hospitalized
Perforation (cecum) reported 3-40% Treatment:
NPO, IV fluids, rectal tube, correct electrolytes Neostigmine is effective in 85-90% of cases
Lasts 1-2 hours, multiple doses may be needed
Cardiac monitor, atropine at bedside

With a variety of medical and surgical conditions

Methylnaltrexone (Relistor): if opiate-induced Colonoscopy is medical therapy fails

Vascular Disease and the Abdomen: 4 Cases

SMA IMA

A. Chronic mesenteric ischemia (Abdominal angina) B. Acute mesenteric arterial thrombosis C. Ischemic colitis D. Acute mesenteric venous thrombosis

Vascular Disease and the Abdomen: 4 Cases

SMA IMA

A. Chronic mesenteric ischemia (Abdominal angina) B. Acute mesenteric arterial thrombosis C. Ischemic colitis D. Acute mesenteric venous thrombosis

Case #1: 60 y/o female 4-6 month hx of worsening postprandial abdominal pain. Described an ache, in midepigastric-central location. Pain starts 15 min. after eating, increases over the next 1-2 hrs., then abates. Worse with large meals. Subsequently, she has lost 20 lbs in 6-8 weeks. PMHx: HTN, hyperlipidemia, tobacco abuse Physical exam: unremarkable Labs, CT scan, upper and lower endoscopy - all normal She has been told she has IBS

A. B. C. D.

Chronic mesenteric ischemia Acute mesenteric arterial thrombosis Ischemic colitis Acute mesenteric venous thrombosis

Chronic Mesenteric Ischemia Also Known as - Abdominal Angina


Etiology: atherosclerotic disease
Involves proximal celiac, SMA, IMA arteries Because of collateral circulation, 2 of 3 involved

Mean age: 60 yrs, Women > men; 3:1 Presentation: post-prandial pain Work-up: Duplex US, CT-A, MRA or conventional angiography Treatment: angioplasty, stenting, surgery

Vascular Disease and the Abdomen: 4 Cases

SMA IMA

A. Chronic mesenteric ischemia (Abdominal angina) B. Acute mesenteric arterial thrombosis C. Ischemic colitis D. Acute mesenteric venous thrombosis

Case #2: 78-y/o male presents to ED with 90-minute history of severe diffuse abdominal pain. Started suddenly at 7:30 pm. (+) nausea, vomiting. No change in bowel/bladder habits. PMHx: CAD, HTN, Afib, frequent falls, early dementia Meds: metoprolol, ASA, donepezil (Aricept) VS: BP = 115/70, P = 110, irreg. RR = 28, afebrile PE: The patient is writhing in bed, very restless, little to no relief with opiate analgesia. Abdomen: soft, non-tender, absent bowel sounds, no rebound or guarding.

A. B. C. D.

Chronic mesenteric ischemia Acute mesenteric arterial thrombosis Ischemic colitis Acute mesenteric venous thrombosis

Acute Mesenteric Arterial Occlusion


Etiology: embolus > thrombosis SMA preferentially involved Presentation: Pain out of proportion to physical findings Mortality: very high

Vascular Disease and the Abdomen: 4 Cases

SMA IMA

A. Chronic mesenteric ischemia (Abdominal angina) B. Acute mesenteric arterial thrombosis C. Ischemic colitis D. Acute mesenteric venous thrombosis

Case #3: 82 y/o NH resident with one week of progressive weakness. Crampy abdominal pain, (+) diarrhea. Rare vomiting and decreased po intake Outpatient identified UTI - Rx with nitrofurantoin (Macrodantin). Vitals: BP = 90/45, P = 115, RR = 28, T = 100.6 PE: Ill-appearing, restless. Mucus membranes: very dry. The abdomen is distended and tympanic; absent BS, Mild left-sided tenderness, but no rebound or guarding. Rectal exam = brown stool, heme(+) Labs: Na+ 146, K+ 2.7, Cl 115, HCO3- 15, BUN 66/Cr 2.2 Wbc = 15.5, H/H = 10.5/31.5 U/A = WBC - TNTC

A. B. C. D.

Chronic mesenteric ischemia Acute mesenteric arterial thrombosis Ischemic colitis Acute mesenteric venous thrombosis

Ischemic Colitis
Etiology: low-flow state
Non-gangrenous - 80-85%; gangrenous - 15-20%

Age: 90% over the age of 60 yrs Location: left colon 75% Presentation: varies on severity
Mild case: crampy abd pain, diarrhea, heme (+) stool Severe: Peritonitis

Work-up: CT, MRA or colonoscopy Treatment:


Mild case: liquid diet, antibiotics Severe: IV fluids, antibiotics, surgery

Vascular Disease and the Abdomen: 4 Cases

A. Chronic mesenteric ischemia (Abdominal angina) B. Acute mesenteric arterial thrombosis C. Ischemic colitis D. Acute mesenteric venous thrombosis

Case #4: 42 y/o female with 7-10 day hx of vague, mid-abdominal pain. (+) nausea and vomiting, (+) anorexia and occasional diarrhea. No fever. PMHx: HTN FHx: Mother PE age 40, Father: Unknown Meds: HCTZ, OCP SHx: married, (+) smokes 1pk/day Vitals: BP = 130/70, P = 105, RR = 28, T = 99.4 PE: Appears uncomfortable, non-toxic. Abdomen: soft, slightly distended and tympanic, rare BS, no rebound/guarding. Rectal exam = brown stool, heme(+) Labs: WBC = 15.5, H/H = 10.5/31.5, HCO3 = 18.

A. B. C. D.

Chronic mesenteric ischemia Acute mesenteric arterial thrombosis Ischemic colitis Acute mesenteric venous thrombosis

Acute Mesenteric Venous Thrombosis


Frequency: 10-15% of all mesenteric ischemia At-risk population: hypercoagulable state Presentation: insidious over 7-10 days Work-up:
X-rays: non-specific findings (ileus)
Late findings: thumbprinting, pneumatosis or portal venous gas

Duplex US: useful if used early CT: may ==> enlarged mesenteric or portal vein and thrombus within the vein
Gas in the wall of the bowel, fat streaking, and thickened bowel wall have also been noted

Treatment: anticoagulation, thrombolytics


Surgery: for bowel infarction, peritonitis

Intestinal (Mesenteric) Ischemia: Remember Mechanisms


A. Low-flow state: ischemic colitis B. Mesenteric venous thrombosis: think - hypercoaguable states C. Arterial occlusion: 1. Atherosclerosis: intestinal angina 2. Thrombo-embolic phenomenon:
Pain out of proportion to physical findings

Pediatric Surgery
7. A 7-month-old male presents with 24 hours of abdominal pain, vomiting, and blood in the diaper. The pain appears episodic and severe interspersed with periods of lethargy. Exam: VSS, afebrile Abdomen: soft, bowel sounds absent. A mass is noted in RUQ and the RLQ feels empty.

The most likely diagnosis is:


A. B. C. D.

Necrotizing enterocolitis (NEC) Meckels diverticulum Intussusception Hypertrophic pyloric stenosis

Intussusception
Sex: Male > female; 3:1 Etiology: unclear. Likely lead point => invagination Peak age: 5-10 months Presentation: Classic triad (Note: triad only present 30% of cases)
A) vomiting, B) abdominal pain, c) currant jelly (bloody) stool

Abdominal pain: colicky, severe, intermittent Abdominal exam R-sided sausage-like mass, Empty RLQ (Dance sign) Diagnosis: U/S, barium enema Treatment: Barium, water-soluble or air-contrast enema

Lead Points: Adults= polyps, cancer Child = idiopathic - ? Peyers patch

Meckels Diverticulum: The Rule of 2s


2% of the population harbor it
Complications occur in minority (4-16%)

2x more common in males Within 2 feet of ileocecal valve Presentation: with one of 2 complications 1) Children: Painless bleeding
Due to heterotopic gastric tissue

2) Adults: Obstruction Diagnosis: technetium-99m pertechnetate scan


known as a Meckels scan (80-90% sensitive)

Hypertrophic Pyloric Stenosis


Etiology: Marked hypertrophy and hyperplasia of the muscular layers
of the pylorus --> leading to narrowing of the gastric antrum.

Sex: Male > female; 4:1 Peak age: 3 weeks (Range: 1-18 weeks) Rare in premature infants Presentation: nonbilious vomiting/regurgitation After which the infant is still hungry Dehydration and poor weight gain can occur Abdominal exam: RUQ olive Diagnosis: Ultrasound Treatment: Surgery

Necrotizing Enterocolitis (NEC)


Incidence: increases with prematurity Presentation: < 30 weeks EGA: ave. time of onset = 20.2 days 31-33 weeks EGA: ave. time of onset = 13.8 days > 34 weeks EGA: ave. time of onset 5.4 days Term infants: usually develop in first 1-3 days of life Presentation: starts with feeding intolerance Later: vomiting, diarrhea, Clinical triad may ultimate develop A) abdominal distension, B) bloody stools, and C) pneumatosis intestinalis*** (pathognomonic) Treatment: NGT decompression, antibiotics, TPN

Answers
1. C 2. A 3. B 4. D 5. B 6. A 7. C

Managing Dysrhythmias
Jonathon Firnhaber, MD, FAAFP

Assistant Professor The Brody School of Medicine at East Carolina University Greenville, North Carolina

Disclosure Statement
Dr. Firnhaber has nothing to disclose.

The AAFP has selected all faculty appearing in this program. It is the policy of the AAFP that all CME planning committees, faculty, authors, editors, and staff disclose relationships with commercial entities upon nomination or invitation of participation. Disclosure documents are reviewed for potential conflicts of interest and, if identified, they are resolved prior to confirmation of participation. Only those participants who had no conflict of interest or who agreed to an identified resolution process prior to their participation were involved in this CME activity.

Learning Objectives
1. Differentiate the diagnosis and management of Mobitz type I and Mobitz type II AV heart block. 2. Analyze the diagnosis and management of common forms of supraventricular arrhythmias. 3. Assess the diagnosis and management of sinus note disease. 4. Determine the diagnosis and management of ventricular tachycardia.

1. A 52-year-old male who is an tennis player has stage 1 hypertension. PMH is benign. His electrocardiogram is shown below.

1. A 52-year-old male who is an avid golfer and tennis player is diagnosed with stage 1 hypertension. PMH is unremarkable. His electrocardiogram was shown previously. Given the ECG findings, which of the following drugs would be unsafe to use in this patient?

A. Dihydropyridine calcium channel blockers B. Non-dihydropyridine calcium channel blockers C. -blockers D. Central 2-agonists E. Cyanide

Sinus Bradycardia, 1st Degree AVB

Both ECG findings are commonly associated with higher degrees of physical conditioning Neither is a contraindication to the use of -blockers or nondihydropyridine CCBs (or any other antihypertensives)

2. A 52-year-old man with a history of COPD and hypertension presents with worsening fatigue. He denies chest pain or shortness of breath.
Exam is notable for a slow, irregular pulse; his lungs are clear. His current medications include lisinopril, chlorthalidone, tiotropium, and ASA.

Which does this rhythm strip show?

A. Mobitz type I AV block B. Mobitz type II AV block C. Blocked premature atrial contraction D. Complete (third-degree) heart block

Mobitz Type I Second-Degree AV Block (Wenckebach)


The PR interval progressively lengthens until a P wave fails to conduct and a beat is dropped.

3. What is the most likely cause of this dysrhythmia?


A. Anterior wall infarction B. Dihydropyridine calcium channel blocker therapy C. Inferior wall ischemia D. Hyperthyroidism

Mobitz Type I Second-Degree AV Block (Wenckebach)


Almost always represents disease of the AV node.
May be seen in athletically fit individuals, especially during sleep.

In the acute setting, inferior wall ischemia is likely.


Inferior wall is supplied by RCA, which also supplies the AV node. Anterior wall is supplied by the left coronary artery, which supplies the His-Purkinje system.

Treatment: the rhythm itself generally does not require treatment; the underlying cause may.

Mobitz Type I Second-Degree AV Block (Wenckebach)


-blockers and non-dihydropyridine CCBs (diltiazem and verapamil) slow conduction in the AV node
Dihydropyridine CCBs (all end in pine) generally do not cause significant AV slowing

Hypothyroidism can cause AV slowing; hyperthyroidism does not

Mobitz Type II Second-Degree AV Block

Characterized by intermittently nonconducted P waves not preceded by PR prolongation and not followed by PR shortening.

Mobitz Type II Second-Degree AV Block


Almost always represents disease of the distal conduction system, below the AV node: His-Purkinje system May progress to third-degree heart block, with no emerging escape rhythm Treatment: permanent pacemaker

Third-Degree AV Block (Complete Heart Block)

Characterized by a regular rhythm with complete AV dissociation. Impulses generated by the SA node do not propagate to the ventricles. Two independent rhythms can be noted on the ECG.

Third-Degree AV Block (Complete Heart Block)


If the result of inferior MI, AV node may recover
Escape rhythm typically originates in AV junction, and is narrow-complex

If the result of anterior MI, distal conduction system is typically permanently damaged
Escape rhythm originates in the ventricles, and is wide-complex

Treatment: permanent pacemaker

4. A 24-year-old college student presents with an intermittent sensation of rapid heartbeat. These episodes may occur at rest or with exertion and seem to start and stop abruptly.
The patient is a nonsmoker and denies alcohol or drug use. There is no history of heart disease.

Which does this rhythm strip show?

A. B. C. D. E.

Atrial flutter Supraventricular tachycardia (SVT) Multifocal atrial tachycardia (MAT) Ventricular tachycardia (VT) Pacemaker-mediated tachycardia (PMT)

Narrow Complex Tachycardia

Sinus tachycardia and SVT are both regular.


Recall sinus tach is a secondary rhythm.

Atrial flutter can be regular or irregular. MAT is always irregular. VT and pacemaker-mediated tachycardia are wide-complex rhythms

5. Which of the following interventions is not appropriate to quickly help define this narrow-complex rhythm?
A. Vagal maneuvers B. IV adenosine C. IV digoxin D. IV -blocker

Narrow Complex Tachycardia


Options to quickly slow AV conduction include: Vagal maneuvers: Valsalva, unilateral carotid massage
IV adenosine: 6 mg bolus; follow with 12 mg if ineffective IV -blocker: metoprolol 5 mg IV diltiazem: 15-30 mg bolus

Ventricular response in sinus tachycardia and atrial flutter gradually slows; ventricular response in SVT abruptly converts to sinus rhythm. Digoxin also slows AV conduction, but because it requires loading over hours, it is not quickly effective.

Supraventricular Tachycardia

SVT is a regular, narrow-complex tachycardia. A vagal maneuver (arrow) results in abrupt termination. An escape beat is also seen (arrowhead)

SVT: Treatment Options


Drug therapy for acute treatment Electrical cardioversion is appropriate if hypotensive or ongoing chest pain. Long-term therapy with digoxin, diltiazem, or -blocker Ablation of the arrhythmia is an alternative.

Atrial Flutter

Atrial flutter is a regular or regularly irregular narrow-complex rhythm that is typically rapid. Vagal maneuver (arrow) slows AV conduction and makes the flutter waves more apparent (arrowheads) The atrial rate is ~300. Conduction is expressed as atrial beats:ventricular beats (eg, 3:1, 2:1).

Multifocal Atrial Tachycardia

MAT is an irregular narrow-complex rhythm with 3 or more P waves of variable morphology. Most common in patients with lung disease; can occur post-MI or with hypokalemia or hypomagnesemia. Rate may be reduced by using IV verapamil. Differences from wandering atrial pacemaker (WAP): significantly increased rate and almost invariable association with severe pulmonary disease.

6. For long-term therapy, the most effective control of heart rate in atrial fibrillation, both at rest and with exercise, occurs with which one of the following?

A. Digitalis B. Calcium channel blockers C. Class 1A antiarrhythmics D. -blockers

Atrial Fibrillation

Atrial fibrillation is an irregularly irregular narrow-complex rhythm that may be rapid. The atrial rate is > 300 bpm. No atrial flutter waves or discrete P waves are noted.

Atrial Fibrillation Therapy


For long-term therapy, -blockers provide the most effective control of heart rate in AF, both at rest and during exercise CCBs, particularly diltiazem, lower rate at rest and with exercise, but overall are not as effective as -blockers Digitalis more effectively controls rate at rest; it may not control rate with exercise Class 1A antiarrhythmics can help maintain sinus rhythm but may increase heart rate

Atrial Fibrillation Therapy


Steps in treatment:
Control rate Select anticoagulation (ASA or warfarin) Consider conversion to sinus rhythm
Medical/electrical

If the ventricular rate exceeds 210 bpm, suspect a pre-excitation bypass tract
Wolff-Parkinson-White syndrome, involving the bundle of Kent, causing a delta wave

7. Your patient with atrial fibrillation asks whether he should take warfarin to reduce his risk of stroke. Which of the following is a component of the CHADS2 score? A. Congestive heart failure B. Hyperlipidemia C. Age > 50 D. Diabetes for > 10 years E. Systolic hypertension

Atrial Fibrillation: CHADS2


Risk < 2 strokes per 100 patient-years with aspirin: little to gain from warfarin. Risk > 4 strokes per 100 patient-years with aspirin: consistent improvement in quality-adjusted survival with warfarin. Between these extremes, risk quantification is critical.

CHADS2 score relates to non-valvular AF.


Not considered are: valvular heart disease, prior peripheral embolism, intracardiac thrombus, hyperthyroidism.

Atrial Fibrillation: CHADS2


Congestive heart failure Hypertension Age > 75 Diabetes Prior Stroke or TIA Score: 0 1 >2 = 1 point = 1 point = 1 point = 1 point = 2 points

Low risk; ASA therapy Moderate risk; ASA or warfarin therapy Moderate-high risk; warfarin therapy

8. Which one of the following statements concerning sinus node disease is correct?
A. Sinus node disease is most commonly due to left anterior descending coronary artery disease. B. Sinus node disease may present with episodic sinus arrest. C. A pacemaker is indicated for all cases of sinus node disease. D. Sinus node disease is commonly associated with hyperthyroidism.

Sinus Node Disease


May present with:
Tachy-brady syndrome Persistent sinus bradycardia Persistent sinus tachycardia Episodic sinus arrest

Most common in patients > 60 years

Sinus Node Disease


Causes include:
Intrinsic aging Superimposed drug effect Right or circumflex coronary artery disease Hypothyroidism

Pacemaker therapy is indicated only in symptomatic patients.

Question 8 Rhythm Strip

Sinus arrest due to sinus node disease. Note the lack of a P wave preceding the pause. Junctional escape beats may be seen (AV nodal origin; narrow complex; no preceding P wave).

9. Which statement concerning ventricular tachycardia (VT) is correct?


A. The presence of fusion beats excludes VT B. VT is considered sustained if pharmacologic intervention is required to convert the rhythm C. Torsades de pointes is typically associated with hypocalcemia D. AV dissociation is one feature of VT

Ventricular Tachycardia (VT)


> 3 beats in a row that originate from the ventricle at a rate of more than 100 bpm Self-termination within 30 seconds: nonsustained VT Duration > 30 seconds: sustained VT (even if it ultimately self-terminates) Signs of VT:
AV dissociation Capture beats Fusion beats Concordance

Ventricular Tachycardia (VT)


Causes:
Hypoxia Electrolyte disturbance Ischemia Drug toxicity Heart failure Prolonged QT interval

Acute therapy:
Lidocaine Amiodarone

VT with AV Dissociation

VT with Fusion Beat (2nd) and Capture Beat (4th)

VT with Fusion Beat

VT with Concordance

Torsades de Pointes
Twisting of the points

Polymorphic VT, characterized by a cyclical progressive change in cardiac axis. Usually nonsustained; may evolve into ventricular fibrillation. Associated with hypomagnesemia, hypokalemia and medications or conditions that prolong the QT interval.

References: Arrhythmias
Gage B. Selecting Patients With Atrial Fibrillation for Anticoagulation. Circulation. 2004;110:2287-2292. Wagner G. Marriotts practical electrocardiography. 11th ed. Philadelphia, PA: Lippincott Williams & Wilkins; 2008

Answers
1. 2. 3. 4. 5. 6. 7. 8. 9. E A C B C D A B D

Trauma and Wound Care


Robert Dachs, MD, FAAFP
Clinical Associate Professor and Director of Research Ellis Hospital Family Medicine Residency Program Albany Medical College Albany, New York

Disclosure Statement
Dr. Dachs has nothing to disclose.

It is the policy of the AAFP that all individuals in a position to control content disclose any relationships with commercial interests upon nomination/invitation of participation. Disclosure documents are reviewed for potential conflicts of interest. If conflicts are identified, they are resolved prior to confirmation of participation. Only participants who have no conflict of interest or who agree to an identified resolution process prior to their participation were involved in this CME activity

Learning Objectives
1. 2. 3. 4. 5. 6.

Review the ABCs of trauma care. Recognize and identify treatment for blunt head trauma. Recognize common injuries associated with child abuse. Evaluate and triage complicated extremity trauma. Develop wound care strategies for common lacerations. Recognize when to administer tetanus prophylaxis.

Trauma Facts
Leading cause of death: up to age 44
MVA (31%), suicide (21%), homicide (14%)

MVA: 50,000 deaths/yr, 40% intoxicated


Management: ABCs
Airway Pearls: Tongue is most common cause of obstruction

AIRWAY:
1. You come upon the scene of an MVA. The only victim is the driver, who has been extricated from the car and is lying on the ground unresponsive. Onlookers are standing around. Your first step is to open the airway. In this scenario, the recommended maneuver to open the airway is:

1. Head-tilt chin-lift maneuver

2. The jaw thrust maneuver


3. Turn the head to the side to avoid vomiting 4. Perform a roadside cricothyroidotomy

Opening the Airway: ACLS Recommendation 1. Head-tilt-chin lift 2. Jaw thrust

-no evidence of trauma - (+) trauma

Class IIB recommendation

Breathing:
2. A 27 y/o male unrestrained driver of car brought to ED

after a rollover MVA. On arrival, he is awake, can clearly state his name, but due to severe tachypnea, cannot give more than one-word answers. Vitals: Pulse = 150, RR = 48 labored, BP = 65/30, O2 sat = 81% on 100% O2. PE: Neck: Trachea deviated to left, (+) JVD Lungs: Absent breath sounds on right, right chest tympany

At this point, you should:


1. Perform a rapid sequence intubation (RSI)
2. Insert a 14-gauge needle in right second intercostal space 3. Insert a chest tube on right side 4. Obtain a STAT upright CXR

Circulation Issues:
When hemorrhagic shock is the issue
Hypotension and Pulse > 100 does not occur until 20% volume loss No tachycardia in patients on B-blockers Monitor urine output: > 0.5cc/kg/hr Universal donor: Type O negative, try to avoid Transfusions can only be given with normal saline
Other causes of shock in trauma: Tamponade, tension pneumothorax, neurogenic

Circulation:
3. A 6 y/o male is rushed to ED after being struck by a car. He is awake and crying. Pale, diaphoretic. Vitals: BP = 65/30, P = 160, RR = 30 Head: no trauma. Neck: immobilized. No JVD, trachea midline. Lungs: clear bilaterally. Heart: RR, no murmur. Abdomen: diffusely tender with guarding and rebound. Right femur with diffuse swelling, tender and deformed.

The initial fluid resuscitation in this patient is:

1. 2. 3. 4.

20 mLs/kg isotonic saline bolus 20 mLs/kg D5 1/2 normal saline bolus 20 mLs/kg 1/2 normal saline bolus 20 mLs/kg 1/4 normal saline bolus

Pediatric Trauma/Fluid Resuscitation


Initial: 20 mL/kg isotonic saline After 3 boluses, if shock persists, Start PRBC 10 cc/kg

3 in 1 Rule
ATLS Text, 7th edition, 2004 Neville KA, et al. Arch Dis Child, 2006

Primary Survey in Trauma


Airway
Breathing

Circulation
Disability Exposure

Head Trauma
4. A 22-year-old male presents to ED after being struck on right side of head with a baseball bat. He walked into the ED, recalled events. Vital signs: stable, GCS = 15. Right scalp with large STS. No neurologic deficits. 15 minutes later, the patient is minimally responsive and the right pupil is now fixed and dilated. STAT CT scan is likely to show:

1. 2. 3. 4.

Right side epidural hematoma Right side subdural hematoma A basilar skull fracture An intracerebral contusion

Head Trauma
Accounts for 50% of all deaths due to trauma A. Intracerebral hemorrhage Often not seen on initial CT (delayed presentation) B. Epidural hematoma 80% due to rupture middle meningeal artery Rare in the elderly, associated with skull fx Lucid interval (talk and deteriorate) C. Subdural hematoma Tear of bridging veins between dura and arachnoid Common in elderly, alcoholics
6x more common than epidural Higher mortality than epidural

Epidural Hematoma

Subdural Hematoma

Convex

Concave, crescent shape

5. Which of the following statements is false?


1. Protective helmets reduce head injury and death in motorcyclists. 2. Protective helmets reduce head and facial injuries in bicyclists. 3. Protective helmets reduce head and facial injury in skiers. 4. IV steroids are useful in the management of head injury.

Head Trauma
Motorcycle helmets decrease head injury (69%) and death (42%)
Cochrane Library 2008, Issue 3

Bicycle helmets decrease head injury (63-88%) and facial injuries (65%)
Cochrane Library 2008, Issue 3

Skiing helmets decrease head injury (60%)


Sulheim, S. et al. JAMA 295:919, Feb 22, 2006

Head Trauma
Steroids provide no benefit, possible harm
20 trials, 12,303 randomized pts. Largest trial associated with increased risk of death Cochrane Library 2009, Issue 2

6. A 4 y/o male is brought to your ED/office after falling in the garage, striking his forehead. No LOC, he has vomited x2. On exam: (+) soft tissue swelling of the forehead. No Battle sign, raccoon eyes or otorrhea or rhinorrhea. He is neurologically intact, appears comfortable, playful. The 2007 AAFP/AAP Guideline on the Management of Minor Closed Head Injury in Children recommends:

A. Observation for 2 hours in a medical professional setting B. Observation for 24 hours in a medical professional setting C. Observation for 24 hours either by medical professionals or reliable family D. The child should undergo CT scan of the head

The 2007 AAFP/AAP Guideline on the Management of Minor Closed Head Injury in Children
2 - 20 year olds With isolated head trauma (not multiple trauma) Normal mental status No focal neurologic findings No evidence of skull fracture

/
No LOC Observe for 24 hrs
(medical or reliable family)

\
(+) LOC < 1 minute Observe is acceptable
OR

No radiology studies

Obtain head CT

Andavoid skull x-rays and MRI

The 2007 AAFP/AAP Guideline on the Management of Minor Closed Head Injury in Children
The rationale for observation alone:
Without LOC: < 1:5000 chance of intracranial injury requiring neurosurgical intervention

What about vomiting, headache, or lethargy?


These signs are of limited predictive value

What about children under age of 2 years?

2001 AAP Guidelines for Minor Head Injury in Children < 2 Years of Age
Literature review says these are clinical predictors of intracranial injury (ICI):
Skull fracture (SF) Scalp swelling (80-100% of SF have swelling) Younger the age, greater risk Inflicted injury (abuse) No clear history of trauma

Not predictors:
Loss of consciousness Vomiting

2001 AAP Guidelines for Minor Head Injury in Children < 2 Years of Age
High risk (any one)
-Depressed mental status -Focal neuro deficits -Signs of skull fracture -Seizure -Irritability -Bulging fontanel -Vomiting > 5x -LOC > 1 min

Low risk (needs all)


-Low energy mechanism (eg, fall from < 3 feet)

-No signs or symptoms (eg, no vomiting, no STS)


-Older age more reassuring (eg, age > 12 months

Go to CT

Observation - (no time frame) OK with reliable parents Observe 4-6 hrs post-injury

Everything else: Intermediate risk / \

-In hospital setting

OR Radiology testing

So Why Not CT Scan Them All?


Risk of sedation Costs Radiation exposure (linked to future cancer)*
Radiation = to 100-150 chest x-rays)

Impairs later (adult) cognitive abilities**


*Mathews JD, et al. Cancer risk in 680,000 people exposed to CT scans in Childhood: data linkage study of 11 million Australians. BMJ, May 22, 2013 **Effect of low doses of ionising radiation in infancy on cognitive function In adulthood: Swedish population based cohort. Hall P, et al BMJ Jan 3, 2004

7. A 45-year-old male driver of car involved in MVC at intersection. Struck on passenger side. Wearing seatbelt. No LOC. Not ambulatory at scene. EMS transports pt. to ED on a backboard and hard (Philadelphia) collar in place. On arrival, the patient has no complaints. VS: normal, not intoxicated, no neuro deficits, no significant injuries other than an abrasion on the L hand. No midline posterior neck tenderness. At this point:

1. The cervical collar can be removed.

2. C-spine x-rays are needed before the collar can be removed. 3. If the C-spine shows no fracture, flexion and extension views should be obtained. 4. Because C-spine x-rays can miss significant injury, a CT scan should be obtained.

NEXUS (C-Spine) Guidelines


1. 2. 3. 4. 5.

No posterior midline cervical tenderness No alcohol intoxication A normal level of alertness No focal neurologic deficits No distracting injuries

If meets all, no C-spine x-ray required!!!


Prospective study: 34,069 pts. at 21 trauma centers==> 100% Sensitive!!!
Hoffman, JR, et al. NEJM, 2000

Neck Trauma
Myth: A soft collar is standard care for whiplash

Richell-Herren, KJ. J Accident Emerg Med 16(9): 363, Sept 1999 4 of 5 studies note increased pain and decreased mobility with collar

(Blunt) Chest Trauma


Chest wall Lungs/pleura
Heart/great vessels

(Blunt) Chest Trauma


Chest wall
Rib fracture: #4-9 most common - painful! Flail chest: 3 ribs with > 2 fractures each

Lungs/pleura
Heart/great vessels

(Blunt) Chest Trauma


Chest wall
Rib fracture: #4-9 most common - painful! Flail chest: 3 ribs with > 2 fractures each

Lungs/pleura
Hemo/pneumothorax Pulmonary contusion-->

Heart/great vessels

(Blunt) Chest Trauma


Chest wall
Rib fracture: #4-9 most common - painful! Flail chest: 3 ribs with > 2 fractures each

Lungs/pleura
Hemo/pneumothorax Pulmonary contusion

Heart/great vessels
Aortic tear -------------> Cardiac tamponade

(Blunt) Chest Trauma:


Commotio Cordis
216 cases in registry Blunt trauma (eg, ball)

Strikes before peak of T wave

===> V Fib

(Blunt) Abdominal Trauma:


#1: Spleen
At risk: splenomegaly
Mononucleosis, malaria, hematologic disease

Kehrs sign: blood irritates diaphragm =>

L shoulder pain
Post-splenectomy vaccines: >14 days before/ Pneumococcal HIB after splenectomy
Meningococcal

#2: Liver

8. A 31-year-old male is brought to the ED after a steel scaffold struck and pinned his left lower extremity for 2 hrs before extrication. On arrival in ED: VS: stable, ABCs are normal. Left leg: hip to ankle is swollen, ecchymotic and tender. Sensation intact. 2+ DP, PT present Labs: H/H=13/37, Urine dipstick: (+) blood Urinalysis: 0-2 RBCs, 0-2 WBCs/hpf This patient should be admitted for the management of:

1. 2. 3. 4.

Rhabdomyolysis Compartment syndrome Renal contusion Hemorrhagic shock

Rhabdomyolysis
First described in WWII London bombings Seen in: Trauma, seizures, burns, drug overdose, exertion, toxin/drug induced Urine dipstick positive 50% (myoglobinuria) Elevated CPK (> 2-3x reference) Complication: Acute renal failure 30-40% Treatment: Crystalloid 500 cc/hr=> Urine output 200-300 cc/hr

Compartment Syndrome
Can occur in anywhere perfusion pressure falls below tissue pressure in any anatomic space > 30 mm Hg*** Classic: extremities, but any compartment susceptible Classic: trauma to extremity - but can occur with exercise Clues: severe pain, decreased sensation, pain on passive stretch, tense extremities 4-6 hours before irreversible damage Do NOT wait for pallor, pulselessness Caution: open fractures are NOT immune from developing compartment syndromes

Wound Care
9. A 37-year-old male in good health presents with a 4 cm laceration to the mid-volar forearm after punching a glass window. His last tetanus shot was 12 years ago.

Which one of the following statements is true?


1. A radial nerve deficit would result in a lack of a pincer grasp 2. On neuro-vascular examination, a median nerve deficit would demonstrate inability to spread his fingers against resistance 3. Tap water irrigation has been shown to result in lower wound infection rates than irrigation with sterile saline. 4. X-rays are only indicated if the glass is known to contain lead

Wound Care:
A. Tetanus Prophylaxis
Clean wound > 10 years since last dose Dirty wound > 5 years since last dose Safe in pregnancy Why add diphtheria coverage? Answer: Approx 200 cases diphtheria/yr

B. Tetanus
Clostridium tetani - anaerobic Gr. (+), ubiquitous, soil 25-50 cases/yr Elderly, immigrants

Tetanus Update: Tdap


2005 - ACIP (Advisory Committee to CDC)
- To replace dT with Tdap - ages 19-64 x1 - Ages 11-18, can give with meningococcal vaccine - Can give to those who have had pertussis - Will injection site reaction occur if pt has had dT within the past 5 years? No

2010 - Recommends for patient > 65 years


If in close contact with infants

2012 - Recommends x1 > 65 yrs of age

Nerve Function: Hand Radial Nerve


Sensory: Posterior hand - thumb to radial 1/2 of ring finger Motor: Wrist and finger extension Test against resistance

Nerve Function: Hand Median Nerve


Sensory: Palmer surface from thumb to radial 1/2 of ring finger Motor: Flexion of wrist and fingers Best test: Make OK sign

OK

Deficit

Nerve Function: Hand Ulnar Nerve


Sensory: little finger and ulnar 1/2 of ring finger Motor: innervates interosseous muscles (intrinsics) Test: Abduction of fingers

Can I Irrigate with Tap Water? Cochrane Review: 2002


Meta-analysis of 3 studies tap water vs. normal saline ***Decreased rate of wound infection in adults with tap water (RR 0.55, 95% 0.31 - 0.97)

Trauma: Wound Care Principles


Irrigate, irrigate, irrigate Pearl: Tap water appears better than saline X-ray if you think any glass is possible Will identify any glass 2 mm or greater Do NOT apply iodophor/chlorhexidine into wound
Who do you place on prophylactic antibiotics? High-risk site - (hand, foot) High-risk mechanism (bites) High-risk patients (immunocompromised, prosthetic valve)

10. Child abuse should be suspected in all of the following EXCEPT:


1. 7 month old with diffuse cerebral and retinal hemorrhages 2. 2 year old with 3 rib fractures after a fall 3. 6 month old with multiple lower ext bruises from falls 4. 20 month old with spiral fracture of distal tibia

Child Abuse
Child abuse - spectrum of injuries are wide - Burns, contusions, fractures, head injury
Shaken baby syndrome: Diffuse cerebral injury with edema, +/- intracerebral bleed, retinal hemorrhages

3 Pearls--1. If they dont cruise, they dont bruise 2. Rib fractures - < age 3, 82% are abuse
62 children, Univ. Colorado, Barsness, KA,et al. J Trauma 2003

3. Undiagnosed ==> > 25% mortality in 2 yrs

Toddlers Fracture Is Not Abuse


Most common fx in age 9 mos - 3 yrs that present with a limp (29 of 100) Spiral fracture of distal tibia Best seen on oblique view May be occult Below knee walking cast x 3 weeks

Answers
1. B 2. B 3. A 4. A 5. D 6. C 7. A 8. A 9. C 10.D

ACS and Hyperlipidemia


Jonathon Firnhaber, MD, FAAFP Assistant Professor The Brody School of Medicine at East Carolina University Greenville, North Carolina

Disclosure Statement
Dr. Firnhaber has nothing to disclose.

The AAFP has selected all faculty appearing in this program. It is the policy of the AAFP that all CME planning committees, faculty, authors, editors, and staff disclose relationships with commercial entities upon nomination or invitation of participation. Disclosure documents are reviewed for potential conflicts of interest and, if identified, they are resolved prior to confirmation of participation. Only those participants who had no conflict of interest or who agreed to an identified resolution process prior to their participation were involved in this CME activity.

Objectives
1. Identify the primary target for cholesterol lowering therapy based on the Adult Treatment Panel III report. 2. Categorize risk stratification for cholesterol therapy based upon LDL-C risk factors and the Framingham Data. 3. Recognize the importance of hyperlipidemia therapy in the metabolic syndrome. 4. Recognize the hyperlipidemia therapy of coronary artery equivalent patients. 5. Relate the importance and role of intensive LDL cholesterol lowering therapy.

Acute Coronary Syndrome


Acute coronary syndrome
Unstable angina (UA) Non-ST-elevation myocardial infarction (NSTEMI) ST-elevation myocardial infarction (STEMI)

1. A 52-year-old man presents to the ED with a 45-minute history of squeezing substernal chest pressure radiating to his left arm. An ECG from a routine physical two months ago was normal.

Question 1 ECG

1. This patients history and ECG are most consistent with which one of the following?
A. B. C. D. Esophageal spasm Acute ST segment elevation MI (STEMI) Acute non-ST segment elevation MI (NSTEMI) Pericarditis

ECG Findings: ST Segment Depression, Inferior and Lateral Precordial Leads

Acute Coronary Syndrome


UA and NSTEMI are closely related, with similar presentations, but with differing severity:
Both cause ST-segment depression or prominent Twave inversion Ischemia severe enough to cause myocardial injury with release of biomarkers (troponin I, troponin T, CK-MB) = NSTEMI

STEMI is distinguished from UA/NSTEMI by presence of ST-segment elevation on ECG (followed by Q wave)
Warrants immediate reperfusion therapy

Acute Coronary Syndrome


Etiology: reduced myocardial perfusion
Reduced O2 supply (more common) Increased O2 demand

Acute Coronary Syndrome


Most commonly due to occlusive thrombus developing on top of a disrupted atherosclerotic plaque
Nearly 50% at sites with < 50% luminal narrowing

Other etiologies:
Spasm at site of atherosclerotic plaque Normal coronary arteries with spasm Prinzmetals angina; transient ST elevation associated Arterial inflammation (Kawasaki disease) Cocaine-induced (treat with NTG and CCB)

2. This patients initial troponin is normal. Which of the following statements about cardiac biomarkers is correct?
A. A single negative troponin excludes MI B. Troponin is a better marker of reinfarction than is CK-MB C. Myoglobin can be detected as early as 2 hours after the onset of myocardial necrosis D. CK-MB and myoglobin rise and fall more slowly than troponin

Troponin
Troponin is the biomarker of choice in the evaluation of ACS. Troponin I and T are equivalent. Detected 3-6 hours after the onset of ischemic symptoms Normal serial troponin levels exclude myocardial infarction, but do not exclude unstable angina. Troponin remains elevated for 7-14 days post-MI

Myoglobin
Earliest marker of MI
Released from damaged muscle more rapidly than CK-MB or troponin Can be detected as early as 2 hours after cardiac necrosis Peaks at about 8 hours; remains elevated < 24 hours

Sensitive but not specific


Cardiac origin must be confirmed with a more cardiac-specific enzyme False positives due to skeletal muscle injury

CK-MB
Remains elevated for 36-48 hours following MI Early peak (12-18 hours) suggests reperfusion CK-MB and myoglobin rise and fall more rapidly than troponin Better for diagnosing reinfarction

3. This patient continues to have pain despite treatment with IV NTG, morphine, -blocker, aspirin, and heparin. A repeat ECG shows persistent ST segment depression. What is the next most appropriate intervention? A. Emergency PCI (percutaneous coronary intervention) B. Fibrinolysis C. Emergency CABG D. Glycoprotein IIb/IIIa agent

2007 ACC/AHA UA/NSTEMI Guideline Revision


Recommends initial invasive strategy in patients with the following characteristics:
Recurrent angina or ischemia at rest or with lowlevel activities despite intensive medical therapy Elevated cardiac biomarkers (TnT or TnI) New or presumably new ST-segment depression Signs or symptoms of HF or new or worsening mitral regurgitation High-risk findings from noninvasive testing

2007 ACC/AHA UA/NSTEMI Guideline Revision


Recommends initial invasive strategy in patients with the following characteristics:
Hemodynamic instability Sustained ventricular tachycardia PCI within 6 months Prior CABG High-risk score (eg, TIMI, GRACE) Reduced left ventricular function (LVEF less than 40%)

ACS Therapy: Fibrinolysis


Fibrinolytic therapy or primary percutaneous coronary intervention (PCI) should be strongly considered in all STEMI patients with a symptom onset within 12 hours

ACS Therapy: Glycoprotein IIb/IIIa Inhibition


Glycoprotein IIb/IIIa receptor blockers
Inhibit platelet aggregation

Tirofiban (Aggrastat) Eptifibatide (Integrilin) Abciximab (ReoPro)

ACS Therapy: PCI with Stent


Bare metal
Bare metal acts as foreign body, increasing risk of in-stent thrombosisclopidogrel + ASA decreases risk; continue 1 month post-stent if no MI (or 12 months if post-MI) Endothelialization may progress to in-stent stenosis

Drug-eluting
Delay endothelialization, maintaining bare metal longer; continue clopidogrel + ASA for 12 months
Sirolimus (Cypher), tacrolimus (Mahoroba), paclitaxel (Taxus)

ACS Therapy: Emergency CABG

Considered only if coronary anatomy is not suitable for PCI

4. A 70-year-old woman with history of hypertension and type 2 diabetes presents to the ED with a 14-hour history of profound shortness of breath, but no chest pain. Her current medications include chlorthalidone, lisinopril, and insulin.

Question 4 ECG

4. This patients history and ECG are most consistent with which one of the following?
A. Unstable angina B. Acute anterior wall ST-segment elevation myocardial infarction C. Pericarditis D. Acute anterior wall ischemia

ECG Findings
Q waves and ST-segment elevation with T-wave inversion, anterior precordial leads (V2 V5)

ACS: STEMI
ST segments:
Elevation occurs immediately post plaque-rupture and is consistent with myocardial injury. Resolution of ST elevation suggests reperfusion. Persistent ST elevation may be seen with aneurysm formation. ST depression indicates myocardial ischemia.

Q waves:
Develop approximately 12 hours post plaque-rupture, and are indicative of (electrically) dead myocardium (MI). Typically permanent.

ACS: STEMI
Anterior/anteroseptal
LAD Leads V1 V4

Lateral
Circumflex Leads V5 V6

Inferior
RCA Leads II, III, aVF

5. Which of the following medications improves survival post-MI?


A. Digoxin B. Dihydropyridine calcium-channel blocker C. Long-acting nitrate D. ACE-inhibitor

Post-MI Survival
ACE-inhibitors, -blockers, statins, and ASA improve survival post MI. Nitrates, clopidogrel, calcium-channel blockers, and digoxin may improve symptoms, but do not affect survival.

ATP IV Basics
4 statin benefit groups
1. 2. 3. 4. Individuals with clinical ASCVD With primary elevations of LDL-C > 190 mg/dL 40-75 yrs with diabetes and LDL 70-189 Without clinical ASCVD or diabetes, age 40-75, LDL-C 70-189, and estimated 10-year ASCVD risk of 7.5% or higher

ATP IV Basics
Why not other approaches?
Treat to target
Current clinical data do not indicate what target should be Unclear magnitude of benefit one target or lower Potential adverse effects from multi-drug therapy

Lowest is best
Doesnt consider potential adverse effects from multi-drug therapy vs. magnitude of ASCVD event reduction

ATP IV Basics
Why not other approaches?
Treat level of ASCVD risk
Current recommendations consider both risk reduction benefits and adverse effects of statin therapy

Lifetime risk
Lack of data with RCTs > 15 years follow up Lack of safety data with statin use > 10 years Lack of data in individuals < 40 years of age

6. Which of the following statements is correct regarding the addition of nonstatin therapy to existing statin therapy?
A. The addition of niacin for individuals with treated LDL of 40-80 provides substantial ASCVD risk reduction. B. The addition of fenofibrate for individuals using maximum tolerated statin intensity provides substantial ASCVD risk reduction. C. The addition of omega-3 fatty acid for individuals using maximum tolerated statin intensity provides substantial ASCVD risk reduction. D. As of yet, there are no data to show that adding a nonstatin drug to high intensity statin therapy will provide incremental ASCVD risk reduction with an acceptable margin of safety.

ATP IV Basics
Current RCT data shows event reduction with maximum tolerated statin intensity rather than with specific targets Specific LDL targets may result in:
Undertreatment of those with high risk but only marginally elevated LDL-C. Diabetics often have lower LDL-C than non-diabetics; goal-directed therapy may encourage use of a lower statin dose than is supported by RCTs Overtreatment with nonstatin therapywhich shows no additional risk reductionto achieve an arbitrary target

ATP IV Basics
In selected individuals who are not in one of the four statin benefit groups, additional factors may be considered, including:
LDL > 160 or other evidence of genetic hyperlipidemia FH premature ASCVD (male < 55, female < 65) hs-CRP > 2 mg/dL CAC score > 300 Agatston units or > 75th percentile Ankle-brachial index (ABI) < 0.9 Elevated lifetime risk of ASCVD

7. For which of the following groups does the ACC/AHA Expert Panel specifically make no recommendation for or against statin therapy?
A. Patients with NYHA Class II-IV ischemic systolic heart failure B. Patients on maintenance hemodialysis C. Patients with untreated LDL-C < 100 mg/dL D. A and B E. A, B and C

ATP IV Basics
There are no good RCTs to support (or refute) the use of statins in: Patients with NYHA Class II-IV ischemic systolic heart failure Patients on maintenance hemodialysis

ATP IV Basics: Statin Intensity vs. Statin Benefit Group


1. Individuals with clinical ASCVD

Age < 75: High-intensity Age > 75: Moderate-intensity


High-intensity

2. With primary elevations of LDL-C > 190 mg/dL 3. 40-75 yrs with diabetes and LDL-C 70-189
Estimated 10-y ASCVD risk > 7.5%: High-intensity Estimated 10-y ASCVD risk < 7.5%: Moderate-intensity

4. Without clinical ASCVD or diabetes, age 40-75, LDL-C 70-189 and estimated 10-year ASCVD risk of 7.5% or higher
Moderate-to-high intensity

ATP IV Basics: statin intensity


High-intensity (> 50% LDL-C reduction)
Atorvastatin 40-80 mg Rosuvastatin 20 mg

Moderate-intensity (30% to < 50% LDL-C reduction)


Atorvastatin 10-20 mg Rosuvastatin 5-10 mg Simvastatin 20-40 mg Pravastatin 40-80 mg Lovastatin 40 mg Fluvastatin 40 mg bid

ATP IV Basics: The Risk Calculator


http://my.americanheart.org/cvriskcalculator
The information required to estimate ASCVD risk includes: age, sex, race, total cholesterol, HDL cholesterol, systolic blood pressure, blood pressure lowering medication use, diabetes status, and smoking status.

References: ACS / Hyperlipidemia


ACC/AHA 2007 Guidelines for the Management of Patients With Unstable Angina/NonST-Elevation Myocardial Infarction: Executive Summary A Report of the American College of Cardiology/American Heart Association Task Force on Practice Guidelines (Writing Committee to Revise the 2002 Guidelines for the Management of Patients With Unstable Angina/NonSTElevation Myocardial Infarction) Circulation 2007;116;803-877. Braunwald E, Antman EM, Beasley JW, et al: ACC/AHA 2002 Guideline Update for the Management of Patients with Unstable Angina and Non-ST-Elevation Myocardial Infarction: A Report of the American College of Cardiology/American Heart Association Task Force on Practice Guidelines. Committee on the Management of Patients with Unstable Angina. American College of Cardiology/American Heart Association, 2002, pp 6-7.

References: ACS / Hyperlipidemia


Jeremias A, Gibson M. Narrative review: alternative causes for elevated cardiac troponin levels when acute coronary syndromes are excluded. Ann Intern Med. 2005; 142(9): 786-91. Miller CD, Roe MT, Mulgard J, et al. Impact of acute beta-blocker therapy for patients with non-ST-segment elevation myocardial infarction. Am J Med. 2007;120:685-692. 2013 ACC/AHA Guideline on the Assessment of Cardiovascular Risk: A Report of the American College of Cardiology/American Heart Association Task Force on Practice Guidelines. Published online before print November 12, 2013, doi: 10.1161/01.cir.0000437741.48606.98

References: ACS / Hyperlipidemia


Poole-Wilson PA, Swedberg K, Cleland JG, et al. Comparison of carvedilol and metoprolol on clinical outcomes in patients with chronic heart failure in the Carvedilol Or Metoprolol European Trial (COMET): randomised controlled trial. Lancet. 2003;362:713. Daniels LB, Maisel AS. Natriuretic peptides. J Am Coll Cardiol. 2007;50(25):2357-2368. Wu AH, Jaffe AS, Apple FS et al. National Academy of Clinical Biochemistry laboratory medicine practice guidelines: use of cardiac troponin and B-type natriuretic peptide or N-terminal proB-type natriuretic peptide for etiologies other than acute coronary syndromes and heart failure. Clin Chem. 2007;1;53(12):2086-96. Hunt SA, American College of Cardiology, et al. ACC/AHA 2005 guidelines update for the diagnosis and management of chronic heart failure in the adult. J Am Coll Cardiol. 2005; 46(6):e1-82.

Answers
1. 2. 3. 4. 5. 6. 7. C C A B D D D

Bites, Stings and. Other Emergency Things


Robert Dachs, MD, FAAFP Clinical Associate Professor and Director of Research Ellis Hospital Family Medicine Residency Program Albany Medical College Albany, New York

Disclosure Statement
Dr. Dachs has nothing to disclose.

It is the policy of the AAFP that all individuals in a position to control content disclose any relationships with commercial interests upon nomination/invitation of participation. Disclosure documents are reviewed for potential conflicts of interest. If conflicts are identified, they are resolved prior to confirmation of participation. Only participants who have no conflict of interest or who agree to an identified resolution process prior to their participation were involved in this CME activity

Learning Objectives
1. Appropriately manage a variety of animal bites. 2. Appropriately manage tick-borne illness 3. Recognize and manage acute allergic reactions. 4. Recognize and determine which toxicology emergencies require specific antidotes. 5. Review current ACLS guidelines.

Animal Bites
1. Which of the following bites has the highest risk of infection?
1. 2. 3. 4.

Cat bite to the hand Human bite to the face Dog bite to the thigh Spider bite to the arm

A. Cat Bite
5 to 18% of all reported bites Puncture wounds 80% of bites become infected - 53-80% with Pasteurella multocida - Watch for bone and joint infection RX: Amoxicillin-clavulanate

B. Human Bite
Watch for closed fist injury High rate of infection, 26-83% polymicrobial
Viridans Streptococci,100% Bacteroides species, 82% S. aureus, 29% Peptostreptococcus, 26%

S. epidermidis, 53% Corynebacterium species, 41%

Eikenella species, 15%

Copious irrigation, avoid closure RX: Amoxicillin-clavulanate x 5 days

C. Dog Bite
80 to 90% of all reported bites (#1) Most common on extremities Only 5% of bites develop infection - Higher rate in hands, deep puncture, older pts.
Pasteurella multocida Bacteroides species Fusobacterium species EF-4 bacteria DF-2 bacteria (Capnocytophaga sp) S. aureus, 29% Fusobacterium Eikenella species, 15%

Primary closure - OK

+/- Amoxicillin-clavulanate

Bites and Stings


2. A 40-year-old male presents with right axillary swelling over the past 3 weeks. Large, multiple lymph nodes are present, but no fever or chills. He has a cat, dog, and parrot in his home.
The most likely diagnosis is:
1. 2. 3. 4.

Cat scratch disease Sporotrichosis Brucellosis Psittacosis

Cat Scratch Disease


22,000 cases/year, 2000 hospitalizations/year Regional lymphadenopathy, 10% suppurative - Axillary/epitrochlear nodes 46% - Cervical 26%, inguinal 17% Bartonella (Rochalimaea) henselae - small Gr (-) rod Diagnosis: Cat scratch and serologic testing (IFA) Rx: Only 1 prospective blinded studyMore rapid resolution of LN size with azithromycin
Bass JW, et al. Prospective randomized double blind placebocontrolled evaluation of azithromycin for treatment of cat-scratch disease. Pediatr Infect Dis J. Jun 1998;17(6):447-52

Self limiting disease, 1 to 2 months

Do NOT I & D!!!

Sporotrichosis
"Rose gardener's disease - fungus Sporothrix schenckii Painless nodular lesions (at the point of entry) also along lymphatic channels.

Psittacosis
Atypical pneumonia Contracted from parrots (and many other species of bird) Chlamydia psittaci

Brucellosis
Febrile illness, myalgias, Due to ingestion of unsterilized milk or meat Brucella - small, Gram Neg(-) coccobacilli

Next Case..
3. A 24-year-old male presents with a 3-day history of a rash that is increasing in size. - It is not (-) painful or tender. - It is flat, oval, 14 cm x 7 cm in size, has central clearing and has no fluctuance. - No associated fever/chills or systemic symptoms. - No new medications. No recall of any insect bite. - He recently vacationed on Marthas Vineyard, MA.

3. A 24-year-old male presents with a 3-day history of a rash that is continuing to increase in size. The rash is not painful or tender. It is flat, oval, 14 cm x 7 cm in size, has central clearing and has no fluctuance. No associated fever/chills or systemic symptoms. No new medications. No recall of any insect bite. He recently vacationed on Marthas Vineyard, MA.
1. 2.

Prescribe azithromycin 500mg qd x 5 days Prescribe doxycycline 100mg BID x 14 days

3.
4.

Obtain a Lyme titer, if positive treat with ceftriaxone. Obtain a Lyme titer, if positive, confirm with a Western blot study and if positive treat with ceftriaxone.

Lyme Disease
Due to spirochete: Borrelia burgdorferi Transmitted by: Deer tick
Ixodes scapularis and Ixodes pacificus
From left to right, an Ixodes scapularis larva, nymph, adult male tick, and adult female tick.

Wormser G P et al. Clin Infect Dis. 2006; 43:1089-1134

Courtesy of the CDC

Lyme Disease
Due to spirochete: Borrelia burgdorferi Transmitted by: Deer tick
Ixodes scapularis and Ixodes pacificus

Stages: Early, Late, Post-Lyme


I. Early Lyme Disease: Erythema migrans Develops 7-14 days after bite (range 3-30 days) Clinical findings are sufficient for the diagnosis of erythema migrans Serologic testing is too insensitive in the acute phase (the first 2 weeks) to be helpful diagnostically
IDSA guideline, 2006

Illustrative Examples of CultureConfirmed Erythema Migrans.

Courtesy of Wikipedia

Courtesy of the CDC/James Gathany

Lyme Disease
Early Lyme Disease
Erythema migrans: present 50-70% of cases Treatment options:
Doxycycline 100 mg BID, 10-21 days Amoxicillin 500 mg TID, 10-21 days Cefuroxime axetil 500mg BID, 10-21 days

Neurologic disease:
Cranial neuropathy (7th nerve), radiculopathy, lymphocytic meningitis

Cardiac disease: think A-V block


IDSA guideline, 2006

Lyme Disease
Late Lyme disease
Lyme arthritis:
Large joints, typically knees

Neurologic disease:
Encephalopathy Peripheral neuropathy

Post-Lyme disease syndromes


unexplained chronic subjective symptoms following treatment
Do not confuse with Jarisch-Herxheimer reaction
IDSA guideline, 2006

Lyme Disease Treatment


Watch for Jarisch-Herxheimer reaction
Fever, chills, myalgias, headache Remember syphilis Treat symptomatically, do not d/c or switch antibiotic

Other Deer Tick-Borne Illnesses


HGA: Human granulocytic anaplasmosis
Previously known as ehrlichiosis
Due to Anaplasma phagocytophilum

Within 3 weeks of tick bite Fever, chills and headache, with Thrombocytopenia, leukopenia, elevated LFTs

Babesiosis
Malaria-like illness with intracellular protazoa Hemolytic anemia, thrombocytopenia, elevated LFTs

All 3 associated with deer tick

Other Tick-Borne Illnesses


Rocky Mountain Spotted Fever
Organism: Rickettsia rickettsii, transmitted by
The American dog tick and The Rocky Mountain wood tick.

90% of cases are April - September. > 50% of cases involve children < 15 years old Symptoms: 5-10 days after tick bite:
Flu-like illness Rash Latermultisystem involvement
Photo courtesy of CDC

Other Tick-Borne Illnesses


Tularemia Bartonella Q fever Relapsing fever STARI-Masters disease Colorado tick fever Tick paralysis

Allergic Reactions
4. A 64-year-old male presents to the ED with diffuse pruritus and erythema along with facial and oral swelling. This occurred 15 minutes after eating peanuts. His blood pressure is 65/35 mm Hg, pulse is 120 bpm.

Allergic Reactions
4. A 64-year-old male presents to the ED with diffuse pruritus and erythema along with facial and oral swelling. This occurred 15 minutes after eating peanuts. His blood pressure is 65/35 mm Hg, pulse is 120 bpm.
This patient is experiencing:
1. 2. 3. 4. Urticaria Angioedema Anaphylaxis Anaphylactoid reaction

Allergy: Reactions
Urticaria (hives) - IgE mediated Angioedema - may be
1) IgE-mediated or 2) Idiopathic (ACE-induced), not true allergy

Result: Swelling of face, neck, and tongue


Anaphylaxis - may occur within seconds to 1 hour
Skin rash, respiratory symptoms, hypotension, GI distress

Anaphylactoid reactions: Non-immunologic (not IgE)-mediated release of granules from cells not true allergy (eg, radiocontrast)

Etiologic Agents
Anaphylactic: IgE-dependent Food Medication Insect venom Latex Exercise Anaphylactoid: Non-IgE/nonimmunologic Opioids ASA and NSAIDs Radiocontrast media

IgE Y

Allergic Reaction 5. The 64-year-old patient with anaphylaxis in the previous case should receive which of the following medications:
1. Epinephrine 2. Epinephrine plus diphenhydramine

3. Epinephrine plus diphenhydramine plus methylprednisolone 4. Epinephrine plus diphenhydramine plus methylprednisolone plus ranitidine

Allergic Reaction: Treatment


Vasoconstrictors: Epinephrine *Mild-moderate: 0.3-0.5 cc 1:1000 solution, SQ or IM
**Severe: 1-5 cc of 1:10,000 solution, IV ***If pt. On B-Blocker - Give glucagon 1-5 mg IV

Epinephrine: How to Administer

0.2 - 0.5 ml SQ/IM (1:1000) (1 mg/mL) Children 0.01 mg/kg

1ug/min (1:10,000) 1 mg in 1L NS at 1 cc/min

Allergic Reaction: Treatment


Vasoconstrictors: Epinephrine *Mild-moderate: 0.3-0.5 cc 1:1000 solution, SQ or IM
**Severe: 1-5 cc of 1:10,000 solution, IV ***If pt. On B-Blocker - Give glucagon 1-5 mg IV

If Epi-Pen is used, refer to ED for follow up Reason: chance of biphasic reaction The second wave

Allergic Reaction: Treatment


Vasoconstrictors: Epinephrine *Mild-moderate: 0.3-0.5 cc 1:1000 solution, SQ or IM
**Severe: 1-5 cc of 1:10,000 solution, IV ***If pt. On B-Blocker - Give glucagon 1-5 mg IV

H1 antagonist: diphenhydramine H2 antagonist: yes, H2-blocker du jour Steroids: - Do nothing for acute episode - May prevent recurrence

Allergic Reactions: Treatment


Antihistamines: Dont forget the H2 blocker
Methods: 91 ED pts. with acute allergic symptoms, randomized to: Results
Diphenhydramine 50 mg IV Diphenhydramine 50 mg IV + ranitidine 50 mg IV + placebo

Urticaria at 2 hrs Need for additional antihistamines

8% 4%

26% 23%

Lin, RY, et al. Ann Emerg Med 36(5): 462, Nov. 2000.

Another Case
6. 57 y/o female presents to ED/office noting that upon awakening her tongue was very swollen. Denies trauma. No fever/chills.

PMHx: HTN, NIDDM. Meds: lisinopril, metformin VS: BP = 160/80, P = 90, RR = 24, T = 98. Exam: Massively enlarged tongue (see photo).

6. 57 y/o female presents to ED/office noting that upon awakening her tongue was very swollen. Denies trauma. No fever/chills. PMHx: HTN, NIDDM. Meds: lisinopril, metformin VS: BP = 160/80, P = 90, RR = 24, T = 98. Exam: Massively enlarged tongue (see photo). The most likely diagnosis is: 1. 2. 3. 4. ACE-induced angioedema Type I hypersensitivity reaction Bee sting to the tongue Scombroid poisoning

ACE-Induced Angioedema
NOT an allergic reaction Due to accumulation of bradykinin (?) Can occur months to years after ACE use Treatment: supportive

Can you give an ARB to a patient with ACE-induced angioedema? In theory.YES


Data: Very limited (on pts. switched to ARB) 1) 3/39 pts. (7.7%) developed AE (CHARM trial)1 2) 2/26 pts. (8%) developed AE2

1 Lancet

2003

2Arch

Intern Med 2004

Bites, Stings and Other Things Bee Stings


Complex venom - Enzymes - Proteins - Serotonin - Acetylcholine - Histamine

Pain Normal Erythema ==> response STS Tingling


Apids Honey bees Bumble bees

2 groups: Vespids Yellow jackets Wasps Hornets

Bee Stings: 4 Potential Reactions


Local reaction: common Toxic reactions: often > 10 stings at once ==> N/V/D, lightheaded, syncope, headache Delayed reaction: 10-14 days later, serumsickness-like ==> malaise, headache, polyarthralgias Anaphylaxis: IgE-mediated

Allergic Reactions: Back to Medical School


Type I (immediate hypersensitivity): antigen
attaches to IgE and IgG4 on mast cells and basophils ==> degranulation release mediators (increased vascular permeability, smooth muscle constriction, etc)

Type II: IgG and IgM Abs react to Ag on cell surfaces


eg, blood transfusion rxn, ITP, hemolytic anemias

Type III - (immune complex): Ag-Ab complex triggers


complement system ==> eg, post-strep GN, serum sickness

Type IV (delayed hypersensitivity): T cell-mediated eg, PPD, poison ivy

Scombroid Poisoning: Pseudo Fish Allergy


Mimics allergic reaction - facial flushing, diaphoresis, hives, edema, diarrhea, peppery taste Occurs minutes to 1-2 hours after eating contaminated fish Classically tuna and mackerel (Scombroidae family), can occur in others Histidine in muscle converted by bacteria to histamine. Rx: H1 and H2 blockers Self-limiting: 4-6 hours

Ciguatera Poisoning
Ingestion of reef fish that have accumulated sufficient amounts of the dinoflagellate
Most common: barracuda, amberjack, grouper, snapper, sturgeon, king mackerel

GI or neurologic symptoms (or a mixed)


Onset 1-6 hrs after eating, lasts weeks-months Cold sensation reversal: perceives cold temperatures as hot sensations (and vice versa) Occurs in 80% of patients and pathognomonic

Environmental Injuries: A. Cold-related Injuries


Chilblains (or pernio): is an abnormal vascular response to cold resulting in inflammatory skin condition with pruritus and/or painful erythematous to violaceous acral lesions Frostnip: superficial freeze injury characterized by lack of extracellular ice crystal formation => pale, painful tissue Resolves with rewarming; no tissue loss

Frostbite: ice crystal formation, (+) tissue loss

Rapid rewarming in circulating water, 104-108F (40-42C)

Environmental Injuries: B. Heat-Related Illness


Heat exhaustion: nonspecific symptoms
Dizziness, weakness, malaise, N/V, HA, diaphoresis Temp: normal - 104F (40C), normal neuro exam

Heat stroke: 2 versions: Classic

Exertional Both (+) CNS dysfunction

Risk factors: 1) Exogenous heat gain 2) Increased heat production 3) Decreased heat dispersion a. Dehydration b. CV disease c. Extreme of age d. Obesity e. Improper clothing f. Skin disease g. Drugs

Environmental Injuries: Heat-Related Illness


Heat exhaustion: nonspecific symptoms
Dizziness, weakness, malaise, N/V, HA, diaphoresis Temp: normal - 104F (40C), normal neuro exam

Heat stroke: 2 versions: Classic: elderly, develops more gradually Delirium/seizures (looks like sepsis)

Typically anhidrosis Exertional

Environmental Injuries: Heat-Related Illness


Heat exhaustion: nonspecific symptoms
Dizziness, weakness, malaise, N/V, HA, diaphoresis Temp: normal - 104F (40C), normal neuro exam

Heat stroke: 2 versions: Classic: elderly, develops more gradually Delirium/seizures (looks like sepsis)

Typically anhidrosis Exertional: younger, rapid onset, high temp Will continue to sweat

Environmental Injuries: Heat-Related Illness


Heat exhaustion: nonspecific symptoms Dizziness, weakness, malaise, N/V, HA, diaphoresis Temp: normal - 104F (40C), normal neuro exam Heat stroke: 2 versions:

Classic Exertional

Treatment for heat stroke: evaporate cooling or immersion cooling Note: antipyretics dont work

Toxicology
7. A 21-year-old college student presents to the ED with friends who report the student swallowed a whole bottle of acetaminophen 1 hour before arrival. They also note the patient has been drinking alcohol. The patient is awake but appears intoxicated.

At this point, you should:

1. 2. 3. 4.

Administer syrup of ipecac Perform a gastric lavage Administer activated charcoal Administer N-acetylcysteine (Mucomyst)

Toxicology
Gastric Decontamination
Syrup of ipecac - No, No, No!!!!!!!
- AAP says do not keep in home (Pediatrics, Nov 2003)

Gastric emptying - 36-40 Fr tube


- Possibly helpful if used within 60 min - Risk for iatrogenic injury (aspiration, esophagus)

Charcoal (best option)


- If given < 30 min, decreases absorption by 70% - If given 30-60 min, decreases absorption by 30% - Dose: 1-2 gm/kg (max 100 gm)

Acetaminophen Toxicity
Max daily dosing: 4 gm per day, toxic dose = 150 mg/kg 2nd most common cause of liver transplantation in US, however, only 4% of those with hepatoxicity develop liver failure 4 clinical phases:
- Phase 1 (0-24 hrs): asymptomatic, nausea/vomiting - Phase 2 (18-72 hrs): RUQ abd pain, N/V, rising LFTs - Phase 3 (72-96 hrs): Abd pain, N/V, jaundice, encephalopathy, renal failure, death - Phase 4 (4 -14 days): resolution

Acetaminophen levels: drawn at 4 hours after ingestion, treatment based on Rumack-Matthew nomogram
Treatment: N-acetylcysteine (NAC)

Toxicology Emergencies
Agent
Acetaminophen Aspirin B-blocker Ca-channel blocker Digitalis Heparin Isoniazid (INH) Opiates Organophosphates TCA

Antidote
N- Acetylcysteine (Mucomyst) Alkaline diuresis Glucagon Glucagon Fab antibodies (Digibind) Protamine Sulfate Pyridoxine (Vit B6) Naloxone (Narcan) Atropine NaHCO3

Common Toxidromes #1. Anticholinergic


Presentation:
Hot as Hades..Hyperthermia Blind as a Bat..Mydriasis Dry as a Bone.Thirst, decreased salivation Red as a Beet..Flushing, vasodilation Mad as a Hatter..Delirium, agitation, confusion

Etiology: Antihistamines, Antiparkinson, Antipsychotics, Antiemetics (phenothiazines), Antidepressants (TCA), Antispasmotics

Common Toxidromes: #2. Serotonin Syndrome


Presentation:
Cognitive-behavior: agitation, anxiety, drowsy, delirium, headache, seizures Autonomic dysfunction: tachycardia, arrhythmias, hyperthermia, HTN, diaphoresis, diarrhea, nausea Neuromuscular: restlessness, tremor, hyperreflexia, dysarthria, ataxia, myoclonic jerks/twitching

Etiology:
Most common: SSRIs, MAOs Especially if combined with: meperidine, cocaine, dextromethorphan, venlafaxine, amphetamine

Serotonin Syndrome: Presentation

Used with permission NEJM

Boyer E and Shannon M. N Engl J Med 2005;352:1112-1120

Serotonin Syndrome: Differential Dx and Differences Between Serotonin Anticholinergic Syndromes


Skin Serotonin Syndrome Anticholinergic Neuroleptic malignant syndrome Diaphoretic Dry Muscular Tone Increased Normal Reflexes Hyperreflexia Normal

Diaphoretic, Lead pipe Bradyreflexia pallor rigid

Toxicology
Treatment for the serotonin syndrome is: Cyproheptadine (Periactin) - Has antiserotonergic properties - Only available orally

Final Topic: BLS and ACLS


Take-Home Messages:
Out-of-hospital arrest: think C-A-B (no longer A-B-C)
Compressions: push hard, push fast V-fib/V-tach - know your algorithm (see next slide)
Defibrillate once (not x3) and return to compressions Amiodarone over lidocaine

Other ACLS meds:


Asystole: No atropine SVT meds: adenosine over Ca++ channel blocker Just say No to NaHCO3.

V-Fib/Pulseless V-Tach |
Defibrillate x 1

I 5 cycles of CPR, if shockable rhythm


1) Defibrillate again (x1) and 2) Give vasopressors (epi or vasopressin)

I 5 cycles of CPR, if shockable rhythm


1) Defibrillate again (x1) and 2) Give antiarrhythmic (amiodarone)

Answers
1. 2. 3. 4. 5. 6. 7.

A A B C D A C

Evaluate Todays Sessions

www.aafp.org/reportcme/boardrev/indianapolis

5:45 pm

Recess

Board Review Express March 13-16, 2014 Indianapolis, Indiana


Day 4 - Sunday, March 16
6:45 - 7:15 am 7:15 - 7:45 am 7:45 - 8:15 am 8:15 - 8:45 am 8:45 - 9:15 am 9:15 - 9:30 am 9:30 - 9:45 am 9:45 - 10:15 am 10:15 - 10:45 am 10:45 - 11:15 am 11:15 - 11:45 am 11:45 - 12:00 pm 12:00 pm Breakfast Provided Behavioral Medicine I - Stanley Oakley, Jr, MD, FAPA Hypertension - Jonathon Firnhaber, MD, FAAFP The Major Arthritides - BW Blount, MD, FAAFP Heart Failure - Jonathon Firnhaber, MD, FAAFP Q&A Break Behavioral Medicine II - Stanley Oakley, Jr, MD, FAFP Endocrine Disease - BW Blount, MD, MPH, FAAFP Behavioral Medicine III - Stanley Oakley, Jr, MD, FAFP Urologic Problems - BW Blount, MD, MPH, FAAFP Q&A Adjourn

Behavioral Medicine I:
Major Depression in Adults and Special Populations
Stanley P. Oakley, Jr, MD, FAPA

Professor Department of Psychiatric Medicine The Brody School of Medicine at East Carolina University Greenville, North Carolina

AAFP Board Review: Express

AAFP Board Review: Supersonic Overdrive

Disclosure Statement
Dr. Oakley has nothing to disclose.

The AAFP has selected all faculty appearing in this program. It is the policy of the AAFP that all CME planning committees, faculty, authors, editors, and staff disclose relationships with commercial entities upon nomination or invitation of participation. Disclosure documents are reviewed for potential conflicts of interest and, if identified, they are resolved prior to confirmation of participation. Only those participants who had no conflict of interest or who agreed to an identified resolution process prior to their participation were involved in this CME activity.

Learning Objectives
1. Recognize the differential diagnosis and clinical presentation of major depression in adults and in special populations (geriatric, pregnant, and postpartum). 2. List the pharmacotherapeutic options for treatment of these conditions.

Most of what follows is true

1. According to the DSM-5 model for diagnosing mental health disorders, major depression is coded on which axis?
A. Axis I B. Axis II C. Axis III D. Axis IV E. None of the above

No Multiaxial System in DSM-5


No axes in the DSM-5, adopted in May of 2013 Single entry diagnosis, with modifiers for severity Aligns with CPT-10

2. Which of the following is NOT true about screening for depression?


A. Brief, 2-question screens for depression are discouraged compared to longer instruments B. The Edinburgh Postnatal Depression Scale should be routinely used at the first postpartum visit C. There are multiple standardized scales that can be used for screening for depression D. The USPSTF recommends screening adults and adolescents for depression

Depression Screening
USPSTF guidelines recommend screening adults for depression. Can use following scales:
Zung Self-Rating Depression Scale Beck Depression Inventory General Health Questionnaire, Center for Epidemiologic Studies Depression Scale SelfCARE (D) Geriatric Depression Scale

But

Depression Screening
A yes response to the following 2 questions may be as effective as using longer screening tools. (USPSTF, 2002)
Over the past 2 weeks, have you ever felt down, depressed, or hopeless? Have you felt little interest or pleasure in doing things?

Depression
Effective management strategies
Collaborative care between primary care MD, psychiatrist, psychologist (cognitive therapy) Combining patient education, automated pharmacy data, & psychiatric intervention Medication counseling by non-MDs (nurses, pharmacists, counselors) Nurse case management (general support, medication counseling, telephone follow-up)

Depression: Screening
USPSTF found evidence is insufficient to recommend for or against routine screening of children (7-11 years old) for depression USPSTF recommends screening adolescents (12-18 years old) and adults for depression in clinical practices with systems in place for diagnosis, treatment, follow-up First postpartum evaluation should include screening for depression Routine use of the Edinburgh Postnatal Depression Scale improves diagnosis rates

3. Which of the following is NOT true about depression and Persistent Depressive Disorder (dysthymia)?
A. The depression associated with dysthymia is not as severe as that of major depressive disorder (MDD) B. Lifetime risk is twice as great for women than men C. Depression is more common in medically ill patients D. D. Dysthymic patients may have periods of normal mood that can last for 1 year

Diagnosis of Major Depression Requires 5 or More of the Following Sx


1) Depressed mood 2) Loss of enjoyment (anhedonia) 3) Weight or appetite change 4) Sleep changes 5) Psychomotor changes (restlessness or slowing) 6) 7) 8) 9) Poor energy Feelings of guilt Poor concentration Thoughts of death

(NOTE: #1 or #2 must be present)

Major Depression

Diagnosis of Persistent Depressive Disorder (Dysthymia)


Requires depressed mood for at least two years with no symptom-free period lasting longer than two months Along with 2 or more of the following:
Appetite or sleep changes Decreased energy or concentration Thoughts of guilt or death Psychomotor changes (restlessness or slowing) Hopelessness

Dysthymia

Epidemiology of Depression
Depression is common. Lifetime risk greater for women (20%25%) than for men (10%) Community prevalence is about 5% Prevalence is higher (15%) in the medically ill.

4. Which of the following is true about treating depression?


A. If a patient does not respond to an SSRI within 2 weeks of starting it, the dose should be increased or a second antidepressant should be added B. The medication of choice for depressed patients with seizures is bupropion C. Memory loss caused by ECT is often short term & reversible D. Antidepressants are contraindicated if the patient is breastfeeding

Etiologies of Depression - Drugs


Narcotics Antineoplastic agents Sedative hypnotics Centrally-acting antihypertensives Steroids Anxiolytics Alcohol

Medical Screening for Depression


History & Physical
(Especially the neurological exam)

CBC CMP Thyroid Panel, TSH RPR

Features of Effective Clinical Management


Patient education (the diagnosis, treatment options, duration of treatment and costs, side effects, goals of the treatment, recurrence and relapse, etc.) Reassurance (such as, depression is a medical illness, not a character defect or weakness; recovery is the rule, not the exception; treatments are effective; an effective treatment can be found for nearly all patients, etc.) Regular monitoring for symptoms and adverse effects Adjustments or changes in the treatment plan if response is lacking or suboptimal

Choice of Antidepressant
All modern antidepressants are equally effective Patient preference Cost History of prior response to specific medication Response of first-degree relative to specific medication Use one antidepressant

Treatment of Depression
Usually begin with selective serotonin reuptake inhibitors (SSRIs)
Once-a-day dosing No titration 80% of the time Few side effects (GI, sexual) Safe in overdose

Selective Serotonin Reuptake Inhibitors (SSRIs)


Fluoxetine (Prozac) 20mg Sertraline (Zoloft) 50mg Paroxetine (Paxil) 20mg Escitalopram (Lexapro) 10mg Citalopram (Celexa) 20mg Fluvoxamine (Luvox) 100mg More activating

More relaxing

Other Antidepressants
Bupropion (Wellbutrin) 150-450 mg
No sexual side effects, not with seizures

Venlafaxine (Effexor XR) 75-225 mg


Mixed nor-epi & serotonin

Nefazodone (Serzone) 50-100 mg


No sexual side effects, sedating

Mirtazapine (Remeron) 15-45 mg


At low doses, sedating, appetite increased

Duloxetine (Cymbalta) 40-60 mg


May be useful if chronic pain is present

New Antidepressants
Vilazodone (Viibryd) 20-40 mg
Serotonin agonist and reuptake blocker

Vortioxetine (Brintellix) 10-20 mg


Serotonin agonist and reuptake blocker

Levomilnacipran (Fetzima) 20-40 mg


SNRI

Treatment (cont.)
If no response to any newer agents, or if chronic pain is a large issue in the depression, consider a tricyclic antidepressant

Tricyclic Antidepressants
Desipramine (Norpramin) 50-150 mg
Less sedation Maximum dosage up to 300 mg/day

Nortriptyline (Pamelor) 50-100 mg


More sedation Maximum dosage up to 125 mg/day

Serum levels available

Monoamine Oxidase Inhibitors (MAOIs)


Generally not used Dietary restrictions; if patients eat foods with tyramine, will get hypertensive crisis Problematic in acute medical crises or emergency room settings Oral drugs are phenelzine (Nardil) and tranylcypromine (Parnate) Transdermal patch selegiline (Emsam)

Antidepressant Side Effects


Tricyclics: anticholinergic effects, narrow-angle glaucoma, caution in bundle branch blocks, weight gain SSRIs: sexual dysfunction; hyponatremia SNRIs: mild anticholinergic, narrow-angle glaucoma, hepatic dysfunction Bupropion: seizure induction (0.4%)

Depression: Terms
Response: at least a 50% reduction in symptoms of depression as assessed by rating scale Remission: resolution of essentially all symptoms Recovery: remission lasting for 6 to 12 months Relapse: worsening before achieving recovery Recurrence: new depressive episode within a few months of recovery

Duration of Treatment
For 1st episodes of depression, treat for 912 months. For recurrent depression, treat for at least 2 years. If patient relapses after successful treatment, > 90% will respond to the same antidepressant.

Augmentation of Antidepressants
Cytomel (T-3) 25-50 mcg q am Lithium carbonate 300-600 mg daily Response usually rapid, 7-9 days Bupropion (STAR*D study) Buspirone (STAR*D)

Treatment of Depression
If no response to multiple antidepressants, consider a trial of electroconvulsive therapy (ECT)
ECT: most effective treatment in patients with severe resistance or psychotic depression
Safe; memory loss is short-term, reversible

Geriatric Depression
ECA study revealed 3% prevalence of depression in the elderly Other studies suggest 10-15% prevalence Co-existence of depression in the medically ill elderly ranges from 25%-50% 500 NC family physicians rated geriatric depression in the top 3 clinical challenges

Geriatric Depression
Geriatric depression may be underdiagnosed by 50% in primary care settings 80% of depression is treated by primary care providers Often co-morbid with generalized anxiety or dementia

Depressive Symptoms in the Young


Sadness Loss of interest (Anhedonia) Guilt Suicidal ideation These are the EMOTIONAL symptoms of depression

Depressive Symptoms in the Old


Loss of interest Insomnia Suicidal ideation Physical complaints These are the SOMATIC (vegetative) symptoms of depression

Suicide in the Elderly


12% of the population is over 65, they compose 25% of successful suicides Increased risks include being single, widowed, or a white male Older patients use more lethal means and are more successful - they dont make gestures

Pseudodementia
Also known as reversible dementia of depression Major depression in the elderly can present with prominent memory complaints Some controversy over this being called a dementia, since no neurons are lost Resolves as depression is treated due to improvement in concentration

Psychotic Depression
Elderly are slightly more likely to develop psychotic depression 3.6% of the depressed elderly in the community develop psychosis This increases to 20-45% of the elderly hospitalized for major depression

Psychotic Depression (continued)


Usually has psychotic delusions; hallucinations not prominent Delusions either mood congruent or mood incongruent Mood congruent delusions usually nihilistic (rotting, diseased) Not responsive to antidepressants alone; requires addition of an antipsychotic for treatment, or ECT

Treatment of Geriatric Depression


Usually begin with selective serotonin reuptake inhibitors (SSRIs)
Once-a-day dosing No titration 80% of the time Few side effects (GI, sexual) Safe in overdose

Geriatric Treatment (continued)


Patient must have an adequate trial of medication Ranges from 4-12(!) weeks, but if no response at all by 4 weeks, it is unlikely response will develop later If partial response at 4 weeks, maximize dose and continue

Geriatric Treatment (continued)


If partial response to SSRI, change to another SSRI If no response to SSRI, try switching to another category

Other Antidepressants Tolerated Well by the Elderly


Bupropion (Wellbutrin) 50-100 mg Venlafaxine (Effexor) 37.5 mg Desvenlafaxine (Pristiq) 50 mg Nefazodone 25-50 mg Mirtazapine (Remeron) 7.5-15 mg Duloxetine (Cymbalta) 20-40 mg

Depression: Special Groups


Pediatrics
Fluoxetine and escitalopram only ones FDAapproved (7-17 years)

Lactation
Antidepressants are NOT contraindicated! In most cases, infant blood concentrations of TCAs and SSRIs have been below the detection limit of commercial labs & well tolerated Fluoxetine: can check infant blood levels at 6 weeks

Antidepressants: Use in the Pregnant Patient


Relapse risk 5x higher if antidepressant is stopped SSRIs are first-line
Risk of PPHN with SSRI use in late pregnancy Paroxetine: changed to pregnancy category D TCAs also considered safe and effective SSRIs appear to have a more favorable side-effect profile than TCAs One small study supports the use of venlafaxine

Informed consent is key

5. Which of the following is NOT true of postpartum depression?


A. The baby blues typically resolve spontaneously by the 1st or 2nd postnatal week B. Patients are predisposed by prior history of depression C. Therapy is more effective than medications

D. A history of bipolar disorder is a risk factor for the development of postpartum psychosis

Postpartum Depression
Baby blues
Frequency range: 26 to 85% Characterized by mild depressive symptoms Lasts 1-2 weeks Treatment: supportive care Increases risk for postpartum major depression (PMD) later in the postpartum period, especially if symptoms were severe

Postpartum Depression
Postpartum Major Depression (PMD) Appears most often within first 3 months after delivery Predisposing factors: history of severe depression, stress, negative experiences during perinatal period, insufficient social support Symptoms same as major depression plus excessive concern and fear about infant Need to rule out thyroid dysfunction

Postpartum Depression: Treatment


Treatment is the same as that for major depression Psychotherapy or pharmacotherapy may be used alone or in combination
No single modality has been shown to be superior

Postpartum Psychosis
Postpartum psychosis: a medical emergency
0.2% occurrence Onset within the first month of delivery Manic in nature; bipolar disorder is a risk factor Inability to sleep, agitation, expansive or irritable mood, avoidance of the infant Delusions or hallucinations often involve the infant; auditory hallucinations "telling" mother to kill her infant are possible May require inpatient treatment with ECT, neuroleptic agents, and/or mood stabilizers

Key Points for the Exam


Screening for depression:
Children = no Adolescents and adults = yes, if systems in place to monitor

Recovery from first MDD: can discontinue medications Adequate trial of antidepressant: 4-6 weeks ECT: memory loss short-term, reversible Lithium: mania without rapid cycling Lactation: antidepressants OK, lithium not Pregnancy: continue treatment to avoid relapse; SSRIs first-line therapy, but avoid paroxetine

Answers
1. 2. 3. 4. 5. E A D C C

Hypertension
Jonathon Firnhaber, MD, FAAFP Assistant Professor The Brody School of Medicine at East Carolina University Greenville, North Carolina

Disclosure Statement
Dr. Firnhaber has nothing to disclose.

The AAFP has selected all faculty appearing in this program. It is the policy of the AAFP that all CME planning committees, faculty, authors, editors, and staff disclose relationships with commercial entities upon nomination or invitation of participation. Disclosure documents are reviewed for potential conflicts of interest and, if identified, they are resolved prior to confirmation of participation. Only those participants who had no conflict of interest or who agreed to an identified resolution process prior to their participation were involved in this CME activity.

Learning Objectives
1. Recognize the initial drug therapy and the compelling indications for alternative drug therapy in hypertension. 2. Cite the common causes for refractory hypertension. 3. Identify the frequent causes and therapy for secondary hypertension.

BP and Cardiovascular Risk


HTN is an independent risk factor for ischemic cardiovascular events. Risk of vascular death increases progressively and linearly from a normal pressure of 115/75 mm Hg. For every 20 mm Hg systolic or 10 mm Hg diastolic increase in blood pressure, the risk of major cardiovascular events and stroke doubles.

JNC-8 Recommendations
In the general population < 60 years, and population > 18 years with CKD, and population > 18 years with diabetes: Initiate treatment at SBP > 140 or DBP > 90 Treat to goal SBP < 140 and DBP < 90

In the general population > 60 years: Initiate treatment at SBP > 150 or DBP > 90 Treat to goal SBP < 150 and DBP < 90

JNC-8 Recommendations
In the general non-black population, including those with diabetes, initial treatment should include: A thiazide-type diuretic, or CCB, or ACEI, or ARB In the general black population, including those with diabetes, initial treatment should include: A thiazide-type diuretic, or CCB

JNC-8 Recommendations
In the population > 18 years with CKD, initial (or add-on) treatment should include an ACEI or ARB to improve kidney outcomes. Do not use an ACEI and an ARB together in the same patient. The main objective of hypertension treatment is to attain and maintain goal BP.
Add and titrate as necessary to meet this objective.

ASH/ISH Recommendations
In patients > 18 years, initiate treatment at BP > 140/90 In patients > 80 years, initiate treatment at BP > 150/90 Initial therapy:
Non-black < 60 Non-black > 60 Black ACEI or ARB CCB or thiazide CCB or thiazide

If initial BP > 160/100, initiate with 2 drugs


CCB or thiazide plus ACEI or ARB

1. Which one of the following is a preferred firstline agent for managing hypertension in patients with stable ischemic heart disease?

A. B. C. D. E.

A thiazide diuretic An angiotensin receptor blocker A -blocker A long-acting calcium channel blocker A long-acting nitrate

Hypertension in IHD
American Heart Association guidelines recommend -blockers and/or ACEIs for hypertensive patients with stable ischemic heart disease. ACEIs are recommended in patients already on -blocker therapy (especially following myocardial infarction), in diabetics, and in patients with left ventricular dysfunction.

Hypertension in IHD
Long-acting CCBs may be used in patients who do not tolerate -blockers; short-acting CCBs should be avoided because they increase mortality. Although ARBs have indications similar to those of ACEIs, the AHA recommends using them only in patients who do not tolerate ACEIs. Long-acting nitrates are used for their antianginal properties and have little role in the management of hypertension.

Compelling Indications for Individual Drug Classes (JNC-7)


Compelling indication
Heart failure Post-MI High coronary disease risk Diabetes Chronic kidney disease Recurrent stroke prevention

Diuretic

BB

ACEI

ARB CCB

Aldo ANT

ASH/ISH: Other Major Conditions


Hypertension plus:
Diabetes: CKD: Clinical CAD: Stroke history: CHF: ACEI or ARB ACEI or ARB -blocker plus ACEI or ARB ACEI or ARB -blocker plus ACEI or ARB, plus diuretic, plus spironolactone, regardless of BP

HTN Treatment in Minority Populations


African-American patients exhibit somewhat reduced BP responses to monotherapy with: ACE-inhibitors ARBs -blockers when compared with diuretics or CCBs ACE-inhibitor-induced angioedema occurs 24 times more frequently in African-American patients with HTN than in other groups.

2. A 48-year-old female presents as a new patient. BP 172/110 in both arms; CV exam unremarkable. BMI: 24.4; she takes no medications. BMP: Cr 0.68 mg/dL, K 3.3 mEq/dL. If the patients hypertension should prove refractory to treatment, which one of the following tests is most likely to reveal the cause of her secondary hypertension?

A. B. C. D. E.

24-hour urine catecholamine level Plasma aldosterone/renin ratio MRA of the renal arteries Echocardiography Sleep study (polysomnography)

Secondary Hypertension
Primary hyperaldosteronism is the most common cause of secondary hypertension in the middle-aged population. Diagnosis is based on the renin/aldosterone ratio. Plasma renin activity is variable and may be misleading. Unprovoked hypokalemia further supports the diagnosis, although is not present in the majority of cases.

Secondary Hypertension
Other considerations include:
Coarctation of the aorta (more common in younger patients)
Echocardiogram

Sleep apnea Pheochromocytoma Hypercortisolism Hyperthyroidism Renal parenchymal disease (children)

Renal Artery Stenosis


Most common cause: Age < 30: fibromuscular disease Age > 30: atherosclerotic disease May present with: Accelerated or resistant HTN, renal dysfunction, flash pulmonary edema Diagnosis: MRA of renal arteries (or CT angiogram) Elevated renin level alone is not diagnostic

3. A 54-year-old male with type 2 diabetes has a BP of 148/94 and creatinine of 1.25 mg/dL. One month after starting lisinopril 20mg/d, his BP is 128/80 and creatinine is 1.5 mg/dL. A repeat creatinine 1 week later is unchanged. What should you do?
A. Continue lisinopril at the same dosage B. Reduce the lisinopril dosage to 10 mg C. Discontinue lisinopril D. Change lisinopril to chlorthalidone E. Change lisinopril to losartan

Treatment-Induced Decline in Renal Function


A 20-30% increase in creatinine, which then stabilizes, represents a hemodynamic change, and not a structural change.
Slight rise in creatinine serves as an indirect indicator that intraglomerular (IG) pressure has been reduced. ACEI/ARB also dilate efferent arteriole, exaggerating decline in IG pressure.

Treatment-Induced Decline in Renal Function


Renal dysfunction associated with antihypertensive treatment is independent of the agent used. If creatinine increases by more than 30%, agent should be discontinued and other causes of renal dysfunction should be evaluated.

4. For which one of the following conditions is a thiazide diuretic an appropriate option for antihypertensive therapy?
A. Gout B. Bipolar disorder treated with lithium C. Diabetes D. Chronic renal insufficiency, with a serum creatinine level of 2.6 mg/dL

Thiazide Diuretics in HTN


Advocated as initial treatment since 1977 (JNC-1) JNC-7: thiazide diuretic should be used in drug treatment for most, either alone or combined with drugs from other classes JNC-8: one of several options for initial treatment

Thiazide Diuretics in HTN


Reduce excretion of:
Calcium (may slow bone demineralization) Uric acid (increasing likelihood of gout) Lithium (increasing risk of lithium toxicity)

Increase excretion of:


Potassium (average decrease of 0.3-0.4 mmol/L; dietary salt restriction can minimize thiazide-induced K loss) Magnesium (complicates correction of hypo-K)

Thiazide Diuretics in HTN


Average increase in glucose attributed to thiazide use: 3-5 mg/dL Presence of diabetes is not a contraindication to use of thiazides Typically considered ineffective when GFR < 30-40 mL/min
Exception is metolazone, which is not useful as monotherapy but improves diuresis when used in conjunction with loop diuretic

Does It Matter Which Thiazide?


Antihypertensive Efficacy of Hydrochlorothiazide as Evaluated by Ambulatory Blood Pressure Monitoring
Systematic review of all the randomized trials that assessed 24-hour BP with HCTZ in comparison with other antihypertensive drugs

J Am Coll Cardiol 2011;57:590600

Findings
Decrease in 24-hour BP with HCTZ dose 12.5 to 25 mg: systolic 6.5 mm Hg, diastolic 4.5 mm Hg This reduction was statistically inferior to:
ACEI ARBs -blockers CCBs (mean BP reduction 12.9/7.7 mm Hg) (mean BP reduction 13.3/7.8 mm Hg) (mean BP reduction 11.2/8.5 mm Hg) (mean BP reduction 11.0/8.1 mm Hg)

HCTZ Dose Titration


There was no significant difference in systolic or diastolic 24-hour BP reduction between HCTZ 12.5 mg (5.7/3.3 mm Hg) and HCTZ 25 mg (7.6/5.4 mm Hg)
With HCTZ 50 mg, the reduction in 24-hour BP was significantly higher (12.0/5.4 mm Hg) and was comparable to that of other agents

HCTZ Dose Titration


All biochemical adverse effects such as hypokalemia, hyponatremia, hyperuricemia, insulin resistance, and visceral fat accumulation are dose dependent and become clinically more significant with daily doses exceeding 25 mg. An additional concern is the risk of sudden cardiac death that has been shown to increase in a dose dependent fashion with HCTZ doses exceeding 25 mg daily.

Messerli Conclusion
HCTZ in its commonly used dose of 12.5 to 25 mg daily lowers BP significantly less well than do all other drug classes as measured in head-to-head studies by ABP monitoring.
Because of such paltry antihypertensive efficacy and the lack of outcome data at these doses, physicians should refrain from prescribing HCTZ as initial antihypertensive therapy.

Other Thiazide Options


Chlorthalidone: 12.5-25 mg daily; maximum dose 50 mg daily Indapamide (Lozol): 1.25-2.5 mg daily; maximum dose 5 mg daily

5. 54-year-old male presents for f/u of HTN. Despite careful adherence, his BP averages 150/90 mm HG. Recent labs are normal: CBC, BMP, UA. Medications: chlorthalidone 12.5 mg/d, carvedilol 25 mg bid, amlodipine 10 mg/d and lisinopril 40 mg/d.
Which one of the following medication changes would be most reasonable?

A. B. C. D.

Adding isosorbide mononitrate Substituting furosemide for chlorthalidone Substituting losartan for lisinopril Adding spironolactone

Resistant Hypertension
Spironolactone is an appropriate choice for treating resistant HTN, even when hyperaldosteronism is not present. Nitrates have some effect on BP but are recommended only for patients with CAD No benefit to switching ACEI to ARB A longer-acting diuretic such as chlorthalidone is also recommended for treating hypertension, particularly in resistant cases with normal renal function

Resistant Hypertension
Persistent HTN despite > 3 drugs Most common cause: poor adherence Suboptimal therapy
Typically inadequate diuresis Move to loop diuretic Add spironolactone Consider vasodilating -blocker (carvedilol, labetalol, nebivolol) Consider clonidine, hydralazine, -blocker

Resistant Hypertension
Exogenous drugs
Caffeine (energy drinks, supplements) Alcohol, nicotine Cocaine NSAIDs OCPs Steroids, erythropoietin Herbal agents

Secondary HTN

Hypertension

References: Hypertension
James PA, Oparil S, Carter BL, et al. 2014 Evidencebased guideline for the management of high blood pressure in adults: Report from the panel members appointed to the eighth joint national committee (JNC 8). JAMA. 2013. http://dx.doi.org/10.1001/jama.2013.284427. Weber MA, Ram CVS, Cohen DL, et al. Clinical Practice Guidelines for the Management of Hypertension in the Community A Statement by the American Society of Hypertension and the International Society of Hypertension. Journal of Hypertension. 2014;32:3-15.

References: Hypertension
Chobanian AV, Bakris GL, Black HR, et al: The Seventh Report of the Joint National Committee on Prevention, Detection, Evaluation, and Treatment of High Blood PressureThe JNC 7 Report. National Heart Lung and Blood Institute (NHLBI), 2003. Palmer BF: Renal dysfunction complicating the treatment of hypertension. N Engl J Med. 2002;347:1256. Sarafidis PA, Bakris GL. Resistant hypertension: an overview of evaluation and treatment. J Am Coll Cardiol. 2008;52:1749-57.

References: Hypertension
ALLHAT Officers and Coordinators for the ALLHAT Collaborative Research Group: Major outcomes in high-risk hypertensive patients randomized to angiotensin-converting enzyme inhibitor or calcium channel blocker vs diuretic: the Antihypertensive and Lipid-Lowering Treatment to Prevent Heart Attack Trial (ALLHAT). JAMA. 2002;288(23):29812997. Use of Diuretics in Patients with Hypertension. N Engl J Med. 2009;361(122):2153-2160. Chobanian AV: Isolated systolic hypertension in the elderly. N Engl J Med. 2007;357(8):789- 796.

References: Hypertension
August P: Initial treatment of hypertension. N Engl J Med. 2003;348(7):610617. Hall WD: A rational approach to the treatment of hypertension in special populations. Am Fam Physician. 1999;60(1):156166. Epstein M: Diagnosis and management of hypertensive emergencies. Clinical Cornerstone. 1999;2(1):4151. Vaughan CJ, Delanty N: Hypertensive emergencies. Lancet. 2000;356:411-417. American Diabetes Association: Hypertension management in adults with diabetes. Diabetes Care. 2004;27(suppl 1):S65S67.

Systolic Hypertension in the Elderly


In the SHEP (Systolic Hypertension in the Elderly Program) study, treatment with chlorthalidone resulted in reduction of:
Stroke incidence: 36% Coronary heart disease: 27% CHF: 55%

Answers
1. 2. 3. 4. 5. C B A C D

The Major Arthritides: Practical, Evidence-Based, Clinical Pointers

B. Wayne Blount, MD, MPH JenCare & Professor Emory University SOM

Learning Objectives
1. Cite a systematic approach to the diagnosis of arthritic disorders. 2. Discuss the Dx & management of RA, OA, & Gout. 3. Identify indications and use of disease modifying anti-rheumatic drugs

Additional slides for your study:


More on above topics Spondyloarthropathies Lyme Pseudogout

Question #1 A 28 YO Hisp F c/o 8-mo Hx of painful, swollen joints.


+ AM stiffness; several joints in hands & arm involved, bilaterally; + fatigue; no wgt loss, fever PE WNL except joints:
Swollen, tender left wrist, 3 MCPs & 2 PIPs; Tender right wrist, left ankle & other MCPs

Question #1 What type of arthritis does she have?


A. Monoarticular B. Crystal-induced C. Symmetric polyarticular D. Asymmetric polyarticular

An Approach to Articular Disease


Look for the pattern: 1 of 3 patterns
Monoarticular Symmetric Polyarticular
Asymmetric Polyarticular

Acute Monoarthritis
Inflammation (swelling, tenderness, warmth) in one joint
Occasionally polyarticular diseases can present with monoarticular onset: (RA, JRA, reactive and enteropathic arthritis, sarcoid, viral, psoriatic arthritis) THE MOST CRITICAL DIAGNOSIS TO CONSIDER: INFECTION!

Monoarticular
Septic Gout CPPD Tumor Trauma Viral

Symmetric Polyarthritis
RA SLE Psoriatic Osteoarthritis Scleroderma Lyme Rheumatic fever Gout CPPD Hepatic

Asymmetric Polyarthritis
HLA B-27 dz: Psoriatic, ankylosing spondylitis, reactive arth, IBD Gout CPPD Lyme Viral

Key Points in the Approach to Arthritides


1. Distinguish arthritis from soft-tissue non-articular syndromes (discrepancy between active and passive ROM suggests periarticular/soft tissue) 2. If the problem is articular, distinguish single-joint from multiple-joint involvement 3. Inflammatory or non-inflammatory disease 4. Always consider septic arthritis!

Question # 2 Which of the following findings is most specifically diagnostic of classic RA? A. B. C. D. High ESR Positive ANA Rheumatoid joint erosions Rheumatoid factor

Answer
C. Rheumatoid joint erosions All of the others are associated with RA, but only joint erosions are seen only in classic RA.

Question # 3 To diagnose RA, one needs which of the following?


A. B. C. D. E. 3/6 criteria fulfilled 5 points total 6 Points total 4/6 criteria fulfilled 5/8 criteria fulfilled

New Criteria for RA


Every patient with > 6 points is unequivocally classified as an RA patient Provided he has synovitis in > 1 joint and given that there is no other diagnosis better explaining the synovitis. 4 areas are covered in the diagnosis: See extra slides

Question #4 In treating RA, DMARDs should be started when?

A. As soon as the diagnosis is made B. After 3 months of therapy with NSAIDs C. Only after NSAIDs have failed D. Only by a rheumatologist

Treatment of Rheumatoid Dz
DMARDs should be started as early as possible to delay dz progression; A Rec Use NSAIDs in lowest dose for relief & cut back when DMARDs work; A Rec No regimen of monotherapy is clearly superior to any other; A Rec, AHRQ, 2007 Exercise is effective in improving function & reducing bone loss; A Rec, Cochrane, 2009 Alt/comp med therapies have NO evidence; A Rec, Cochrane: Diet, U/S, balneotherapy, acupuncture

Treatment of Rheumatoid Dz
Corticosteroids are effective but have high toxicity. Use lowest dose possible for shortest time. A Rec Combo Rx more effective than monotherapy. A Rec,
AHRQ, 2007 & Annals of Rheumatic Dis, 2009

But do not combine biologic agents; combine with MTX Cure is elusive; True remission by only a minority Current use & cumulative exposure of oral steroids may increase risk of M.I. in R.A. (up 68%)

Psoriatic Arthritis

Psoriatic Arthritis
Prevalence of arthritis in psoriasis: 5-20% Psoriasis usually precedes PSA - 75% In 15 - 20%, arthritis precedes psoriasis Nail changes common Usually insidious, but 1/3 have acute onset Enthesopathy at Achilles Dactylitis Psoriatic plaques Scalp, extensor surfaces, natal cleft, umbilicus

Psoriatic Arthritis
Subtypes: Asymmetric, oligoarticular-associated dactylitis Predominant DIP involvement nail changes Polyarthritis RA-like lacks RF or nodules Arthritis mutilans destructive erosive hands/feet Axial involvement spondylitis HIV-associated more severe

Diagnosis of Psoriatic Arthritis


Point system: Need to have established inflammatory articular disease with > 3 points from: Current psoriasis: 2 points A Hx of psoriasis (when it is not current): 1 point A family Hx of psoriasis (when not current & no personal Hx.): 1 point Dactylitis: 1 point RF negative: 1 point Nail dystrophy: 1 point Juxta-articular new bone formation: 1 point No specific lab findings

Treating Psoriatic Arthritis

Physical therapy: Start early! Only 2 pharm agents have well-demonstrated efficacy: Parenteral, high-dose Methotrexate Sulfasalazine Cochrane, 2009 Can try Cyclosporine, leflunomide, and TNFs

Osteoarthritis

A chronic joint disorder in which there is progressive softening and disintegration of articular cartilage accompanied by new growth of cartilage and bone at the joint margins (osteophytes) and capsular fibrosis

Osteoarthritis Classification
Primary or idiopathic
Secondary
Infection Dysplasia Perthes SUFE Trauma AVN

Osteoarthritis Etiology

Genetic Metabolic Hormonal Mechanical Aging

3 Osteoarthritis Mechanisms
1. Disparity between stress applied to articular cartilage & strength of articular cartilage.
2. Increased stress (F/A) Increased load eg, BW or activity Decreased area eg, varus knee or dysplastic hip 3. Weak cartilage

Arthritis Symptoms Pain Swelling Stiffness Deformity Instability Loss of function Chronicity

Osteoarthritis X-ray Changes

Joint space narrowing Subchondral sclerosis Osteophytes Cysts

Osteoarthritis X-ray Changes

Osteoarthritis Non-surgical Treatment Analgesia: Acetaminophen Topical NSAIDs Walking aids Weight Loss Physiotherapy; esp aquatics Altered activity
Disease modifying drugs ?

Arthritis Surgical Treatment

Arthroscopy Osteotomy Arthrodesis Excision arthroplasty Replacement arthroplasty

Question #5 What is the definition of gout?

A. B. C. D.

Serum urate > 6.0 mg/dL Deposition of crystals in tissues Serum urate > 6.8 mg/dL Having podagra

Gout
Caused by monosodium urate crystals Most common type of inflammatory monoarthritis Typically: first MTP joint, ankle, midfoot, knee Pain very severe; cannot stand bed sheet May be with fever and mimic infection The cutaneous erythema may extend beyond the joint and resemble bacterial cellulitis

GOUT: A Chronic Disease of 4 stages. Each stage needs treatment.


Asymptomatic hyperuricemia
Acute Flares of crystallization Intervals between flares Advanced Gout & Complications
See Extra slides for treatment

Summary
Diagnostic approach to Arthritis Rheumatoid Arthritis DMARDs Psoriatic Arthritis Osteoarthritis Gout

Extras

Bibliography
1. Gill JM, Quisel AM, Occa PV, Walters DT. Diagnosis of systemic lupus erythematosus. AFP. 2003; 68:2179-86. 2. Siva C, Velasquez C, Mody A, Brasington R. Diagnosing acute monoarthritis in adults. AFP. 2003;68:83-90. 3. Richie AM, Francis ML. Diagnostic approach to polyarticular joint pain. AFP. 2003; 68:1151-60. 4. Lane SK, Gravel JW. Clinical utility of common serum rheumatologic tests. AFP. 2002;65:1073-80. 5. Cochrane review 6. Rindfleisch JA, Muller D. Diagnosis & Management of Rheumatoid Arthritis. AFP. 2005;72:1037-47. 7. Westlake SL et al. The Effect of Methotrexate on Cardiovascular Disease in Patients with Rheumatoid Arthritis. Rheumatology. 2010;49:295-307. 8. Avina-Zubieta J. Rheumatology, Jan, 2013. 9. British Soc of Rheumatology; The 2012 guideline for the treatment of psoriatic arthritis. Rheumatology. 2013;52:1754-7. Treatment of Knee Osteoarthritis 10. Erika Ringdahl,and Sandesh Pandit, Am Fam Physician 2011 Jun 1;83(11):1287-1292.

Bibliography
ACR Guidelines for Pharmacologic and Nonpharmacologic Treatment of Gout. Available at: http://onlinelibrary.wiley.com/doi/10.1002/acr.21772/pdf ACR Recommendations on Therapies for Osteoarthritis of the Hand, Hip, and Knee. Available at: http://ql3dq7xx6q.search.serialssolutions.com/?sid=Entrez:PubMed&id=pmid:225 63589 Implementing AHRQ Effective Health Care Reviews: Helping Clinicians Make Better Treatment Choices Analgesics for Osteoarthritis. Am Fam Physician. 2013 Mar 1;87(5):354-356.

Answers
1. C 2. C 3. C 4. A 5. B

Extras

1st Area to Dx R.A.


Joint involvement : designating the metacarpophalangeal joints, proximal interphalangeal joints, the interphalangeal joint of the thumb, second through third metatarsophalangeal joint and wrist as small joints Shoulders, elbows, hip joints, knees, & ankles as large joints

1. Involvement of
1 large joint = 0 points 2-10 large joints = 1 point 1-3 small joints (with or without involvement of large joints) = 2 points 4-10 small joints (with or without involvement of large joints) = 3 points > 10 joints (with involvement of at least 1 small joint) = 5 points

2. Serological Parameters Including RF & Anti-Citrullinated Protein Antibody (ACPA)


Negative RF and negative ACPA: 0 points Low-positive RF or low-positive ACPA: 2 points High-positive RF or high-positive ACPA: 3 points

3. R.A. Acute Phase Reactants


1 point for elevated erythrocyte sedimentation rate (ESR), or elevated CRP value (c-reactive protein)

4. R.A. Duration of Arthritis


1 point for symptoms lasting six weeks or longer

Use of Rheum Blood Tests


Selective ordering Interpreted cautiously Interpret within individual patients clinical situation Most helpful to confirm your clinical diagnosis Ordering panels is discouraged

Non-Rheumatic Diseases with Positive RF


Hepatitis C Mixed cryoglobulinemia Sarcoidosis Pulmonary fibrosis Infections Aging < 70% 90% 5-30% 20% Varies 5%

ANA
Reported as titers: > 1:320 more likely to be true dz Titers of < 1:40 unlikely to have a rheumatologic dz ANA pattern is more specific for dz Best for SLE, drug-induced lupus, Sjgren, scleroderma & MCTD

Disease-Specific ANAs
Disease SLE RA Scleroderma MCTD Polymyositis Sjgren Wegener ANA Assoc Anti-Smith RF Anti-centromere Anti-U1RNP Anti-Jo-1 Anti-SSA & anti-SSB c-ANCA & p-ANCA

Chromatin Antibodies
Anti-dsDNA: Rule in SLE Anti-histone: Rule out drug-induced lupus Anti-Smith: R/I SLE Anti-Ro: Assoc with Sjgren Anti-centromere: Assoc with scleroderma c-ANCA: Sens & spec for Wegener

ESR
Low Specificity Correlates with clinical activity in RA Best when used for PMR: Sens = 80% TA: Sens = 95%
Usually quite high vs elevated (NOT 20-50)

Indications for Arthrocentesis


Synovial Fluid Analysis: The single most useful diagnostic study in initial evaluation of monoarthritis
1. 2. 3. 4. Suspicion of infection Suspicion of crystal-induced arthritis Suspicion of hemarthrosis Differentiating inflammatory from noninflammatory arthritis

Tests to Perform on Synovial Fluid


Gram stain and cultures Total leukocyte count/differential Inflammatory vs. non-inflammatory Polarized microscopy to look for crystals Not really routinely: Chemistry (glucose, total protein, LDH) unlikely to yield helpful information beyond the previous tests.

Synovial Fluid Analysis


Joint Fluid
Normal NonInflammatory Inflammatory Septic

Appearance
Clear/Yellow Clear/Yellow Turbid/Yellow Pus

Cell Count
<200 WBCs <2000 WBCs <50,000 WBCs >50,000 WBCs

Other Tests Indicated for Acute Arthritis


1. Almost always indicated:
Radiographs CBC ESR/CRP

2. Indicated in certain patients:


Cultures

3. Rarely indicated:
Serologic: ANA, RF, HLA-B27 Serum uric acid level

Septic Joint

Most articular infections a single joint 15-20% cases polyarticular Most common sites: knee, hip, shoulder 20% patients afebrile Joint pain is moderate to severe Joints visibly swollen, warm, often red Comorbidities: RA, DM, SLE, cancer, etc.

Septic Joint - Nongonococcal

80-90% monoarticular Most develop from hematogenous spread Most common:


Gram positive aerobes (80%) Majority with Staph. aureus (60%) Gram negative 18%

Likely Causes of Septic Arthritis


Gram Stain Pt Characteristic Organism of Concern

No bacteria
No bacteria No bacteria

Young, healthy
Hx of RA Immunosupression, IV drugs, Hx gm- infection Recent cat/dog bite

GC, Staph
Staph Staph, Strep, Pseudomonas, fungal Pasteurella multocida

No bacteria or Gm -

Gm+
Gm- diplococci Gm Gm No bacteria No bacteria

None
None None SLE or sickle cell Hx prosthetic joint HX fresh/salt H20 exposure + injury; chronic swelling

Staph/Strep
GC (consider meningococcemia) Rx for possible Pseudomonas Include coverage for Salmonella & Pseudomonas Staph. epidermidis, Staph. aureus Mycobacterium marinum

Pseudogout
Can cause monoarthritis; clinically indistinguishable from gout. Often precipitated by illness or surgery. Pseudogout is most common in the knee (50%) and wrist. Reported in any joint (including MTP). CPPD disease may be asymptomatic (deposition of CPP in cartilage).

Inflammatory Vs. Noninflammatory


Feature Morning stiffness Fatigue Activity Rest Systemic Corticosteroid Inflammatory >1h Profound Improves Worsens Yes Yes Mechanical < 30 min Minimal Worsens Improves No No

Temporal Patterns in Polyarthritis


Migratory pattern: Rheumatic fever, gonococcal (disseminated gonococcemia), early phase of Lyme disease Additive pattern RA, SLE, psoriasis Intermittent: Gout, reactive arthritis

Patterns of Joint Involvement


Symmetric polyarthritis involving small and large joints: viral, RA, SLE, one type of psoriatic (the RA-like). Asymmetric, oligo- and polyarthritis involving mainly large joints, particularly lower extremities, especially knee and ankle: reactive arthritis, one type of psoriatic, enteropathic arthritis. DIP joints: Psoriatic.

Gout Treatment Goals


Rapidly end acute flares Protect against future flares Reduce chance of crystal inflammation
Prevent disease progression Lower serum urate to deplete total body urate pool Correct metabolic cause

Which 2 Pathologic Mechanisms Cause Hyperuricemia? Overproduction Underexcretion Which one is the predominant cause (in 90% of patients)?

Underexcretion

Hyperuricemia & Gout

Hyperuricemia caused by Overproduction Underexcretion

No gout w/o crystal deposition

2nd Stage: Acute Flares

2nd stage of gout is heralded by the 1st acute attack.


90% of 1st attacks are monoarticular; Any joint is a possibility 50% are podagra

Untreated? Resolves in 3-10 days

3rd Stage: Intervals Sans Flares


Asymptomatic

If untreated, may advance


Intervals may shorten Crystals in ASx joints Body urate stores increase

4th Stage: Advanced Gout


Chronic arthritis
X-ray changes Tophi develop Acute flares continue with upper extremities more involved Avg. time from initial attack to chronic gout is 11.6 yrs.

X-rays

Calcified, overhanging edge is typical of gout

Diagnosing Gout
Hx & P.E.
Synovial fluid analysis ? Clinical Diagnosis ?

Not Serum Urate: Not reliable


May be normal with flares May be high with joint Sx from other causes

Synovial Fluid Analysis (Polarized Light Microscopy) The Gold standard Crystals intracellular during attacks Needle & rod shapes Strong negative birefringence

?Clinical Dx?

Typical presentation Use colchicine in a typical presentation (Familial Mediterranean fever) Ultrasonography use is increasing MRI & CT?

Treatment Goals

Rapidly end acute flares


Protect against future flares Reduce chance of crystal inflammation

Prevent disease progression Lower serum urate to deplete total body urate pool Correct metabolic cause

Ending Acute Flares

Control inflammation & pain, & resolve the flare Not a cure Crystals remain in joints Dont try to lower serum urate during a flare Choice of med not as critical as alacrity & duration (?) EBM At least 3 days; usually 5-7 days

MED Considerations
Colchicine:
Not as effective late in flare Only 1 agent on US market now: $$ Contraindicated in dialysis pts. Cautious use in: renal or liver dysfunction; active infection, age > 70

Numerous meds increase serum colchicine: Statins, digoxin, macrolides, -azoles, CCBs, grapefruit *Loading dose = 1.2 mg; then 0.6 mg 1 hr. later

MED Considerations
Corticosteroids:
Worse glycemic control Oral, intra-articular (esp in monoarticular flare), or parenteral May need to use mod-high doses. New guidelines suggest 10 mg/day. I disagree: Needs to be higher: > 20mg Useful in patients who have contraindications to NSAIDs & colchicine

Treatment Goals
Rapidly end acute flares
Protect against future flares Reduce chance of crystal inflammation

Prevent disease progression

Lower serum urate to deplete total body urate pool Correct metabolic cause

When starting a urate-lowering agent, what is the recommended amount of time to overlap with a flare preventative?

A. 2 weeks B. 1 month C. 2 months D. 4 months E. 6 months

Protection vs. Future Flares


Colchicine : 0.6-1.2 mg/day (0.3 if CRI) Low-dose NSAIDs (eg, 25 mg of indomethacin or 250 mg of naproxen) Both decrease freq & severity of flares Prevent flares with start of urate-lowering RX
Best with 6 mos of concommitant RX; > 3 months w/o flare

EBM: B
Wont stop destructive aspects of gout

Use of Colchicine
Only drug approved by FDA for preventing acute flares Used as 0.6 mg q day or BID for 6 months EBM: B; (3% flares vs. 40%) A.E.s: Diarrhea, LFTs, HA *Start the prophylactic dose 12 hrs. after the 2nd dose for the acute flare

New Evidence for Preventing Flares


Cherry intake lowers risk for flares by 35% Cherry extract intake lowers risk for flares by 45% Allopurinol alone reduces risk by 53% Allopurinol & cherries together reduced it by 75% ? Anti-inflammatory &/or reduce urate reabsorption in kidneys
Arthritis & Rheumatism, Sept 28, 2012

Treatment Goals
Rapidly end acute flares Protect against future flares Reduce chance of crystal inflammation

Prevent disease progression Lower serum urate to deplete total body urate pool Correct metabolic cause

In relation to an acute flare, when does one start a urate-lowering medicine?

A. Right away B. At end of acute flare C. 3-5 days after end of flare D. 1-2 weeks after end of flare

Urate-Lowering Therapy (ULT)


Not to be started during an acute attack. 1-2 weeks after acute attack resolves. Prophylaxis starts simultaneously or before Difference of opinion about who should start ULT Everyone with gout? Not in pts with only 1 attack & no complications (tophi, CRI, stones, or diuretic use) Shared decision* Definitely all pts with 3 attacks or tophi or urolithiasis

In treating to goal, the target serum urate level is A. < 7.0 mg/dL B. < 6.8 mg/dL C. < 6.5 mg/dL D. < 6.2 mg/dL E. < 6.0 mg/dL

Conservative Measures to Lower Urate Diet: reduce purines Alcohol: Cut The above 2 measures will lower urate by 10-15% Avg. starting level is 9.5 10. Rarely get to < 6.0 with these measures; but they help.

Which of the following is not an uricosuric agent?

A. Probenecid B. Candesartan C. Fenofibrate D. Losartan

Uricosuric Agents
Probenecid: Only FDA-approved one Losartan & fenofibrate for mild disease
Increased secretion of urate into urine

Reverses most common physiologic abnormality in gout (90% pts are underexcretors) 1/3 pts discontinue it. Not in pts with lithiasis

Xanthine Oxidase Inhibitor Allopurinol, oxypurinol, or febuxostat: Block conversion of hypoxanthine to uric acid Effective in overproducers May be effective in underexcretors Can work in pts with renal insufficiency

Allopurinol
Allopurinol may lower all-cause mortality & C.V. events; EBM: C In CKD, allopurinol may slow progression of CKD; EBM: C Adjust dose in CKD Remember AEs; esp T.E.N.S. If rash, stop med and come in. (!CKD & Diuretics!) Lowering sUA is dose dependent:
Achieved goal sUA: 26% @ 300mg/day vs 78% @ 300mg BID

Using Allopurinol Treat to Goal


*Start at 100 mg/day Gradually titrate up by 50-100 mg/day every 2 5 weeks Slower titration in CKD Treat to goal (Lowest dose that gets < 6.0) Too-fast lowering of urate can trigger a flare

Using Allopurinol Treat to Goal


Goal is Serum Urate < 6 mg/dL
*Most patients will need > 300 mg/day of allopurinol to achieve this goal

New Recommendation
To reduce allopurinol toxicity, consider HLA-B*5801 screening pts @ high risk:
Koreans > stage 3 CKD & All pts. of Han Chinese & Thai descent

New Recommendation To get to goal, can use combination of xanthine oxidase inhibitor and uricosuric.

Sites
Can occur in other joints, bursa, & tendons

PREVENT DISEASE PROGRESSION


Lower urate to < 6.0 mg/dL: This depletes total body urate pool & deposited crystals EBM A
Rx is lifelong & continuous Med choices: Uricosuric agent Xanthine oxidase inhibitor Uricase

Viral Arthritis
Younger patients Usually presents with prodrome, rash History of sick contact Polyarthritis similar to acute RA Prognosis good; self-limited Examples: Parvovirus B-19, rubella, hepatitis B and C, acute HIV infection, Epstein-Barr virus, mumps

Reactive arthritis

Ankylosing spondylitis

Psoriatic arthritis

Spondyloarthropathy
Arthritis associated with inflammatory bowel disease

Undifferentiated spondyloarthropathy

Inflammatory Back Pain


Onset of back discomfort before age 40 Insidious onset > 3 months duration Morning stiffness in the back Improvement with exercise If 4 of these are met, AS is diagnosed

Ankylosing Spondylitis

0.2% of population Mainly affects spine and SI joints Male > female HLA B27 in 90% Synovitis Enthesopathy

Ankylosing Spondylitis Hips and Knees

Flexion deformities Arthritis with large osteophytes Ankylosis

Ankylosing Spondylitis Diagnosis

Rome criteria
A - bilateral SI B - stiff lumbar spine - stiff thoracic spine - decreased chest expansion (< 7cm) - > 3/12 LBP - Iritis

A+ 1xB or 4xB

Ankylosing Spondylitis X-ray Changes

Joint space narrowing Large osteophytes Heterotopic bone Ankylosis

Reactive Arthritis Triggered by specific gut or genito-urinary tract


infections Salmonella, Yersinia, Campylobacter, Shigella Chlamydia Joint symptoms appear 1-3 week later Oligoarthritis; usually lower extremity Enthesitis Frequent association with extra-articular findings A proportion evolve into chronic spondyloarthropathy

Extra-Articular Features of Reactive Arthritis


Dont be put off if they are not present Ask about GI disturbance - even mild Ask about conjunctivitis Take a sexual history (with explanation) Examine eyes and skin (soles/external genitalia) Look for enthesitis

Septic Joint - Gonococcal


Most common cause of septic arthritis Often preceded by disseminated gonococcemia Sexually active individual, 5-7 days h/o fever, chills, skin lesions, migratory arthralgias and tenosynovitis persistent monoarthritis Women often menstruating or pregnant Genitourinary disease often asymptomatic

Lyme Arthritis
Erythema migrans 7-10 days after Borrelia burgdorferi tick bite Early dissemination Migratory arthralgias, fever, systemic complaints Late dissemination/chronic disease Migratory oligoarthritis Carditis Neurological

Practically speaking, which one of the following is not an appropriate way to diagnose gout? A. Response to colchicine. B. Symptomatic joint aspiration C. Asymptomatic joint aspiration D. Serum urate level E. Clinical presentation Ans: D

Which of the following is the recommended 1st line Rx for O.A.?


A. Aspirin B. Indomethacin C. Acetaminophen D. Ibuprofen E. Low-dose corticosteroid

Which of the following is not typically in the differential for symmetric polyarthritis?

A. Rheumatoid Arthritis B. Gout C. Septic arthritis D. Calcium pyrophosphate disease E. Scleroderma

Ans: C

Which of the following is not true when using DMARDs?


A. Combination therapy may be better than monotherapy. B. Methotrexate is the recommended 1st DMARD to be used. C. If using combined therapy, using 2 TNFs is recommended. D. Exercise can add to the efficacy of DMARD treatment. Ans: C

DMARDs
Disease-modifying anti-rheumatic drug Immunosuppressants Anti-malarials TNF Cytokine Inhibitors D-penicillamine: High toxicity Gold: Limited by adverse effects Steroids: Assoc with increased CV risk. Used today only as an adjunct; systemic or intra-articular

Use with contraception TNF inhibitors: Increase in skin cancer & opportunistic infections Biologics are TNFI, T-cell I, B-cell modulators and interleukin modifiers

DMARDs
TNF-a inhibitors may be the most powerful DMARDs for overall efficacy in RA, JRA*, Ankylosing Spondylitis, & Behcets Reduce disease activity, retard progression, & improve QOL But watch for adverse effects
Int J Clin Rheum. 2010;5:101-15.

DMARDs
MTX can achieve disease control in about 1/3 of RA patients. The other 65% are candidates for biologics.
ACR Conf 2012

Also newest rec is that one should not start a biologic for RA w/o 1st a trial of MTX. 3/2013 Arthritis Care & Research

Arthritis of SLE
Musculoskeletal manifestation 90%. Most have arthralgia. May have acute inflammatory synovitis RA-like. Do not develop erosions. Other clinical features help with DD: malar rash, photosensitivity, rashes, alopecia, oral ulceration.

Heart Failure
Jonathon Firnhaber, MD, FAAFP Assistant Professor The Brody School of Medicine at East Carolina University Greenville, North Carolina

Disclosure Statement
Dr. Firnhaber has nothing to disclose.

The AAFP has selected all faculty appearing in this program. It is the policy of the AAFP that all CME planning committees, faculty, authors, editors, and staff disclose relationships with commercial entities upon nomination or invitation of participation. Disclosure documents are reviewed for potential conflicts of interest and, if identified, they are resolved prior to confirmation of participation. Only those participants who had no conflict of interest or who agreed to an identified resolution process prior to their participation were involved in this CME activity.

Objectives
1. Establish the diagnosis and therapy of acute coronary syndrome. 2. Formulate the indications of fibrinolysis, glycoprotein llb/lla agents, and angioplasty. 3. Analyze the difference between systolic and diastolic cardiac dysfunction. 4. Differentiate the therapies that improve survival in heart failure and those that only improve symptoms. 5. Review the role of neurohormonal activation in systolic heart failure.

1. Which one of the following statements regarding systolic heart failure is correct?
A. Left ventricular ejection fraction is < 40%. B. Systolic dysfunction produces different symptoms than diastolic dysfunction. C. Systolic dysfunction results from increased ventricular stiffness. D. Systolic dysfunction and diastolic dysfunction do not coexist in the same patient.

Systolic Heart Failure


Systolic dysfunction: ejection fraction < 40% Systolic dysfunction may produce the same symptoms as diastolic dysfunction. Diastolic dysfunction results from increased ventricular stiffness and decreased ventricular compliance. Both systolic and diastolic dysfunction may be present in the same patient.

Systolic Heart Failure Treatment


Low cardiac output triggers neurohormonal activation, which ultimately results in premature apoptosis of cardiac myocytes.
Preload reduction
Diuretics, nitrates

Afterload reduction
ACEI, ARB, hydralazine, nitrates

Sympathetic blockade
-blockers

Aldosterone-antagonist therapy
Spironolactone, eplerenone (Inspra)

2. A 70-year-old woman with hypertension presents with progressive dyspnea. Rales at the lung bases are noted, and chest x-ray shows pulmonary edema with a normal size heart. Which of the following statements is true?
A. Viral cardiomyopathy presents with diastolic dysfunction. B. Treatment of diastolic heart failure requires high-dose diuretics. C. The ejection fraction with diastolic heart failure is > 40%. D. Digoxin therapy should be considered in the majority of patients with diastolic heart failure.

Diastolic Heart Failure


Careful preload reduction is appropriate; patients tend to be volume sensitive and can develop hypotension with excessive diuresis.
Recall that ventricular filling occurs during diastole. Impaired ventricular relaxation limits diastolic filling, decreases stroke volume, and ultimately decreases cardiac output (CO). Elevated heart rate further decreases diastolic filling time (decreasing CO). Diminished preload also decreases ventricular filling.

Diastolic Heart Failure


Careful decrease in heart rate, using blocker or non-dihydropyridine CCB is appropriate. If rapid atrial fibrillation/flutter is present, digoxin is indicated for rate control.
Larger doses of -blocker or non-dihydropyridine CCB may have excessive negative inotropic effect.

3. Which of the following is considered the single most useful diagnostic study in the evaluation of patients with heart failure? A. B. C. D. Brain-natriuretic peptide (BNP) ECG Chest x-ray 2-D echocardiogram with Doppler flow studies E. Nuclear stress test

Echocardiogram in HF
Three fundamental questions:
1. Is the LV ejection fraction (EF) preserved or reduced? 2. Is the structure of the LV normal or abnormal? 3. Are there other structural abnormalities such as valvular, pericardial, or right ventricular abnormalities that could account for the clinical presentation?

4. A 56-year-old male with longstanding hypertension and a 30-pack-year smoking history has a 2-day history of dyspnea on exertion. Physical examination is only notable for rare crackles at the bases. Which one of the following serologic tests would be most helpful for detecting left ventricular dysfunction? A. B. C. D. E. Brain-natriuretic peptide (BNP) Troponin-T C-reactive protein D-dimer Cardiac interleukin-2

BNP and Heart Failure


BNP is secreted from the ventricles in response to ventricular volume expansion and pressure overload. Release is directly proportional to ventricular dysfunction and correlates with end-diastolic pressure. BNP undergoes partial renal excretion; levels are inversely proportional to creatinine clearance.

BNP and Heart Failure


BNP < 100 excludes CHF as a cause of dyspnea BNP > 400 confers a 95% likelihood of CHF BNP 100400 requires further investigation
Pulmonary disorders that can elevate BNP include lung cancer, cor pulmonale, and pulmonary embolus Other causes of BNP elevation include cirrhosis with ascites, primary hyperaldosteronism, Cushing disease These disorders do not cause the same extent of BNP elevation as LV dysfunction

5. Which one of the following is preferred for chronic treatment of congestive heart failure due to left ventricular systolic dysfunction? A. Diuretics B. Digoxin C. Calcium channel blockers D. ACE inhibitors E. Hydralazine plus isosorbide dinitrate

ACE Inhibitors and Heart Failure


ACEIs are preferred for CHF due to LV systolic dysfunction because they offer the greatest reduction in mortality.

Other Treatments for Heart Failure


Hydralazine/isosorbide is a reasonable alternative. Diuretics typically should not be used as monotherapy. Digoxin may help with symptoms and decreases hospitalizations, but has not been proven to affect mortality.
Aim for a serum level of < 1.0 ng/mL

6. A 62-year-old African-American male is admitted to the hospital for the third time in 6 months with heart failure. He has dyspnea with minimal activity. Echocardiography reveals an ejection fraction of 40%. Which one of the following combinations of medications is most appropriate for long-term management of this patient?
A. B. C. D. Enalapril plus digoxin Hydralazine plus isosorbide dinitrate Losartan plus amlodipine Spironolactone plus bisoprolol

Vasodilators in Heart Failure


Hydralazine plus isosorbide dinitrate reduces mortality rates and improves quality-of-life measures
When standard treatment with diuretics, blockers, and an ACEI (or ARB) is insufficient to control symptoms or cannot be tolerated Particularly effective in African-Americans with NYHA class III or IV heart failure

Vasodilators in Heart Failure


Dihydropyridine calcium channel blockers (amlodipine, felodipine, etc.)
No direct role in the treatment of heart failure May be used for blood pressure lowering if standard HF therapy does not provide adequate control

7. A 55-year-old male has New York Heart Association class III chronic systolic heart failure due to hypertensive cardiomyopathy. Which one of the following is contraindicated in this patient?
A. B. C. D. E. Carvedilol Digoxin Ramipril Spironolactone Verapamil

Good and Bad Drugs for HF


ACEIs and -blockers improve mortality in heart failure (HF).

Aldosterone antagonists, including spironolactone and eplerenone


Reduce all-cause mortality Improve ejection fractions in systolic HF

Good and Bad Drugs for HF


Digoxin and furosemide improve symptoms and reduce hospitalizations in systolic HF.
Furosemide may decrease mortality.

Verapamil, due to its negative inotropic effect, is associated with worsening heart failure and an increased risk of adverse cardiovascular events.

8. Which one of the following agents is most appropriate for use in a hypertensive patient with CHF?

A. B. C. D.

Nebivolol (Bystolic) Amlodipine (Norvasc) Carvedilol (Coreg) Aliskiren (Tekturna)

-blockers in CHF
Three have sufficient data to support their use: Metoprolol succinate Carvedilol Bisoprolol

9. Which one of the following is considered a contraindication to the use of -blockers for congestive heart failure?
A. Mild asthma B. Symptomatic heart block C. New York Heart Association (NYHA) Class III heart failure D. NYHA Class I heart failure in a patient with a history of a previous myocardial infarction E. An ejection fraction < 30%

-blockers and Heart Failure


Mortality rates are improved in HF patients who receive -blockers in addition to ACEIs and diuretics. -blockers decrease mortality in patients with prior MI, regardless of NYHA classification

-blockers and Heart Failure


-blockers are effective in patients with NYHA Class II or III heart failure. There is no absolute threshold ejection fraction. -blockers should be started when the patient is stable and euvolemic.

-blockers and Heart Failure


Contraindications to -blocker use include:
Hemodynamic instability Heart block Bradycardia Severe asthma

-blockers may be initiated in patients with mild asthma or COPD, as long as they are monitored for potential complications.

References: Heart Failure


ACC/AHA 2007 Guidelines for the Management of Patients With Unstable Angina/NonST-Elevation Myocardial Infarction: Executive Summary A Report of the American College of Cardiology/American Heart Association Task Force on Practice Guidelines (Writing Committee to Revise the 2002 Guidelines for the Management of Patients With Unstable Angina/NonSTElevation Myocardial Infarction) Circulation 2007;116;803-877. Braunwald E, Antman EM, Beasley JW, et al: ACC/AHA 2002 Guideline Update for the Management of Patients with Unstable Angina and Non-ST-Elevation Myocardial Infarction: A Report of the American College of Cardiology/American Heart Association Task Force on Practice Guidelines. Committee on the Management of Patients with Unstable Angina. American College of Cardiology/American Heart Association, 2002, pp 6-7.

References: Heart Failure


Krishnaswamy P, Lubien E, Clopton P, et al. Utility of betanatriuretic peptide levels in identifying patients with left ventricular systolic or diastolic function. Am J Med. 2001;111(4):274-279. Maisel AS, Mehra MR. Understanding B-type natriuretic peptide and its role in diagnosing and monitoring congestive heart failure. Clin Cornerstone. 2005;7(Suppl 1):S7-S17. Jeremias A, Gibson M. Narrative review: alternative causes for elevated cardiac troponin levels when acute coronary syndromes are excluded. Ann Intern Med. 2005; 142(9): 786-91. Miller CD, Roe MT, Mulgard J, et al. Impact of acute betablocker therapy for patients with non-ST-segment elevation myocardial infarction. Am J Med. 2007;120:685-692.

References: Heart Failure


Poole-Wilson PA, Swedberg K, Cleland JG, et al. Comparison of carvedilol and metoprolol on clinical outcomes in patients with chronic heart failure in the Carvedilol Or Metoprolol European Trial (COMET): randomised controlled trial. Lancet. 2003;362:7-13. Daniels LB, Maisel AS. Natriuretic peptides. J Am Coll Cardiol. 2007;50(25):2357-2368. Wu AH, Jaffe AS, Apple FS et al. National Academy of Clinical Biochemistry laboratory medicine practice guidelines: use of cardiac troponin and B-type natriuretic peptide or N-terminal proB-type natriuretic peptide for etiologies other than acute coronary syndromes and heart failure. Clin Chem. 2007;1;53(12):2086-96. Jessup M, Silver MA, Stevenson LW, et al. 2009 focused update: ACCF/AHA Guidelines for the Diagnosis and Management of Heart Failure in Adults: a report of the American College of Cardiology Foundation/American Heart Association Task Force on Practice Guidelines: developed in collaboration with the International Society for Heart and Lung Transplantation. Circulation. 2009;119:19772016.

Answers
1. 2. 3. 4. 5. A C D A D

6. 7. 8. 9.

B E C B

Behavioral Medicine II:


Bipolar Disorders and Anxiety Disorders Stanley P. Oakley, Jr, MD, FAPA

Professor Department of Psychiatric Medicine The Brody School of Medicine at East Carolina University Greenville, North Carolina

Disclosure Statement
Dr. Oakley has nothing to disclose.

The AAFP has selected all faculty appearing in this program. It is the policy of the AAFP that all CME planning committees, faculty, authors, editors, and staff disclose relationships with commercial entities upon nomination or invitation of participation. Disclosure documents are reviewed for potential conflicts of interest and, if identified, they are resolved prior to confirmation of participation. Only those participants who had no conflict of interest or who agreed to an identified resolution process prior to their participation were involved in this CME activity.

I always thought that the brain was the most important organ in the body, until I realized who was telling me that.

E. Phillips

Learning Objectives
1. Recognize the differential diagnosis and clinical presentation of Bipolar Disorders and Anxiety Disorders 2. List the treatment options for Bipolar Disorders and Anxiety Disorders

1. Which of the following is NOT true about bipolar disorder?


A. Kindling is the acceleration of untreated mood swings B. Bipolar disorder occurs equally in men and women C. Bipolar II is characterized by depression alternating with mania D. Hypomania is milder than mania, without gross lapses in impulse control or reality testing

Bipolar Disorder
Is lifelong and chronic Mood becomes elevated or depressed without obvious trigger Affects 1 in 60 to 1 in 100 people Onset frequently in late teens/early 20s Onset of new manic episode after age 65 extremely rare (rule out medical causes)

Bipolar Illness
1st degree relatives of bipolar patients are bipolar 50% of the time Sex distribution equal for Bipolar I Bipolar II more common in women

Diagnosis of Bipolar Disorder


Requires a period of increased activity, with an abnormally elevated, expansive, or irritable mood Along with 3 or more of:
Grandiosity or euphoria Decreased need for sleep Pressure to keep talking Flight of ideas or racing thoughts Distractibility Psychomotor agitation Engaging in unrestrained buying sprees, sexual activity, substance use, etc.

Pitfalls of Diagnosis
Only one manic episode required for diagnosis of bipolar (depressed episode not necessary) Ask patients presenting with depression about symptoms of mania Can use the Mood Disorder Questionnaire (http://www.dbsalliance.org/pdfs/MDQ.pdf) during depressed or manic phase to assist with diagnosis

Types of Bipolar Disorders


Bipolar IFull manic episodes (at least 1 week duration) Bipolar IIHypomanic episodes (at least 4 days duration) Cyclothymia Unspecified Bipolar Disorder (was Bipolar Disorder NOS in DSM-IV)

Bipolar Disorders

Bipolar I

Bipolar II

Cyclothymia

2. Which is NOT true about treating bipolar disorder?


A. Valproic acid is most effective for acute mania with rapid cycling B. Women with bipolar disorder may continue on lithium if they become pregnant C. Lithium is effective in 80% of cases D. Valproic acid is the most prescribed mood stabilizer

Need for Treatment


Untreated mood swings tend to accelerate and become rapid cycling (known as kindling) To prevent suicide To prevent patients embarrassment and disruption of lives (many patients will remember manic actions)

Mood Stabilizers (Used for Bipolar I, II, & Cyclothymia)


Lithium
Usual dose 300 mg three or four times a day Usually works in 9-10 days

Anticonvulsants
Valproic acid Carbamazepine Lamotrigine Oxcarbazepine (NOT phenytoin, phenobarbital or gabapentin)

Lithium
Is a salt, not a drug with a complex structure Is excreted by kidney unchanged by body Works in 80% of Bipolar I cases May prevent depressive swings, but not as effective for treating them; may need to add antidepressant

Side Effects of Lithium


Fine tremor Contraindicated in pregnancy
(Ebsteins anomaly in 1st trimester)

If toxic:
Nausea & vomiting Diarrhea Ataxia Coma & death

Increased urination

Lithium
Lithium is less effective in:
Rapid cycling bipolar illness Poorly compliant patients

Rapid discontinuation can cause relapse Lithium is dialyzable if patient is toxic

Lithium Levels
Must check 5 days after starting or changing dose Seek to maintain 0.8-1.0 mEq/L levels to prevent relapse (acute mania may require levels of 1.2)

Lithium Monitoring
Long-term effects on thyroid (reversible) and kidney (irreversible). Check prior to starting and every 6-12 months Avoid dehydration and diuretics, careful with NSAIDs (ibuprofen, naproxen, etc.) EKG for patients over age 40 (can rarely cause junctional rhythm)

Valproic Acid (Depakote)


Is most frequently prescribed mood stabilizer May be more effective for rapid cycling bipolar disorders (> 4 swings per year) Loading dose is 10 mg per pound per day
Ex: 150 lb. man = 1500 mg per day = 500 mg 3 times/day

Usually start at 1/2 to 2/3 daily dose as outpatient

Side Effects of Valproic Acid


GI upsettake with food Liver dyscrasiascheck liver panels Occasional sedation Usually well tolerated Levels availabletherapeutic levels between 50-100 mcg/mL

Carbamazepine (Tegretol)
May also be more effective for rapid cycling bipolar disorders Starting dose 100-200 mg three times a day May cause blood dyscrasiasmust check blood counts Levels availablewant levels between 8-12 mcg/mL

Other Anticonvulsants
Less proven, but may be effective if previous meds fail:
Oxcarbazepine (Trileptal) Topiramate (Topamax)usually used with another mood stabilizer ?Gabapentin (Neurontin)?poor evidence

Bipolar Disorder: Treatment


Lamotrigine (Lamictal) Bipolar depression Rash in 5%; may progress to Stevens-Johnson syndrome Atypical antipsychotics Olanzapine (Zyprexa), quetiapine (Seroquel), risperidone, ziprasidone (Geodon), aripiprazole (Abilify) Antidepressant effects; also effective in mania Can use lower doses for bipolar than psychosis Weight gain, lipid/glucose abnormalities

Anxiety Disorders
Panic disorder Agoraphobia Specific phobia Social anxiety disorder Generalized anxiety disorder

3. A patient presents with panic attacks 2 to 3 times a week. She can be treated with:
A. B. C. D. E. Quetiapine Buspirone Propranolol Fluoxetine Bupropion

Panic Disorder
Can occur with or without agoraphobia Agoraphobia is now a separate diagnosis in DSM-5 Is a discrete, unprovoked psychophysiological event Almost always (90%) comorbid with another illness Female:male ratio is 2:1

Symptoms of Panic Attacks


Sudden onset and escalation of extreme anxiety, fear, and apprehension Accompanied by somatic complaints such as feeling dizzy, lightheaded, faint, tremulous, short of breath, and sweating Patients often state, I am about to die, or I am going crazy. 25% will have nocturnal attacks

Treatment of Panic Disorder


SSRIs
Best tolerated Good response reported with all SSRIs

Venlafaxine (Effexor) Tricyclics (imipramine & desipramine)


Patients sensitive to activating effects Start with geriatric" doses, 10-25 mg

Treatment of Panic Disorder


MAOIs
May be more effective than tricyclics 3rd or 4th line due to dietary restrictions and risk of hypertensive crisis

Buspirone (Buspar) is not effective Should treat for at least 12 months Psychotherapy helpful if agoraphobia does not respond to drug treatment of actual panic attacks

Specific Phobia
11% lifetime prevalence Some phobias common and culturally or family related Little comorbidity Often dont seek help Treated with cognitive behavioral therapy and graded desensitization

Social Anxiety Disorder


Known as Social Phobia in DSM-IV Fear of performance situations
Speeches and presentations Meeting new people Eating in crowded places

May affect 2% of the population

Treatment of Social Anxiety Disorder


SSRIs surprisingly effective Phenelzine (MAOI) Buspirone not effective Beta blockers reduce tremors, sweating, etc., but do not help subjective anxiety

4. In RCTs, drug vs. placebo, which of the following has not been shown to be beneficial in the treatment of generalized anxiety disorder? A. Beta blockers B. Benzodiazepines C. Antidepressants D. Cognitive therapy

Generalized Anxiety Disorder (GAD)


Involves excess anxiety and worry for more than 6 months Accompanied by at least 3 physical symptoms: restlessness, fatigue, poor concentration, muscular tension, irritable bowel symptoms, sleep disturbance, etc.

Generalized Anxiety Disorder (GAD)


Female:Male ratio is 2:1 GAD patients say they have been anxious all of their lives and that they worry about everything Many GAD patients are shy, compliant, perfectionist, and are concerned with their own failures and imperfections.

Generalized Anxiety Disorder (GAD)


Major differential diagnosis involves major depression, frequently comorbid Is chronic; 50% still diagnosed GAD at 5 year follow-up

Treatment of GAD
Must individualize treatment SSRIs effective for generalized anxiety Venlafaxine also useful Benzodiazepines useful for immediate relief, or if above meds fail. Beta blockers do not relieve generalized anxiety

Other Treatments for GAD


Buspirone (Buspar) may be effective, especially if used as a 1st agent
Not a controlled substance, does not prevent ETOH or BZD withdrawal Delayed action, 1-2 weeks Daily bid dosing, not prn

Key Points for the Exam


Strong genetic component in bipolar disorder Lithium contraindicated in 1st trimester Anticonvulsants used most often for bipolar treatment Panic attacks respond to SSRIs, SNRIs, and tricyclics

Answers
1. 2. 3. 4. C B D A

Personality Disorders
Pattern of behavior that causes problems Behavior is rigid and lifelong Patient experiences little anxiety (ego-syntonic) Seldom seek treatment for personality disorder Etiology unclear
Maladaptive patterns are the result of dysfunctional early environments that prevent the evolution of adaptive patterns Some evidence points to a genetic basis Diagnosis should not be made until age 18 years and personality development is complete

Personality Disorders: Cluster A (Odd, Eccentric)


Paranoidsuspicious Schizoidloner Schizotypalodd loner General principles Uncomfortable in interpersonal situations, emotionally distant, hard to engage Do not respond appropriately to cues May not come in due to fear of personal contact Respect patient's need for personal distance Convey info in a clear, straightforward fashion Do not directly challenge strange ideas about health

Personality Disorders: Cluster B (Dramatic, Emotional, Erratic)


Antisocial: breaks rules, no guilt Borderline: suicidal gestures Histrionic: dramatic, somatic complaints Narcissistic: insensitive to others feelings General tendencies
Demanding, manipulative, unstable Personally inappropriate; cross boundaries

Physicians must be aware of their own emotions


Avoid familiarity; set clear limits Schedule regular visits

Personality Disorders: Cluster C (Anxious, Fearful)


Avoidantshy Dependentstays in harmful relationships Obsessive/compulsive (not OCD) perfectionistic, isolation of affect General principles: Underlying anxiety may significantly interfere with the doctor-patient relationship Provide reassurance; validate concerns; encourage participation, symptom reporting Schedule regular visits Set reasonable limits on time, expectations

Personality Disorders: Treatment Options


Psychotherapy: core treatment
Psychodynamic Interpersonal Cognitive Group Dialectical (skill-based, coping)

Medication: symptomatic
Antidepressants: SSRIs, nefazodone, mirtazapine; avoid TCA/MAOIs (OD risk) Anticonvulsants (divalproex) Antipsychotics (atypical)

Schizophrenia
Rates are equal in males and females
Onset in the teens and 20s 2% prevalence 10% lifetime suicide risk

Requires 6-month history of impaired functioning


Work, education, self-care

Characterized by disorganized thought (bizarre speech, behavior), loose associations


Word saladwords do not relate

Schizophrenia
Symptoms can be positive (eg, hallucinations) or negative (eg, social withdrawal) Social/family interventions improve outcomes Treatment: dopamine D2 blockers (atypical antipsychotics)
Noncompliance rates are high and equivalent across medication classes Delays in treatment worsen long-term outcomes

Atypical Antipsychotics
Clozapine (Clozaril) 100-900 mg/day Olanzapine (Zyprexa) 2.5-20 mg/day Quetiapine (Seroquel) 25-1000 mg/day Risperidone (Risperdal) 1-6 mg/day Ziprasidone (Geodon) 20-200 mg/day Paliperidone (Invega) 3-12 mg/day Aripiprazole (Abilify) 10-30 mg/day Iloperidone (Fanapt) 12-24 mg/day Lurasidone (Latuda) 40-80 mg/day Asenapine (Saphris) 10-20 mg/day

Atypical Antipsychotics
Have low D2 receptor binding and blockade, plus serotonin receptor blockade Fewer extrapyramidal side effects Less increase in prolactin Reduced risk of tardive dyskinesia Increased risk of weight gain Increased risk of metabolic syndromes

Key Points for the Exam


Personality disorders: psychotherapy firstline treatment Seldom seek treatment for personality disorders; are ego-syntonic 2nd generation antipsychotics (atypicals) reduce the risk of TD and EPS; increase risk of metabolic syndrome

Endocrine Disease: Practical, Evidence-Based, Clinical Pointers


B. WAYNE BLOUNT, MD, MPH JenCare & Emory

Learning Objectives
1. Identify the diagnosis and management of the common types of hypothyroidism, hyperthyroidism, & thyroid nodules. 2. Discuss the work-up of pituitary masses. 3. Additional slides for your study:
Adrenal Problems Parathyroid Problems Male Hypogonadism

Dr. Blount has nothing to disclose.

The AAFP has selected all faculty appearing in this program. It is the policy of the AAFP that all CME planning committees, faculty, authors, editors, and staff disclose relationships with commercial entities upon nomination or invitation of participation. Disclosure documents are reviewed for potential conflicts of interest and, if identified, they are resolved prior to confirmation of participation. Only those participants who had no conflict of interest or who agreed to an identified resolution process prior to their participation were involved in this CME activity.

Question 1
17 YO WF c/o swelling in her neck x 2-3 wks. 5-lb wgt gain and somewhat tired. ROS neg
PE: 510, 155 lb, BP 132/80, P 80, AF, thyroid diffusely enlarged, smooth & NT; remainder WNL

Question 1
What single test would you order for this patient? A. T4 B. RT3U C. TSH D. Ultrasound

Hypothyroidism
Female-male = 6:1 Prevalence: 1 in 300 people
In US

Causes:
Hashimotos* Ablation 2ndary: Lithium, interferon amiodarone Transient causes Central causes

Presentation
Fatigue** Weight Gain Dry Skin Cold Intolerance** Hair Loss Coarse hair Hoarseness* Goiter Slow DTRs* Constipation Depression Concentration loss* Myalgia* Hyperlipidemia* Macroglossia Bradycardia Lateral eyebrow thinning*

Diagnosis
Hx & PE: Look for presentations Lab: TSH Thyroid antibodies?
Others C/W Dx: High CPK, LDL, TGs. Proteinuria, Normocytic anemia

Question 2 Which of the following is true?


A. All brands of levothyroxine are bioequivalent B. The usual starting dose in the nonelderly is 1.6 mcg/kg/day C. The usual starting dose in the nonelderly is 1.2 mcg/kg/day D. You should recheck a TSH 3 weeks after a dosage change

Treatment
Start @ 1.6 mcg/kg/day
Start lower in the elderly (1.0-1.25) (Even 25 50 mcg/day) Re-evaluate 5-6 wks after dosage change Different products = different bioavailability

Treatment Principles
Avoid desiccated thyroid (doesnt work in many) Avoid triiodothyronine (doesnt work in many) Too much causes osteoporosis & A Fib Take on fasting stomach & wait 30 mins before eating. (Can do a weekly dosing) Watch other interactions:
Iron, Carafate, cholestyramine, antacids, anticonvulsants, grapefruit, amiodarone, lithium, SSRIs, retinoids

Treatment Principles
If TSH WNL but patient not feeling well, consider:
A. Getting TSH to < 2.5 If still not to par, B. Problems with conversion of T4 to T3

Nutrient deficiency: Heavy metals: selenium, chromium, zinc, iron, copper, mercury, lead Iodine Vitamins: A, B2, B6, B12, D, E

Conversion Problems
Meds: steroids
OCPs Beta blockers Lithium SSRIs Phenytoin Iodinated contrast agents Theophylline Chemotherapy Fluoride Opiates Estrogen

Conversion Problems
Stress Aging ETOH Fasting Radiation Cruciferous vegetables (in excess) Receptor antibodies Low ferritin Pesticides Soy (excess) Hemochromatosis Smoking Kidney dz

Question 3
42 YO WM c/o fatigue, wgt loss, voracious appetite, hand tremor, HAs, decreased exercise tolerance; all x 4 weeks
PE: 60, 150 lb, thyroid diffusely large and NT, + fine hand tremor

Which of the following lab tests are indicated?


A. B. C. D. E. TSH T4 T3 Thyroid antibodies All of the above

Hyperthyroidism
Female-Male = 8:1 Prevalence: 0.2 % Causes:
Graves Multinodular goiter Adenoma Thyroiditis Ingestion

Presentation
Nervousness Palpitations Heat intolerance Tremor Fatigue* Insomnia HA* Irritability* Weight loss Increased appetite Hyperdefecation Mental changes* DOE*

Diagnosis
Hx: Ask about presenting Sx
PE: Wgt Pulse CV Eye BP Thyroid Neuromuscular Skin

Workup of Hyperthyroidism
TSH (Duh!) Free T4 & T3 A Rec CBC B Rec Radioactive uptake scan A Rec Maybes: ESR, ultrasound, thyroid antibodies C Rec All of above are after a good Hx & PE

Question 4
Patient lab results:
Low TSH High T3 & T4 Scan: Diffuse increased uptake

The most likely diagnosis is

A. B. C. D.

Hashimotos Graves Multinodular goiter Ingestion

E. Amiodarone administration

Graves Disease
Most common cause of hyperthyroidism Caused by TSH receptor-stimulating antibodies.
Other Causes of Hyperthyroidism:
Hashimotos Amiodarone Postpartum thyroiditis Metastatic thyroid cancer Iodine Hyperemesis gravidarum

Differential Diagnosis
Graves
TSH Scan Low Diffuse uptake High

Adenoma
Low Nodule

Multinodular
Low Multiple nodules High

T4

High

Question 5
The preferred definitive treatment for Graves disease is: A. Surgery B. Radioactive iodine ablation C. Antithyroid drugs D. Close monitoring

Graves Disease
Radioactive iodine is the TOC: A Rec
Except perhaps in cases with ophthalmopathy: B Rec

Surgery is uncommon today

Drugs: PTU or methimazole; & beta blockers Insufficient evidence for Chinese herbal meds: I Rec, Cochrane 2007

Graves Disease Treatment


Methimazole much safer than PTU With PTU risk of serious liver injury is:
Adults: 1 : 10,000 Peds: 1 : 2,000

PTU now considered a 2nd-line agent


EXCEPT during pregnancy & lactation

A Rec FDA: June, 2009


Monitor aminotransferases & CBC in both: C Rec

Question 6
46 YO WF c/o neck mass x 6 wks
ROS negative PE: All WNL except palpable 2-cm firm mass in right lobe of thyroid

What is the initial diagnostic test for this patient?


A. B. C. D. TSH Fine-needle aspiration Nuclear thyroid scan Surgery

Thyroid Nodules
Work up all nodules
1 in 20 is malignant

Start with TSH* SOR A TSH results determine further workup

Nodule Workup
IF LOW TSH
? Ultrasound

NML OR HIGH TSH

I-123 SCAN

FNA

COLD

HOT

FNA

ENDOCRINE OR SURGERY

How Do We, FM, Encounter Pituitary Problems?


Symptom presentation
Neuro Sx Hormonal abnormalities

The Incidentaloma
(Nonfunctioning pituitary tumors)

Clinical Presentation

Most common are endocrine abnormalities hyper-/hypo-secretion of pituitary hormones: Sx depend on hormone secreted HA Vision changes bitemporal hemianopsia and superior temporal defects

Endocrine-Active Pituitary Adenomas


Prolactin Amenorrhea, galactorrhea, impotence Growth hormone Gigantism and acromegaly Corticotropin Cushings disease TSH - Hyperthyroidism

Differential Diagnosis of a Sellar Mass


Benign Tumors
Pituitary adenoma (most common sellar mass) Craniopharyngioma Meningiomas

Malignant Tumors
Primary
Germ cell tumor Sarcoma Chordoma Pituitary carcinoma

Metastatic
Lung Breast

So, How Do We Evaluate These Pituitary Masses?


Radiologically: MRI with and without gadolinium
Hormonally: Oversecretion Undersecretion

Summary
3 Major Thyroid Problems
Use TSH to start diagnosis in all 3

Pituitary Masses: The Work-up Extras: Subclinical Disease The Thyroid in Pregnancy Male Hypogonadism The Parathyroid Disorders Adrenal Disorders

Bibliography
Am Thyroid Assoc Management Guidelines For Thyroid Nodules. 2009 Bahn, R et al. Approach to the Patient with Nontoxic Multinodular Goiter. J Clin Endocinol Metab, May 2011, 96: 1202-12. Gaitonde DY, et al. Hypothyroidism: An Update. Am Fam Physician. 2012;86:244-51. Barbesino G. Drugs affecting thyroid function. Thyroid. 2010;20:763-70. Wiersinga WM. Thyroid disease manager. http://www .thyroidmanager.org/chapter/adult-hypothyroidism/ Levy EG, et al. Algorithms for diagnosis & management of thyroid disorders. www.thyroidtoday.com/ExpertOpinions

Answers
1. 2. 3. 4. 5. 6. C B E B B A

EXTRAS

Good Reasons for Endocrine Consult


Heart Disease Age < 18 Myxedema Coma Pregnancy Unresponsive to therapy Presence of nodule MEN syndrome

Common ECG Findings in HypOthyroidism


Bradycardia Flattened T Waves Low voltage

Myxedema Coma (HypOthyroidism)


Rare: 0.22 per million Labs: Hyponatremia, hypercapnia, hypoxia, anemia; High: CPK, prolactin, & lipids

Other Causes of Hyperthyroidism


Hashimotos will have tender neck, fever, dysphagia, high ESR or CRP Postpartum thyroiditis:
Prevalence = 7.5% Usually within 1 yr of parturition

Graves Disease Treatment


In pregnancy, PTU is TOC in 1st Trimester Switch to methimazole in 2nd & 3rd trimesters If breastfeeding, PTU is preferred as less is transferred thru milk C Rec; Cooper & Rivkees. J Clin Endocrinol Metab.
2009;94:1881-2.

Graves Pharmacotherapy
Joint decision-making Medical treatment is for 18 months and then attempt to wean: B Rec, Cochrane, 2009 Success rate of 30%-50%, but Recurrence in those is 50%

Risk for Cancer in a Nodule


Children Age < 30 or > 60 Radiation Rapid growth Obstructive Sx Cvcl lymphadenopathy Males Fam Hx Hardness > 4 cm Voice Diff

Sonographic Evidence of Cancer in a Nodule

PPV (%)

Taller than wide Solid appearance Microcalcifications Irregular margins

59.8 49.4 38.6 28.2

Purely cystic nodule is highly unlikely (< 2%)

What to Do with FNA Results


Result
Benign Malignant

Plan
Observe Excision

Indeterminate
Nondiagnostic

Excision
Repeat FNA

Positive FNA?
5% of samples Types of cancer
Papillary: Most common Medullary Anaplastic: Usually older pts

Thyroid Cancer Rx ?
Refer Surgery

Metastasis determines prognosis

Workup of Multinodular Thyroid for Cancer


Dynamic Contrast Medium-Enhanced MRI (DCE-MRI) is more accurate than FNA in detecting cancer in a multinodular gland: B Rec, Tezelman. Archives of Surgery, 2007 Negative Predictive Value = 100%
For FNA, its 58% PPV: DCE-MRI = 78.5%; FNA = 100% Diagnostic accuracy: DCE = 90%; FNA = 71%

If You Cannot Do a DCE-MRI for Multinodular Goiter


Start with a TSH Get an ultrasound FNA anything suspicious found on U/S If unsure, refer

Question
Which of the following organizations recommends screening for ASx thyroid dz? A. B. C. D. USPSTF AAFP ACP AACE

Only the ACP & Am Thyroid Assoc

ACP: Only in women > 60 YO Not in men ATA: All adults > 35 YO q 5 years USPSTF & AHRQ give it an I recommendation

Subclinical Hypothyroidism
Prevalence: 5%-17%

Risk for progression to overt dz: 8%-18% Look for Sx Treat if TSH > 10, attempting conception, or + thyroid peroxidase Ab Be observant for overtreatment: Osteoporosis, A Fib Treatment does NOT result in improved survival or morbidity, nor QOL nor Sx: A Rec, Cochrane, 2007

Subclinical Hyperthyroidism
Subclinical hyperthyroidism: Any antithyroid drug is effective A Rec, Nygaard;
AFPs Clinical Evidence Concise. 2007;76:1014-7.

Prevalence: 0.1%-6% Risk higher in women, age > 60, + antibodies Higher osteoporosis, death from CV causes, A Fib Joint decision-making for treatment or not

The Thyroid in Pregnancy


Pregnancy has profound impact on the thyroid. Pregnancy can be called a stress test for the thyroid. Esp. hypothyroidism during pregnancy is harmful to maternal & fetal health & to childs future intellectual development

The Thyroid in Pregnancy


(Nice to Know, not on Test)
Thyroid can increase 10% in size. A 50% increase in thyroid hormones and in iodine need. 10% of gravid women in 1st trimester will be + for thyroid peroxidase or thyroglobulin Abs.
16% of them have hypothyroidism 33-50% develop pp thyroiditis

The Thyroid in Pregnancy


Levothyroxine is indicated with overt hypothyroidism Levothyroxine is indicated for subclinical hypothyroid (SbHypo) with + TPO Abs Gravid women with SbHypo not treated should have TSH & T4 checked q 4 weeks til 16-20 wks & then > once from 26-32 wks

The Thyroid in Pregnancy


Treatment not needed for isolated low T4 Women already on levothyroxine should increase dose by 25-50% @ pregnancy. Go to 9 doses/week Antithyroid meds are NOT indicated for women with gestational hypERthyroidism For Graves, use PTU in 1st trimester, then methimazole

The Thyroid in Pregnancy


During PP thyroiditis toxic phase, dont need antithyroid meds. Check TSH q 2 months after toxic phase Can try to wean off replacement @ 6-12 months after starting Rx No radioactive iodine scanning during pregnancy

Pituitary Masses
What is the differential diagnosis of a lesion within the pituitary? What is the prevalence of previously unrecognized pituitary tumors? What is the appropriate evaluation of this abnormality?

Pituitary Masses
10-15% of all primary brain tumors 20-25% of pituitary glands at autopsy found to have adenomas 70% of adenomas are endocrinologically secreting (? study bias?) 25% of those with MEN-I develop pituitary adenomas Etiology is unknown Not associated with environmental factors

Radiological Evaluation of a Sellar Mass

MRI with and without gadolinium


Gadolinium contrast Normal pituitary takes up gadolinium more than does CNS tissue Microadenomas often take up gadolinium less than normal pituitary

CT
Calcification in a craniopharyngioma or a meningioma is seen better by a CT than by MRI

PET Scan
Uptake by adenoma was 2-3 X greater than by craniopharyngiomas or meningiomas

Hormonal Evaluation of a Sellar Mass


Evaluation for potential pituitary hormone oversecretion Prolactin IGF-1 or growth hormone, 2 hours post 100 gm glucose load 24 hour urine free cortisol or 1 mg overnight dexamethasone suppression test Free T4, TSH

Hormonal Evaluation of a Sellar Mass


Testing for Pituitary Undersection Free T4 and TSH Serum cortisol and ACTH A(8:00 and 9:00 AM) Cortisol less than 3 meg/dL on two occasions, suggests adrenal insufficiency Cortisol > 10 meg/dL is rarely associated with adrenal insufficiency Cortisol of 3-10 meg/dL needs further testing Insulin tolerance test Metyrapone test

Hormonal Evaluation of a Sellar Mass


Testing for Pituitary Hormone Undersecretion Free testosterone, LH (male) Estradiol, FSH (female) Growth hormone deficiency If ACTH, TSH and gonadotropin deficiency exists and baseline GH is low, no further testing needed Otherwise stimulation with two agents Insulin, clonidine

Male Hypogonadism
Decrease in one or both of the two major functions of the testes. Hypogonadism Pathology Gonadotropins Testosterone Sperm count

Primary
Secondary

Testicular failure
Gonadotrope failure

Elevated
Low or low nl

Low/low nl
Low/low nl

Low/low nl
Low/low nl

Male Hypogonadism
Symptoms
Decreased libido Decreased energy Decreased sexual hair Infertility Erectile dysfunction Loss of muscle mass Decreased bone density

Male Hypogonadism
Primary
Chromosomal abnormalities
Klinefelter syndrome XXY

Infections Trauma Cryptorchidism Drugs Chemotherapy

Male Hypogonadism
Secondary
Pituitary tumors Infiltrative disease Idiopathic hypogonadotropic hypogonadism Kallmann Syndrome

Androgen resistance
5 alpha reductase deficiency Androgen receptor abnormalities

Male Hypogonadism
Total testosterone (AM draw) Abnormal Repeat testosterone (consider Free T), FSH, LH Low T, Low FSH, LH Secondary hypogonadism FSH, LH Low T, High

Prolactin, MRI, T4, TSH Primary hypogonadism

Male Hypogonadism

Therapeutic options
Androgen replacement
Testosterone enanthate or cypionate
Transdermal delivery Androgen 5 mg patch daily AndroGel 1 % 5 mg daily

Stimulating spermatogenesis
hCG, and hMG GnRH

History
Symptom onset Testicular size Breast enlargement Behavioral abnormalities Chemotherapy or radiation therapy Alcoholism Visual field defects Medications

Examination
Testicular size Pubic hair Gynecomastia Muscle mass Body proportions Fundoscopy & visual fields screening

Laboratory Testing
Testosterone Semen analysis Low sperm ct Low sperm ct Gonadotropins (LH/FSH) Elevated Low/low nl Diagnosis

Low/low nl Low/low nl

Primary hypogonadism Secondary hypogonadism

Evaluation of Men with Androgen Deficiency


Confirmed low testosterone

Check LH+FSH (SA if infertility)

High gonadotropins 1o

Low/low nl gonadotropins 2o

Karyotype

Prolactin, other pituitary hormones, iron studies, sella MRI

Testosterone Replacement
Primary goal is to restore testosterone levels to the laboratory reference range Prescribe only for patients with confirmed hypogonadism Role in treating decline in testosterone levels with aging uncertain Multiple preparations
Oral Intramuscular Transdermal Buccal

Oral Testosterone Preparations Alkylated testosterone more slowly metabolized by liver than native testosterone May not induce virilization in adolescents Untoward effects
Cholestatic jaundice Peliosis hepatis Hepatocellular carcinoma

Intramuscular Testosterone
Enanthate and cypionate esters of testosterone Lipophilic, leading to sustained release from muscle depots Side effects related to dosing or administration Regimens of 100 mg q wk to 300 mg q 3 wks acceptable Goal is a mid-cycle level near the middle of the laboratory reference range

Transdermal Testosterone
Patch (Androderm)
Apply to skin of upper arms and torso Delivers 5 mg testosterone/24 hr in continuous manner Approximately 1/3 of patients develop significant contact dermatitis

Gels (AndroGel, Testim)


Apply to skin of upper arms and torso Usually dosed as 5.0 g or 10.0 g of gel to deliver 50 mg or 100 mg testosterone, respectively in a continuous manner Reports of contact dermatitis and gel odor uncommon

Desirable Effects of Testosterone Therapy


Virilization (incompletely virilized men) Increased libido and energy Improved erectile function? Increased muscle mass and strength (8-10 wks) Increased bone mass (full effect ~ 24 mo)

Untoward Effects of Testosterone Therapy


Pain at injection site (IM preparations) Contact dermatitis (patch >> gel) Acne or oily skin Gynecomastia Aggressive behavior (adolescents) Short stature (adolescents) Increased prostate volume/PSA Urinary retention (BPH exacerbation) Sleep apnea Erythrocytosis

Contraindications to Testosterone Therapy


Very high risk of adverse outcomes Prostate cancer Breast cancer High risk of adverse outcomes Undiagnosed prostate nodule Unexplained PSA elevation BPH with severe urinary retention Erythrocytosis NYHA Class III or IV heart failure

Pre-treatment Screening
Digital rectal exam History of urinary retention (urodynamic studies, bladder US PRN) History of sleep apnea symptoms (polysomnography PRN) PSA (urology referral if > 4 ng/mL) CBC

Treatment Monitoring
Serum testosterone
IM testosterone: midpoint between injections, level near middle of reference range Patch: 3-12 hrs after applying new patch Gel: timing not critical Buccal pellet: immediately before or after new pellet

Prostate
DRE @ 3 months, then annually PSA @ 3 months, then annually Prostate biopsy if PSA > 4 ng/mL, PSA increases by > 1.4 ng/mL in 12 months, or PSA velocity > 0.4 ng/mL/yr

Red cell mass


CBC at 3 months, then annually If Hct > 54%, stop therapy, monitor for return to reference range, then resume therapy at a lower dose

Pituitary Masses

Disorders of the Pituitary with Oversection


Prolactinoma Cushings syndrome Acromegaly Gonadotroph Adenomas

Symptoms of Hyperprolactinemia
Women More gonads and breast Hypogonadal Infertility Oligomenorrhea Men More mass effect Hypogonadal Impotence Infertility

Amenorrhea
Galactorrhea Mass effect Headache Visual defects Hypopituitarism rare

Mass effects
Headache Visual defects Hypopituitarism (rare, but more common than in women)

Hyperprolactinemia Differential Diagnosis


Physiologic Causes
Pregnancy Pregnancy Pregnancy Nipple stimulation Emotional or physical stress

Hyperprolactinemia Differential Diagnosis


Associated disease states
HYPOTHYROIDISM HYPOTHYROIDISM HYPOTHYROIDISM Chronic Renal Failure Cirrhosis Adrenal insufficiency Chest wall lesions/Spinal cord lesions

Hyperprolactinemia Differential Diagnosis


Medications (Prolactin levels usually < 100)
Neuroleptics Tricyclic antidepressants Cocaine Monomine-oxidase inhibitors Opioids SSRIs

Metoclopramide
Verapamil Cimetidine

High estrogen
Methyldopa

Hyperprolactinemia Differential Diagnosis


Primary pituitary disease Prolactinoma Acromegaly Cushings syndrome Empty sella syndrome Lymphocytic hypophysitis

Prolactinoma
40-50% pituitary adenomas Rarely increase in size Clinical features 18% spontaneous miscarriages, corrected with treatment BMD 20-25% Gynecomastia Laboratory features Serial prolactin levels Macroprolactin

Prolactinoma - Diagnosis
Prolactin correlates with tumor size Rule of 200s Prolactin level > 200 is almost always a prolactinoma
Normal is < 25

Thorough history and physical exam to consider Differential Dx Pregnancy test Thyroid function studies (TSH and Free T4) MRI with contrast or CT scan with coronal cuts Formal visual field examination if > 10 mm in size Evaluation of remainder of pituitary function, if indicated

Treatment
Aims: fertility, potency, bones, tumor size Prolonged drugs may result in reversal (10-15%) Dopamine agonists: Bromocriptine, quinagoline, cabergoline (long-acting, but not for women who desire pregnancy) Side effects differ between drugs Rapid reduction of prolactin levels Tumor shrinkage in 70-90%

Prolactinoma - Treatment
Indication Tumor size - Macroadenoma (>10 mm in size) 94% of microadenoma remain unchanged at 6 years of follow up Symptoms Asymptomatic microadenoma may be followed by MRI at baseline, 1 year and 2 years Serial PRL values

Prolactinoma - Treatment
Symptomatic patients, Macroadenoma or enlarging microadenoma Medical therapy Treatment of choice
Dopaminergic agonist
Bromocriptine Carbergoline

Surgical removal (70% cure): Rsvd for med failures Transphenoidal hypophesectomy Complete ablation difficult with large tumor Pituitary irradiation

Cushings Syndrome
10-15 per million in general population Higher prevalence in patients with:
Diabetes Obesity Hypertension Osteoporosis

Peak Incidence in 25 40 yo No evidence-based guidelines

Cushings Syndrome
Full-blown syndrome fatal 50% at 5 yr untreated May have few typical features Usually microadenoma Good discriminants: Central obesity Ecchymoses * Plethora * Proximal weakness * Osteopenia/osteoporosis Hypertension WBC > 11.0 Purple striae > 1cm wide * Other features: myopathy, hirsutism, opportunistic infections, loss of libido (male) * = Most specific signs

Cushings Syndrome
Clinical Characteristics of Cushing Syndrome Obesity (centripetal) 94% Facial plethora 84% Hirsutism 82% Menstrual disorder 76% Hypertension 72% Muscular weakness 58% Back Pain 58% Striae 52% Acne 40%

Cushings Syndrome
Clinical Characteristics of Cushings Syndrome Psychological symptoms 40% Bruising 36% Congestive heart failure 22% Edema 18% Renal Calculi 16% Headache 14% Polyuria/Polydipsia 10% Hyperpigmentation 6%
Diagnosis is usually delayed because Sx are nonspecific

Laboratory Diagnosis
1st step is to determine hypercortisolemia Cortisol levels (circadian cycling) AM cortisol may be normal Raised midnight cortisol 24-hr urinary free cortisol Not affected by obesity, drugs, medical conditions Need to measure creatinine (ratio unreliable) 4x normal unequivocal, lower uncertain Midnight salivary cortisol Raised in medical/psychiatric illness

Laboratory Diagnosis
Dexamethasone suppression testing
Corticotrophs retain some sensitivity to glucocorticoid feedback
0.5mg q 6h for 2 days (low dose) 2mg q 6h for 2 days (high dose) suppresses in pituitary CS

Sensitivity & specificity vary widely

2nd step is to determine the source of the hypercortisolemia

Diagnosis
Inferior petrosal sinus sampling
Best test for localising ACTH-dependent Cushings Cushings syndrome: petrosal > peripheral ACTH Ectopic ACTH: petrosal = peripheral Diagnostic accuracy better when CRF given ( ACTH in Cushings syndrome)

MRI of head; if negative then adrenal ACTH level

Diagnosis
3rd step is to exclude exogenous exposure 4th step: exclude physiologic causes Do one of the above tests TWICE Variability of levels and suboptimal sensitivity & specificity Last step: refer

Addisons Disease
Primary: Atrophy or destruction of adrenal glands. 2ndary: Inadequate secretion of ACTH from pituitary Diagnostic tests:
Serum electrolytes Blood glucose CBC CT scan of adrenals MRI of adrenals

Addisons Disease
Clinical presentation
Malaise, fatigue Hyperpigmentation Low Blood Pressure Weight loss N&V Muscle cramps Irregular menses Salt craving

Treatment
Replace: Cortisol Aldosterone

Parathyroid
Hyperparathyroid
Overproduction of PTH from glands
Most often a tumor Can be due to severe CKD

Hypoparathyroid
Only 900 cases per year in U.S. I would not expect any questions on such an uncommon disease Treatment is to restore calcium & mineral balance thru Ca+++ & Vit D supplements

Hyperparathyroid Presentation:
Bone pain Depression Frequent urination Kidney Stones Nausea Loss of appetite

Treatment:
Locate & remove tumor surgically

Behavioral Medicine III:


ADHD, Autistic Spectrum Disorders, and OCD
Stanley P. Oakley, Jr, MD, FAPA

Professor Department of Psychiatric Medicine The Brody School of Medicine at East Carolina University Greenville, North Carolina

Disclosure Statement
Dr. Oakley has nothing to disclose.

The AAFP has selected all faculty appearing in this program. It is the policy of the AAFP that all CME planning committees, faculty, authors, editors, and staff disclose relationships with commercial entities upon nomination or invitation of participation. Disclosure documents are reviewed for potential conflicts of interest and, if identified, they are resolved prior to confirmation of participation. Only those participants who had no conflict of interest or who agreed to an identified resolution process prior to their participation were involved in this CME activity.

Just read part of an incredible synopsis of an article about Attention Deficit something or other.

C. OBrien

Learning Objectives
1. Recognize the characteristics of ADHD and autistic spectrum disorders in the child and adult. 2. Cite the management of these conditions with the following: a. Behavioral therapy b. Pharmacotherapy

3. Recognize the clinical presentation of obsessive compulsive disorder


4. List the treatment options for obsessive compulsive disorder

1. Which of the following comorbidities is most common in boys with a diagnosis of ADHD?
A. Oppositional defiant disorder (ODD) B. Conduct disorder

C. Bipolar disorder
D. Learning disability

2. Which of the following would NOT be consistent with a diagnosis of ADHD?


A. Hyperactive-impulsive or inattentive symptoms causing impairment before age 7 years B. Impairment from symptoms is observed only at school, and not at home

C. Clear evidence of a clinically significant impairment at a teens part-time job


D. Six or more criteria present for > 6 months

Attention-Deficit/Hyperactivity Disorder (ADHD)


Affects 5%-8% of school-aged children Persists into adulthood (3-4% of adults) Specify:
Predominantly Inattentive (DSM-5, 314.00) Predominantly Hyperactive/impulsive (DSM-5, 314.01) Combined (DSM-5, 314.01)

Diagnosis of ADHD
Six or more symptoms present Causes impairment in 2 or more settings:
School, work, or home/personal life Frequently comorbid with other childhood disorders

Inattentive Symptoms
Makes careless mistakes Difficulty sustaining attention Does not listen Fails to finish tasks Poor organization Loses important belongings Distractible Forgetful Avoids jobs that require sustained mental effort

Hyperactive Symptoms
Fidgets Difficulty sitting still Constantly restless Constantly driven Talks excessively Interrupts conversations Cant wait turn

3. Which is true about adults with ADHD?


A. They are unlikely to have their symptoms confirmed by spouses, coworkers, or employers B. They are more likely to be inattentive than hyperactive
C. They have a lower incidence of substance abuse than age-matched peers D. They are unlikely to have children with ADHD

Adult ADHD
Strong genetic basis
70% heritability (among highest for mental health disorders)

Is more likely to be inattentive type (hyperactive type is picked up in childhood) Can be comorbid with impulse disorders (gambling, substance abuse)

Adult Symptoms of ADHD


Poor job performance Frequent changing of jobs Career/academic underachievement Poor daily management
Paying bills, completing chores

Chronic stress from failures Relationship difficulties from inattention and forgetfulness

Diagnosis of ADHD
Meets DSM-5 criteria Various checklists
Conners Comprehensive Behavior Rating Scales Vanderbilt Rating Scale Wender Utah Rating Scale (for adults) Brown ADD Rating Scales

Formal psychological testing for ADHD Therapeutic trials dont work

Differential Diagnosis & Comorbidity


Newer estimates of comorbidity rates: Non-comorbid ADHD: 30% ODD: 60% males, 30% females Conduct disorder: boys > girls Depression: 30-40% Bipolar disorder: 20% Anxiety: up to 25% Learning disabilities: up to 30%

4. Research has shown that the optimal treatment approach for children with ADHD is:
A. B. C. D. Medication plus behavior modification Medication alone Behavior modification alone Changing the childs nutritional habits

Treating ADHD: Evidence-based Medicine


The optimal treatment approach for children with ADHD is medication with behavior modification
Multimodal Treatment Study of Children with ADHD: 600 children, ages 7-9 years Randomly assigned to 4 groups (medication, behavior modification, combined, neither) Combined approach superior in all areas Medication alone was superior to behavior modification alone

Stimulants
Greater than 80% response rate Stimulants improve ADHD by:
Blocking reuptake of dopamine and norepinephrine at the presynaptic neuron Amphetamines directly release catecholamines Inhibiting monoamine oxidase

Goal is to decrease inattention, impulsivity, hyperactivity FDA indication for ADHD

Use of Stimulants for ADHD


Schedule II controlled substance Also used for narcolepsy Need psychological evaluation to confirm ADHD and rule out learning disorder prior to use Side effects include insomnia, weight loss, and tics

Treatment of ADHD: StimulantsMethylphenidate


Short-acting methylphenidate
Methylphenidate Ritalin Focalin

Long-acting methylphenidate
Concerta Ritalin LA, Ritalin SR Metadate CD

Transdermal form available

ADHD Stimulants-Amphetamines
Short-acting amphetamine
Dextroamphetamine Adderall (mixture of amphetamine salts)

Long-acting amphetamine
Dexedrine Spansule Adderall XR Lisdexamfetamine (Vyvanse)

Side Effects of Stimulants


Anorexia Insomnia Weight loss, probably no effect on growth Irritability, dysphoria, withdrawal Headaches Abdominal pain Tics (4% incidence per year is baseline for this population)

5. A 13-year-old female with a history of anorexia nervosa is diagnosed with ADHD, inattentive type. What is the most reasonable FDA-approved treatment option?
A. Methylphenidate B. Bupropion C. Amphetamine/dextroamphetamine (Adderall) D. Atomoxetine (Strattera)

Nonstimulant Drugs for ADHD


Bupropion (not with seizures) Atomoxetine (Strattera)
Not a controlled substance Is a norepinephrine reuptake inhibitor Good for patients who find stimulants too activating, or patients with substance abuse history FDA indication for ADHD

Nonstimulant Drugs for ADHD


Alpha 2 agonists Clonidine Guanfacine Imipramine

Nonpharmacologic Treatment for ADHD


ScheduleKeep same routine Organize home and office items Use notebook organizers For adults, the book Driven to Distraction For children, clear and consistent guidance, rewards for following rules and successes

6. Which of the following is FALSE about the current recommendations regarding adverse cardiac outcomes with ADHD medications?
A. Laboratory testing prior to starting medication is at the physicians discretion B. All patients should have an EKG performed prior to starting medication for ADHD

C. Patients with family histories of sudden cardiac death should have an echocardiography performed prior to starting ADHD medication D. The risk of sudden cardiac death is higher in children treated with stimulants than in the general population

Cardiac Recommendations
AAP did not support the AHA recommendation that EKG be performed in ALL patients in advance of ADHD medication use Risk of sudden cardiac death ADHD medication greater than general population Testing (EKG, echo) should be performed: Family history of sudden cardiac death Patient report of chest pain, shortness of breath, syncope/dizziness before/after medication use Abnormal examination findings (initial and f/u) Laboratory testing: at physicians discretion

Differentiating Bipolar Disorder and ADHD


Get a good history, including family history ADHD is known to be developmental, and symptoms can be seen in infancy ADHD is known to be continual, not episodic Mood symptoms can be secondary to frustration from ADHD, with short-lived tantrums Grandiosity needs to be seen in the context of development

Differentiating Bipolar Disorder and ADHD


Bipolar disorder is more common after the age of 12 Bipolar disorder is usually episodic Relatives of children with ADHD rarely have bipolar disorder Relatives of children with bipolar disorder frequently have bipolar disorder Bipolar disorder is not a label to use casually, especially when criteria are not clear in children

7. Which is true of autism spectrum disorders?


A. Onset always occurs in infancy
B. Children with Aspergers syndrome have the same level of intellectual disability as those with autistic disorder C. Autistic disorder is equally as prevalent in boys as in girls D. Autistic disorder has an earlier onset than Aspergers syndrome

Autism Spectrum Disorders


Neurological disorders usually evident by age 3 years Difficulty in talking, playing with other children, and relating to others, including family Characterized by severe and pervasive impairment in several areas of development:
Social interaction skills Communication skills Stereotyped behavior, interests, and activities

Autism Spectrum Disorders


In DSM-5, four separate diagnoses are combined into a single condition known as Autism Spectrum Disorders They previously were:
Autistic Disorder Aspergers Disorder Childhood Disintegrative Disorder Retts Syndrome

Autism Spectrum Disorders


Autistic disorder (early infantile or childhood autism)
4 times more common in boys than in girls Moderate to severe range of communication, socialization, and behavior problems Most also have intellectual disability

Rett syndrome (primarily females)


Development is normal in the first 6-18 months Regression or loss of abilities Meaningless gestures or movements

Childhood disintegrative disorder (rare)


Regression in multiple areas of function following a period of at least 2 years of normal development Onset by age 10 years

Aspergers Syndrome Later onset than autistic disorder Characteristics


Autism Spectrum Disorders, continued


Lack of social skills (poor eye contact, anxiety) Difficulty understanding subtleties used in conversation Difficulty with social relationships Poor coordination Poor concentration Restricted range of interests

Average to above average intelligence/language skills


Incorrectly referred to as high-functioning autism

Obsessive Compulsive Disorder (OCD)


ObsessionsRecurring, unwanted thoughts CompulsionsRepetitive behaviors that reduce anxiety caused by obsessive thoughts

Obsessive Compulsive Disorder (OCD)


Occurs in 1% of adults Obsessive thoughts produce anxiety, leading to repetitive actions (compulsions) that reduce anxiety Patients are aware that these are irrational, but if stopped will lead to incapacitating anxiety

OCD Comorbidities
1/3 of patients will have major depression 2/3 of patients will have history of depression at some time 6% will be delusional and have no insight

Biology of OCD
Strong genetic component (twin studies) Involves serotonin system of the brain (since only serotonergic drugs are effective)

Common Symptoms
Obsessions Contamination 48% Doubt 47% Symmetry 45% Fear of aggression 36% Somatic obsessions 35% Sexual obsessions 22% Compulsions Checking Washing Need to confess thoughts or guilt Need for symmetry Counting 62% 46% 41% 40% 30%

(Hoarding is now a separate disorder in DSM-5)

Treatment of OCD
Use high-dose SSRIs first If SSRIs fail, use clomipramine (tricyclic) 50% symptom relief is a good medication response Behavioral and cognitive therapies may help 90% of patients relapse if treatment is stopped

Treatment of OCD
High-dose SSRIs
Fluvoxamine 300 mg/day Paroxetine 60 mg/day Sertraline 200 mg/day Fluoxetine 80 mg/day

Clomipramine (tricyclic)
300 mg/day

Treatment of OCD
Drugs can take up to 10 weeks to fully maximize effects Behavioral treatments focus on thought stopping, and flooding (repeating the obsessive thought to desensitize)

Key Points for the Exam


ODD: most common ADHD comorbidity ADHD has to be present in more than 1 setting Medication plus behavior modification: best ADHD treatment ADHD adults are primarily inattentive Aspergers is not high-functioning autism OCD requires continued treatment, usually with high-dose serotonergic agents

Answers
1. 2. 3. 4. 5. 6. 7. A B B A D B D

Oppositional Defiant Disorder (ODD)


Pattern of negativism, hostility, defiance Angry/irritable mood:
Often loses temper Touchy & easily annoyed Angry & resentful

Argumentative/defiant
Often argues with adults Deliberately annoys Blames others Actively defies rules

Vindictiveness
Spiteful and vindictive, holds grudges

Treatment of ODD
There is no medication for ODD There is medication for co-morbid conditions that may exacerbate ODD Parent training, behavioral interventions

Conduct Disorder
A repetitive and persistent pattern of behavior in which the basic rights of others or major societal norms are violated for at least 6 months.

Conduct Disorder
Aggression to people and animals Destruction of property Deceitfulness or theft Serious violations of rules The disturbance causes clinically significant impairment Difficult to treat
May require off-label mood stabilizers May require inpatient treatment, if severe

Specific Learning Disorder


Significant discrepancy between school level, chronological age, intelligence, and academic performance or achievement This discrepancy significantly interferes with academic and/or social functioning Treatment: educational modifications Medication: only if a comorbidity exists that requires medication use

Common Learning Disorder Deficits


Developmental speech/language disorders
Articulation, receptive/expressive language

Academic skills disorders


Dyslexia (reading), dyscalculia (math), dysgraphia (writing)

Nonverbal learning disability


Visual-spatial, visual-motor, sensory, motor

Central auditory processing deficit


Distorted or incomplete auditory messages

Urologic Problems: Practical, Evidence-Based, Clinical Pointers


B. Wayne Blount, MD, MPH JenCare & Emory

Learning Objectives
1. Know the work-up & Dx of hematuria. 2. Know the signs, Sx, Dx, & Rx of prostatitis and BPH. 3. Know the USPSTF recommendation for screening for prostate cancer. 4. Know the work-up & Dx of urolithiasis U.I. will be covered in the Geriatrics session & UTI in STIs. Additional slides for your study: More details on the above topics + testicular pain.

Question # 1 Hematuria is defined as 2 of 3 samples with:

A. B.

Any number of RBCs per hpf. More than 3 RBCs per hpf.

C.
D. E.

More than 30 RBCs per hpf.


3+ blood on urine dipstick. Visibly red urine.

Definitions
Hematuria is defined as three or more RBCs per high-powered field on urine microscopy, from 2 of 3 specimens.

Take-Home Point #1: Positive dipsticks for blood should get microscopic confirmation.

Classification
CLINICAL
Gross Frankly bloody Macroscopic Red urine

Microscopic
Not discolored

PATHOPHYS
Glomerular Non-Glomerular

Age and Hematuria


Age (yr) Common
Glomerulopathy (IgA, Alport syndrome, thin BM disease, APSGN) Hypercalciuria with stones Congenital obstructive anomalies UTIs Sickle cell disease Viral infection

Uncommon
Factitious Fever HUS Hemophilia HSP Schistosomiasis AVMs or fistulae DIC Goodpastures syndrome Loin pain-hematuria syndrome Renal infarction Renal vein thrombosis Schistosomiasis Medullary sponge kidney
AVMs or fistulae Cyclic hematuria in women Endometriosis TTP Renal vein thrombosis Toxins (cantharidin, djenkol bean) LP-HS

0-15

15-50

Calculi Menstrual contamination Exercise UTIs PKD Sickle cell disease Intercourse Papillary necrosis BPH Cancer (renal, ureteral, bladder, prostate) Overanticoagulation PKD Prostatitis

>50

Take-Home Point #2:

Most serious hematuria is going to be due to: 1. Infection (UTI, prostatitis) 2. Stones 3. Malignancy (anywhere along the urinary system)

Take-Home Point #3:


Look for typical clusters of symptoms and signs to quickly and roughly differentiate between infection, stones, and cancer.

If still unsure

Back to the Microscope!

Is it glomerular or non-glomerular?

Glomerular: acanthocytosis (acanthothorn or spike) or casts.


Non-glomerular: isomorphic RBCs.

Question #2

62 YO WM C/O 2 months worsening difficulty starting urination with less force of stream & some dribbling.

PMH: Negative PE: 30 cc prostate; rest: WNL

The next most appropriate step is

A. Diagnose prostate cancer B. Diagnose acute prostatitis C. Diagnose benign prostatic hyperplasia D. Check a PSA level

Benign Prostatic Hyperplasia


Prevalence: 8% 31-40; 45% 51-60, & > 80% 80 YO Sx: LUTS: Reduced force of stream, hesitancy, terminal dribbling, sense of incomplete emptying, urgency, nocturia, frequency Complications: Acute urinary retention, recurrent UTIs, hydronephrosis, & renal failure

Benign Prostatic Hyperplasia


Lifetime risk of surgery = 29% 2 components: Dynamic muscle tension & bulky structural Use AUA symptom scoring index* (Level C Rec)
Mild (score < 7): Watchful waiting Moderate (score 8-19): Medical Rx Severe (score > 20): Surgery
See www.aafp.org/afp/20020701/77.html & end of H.O.

Question # 3
Which of the following meds would be inappropriate for this patient?
A. Saw palmetto B. Alpha-1 antagonist C. 5-Alpha reductase inhibitor D. Ciprofloxacin

BPH Meds
Alpha-1 antagonists (5 approved): Similar efficacy; different side effect profiles: terazosin & doxazosin more SEs; dynamic component; they work: Level A
Rec; Cochrane, 2008

5-Alpha reductase inhibitors (3 approved): Reduce size; need 6-12 MOs Rx for full effect; 2 approved with similar efficacy & S.E.s; They work: Level A Rec; Cochrane, 2008 NNT for hematuria = 2; NNT to prevent a TURP = 6

BPH Meds
Combination Rx MAY help: Level B Rec, Cochrane; (esp when > 30 cc volume) Saw palmetto is controversial & is no better than placebo (8/12)

Other BPH Treatments


TUMT: transurethral microwave thermotherapy Is effective when there is: No urinary retention No previous prostate procedure Prostate volumes between 30-100 mL Not as effective as TURP

Cochrane, 2007
Tadalafil (Cialis) Reason for improvement is unknown. Level B. Not for use with alphablockers

Question # 4
54 YO WM C/O 6 days of perineal pain, urgency & frequency, fever & myalgias. Never had before. The most likely Dx is:
A. Acute prostatitis B. Cystitis C. Chronic bacterial prostatitis D. Chronic nonbacterial prostatitis E. Urethritis

Prostatitis
Acute bacterial Chronic bacterial : in extra slides Chronic nonbacterial (CPPS): inflammatory & noninflammatory

Acute Prostatitis
A type of UTI Sx: Fever, chills, LBP, perineal pain, dysuria, urgency, frequency, myalgias, ? obstructive Sx PE: Tender, warm, swollen, firm & irregular UA & C&S sans massage

Acute Prostatitis Rx
Meds: Level C Rec
Tetracyclines TMP-SMX Quinolones

Duration: 3-4 weeks; Level C Rec

Erectile Dysfunction
ED is a robust predictor of all-cause mortality & CV events in men
Hazard ratio for mortality = 2.04 Hazard ratio for CV event = 1.62 With a dose-response increase with ED severity

Bohm, Circulation, March 15, 2010

Question #5 Which of the following is correct?


A. The 1st line treatment for E.D. is an oral phosphodiesterase 5 (PDE5) inhibitor. B. The 1st line treatment for E.D. is yohimbine. C. The 1st line treatment for E.D. is a vacuum device. D. The 1st line treatment for E.D. is weight loss.

Erectile Dysfunction
1st line therapy should consist of oral phosphodiesterase-5 (PDE5) inhibitors: NNT = 2.1: Level A Rec, Cochrane, 2007 PDE5 inhibitors are most effective in ED assoc with DM, spinal cord dysfunction, and ED caused by antidepressants: Level A Rec, Cochrane, 2007 PDE5 inhibitors can help in ED in nerve-sparing prostatectomy: Level B Rec, Bandolier, 2005 PDE5 inhibitor efficacy & side effects among the 3 are similar, but drop-out rates are lower for sildenafil: Level A Rec, Bandolier, 2005

Phosphodiesterase-5 Inhibitors: Adverse Effects


Vision disturbances Priapism Angina Sudden, permanent sensorineural hearing loss (May, 2010)

Erectile Dysfunction
Vacuum devices: Level B Rec, Bandolier, 2005 Yohimbine: NNT = 6.4: Level A Rec, Bandolier, 2000 Testosterone works in men with low testosterone (< 12 nmol/L), NNT = 2.1: Level A Rec, Bandolier, 2005 Alprostadil works: NNT = 3.5; Is not a 1st-line agent 2/2 side effects: Level A Rec, Bandolier, 2005 We dont know about apomorphine, phentolamine, or intracavernosal VIP: Level I Rec, Bandolier

Erectile Dysfunction: What Else Does & Doesnt Work


Trazodone: NO. Level A Rec, Bandolier, 2005 Fibrates & statins may contribute to ED: Level B Rec, Bandolier, 2007 Having a BMI > 30 is a risk factor for ED: Level B Rec, Bandolier, 2000 Losing weight in obese patients improves erectile function: Level B Rec, Bandolier, 2005

Prostate Cancer

Prostate Cancer Screening EBM


USPSTF : There is evidence against routine screening for prostate cancer by PSA or DRE. 2012 D Rec

This service has no benefit, or its harms outweigh its benefits.

Kidney Stones RISK FACTORS


Male Gender Age (to 65) Low urine vol. Situational Geography Heredity Diet (high Na) Low urine volume Meds

PRESENTATION
Abdominal Pain Renal Colic: Sudden; not relieved Hematuria

Relationship of Stone Location


to Symptoms
Vague flank pain, hematuria Renal colic, flank pain, upper abdominal pain Renal colic, dysuria, urinary frequency, anterior abdominal, & flank pain Renal colic, anterior abdominal & flank

Work-Up
History P.E. U.A. Imaging Labs

Imaging modality
Noncontrast helical computed tomography

Sensitivity (%)
95 to 100

Specificity (%)
94 to 96

Advantages
Most sensitive & specific radiologic test (i.e., facilitates fast, definitive diagnosis) Indirect signs of the degree of obstruction Provides information on nongenitourinary conditions

Limitations
Less accessible and relatively expensive No direct measure of renal function.

Management
(3 Principles)
Recognize emergencies Adequate analgesia Impact of size and location on Hx & Rx

Prevention (of Recurrences) Need to analyze calculi Labs

Summary
Hematuria BPH Prostatitis Prostate cancer Urolithiasis Extras: Scrotal pain

Bibliography
1. USPSTF. Screening for Prostate Cancer. 2005. 2. Stevermer JJ, Easley SK. Treatment of Prostatitis. AFP. 2000;61:301522. 3. NJAFP. CME Report: Diagnosis & Management of Overactive Bladder in Family Medicine. 2007; 2:1-14. 4. Stendardo S. Caring for Patients Who have BPH. AAFP CME Bulletin. 2007;6:1-6. 5. Ciftci, AO. Clinical Predictors for Diff. Diagnosis of Acute Scrotum, European J. of Ped. Surgery. Oct 2004. 6. Luzzi GA. Acute Epididymitis. BJU International. May 2001. 7. Fleisher G, Ludwig S, Henretig F. Textbook of Pediatric Emergency Medicine. 2006. 8. Sudakoff GS, et al., Multidetector CT Urography as the Primary Imaging Modality for Detecting Urinary Tract Neoplasms in Patients with Asymptomatic Hematuria, J Urol. 2008 March, 179(3).

Bibliography
9. Jang T, Schaeffer A. Chronic Prostatitis. (Clinical Evidence Concise). AFP. Aug 1, 2005. 10. Amer. Cancer Soc. Guideline for the Early detection of prostate cancer: Update 2010. Cancer J Clin. 2010. 11. Wilbur J. Prostate Cancer Screening. Am Fam Physician. 2008;78:1377-84. 12. Edwards J. Diagnosis & Management of Benign Prostatic Hyperplasia. Am Fam Physician. 2008;77:1403-10. 13. JFP. June 2012; vol 61:S1-S10. 14. Beers MH, et al., Merck Manual of Diagnosis and Therapy (18th print and online editions), Chapter 226: Approach to the Genitourinary Patient: Isolated Hematuria.

15. Cohen RA and Brown RS, Microscopic Hematuria, New England Journal of Medicine. 348:23, 5 June 2003.
16. Sharp VJ et al. Testicular Torsion. Am Fam Phys. 2013;88:835-40.

Answers
1. 2. 3. 4. 5. B C D A A

EXTRAS

Not All Red Urine is Hematuria

If the urine is visibly red, tea- or cola-colored, but there are < 3 RBCs/hpf, consider:

Hemoglobinuria (false + dipstick) Myoglobinuria (false + dipstick) Beeturia

Rhubarburia
Medications (phenazopyridine, methyldopa, senna, others) Porphyria

RED FLAGS
Smoking history History of persistent irritative voiding symptoms

Occupational exposure to chemicals or dyes History of recurrent or (benzenes or aromatic chronic urinary tract amines) infection History of gross hematuria Analgesic abuse

Age >40 years (>50, some History of pelvic irradiation sources say) History of urologic disorder or disease (not simple UTIs)

Source: Urology 2001;57(4)

Physical Examination
Vitals
Fever? (pyelo) HTN? (glomerulonephritis)

Extremities
Edema? (glomerulonephritis) rashes? (HSP, CTD, SLE)

Heart
New murmur? (endocarditis)

Rectal
BPH? nodules? (cancer) tenderness? (prostatitis, endometriosis)

Lungs
Crackles, rhonchi? (Goodpastures syndrome)

Abdomen
Masses? (cancer, obstruction) bruits? (renal ischemia)

A 7-year-old boy presents 2 weeks after an episode of pharyngitis because his mother noticed his urine was red. He has mild edema on examination.
A. Schistosomiasis B. Goodpastures syndrome C. Post-streptococcal glomerulonephritis D. Prostatitis

A 38-year-old woman with chronic pelvic pain presents with macroscopic hematuria. She has no fever, dysuria, or flank pain. She notes that her urine only turns dark red with or soon after her menstrual cycle.

A. Endometriosis B. Exercise-induced hematuria C. Polycystic kidney disease D. Polycystic ovarian disease E. Both B and C

If Its Glomerular...

Again: acanthocytes or casts in the sediment...


If no protein or renal failure, youre done for now.

But follow-up regularly!

If protein or renal failure, refer to nephrology! (Renal biopsy likely.)

If Its Non-Glomerular...
Regular-appearing, isomorphic RBCs.
Ask: Where is the bleeding from? Step 1: CT urogram. Look for the big anatomical lesions.

If no lesion, then- Step 2: Urine cytology (3 first AM samples) if abnormal, go to cystoscopy. Step 3: Is the patient high risk for malignancy--over 40, toxic exposures, irradiation, etc.? If yes, go to cystoscopy & consider repeat cytology at 6, 12, 24 and 36 months.

Take-Home Point:
A. Glomerular or Not? B. Glomerular - refer if protein or renal failure. C. NonGlomerular - do a CT-U, then cytology (if needed), then see how worried you still are.

The Take-Home Points for Hematuria


Positive dipsticks for blood should get microscopic confirmation R/O myo- or hemoglobinuria and decide glomerular vs. nonglomerular.

Top 3 suspects are infection, stones, and malignancy.


Look for illness patterns: Unilateral flank pain, afebrile, N/V (stones)

Obstructive Sxs, fever, prostate tenderness (prostatitis)


CVAT, fever, dysuria (pyelo)

If no easy answer, ask: Glomerular or not?

Glomerular - protein or renal dz? If so, refer to nephrology.


Not - 1. CT-U; 2. Cytology; 3. Cystoscopy.

BPH Symptom Score


Not at all < 1 in 5 times < half the time About half the time > half the time Almost always

1. In the past month, how often have you had a sensation of not emptying your bladder completely after you finished voiding? 2. In the past month, how often have you had to urinate again less than 2 hours after you finished urinating before? 3. In the past month, how often have you found you stopped and started again several times when you urinated? 4. In the past month, how often have you found it difficult to postpone urination?

BPH Symptom Score


Not at all < 1 in 5 times < half the time About half the time > half the time Almost always

5. In the past month, how often have you had a weak urinary stream? 6. In the past month, how often have you had to push or strain to begin urination? 7. In the past month, how many times did you typically get up to urinate from the time you went to bed until you arose in the morning?

Scoring key: 0 to 7 = mild; 8 to 19 = moderate; 20 or more = severe.


www.aafp.org/afp/20020701/77.html

BPH Meds
Terazosin, doxazosin, & alfuzosin work on all 3 receptor subtypes Tamsulosin relatively selects A & D Silodosin is relatively selective for A receptors Most common AE is dizziness: 2-14%; except for silodosin: Abnormal ejaculation in 10-22%

5-ARIs
Not a firm recommendation but usually use 5-ARIs when prostate > 30 mL or PSA > 1.5 ng/dL AEs are primarily sexually related: dec. libido, abnormal ejaculation, & erectile dysfxn

Erectile Dysfunction
Definition: The inability to achieve or maintain an erection sufficient for satisfactory sexual performance 18 million men in US

Erectile Dysfunction
Principal neurotransmitter for erection is nitric oxide Regulated by cGMP Return to flaccidity, cGMP is hydrolyzed to GMP by PDE5 ERGO: The PHE5 inhibitors

Chronic Bacterial Prostatitis Source of recurrent UTIs Similar Sx as Acute with ASx intervals WBCs + on pre- & post-massage UAs

C&S neg on pre- & + on post-massage

Chronic Bacterial Prostatitis Rx TMP-SMX as 1st line: Level C Rec Quinolone for Rx failures: Level C Rec Rarely: TUP of infected tissue for very Sx complete failures on Abx

Additional Question: Which of the following is not true for Erectile Dysfunction?
A. PDE5 inhibitors work in men with nervesparing prostatectomy. B. PDE5s can cause a sudden, permanent sensorineural hearing loss. C. Yohimbine works for ED. D. Statins can improve ED. Ans: D

You get a PSA result of 5.6 on a 51 YO WM. What is the next evidence-based step?
A. Repeat the PSA now B. Repeat the PSA with a free % C. Refer to urology D. Repeat the PSA in 1 year E. Do nothing

? Positive Screen ?
Biopsy is the gold standard Transrectal BX (TRUS): Office procedure sans sedation or analgesia

Prostate Cancer
Most common solid tumor among Am. men Around 200,000 diagnoses per year Risks: Age, family Hx, & race Protection?: ?Lycopenes? Level C evidence; selenium: Level B Rec; Vit E: Level I Rec 5-Alpha reductase inhibitors: Yes: Level B Rec, Prostate Cancer Prevention Trial

Prostate Cancer Screening EBM


ACP: Discuss with patients and individualize decision. C Rec, 2008 AUA: Screen > age 50 & with 10-yr life expectancy. C Rec, 2008 ACS: Discuss with asymptomatic men with a life expectancy of > 10 years. Use informed, shared decision-making. I, 2010

Screening (Extra Info)


If screen, following are suggestions: Average risk men: annual PSA (with or without DRE) starting @ 50, until life expectancy < 10 yrs. PSA cutoff of 4 C Rec African-American men & men with + F.Hx: As above, starting @ 45 Newer PSA assays have not resulted in better patient outcomes. B Rec If PSA < 2.5, can screen Q 2 yrs

Patient Discussion Points (Extra Info)


PC is an important health concern Benefits of screening & aggressive Rx are not proven DRE & PSA have both false +s & -s High risk for further invasive evaluation Rxs associated with significant morbidity Early detection may save lives & avert cancer-related morbidity, but there is no proof for that

Kidney Stone Labs


All Patients

CBC UA BMP

Urine C & S Ca PO4 Stone Analysis Urate Vitamin D

Kidney Stone Labs


Circumstantial

Hypercalcemia: PTH Abnormal Albumin: Ionized Calcium Hyperoxaluria: Oxalate level Sarcoidosis: ACE level & calcitriol

Hospitalization for Stones ?


Emergencies Refractory nausea Debilitation Extremes of age Refractory pain

Analgesia
NSAIDs: also spasmolytic Narcotics No NSAIDs < 3 days before lithotripsy (ASA < 7 days) Ketorolac

Manage the Stone


After adequate analgesia and ruling out emergencies

Principles here are stone size and location

Suggestions SOR: C
Stones < 4 mm Passage in 1-2 wks (Most in 4-6 weeks) Analgesia Strain urine F/U KUB q 1-2 wks Urology if not passed in 2 wks. (certainly 4 wks as comps 3x) RTC signs of sepsis

Stones < 5 mm or distal stones


Medical therapy: alpha-blocker, &/or nifedipine SOR: A

Stones > 10 mm
Urologic Consultation SOR: C

Stones 5 - 10 mm
Decide based on other parameters

Other Parameters
Location Composition Occupation Larger size

Location
Renal stones usually can be followed Distal ureteral stones will usually pass

Composition
Staghorn renal calculi to urology (assoc. with infections and kidney damage)

Occupation
Pilots cannot fly even with an asymptomatic stone

Larger Size
Renal calculi of 5 mm 2 cm: Extra corporeal lithotripsy Lower pole stones 5 mm 1 cm: ECL Ureteral stones 5 mm 1 cm: ECL Larger than 2 cm or when ECL contraindicated or not effective: Renal & proximal ureteral stones: Percutaneous nephrolithotomy

To Urology
Obstructed Infection Renal injury Solitary kidney

SOR: C

A Suggestion
Patient with abdominal pain

History and physical examination Renal colic suspected Diagnostic imaging

Patient is pregnant, or cholecystitis or gynecologic process is suspected

Patient has history of radiopaque calculi

All other patients

Ultrasound Examination

Plain-film radiography

Intravenous pyelography if CT is not available

Noncontrast helical CT

Stone detected

Stone not detected

Stone detected

Stone not detected

Clinical suspicion of urolithiasis

24-hr. Urinary Tests


Volume Calcium Oxalate Citrate pH Urate Creatinine Not done during acute stone

Do Twice

More Extensive Testing


Children Solitary kidney CRI/CKD Residual stone burden Infected stones Gout Intestinal disease

revention

All patients : 2-3 L water q day, 8-12 oz QHS (urine volume = 2 L/day) B rec NaCL (2g) C Rec unless Animal protein (8 oz) dietary excesses Oxalate Calcium in diet B rec (to 1200 mg/day)

Risk for Renal Failure


Hereditary stone diseases Struvite stones Infection associated calculi & obstruction Frequent relapses No. of urologic interventions Stone size

Causes of Scrotal Pain and Swelling


Pain
Testicular torsion Torsion of appendix testis Epididymitis Trauma Orchitis and others

Swelling
Hydrocele Varicocele Spermatocele Tumor

Torsion
Inadequate fixation of testes to tunica vaginalis at gubernaculum Torsion around spermatic cord
Venous compression to edema to ischemia

Epidemiology
Accounts for 30% of all acute scrotal swelling Bimodal ages neonatal (in utero) and pubertal ages
65% occur in ages 12-18yo

Incidence 1 in 4000 in males < 25 yo Increased incidence in puberty due to inc weight of testes

Predisposing Anatomy
Bell-clapper deformity
Testicle lacks normal attachment at vaginalis Increased mobility Transverse lie of testes Typically bilateral Prevalence 1/125

Torsion: Clinical Presentation


Abrupt onset of pain usually testicular, can be lower abdominal, inguinal Often < 12 hrs duration May follow exercise or minor trauma May awaken from sleep
Cremasteric contraction with nocturnal stimulation in REM

Up to 8% report testicular pain in past May have N&V

Torsion: Examination
Edematous, tender, swollen Elevated from shortened spermatic cord
Horizontal lie common (PPV 80%) Reactive hydrocele may be present

Cremasteric reflex absent in nearly all (unreliable in < 30 mo old) (PPV 95%) Prehns sign (elevation relieves pain in epididymitis and not torsion) is NOT reliable

Intermittent Torsion
Intermittent pain/swelling with rapid resolution (seconds to minutes) Long intervals between symptoms PE: testes with horizontal lie, mobile testes, bulkiness of spermatic cord (resolving edema) Often evaluation is normal if suspicious need GU follow-up

Diagnosis Time is Testicle Ideally -- prompt clinical diagnosis Imaging: 2ndary to clinical exam
Color doppler decreased intratesticular flow
False + in large hydrocele, hematoma Sens 69-100% and Spec 77-100% Lower sensitivity in low-flow pre-pubertal testes

Nuclear technetium-99 radioisotope scan


Show testicular perfusion 30-min procedure time Sens and spec 97-100%

Management
Detorsion within 6hr = 100% viability
Within 12-24 hrs = 20 50 % viability After 24 hrs = 0 - 10% viability

Surgical detorsion and orchiopexy if viable


Contralateral exploration and fixation if bell-clapper deformity

Orchiectomy if non-viable testicle

Never delay surgery on assumption of nonviability, as prolonged symptoms can represent periods of intermittent torsion

Manual Detorsion
If presents before swelling Appropriate sedation In 2/3rds of cases testis torses medially, 1/3rd laterally Success if pain relief, testes lower in scrotum Still need surgical fixation

Neonatal Torsion
70% prenatal, 30% post-natal Post-natal typically 7-10 days after birth Unrelated to gestation age, birth weight Post-natal presents in typical fashion
Doppler U/S and radionucleotide scans less accurate with low blood flow in neonates Surgical intervention if post-natal

Prenatal torsion presents with painless testicular swelling, rare testicular viability
Rare intervention in prenatal torsion

Torsion of Appendix Testis


Appendix testis
Small vestigial structure, remnant of Mullerian duct Pedunculated, 0.3 cm long

Other appendix structures Prepubertal estrogen may enlarge appendix and cause torsion

Torsion of Appendix Testis


Peak age 3-13 yo (prepubertal) Sudden onset, pain less severe Classically, pain more often in abd or groin Non-tender testicle Tender mass at superior or inferior pole May be gangrenous, blue-dot (21% of cases) Normal cremasteric reflex, may have hydrocele Inc or normal flow by doppler U/S

Torsion of Appendix Testis


Management supportive Analgesics, scrotal support to relieve swelling Surgery for persistent pain No need for contralateral expl.

Epididymitis
Inflammation of epididymis Subacute onset pain, swelling localized to epididymis, duration of days With time, swelling and pain less localized Testis has normal vertical lie Systemic signs of infection
Inc WBC and CRP, fever + in 95%

Cremasteric reflex preserved Urinary complaints: discharge/dysuria PPV 80%

Epididymitis
Sexually active males Chlamydia > N. gonorrhea > E. coli Less commonly pseudomonas (elderly) and tuberculosis (renal TB)

Young boys, adolescents often postinfectious (adenovirus) or anatomic


Reflux of sterile urine through vas into epididymis 50-75% of prepubertal boys have anatomic cause by imaging

Epididymitis Diagnosis
Leukocytosis on UA in ~40% of patients PCR Chlamydia + in 50%, GC + in 20% of sexually active 95% febrile at presentation Doppler and nuclear imaging show increased flow If Hx consistent with STD, CDC recommends:
Cx of urethral discharge, PCR for C and G Urine culture and UA Syphilis and HIV testing

Epididymitis Treatment
Sexually active treat with ceftriaxone/ doxycycline or ofloxacin Pre-pubertal boys Treat for co-existing UTI if present Symptomatic Tx with NSAIDs, rest Referral all to GU for studies to rule out VUR, post urethral valves, duplications
Negative culture has 100% NPV for anomaly

Evaluate Todays Sessions

www.aafp.org/reportcme/boardrev/indianapolis

Anda mungkin juga menyukai